Advanced Trauma & Orthopaedics Interactive Study Guide for PLAB/OSCE/MLA

Approach to a Swollen Joint

A swollen joint is a common presentation requiring a systematic approach to differentiate between various causes, including inflammatory, septic, crystal-induced, traumatic, and degenerative conditions. Accurate diagnosis is key to appropriate management and preventing long-term joint damage.

Overview of a Swollen Joint

Joint swelling (effusion or synovial hypertrophy) can be caused by a multitude of conditions. The key is to differentiate urgent conditions like septic arthritis from chronic ones. Important considerations include the acuity of onset, number of joints involved (mono-, oligo-, or polyarthritis), and associated systemic features.

Common Causes of Monoarthritis (Single Swollen Joint):

  • Septic Arthritis: Bacterial infection of the joint. Orthopaedic emergency.
  • Crystal Arthropathy: Gout (monosodium urate crystals), Pseudogout (calcium pyrophosphate dihydrate crystals).
  • Trauma: Haemarthrosis, ligamentous injury, fracture.
  • Reactive Arthritis: Often post-infectious (e.g., GI or GU infection).
  • Early presentation of Inflammatory Arthritis: Rheumatoid arthritis, psoriatic arthritis, spondyloarthropathies.
  • Osteoarthritis: Can have effusions, especially after overuse.
  • Haemarthrosis: Bleeding into joint (trauma, coagulopathy, anticoagulants).
🚨 Red Flag: Septic Arthritis

A hot, swollen, tender joint with restricted movement and systemic signs (fever, rigors) must be considered septic arthritis until proven otherwise. This requires urgent joint aspiration and antibiotic therapy. Delay can lead to irreversible joint destruction and sepsis.

🧠 Approach to Swollen Joint: “IOPRA”

Remember these key areas in assessment:

Infection (Septic Arthritis – RULE OUT FIRST!)
Osteoarthritis (Degenerative)
Pseudogout/Gout (Crystal Arthropathies)
Rheumatoid/Reactive/Psoriatic (Inflammatory Arthritis)
Abnormal bleeding/Trauma (Haemarthrosis/Injury)

History Taking for a Swollen Joint

A detailed history is crucial to narrow down the differential diagnosis.

  • Onset: Acute (hours/days – suggests crystals, sepsis, trauma) or Chronic (weeks/months – suggests OA, RA).
  • Number of joints involved:
    • Monoarthritis (1 joint)
    • Oligoarthritis (2-4 joints)
    • Polyarthritis (≥5 joints)
  • Pattern of involvement: Symmetrical vs. Asymmetrical; Additive vs. Migratory.
  • Pain: SOCRATES (Site, Onset, Character, Radiation, Associated symptoms, Timing, Exacerbating/Relieving factors, Severity). Night pain? Rest pain?
  • Stiffness: Morning stiffness duration (>30-60 mins suggests inflammatory). Stiffness after inactivity (“gelling” in OA).
  • Swelling: Onset, progression, timing.
  • Trauma: Any recent or past injury to the joint? Mechanism of injury?
  • Locking/Giving Way: Suggests internal derangement (e.g., meniscal tear, loose body).
  • Systemic Symptoms:
    • Fever, chills, rigors, malaise: Suggests infection (septic arthritis).
    • Weight loss, fatigue, night sweats: Could be systemic inflammatory disease or malignancy.
    • Rashes, nail changes (pitting, onycholysis – psoriatic arthritis).
    • Eye symptoms (uveitis, conjunctivitis – spondyloarthropathies, reactive).
    • Oral ulcers.
    • Recent GI/GU infection (dysuria, diarrhoea – reactive arthritis).
  • Past Medical History: Previous similar episodes, known arthritis (OA, RA, gout), diabetes, psoriasis, IBD, recent infections, coagulopathy, previous joint surgery.
  • Drug History: Diuretics (can precipitate gout), anticoagulants (haemarthrosis), steroids, immunosuppressants. Allergies.
  • Family History: Arthritis, gout, autoimmune diseases.
  • Social History: Occupation, hobbies, impact on daily activities, alcohol intake.

A 60-year-old man presents with sudden onset of severe pain, redness, and swelling in his right great toe (1st MTP joint) that started overnight. He is unable to bear weight. He had a similar episode 2 years ago. He takes bendroflumethiazide for hypertension. What is the most likely diagnosis based on this history?

The history is highly suggestive of Acute Gout (podagra).

Key pointers: Sudden onset, severe pain, involvement of 1st MTP joint, previous similar episodes, male sex, and use of a thiazide diuretic (which can increase uric acid levels).

Examination of a Swollen Joint (OSCE Steps)

Follow the “Look, Feel, Move, (Special Tests), (Measure)” approach. Always compare with the contralateral joint.

  1. Introduction & Consent: WIPER (Wash hands, Introduce, Permission, Expose adequately, Reposition). Ask about pain before starting.
  2. General Inspection: Patient’s general well-being, gait (if able to walk), use of walking aids, body habitus.
  3. LOOK (from front, sides, back):
    • Swelling: Localized or diffuse? Obvious effusion?
    • Erythema (Redness): Suggests acute inflammation (sepsis, crystals).
    • Deformity: Valgus/varus, fixed flexion, subluxation.
    • Scars: Previous surgery or trauma.
    • Skin changes: Rashes (psoriasis), nodules (rheumatoid nodules, tophi in gout), sinuses.
    • Muscle Wasting: Quadriceps wasting with knee problems.
    • Alignment.
  4. FEEL:
    • Temperature: Use back of hand to compare with contralateral joint and surrounding skin. Warmth suggests inflammation.
    • Tenderness: Palpate joint lines, bony prominences, ligaments, tendons. Localize point of maximal tenderness.
    • Effusion: Test for fluctuance, bulge sign, patellar tap (for knee).
    • Synovial Thickening (Bogginess): Suggests chronic synovitis.
    • Crepitus on movement.
    • Palpate for specific structures (e.g., Baker’s cyst in knee, tophi around joints).
  5. MOVE:
    • Active Range of Motion (AROM): Ask patient to move the joint through its full range. Note any pain or restriction.
    • Passive Range of Motion (PROM): Gently move the joint yourself. Note range, pain, end-feel. Compare active vs. passive (if active restricted but passive full, may suggest muscle weakness/pain inhibition rather than true joint block).
      • If significantly painful, defer full PROM.
    • Pain on movement (especially at extremes of range).
  6. SPECIAL TESTS (Joint Specific):
    • Knee: McMurray’s (menisci), Lachman’s/Anterior Drawer (ACL), Posterior Drawer (PCL), Varus/Valgus stress (collaterals).
    • Shoulder: Rotator cuff tests (e.g., Jobe’s, lift-off), impingement tests (e.g., Hawkins-Kennedy, Neer’s).
    • Hip: Trendelenburg, Thomas test, FABER test.
    • Only perform relevant tests based on history and other findings. Do not perform on acutely inflamed joint if it causes excessive pain.
  7. MEASURE (if relevant): Limb length, thigh/calf circumference (for wasting/swelling).
  8. Neurovascular Examination: Distal pulses, sensation, power if suspicion of neurovascular compromise or nerve involvement.
  9. Examine Joint Above and Below.
  10. Systemic Examination (if indicated): Look for signs of systemic disease (e.g., psoriatic skin/nail changes, rheumatoid nodules, tophi elsewhere, uveitis signs). Lymph node examination. Temperature.
  11. Summarise Findings.

On examining a patient’s swollen right knee, you find it is warm, erythematous, and exquisitely tender to touch. Active and passive movements are severely restricted due to pain. What is your immediate concern and next step in the examination process (assuming you’ve done Look and Feel)?

Immediate Concern: Septic arthritis is the primary concern given the hot, red, tender, and severely restricted joint.

Next Step in Examination:

  • Carefully assess for a significant effusion (patellar tap, bulge sign).
  • Forgo aggressive ‘Move’ and ‘Special Tests’ on this acutely inflamed joint to avoid causing unnecessary pain or potential spread of infection. Note the severe restriction on attempted active/gentle passive movement.
  • Proceed quickly to systemic assessment: measure temperature, check for signs of sepsis (tachycardia, hypotension).
  • Briefly examine other joints for involvement.
  • Check skin for portals of entry for infection.
  • The priority shifts towards urgent investigation (joint aspiration).

Investigations for a Swollen Joint

Investigations are guided by the history and examination findings.

1. Synovial Fluid Aspiration (Arthrocentesis):

Crucial, especially if septic arthritis or crystal arthropathy is suspected. Send for:

  • Gram Stain & Culture (MC&S): Essential to identify bacteria in septic arthritis.
  • Crystal Microscopy (Polarised Light):
    • Gout: Negatively birefringent, needle-shaped monosodium urate (MSU) crystals.
    • Pseudogout: Positively birefringent, rhomboid-shaped calcium pyrophosphate dihydrate (CPPD) crystals.
  • White Cell Count (WCC) and Differential:
    • Normal: <200 cells/µL (<25% PMNs)
    • Non-inflammatory (e.g., OA, trauma): 200 – 2,000 cells/µL (<25% PMNs)
    • Inflammatory (e.g., RA, gout, pseudogout, reactive): 2,000 – 50,000 cells/µL (>50% PMNs)
    • Septic: Usually >50,000 cells/µL (>75% PMNs), but can be lower, especially if early or immunocompromised. Any suspicion of sepsis warrants treatment even if WCC lower.
  • Appearance: Clarity, colour, viscosity. (Septic fluid often purulent, turbid, low viscosity).
  • Glucose (low in sepsis), Protein.

2. Blood Tests:

  • Full Blood Count (FBC): WCC (raised in infection/inflammation), platelets.
  • Inflammatory Markers: Erythrocyte Sedimentation Rate (ESR), C-Reactive Protein (CRP) – elevated in inflammation/infection.
  • Urea & Electrolytes (U&Es): Renal function (important for drug dosing, can be affected by NSAIDs or sepsis).
  • Serum Uric Acid: Often elevated in gout (but can be normal during acute attack). Not diagnostic alone.
  • Blood Cultures: If septic arthritis suspected (may be positive in ~50% of cases).
  • Rheumatoid Factor (RF) & Anti-CCP Antibodies: If RA suspected.
  • ANA, dsDNA: If SLE suspected.
  • HLA-B27: If spondyloarthropathy suspected (e.g., ankylosing spondylitis, reactive arthritis).
  • Coagulation screen if haemarthrosis or pre-aspiration.

3. Imaging:

  • X-rays of the affected joint (AP and Lateral views):
    • May be normal in early arthritis (septic, crystal, inflammatory).
    • Can show fractures, dislocations, OA changes (osteophytes, joint space narrowing, subchondral sclerosis/cysts).
    • Gout: Erosions (punched-out, sclerotic margins, overhanging edges – “rat-bite” erosions) in chronic gout. Soft tissue swelling.
    • Pseudogout: Chondrocalcinosis (calcification of cartilage).
    • RA: Erosions (marginal), joint space narrowing, periarticular osteopenia.
    • Septic Arthritis: Soft tissue swelling, joint space widening (early), joint space narrowing/destruction (late).
  • Ultrasound (USS):
    • Useful for detecting effusions, synovitis, erosions, bursitis, tendinopathy.
    • Can guide joint aspiration, especially for difficult-to-access joints (e.g., hip).
    • Can show crystal deposits (e.g., “double contour sign” in gout).
  • MRI: More detailed assessment of soft tissues (ligaments, menisci, cartilage, synovium), bone marrow oedema, early erosions, infection (osteomyelitis). Not usually first-line for acute swollen joint unless specific indication (e.g., suspected internal derangement, osteomyelitis).
  • CT Scan: Good for bony detail, complex fractures, loose bodies.

OSCE Tips for a Swollen Joint Station

  • Safety First: Always ask about pain before touching. Be gentle, especially with an acutely inflamed joint.
  • WIPER: Wash hands, Introduce, Permission, Expose (joint above and below), Reposition.
  • Structure: Consistently use “Look, Feel, Move.” Verbalize your findings as you go.
  • Compare Sides: Crucial for identifying asymmetry in swelling, temperature, ROM, etc. Examine the “normal” side first.
  • Red Flags for Sepsis: Actively state if you suspect septic arthritis and why (hot, red, swollen, tender, restricted movement, systemic symptoms). Emphasize the urgency.
  • Joint Aspiration: Be prepared to discuss the indications, contraindications, procedure (briefly, aseptically), and what tests to send the fluid for. Mentioning “under aseptic technique” is key.
  • Differential Diagnosis: Offer a sensible list of differentials based on your findings.
    • “Given the acute onset, single joint involvement, warmth, and erythema, my top differentials would include septic arthritis and crystal arthropathy like gout. I would also consider trauma if the history suggested it.”
  • Management Principles: Suggest initial investigations (bloods, joint aspiration, X-ray). For septic arthritis, state “urgent admission, IV antibiotics after joint aspiration, and orthopaedic consultation.”
  • Key Phrases:
    • “I’m now going to examine your [joint]. Please let me know if you feel any pain.”
    • “On inspection, there is visible swelling and erythema of the [joint].”
    • “On palpation, the joint feels warm compared to the other side, and there is tenderness over the joint line.”
    • “Active range of motion is restricted in [movement] due to pain.”
    • “My main concern here would be septic arthritis, which is an orthopaedic emergency.”
    • “I would like to perform an urgent joint aspiration for Gram stain, culture, and crystal microscopy.”
  • Common Pitfalls:
    • Not considering septic arthritis or not highlighting its urgency.
    • Performing aggressive movements on a clearly acutely inflamed joint.
    • Forgetting to compare with the contralateral side.
    • Not exposing the joint adequately.
    • Vague summary or management plan.

Flashcards: Swollen Joint

Click on each card to reveal the answer.

What are the classic features of Gout crystals under polarised light microscopy?

(Click to flip)

Answer

Negatively birefringent, needle-shaped monosodium urate (MSU) crystals.

(Yellow when parallel to compensator axis, blue when perpendicular).

List 3 red flag symptoms/signs for Septic Arthritis.

(Click to flip)

Answer

1. Fever/rigors/systemic upset.

2. Hot, red, swollen, and exquisitely tender joint.

3. Severely restricted active AND passive range of motion due to pain.

(Also: recent infection, IV drug use, immunosuppression, prosthetic joint).

What is the typical synovial fluid White Cell Count (WCC) in Septic Arthritis?

(Click to flip)

Answer

Usually >50,000 cells/µL with >75% polymorphonuclear neutrophils (PMNs).

However, a lower count does not exclude sepsis, especially early on or in immunocompromised patients.

What are the Kocher criteria used for, and what are its components? (Paediatric context)

(Click to flip)

Answer

Used to differentiate septic arthritis from transient synovitis in children with hip pain.

Components (1 point each):

  1. Fever >38.5°C
  2. Non-weight bearing on affected side
  3. ESR >40 mm/hr
  4. WBC >12,000 cells/mm³

(CRP >20 mg/L can be used as an alternative to ESR).

What is the “double contour sign” on ultrasound indicative of?

(Click to flip)

Answer

It is indicative of Gout. It represents monosodium urate (MSU) crystal deposition on the surface of hyaline cartilage.

What is Chondrocalcinosis, and which crystal arthropathy is it associated with?

(Click to flip)

Answer

Chondrocalcinosis is the calcification of hyaline and/or fibrocartilage.

It is classically associated with Pseudogout (Calcium Pyrophosphate Dihydrate – CPPD crystal deposition disease).

Name two common joints affected by Pseudogout.

(Click to flip)

Answer

Commonly affected joints include:

1. Knees (most common)

2. Wrists

(Also shoulders, ankles, elbows).

What is the most common organism causing septic arthritis in adults?

(Click to flip)

Answer

Staphylococcus aureus is the most common causative organism in adults.

(Neisseria gonorrhoeae in sexually active young adults; Streptococcus species also common).

Swollen Joint Quiz

Test your knowledge with these OSCE/PLAB2/MLA-style questions.

1. A 70-year-old woman presents with an acutely swollen, warm, and tender right knee. She has a fever of 38.5°C. Joint aspiration is performed. Which synovial fluid finding is MOST suggestive of septic arthritis?

A. Clear, yellow fluid with WCC 1,000 cells/µL.
B. Negatively birefringent crystals under polarised light.
C. Turbid fluid with WCC 80,000 cells/µL, 90% neutrophils.
D. Blood-stained fluid with WCC 500 cells/µL.
Explanation: A high synovial fluid WCC (typically >50,000/µL) with a predominance of neutrophils (>75-90%) is highly suggestive of septic arthritis. Turbid appearance also points to infection. Option B suggests gout. Option A is non-inflammatory. Option D suggests haemarthrosis.

2. A 30-year-old man develops acute pain and swelling in his left knee and right ankle 2 weeks after an episode of urethritis. He also reports eye redness. What is the most likely diagnosis?

A. Gout
B. Reactive Arthritis
C. Rheumatoid Arthritis
D. Septic Arthritis
Explanation: The triad of arthritis (oligoarthritis), urethritis (or cervicitis), and conjunctivitis/uveitis following an infection (often GI or GU) is characteristic of Reactive Arthritis (formerly Reiter’s syndrome).

3. During a knee examination for swelling, you gently push medially on the suprapatellar pouch, then stroke laterally, and observe for a fluid wave or bulge on the medial side. What is this sign called?

A. Patellar Tap
B. Bulge Sign (or Wipe Test / Stroke Test)
C. McMurray’s Test
D. Lachman’s Test
Explanation: The Bulge Sign (also known as wipe test or stroke test) is used to detect small knee effusions. Patellar tap detects larger effusions. McMurray’s tests for meniscal tears. Lachman’s tests for ACL integrity.

4. Which of the following conditions is classically associated with chondrocalcinosis seen on an X-ray?

A. Gout
B. Rheumatoid Arthritis
C. Pseudogout (CPPD Disease)
D. Osteoarthritis
Explanation: Chondrocalcinosis refers to calcification of articular cartilage and is a hallmark radiological finding in Pseudogout, caused by calcium pyrophosphate dihydrate (CPPD) crystal deposition.

5. A patient with suspected septic arthritis of the knee requires urgent joint aspiration. Which of the following is a key principle of this procedure?

A. Administering IV antibiotics at least 1 hour before aspiration.
B. Performing the procedure under strict aseptic technique.
C. Using a large bore needle primarily to relieve pressure.
D. Aspirating only a small amount of fluid (1-2ml) to avoid discomfort.
Explanation: Strict aseptic technique is paramount during joint aspiration to avoid introducing infection. Ideally, aspiration for culture should be done BEFORE antibiotics, if possible without significant delay. Sufficient fluid should be aspirated for all necessary tests. Needle size depends on the joint and anticipated fluid viscosity.

6. A 55-year-old man presents with recurrent, exquisitely painful monoarthritis of the first metatarsophalangeal joint. Which investigation is most diagnostic during an acute attack?

A. Serum uric acid level.
B. Synovial fluid microscopy for crystals.
C. X-ray of the affected joint.
D. C-Reactive Protein (CRP).
Explanation: The gold standard for diagnosing gout is the identification of monosodium urate (MSU) crystals in synovial fluid via polarised light microscopy. Serum uric acid can be normal or low during an acute attack. X-rays may be normal acutely or show chronic changes. CRP is non-specific.

7. Which pattern of joint involvement is most typical of early Rheumatoid Arthritis?

A. Asymmetrical oligoarthritis of large, weight-bearing joints.
B. Symmetrical polyarthritis affecting small joints of hands (MCPs, PIPs) and feet (MTPs).
C. Acute monoarthritis of the first MTP joint.
D. Predominant involvement of the sacroiliac joints and spine.
Explanation: Rheumatoid Arthritis classically presents as a symmetrical polyarthritis affecting the small joints of the hands (MCPs, PIPs, wrists) and feet (MTPs). DIP joints are usually spared. Option A is more like spondyloarthropathy/reactive. Option C is gout. Option D is spondyloarthropathy.

8. “Look, Feel, Move” is a systematic approach to joint examination. Under which heading would you typically assess for an effusion using the patellar tap?

A. Look
B. Feel
C. Move
D. Special Tests
Explanation: Assessing for an effusion, including techniques like the patellar tap or bulge sign, is part of the “Feel” component of the joint examination, as it involves palpation and manipulation to detect fluid.

9. A patient presents with acute swelling and pain in the knee after a twisting injury playing football. There was an audible “pop”. On examination, there is a significant effusion and a positive Lachman’s test. What is the most likely structure injured?

A. Medial Collateral Ligament (MCL)
B. Posterior Cruciate Ligament (PCL)
C. Anterior Cruciate Ligament (ACL)
D. Lateral Meniscus
Explanation: A history of a twisting injury, an audible “pop”, rapid effusion (often haemarthrosis), and a positive Lachman’s test (or anterior drawer test) are highly suggestive of an Anterior Cruciate Ligament (ACL) rupture.

10. Which of these is NOT a typical feature of Osteoarthritis on an X-ray?

A. Joint space narrowing.
B. Osteophytes.
C. Periarticular erosions.
D. Subchondral sclerosis.
Explanation: The typical X-ray features of Osteoarthritis include: Loss of joint space (narrowing), Osteophytes (bony spurs), Subchondral sclerosis (increased bone density beneath cartilage), and Subchondral cysts. Periarticular erosions are characteristic of inflammatory arthritides like Rheumatoid Arthritis or Gout.

Back Pain Assessment

Back pain is an extremely common presentation. The majority of cases are non-specific (mechanical) low back pain, but it’s crucial to identify “red flag” features suggesting serious spinal pathology (e.g., cauda equina syndrome, malignancy, infection, fracture) and differentiate from inflammatory causes.

Overview of Back Pain Assessment

Back pain is a leading cause of disability worldwide. Most episodes are self-limiting and mechanical in nature. However, a thorough assessment is essential to identify serious underlying pathology (“red flags”) or specific conditions requiring targeted treatment.

Classification of Back Pain:

  • Non-specific (Mechanical) Low Back Pain (~90%): Pain related to spinal structures (muscles, ligaments, facet joints, intervertebral discs) without specific pathology identifiable. Often improves with conservative measures.
  • Nerve Root Pain / Radiculopathy (~5-10%): Pain radiating along a dermatome due to nerve root compression or irritation (e.g., disc herniation, foraminal stenosis). Commonly called “sciatica” if L4-S3 roots affected.
  • Serious Spinal Pathology (“Red Flags”) (~1-2%):
    • Cauda Equina Syndrome (CES) – Orthopaedic/Neurosurgical Emergency!
    • Spinal Malignancy (primary or metastatic)
    • Spinal Infection (e.g., discitis, vertebral osteomyelitis, epidural abscess)
    • Spinal Fracture (traumatic or osteoporotic vertebral compression fracture)
    • Significant spinal stenosis causing neurogenic claudication.
  • Inflammatory Back Pain (e.g., Ankylosing Spondylitis, Spondyloarthropathies): Characterized by insidious onset, morning stiffness, improvement with exercise but not rest, often nocturnal pain.
🚨 Cauda Equina Syndrome (CES)

CES is a surgical emergency caused by compression of the cauda equina nerve roots. Key symptoms include: severe low back pain, bilateral sciatica, saddle anaesthesia/paraesthesia, bladder dysfunction (retention, incontinence, loss of sensation), bowel dysfunction (incontinence, loss of sensation/tone), and lower limb weakness/sensory loss (often bilateral and can be asymmetrical). Suspected CES requires URGENT MRI and surgical referral.

🧠 Back Pain Red Flags: “TUNA FISH” (adapted)

A mnemonic for common red flags (many exist, this is one example):

Trauma (significant) / Thoracic pain
Unexplained Weight Loss
Neurological Deficit (progressive, or CES symptoms)
Age <20 or >55 (new onset)
Fever / IV Drug Use / Immunosuppression (risk of infection)
Inflammatory features (morning stiffness >30min, improves with exercise)
Steroid use (long-term) / History of Cancer
History of Cancer / Saddle anaesthesia / Sphincter disturbance

History Taking for Back Pain

A focused but comprehensive history is key to identifying red flags and guiding diagnosis.

Pain Assessment (SOCRATES):
  • Site: Localized, diffuse? Lumbar, thoracic, cervical?
  • Onset: Sudden (trauma, disc prolapse) or gradual (degenerative, inflammatory, tumour)?
  • Character: Sharp, dull, aching, burning, shooting?
  • Radiation:
    • Sciatica: Radiating down back of leg, often below knee, in a dermatomal pattern (suggests L4, L5, S1 nerve root).
    • Groin/anterior thigh pain: May suggest upper lumbar roots (L1-L3) or hip pathology.
    • Non-radicular referred pain: Dull ache into buttock/thigh, not dermatomal.
  • Associated Symptoms: Numbness, paraesthesia, weakness in legs? Sphincter disturbance?
  • Timing: Constant or intermittent? Worse at night (malignancy, infection, AS)? Worse in morning with prolonged stiffness (inflammatory)?
  • Exacerbating/Relieving Factors: Worse with movement/activity (mechanical)? Better with exercise, worse with rest (inflammatory)? Worse with coughing/straining (disc)? Worse with walking, relieved by leaning forward/sitting (spinal stenosis – neurogenic claudication)?
  • Severity: Pain score (0-10). Impact on daily life.
Red Flag Questions (Systematic Enquiry):

Cauda Equina Syndrome (CES):

  • Bilateral leg pain/sciatica?
  • Saddle anaesthesia or paraesthesia (numbness/tingling around genitals/buttocks)?
  • Bladder dysfunction: Difficulty initiating micturition, urinary retention, overflow incontinence, loss of sensation of bladder fullness?
  • Bowel dysfunction: Faecal incontinence, loss of anal tone/sensation?
  • Sexual dysfunction (new onset)?
  • Progressive or severe bilateral lower limb weakness or sensory loss?

Malignancy/Infection:

  • Age >50 years?
  • History of cancer (prostate, breast, lung, kidney, thyroid are common bone mets)?
  • Unexplained weight loss?
  • Pain worse at night or at rest? Non-mechanical pain?
  • Fever, chills, sweats? Recent infection?
  • IV drug use? Immunosuppression (steroids, HIV)? Recent spinal procedure?
  • Thoracic pain (higher suspicion for sinister pathology)?

Fracture:

  • Significant trauma (fall from height, RTA)?
  • Minor trauma or even just strain in elderly or osteoporotic patients?
  • History of osteoporosis? Long-term steroid use?
  • Sudden onset severe pain, point tenderness?
Inflammatory Features (suggesting Spondyloarthropathy, e.g., Ankylosing Spondylitis):
  • Insidious onset, often age <40 years.
  • Morning stiffness >30 minutes.
  • Improves with exercise/activity, NOT with rest.
  • Pain at night (may wake patient in second half of night).
  • Alternating buttock pain.
  • Associated features: Peripheral arthritis, enthesitis (e.g., Achilles tendonitis, plantar fasciitis), uveitis, psoriasis, IBD, family history.
Yellow Flags (Psychosocial factors predicting chronicity/disability):
  • Beliefs: Expectation of persistent pain, catastrophizing.
  • Behaviour: Fear avoidance, reduced activity levels.
  • Mood: Depression, anxiety.
  • Work: Dissatisfaction, absence from work.
  • Social: Lack of support, compensation issues.
Past Medical History, Drug History, Social History:
  • Previous episodes of back pain, treatments tried.
  • Other medical conditions, medications (especially anticoagulants, steroids).
  • Occupation (manual labour, sedentary), hobbies, functional impact.

Examination of the Spine (OSCE Steps – focusing on Lumbar Spine)

Always explain what you are doing and ask for consent. Expose back adequately (patient in underwear or gown open at back).

  1. General Inspection: Patient standing. Observe posture, gait (antalgic?), ability to undress, general comfort.
  2. LOOK (from back, side, front if needed):
    • Spinal Alignment: Scoliosis (lateral curvature), kyphosis (excessive thoracic curve), lordosis (excessive lumbar curve – may be lost in spasm/AS). Pelvic tilt.
    • Scars: Previous surgery.
    • Skin Changes: Rashes (herpes zoster), naevi, hair patches (spina bifida occulta), café-au-lait spots (NF1).
    • Muscle Wasting: Gluteal, paraspinal.
    • Swelling (rare).
  3. FEEL (Patient standing or prone):
    • Palpate Spinous Processes: For tenderness (fracture, infection, mets), step deformity (spondylolisthesis).
    • Palpate Paraspinal Muscles: For tenderness, spasm.
    • Palpate Sacroiliac Joints (SIJs): For tenderness.
    • Temperature (if infection suspected).
  4. MOVE (Patient standing): Assess active range of motion of lumbar spine. Ask patient to:
    • Flexion: “Try to touch your toes, keeping knees straight.” Note range, pain. (Schober’s test can quantify flexion).
    • Extension: “Lean backwards as far as you can.” Note range, pain.
    • Lateral Flexion (Side Bending): “Slide your hand down the side of your leg.” Both sides.
    • Rotation (Thoracolumbar): “Turn your shoulders to the right/left, keeping hips still.” (Stabilize pelvis).
  5. NEUROLOGICAL EXAMINATION (Lower Limbs – Patient ideally supine/sitting): Essential to detect radiculopathy or myelopathy/CES. Compare sides.
    • Tone: Assess at knee and ankle.
    • Power (MRC Scale 0-5): Test major myotomes:
      • Hip Flexion (L1, L2, L3 – Iliopsoas)
      • Knee Extension (L3, L4 – Quadriceps)
      • Knee Flexion (L5, S1 – Hamstrings)
      • Ankle Dorsiflexion (L4, L5 – Tibialis Anterior)
      • Ankle Plantarflexion (S1, S2 – Gastrocnemius, Soleus)
      • Great Toe Extension (L5 – Extensor Hallucis Longus)
    • Reflexes:
      • Knee Jerk (L3, L4)
      • Ankle Jerk (S1)
      • Plantar Response (Babinski sign – UMN lesion if upgoing)
    • Sensation (Light touch, pinprick): Test key dermatomes:
      • L1: Groin
      • L2: Anterior mid-thigh
      • L3: Medial knee / distal thigh
      • L4: Medial malleolus / medial aspect of lower leg
      • L5: Dorsum of foot / great toe / lateral aspect of lower leg
      • S1: Lateral malleolus / lateral border of foot / sole
      • Saddle Area (S2-S5): Crucial if CES suspected. Ask about altered sensation when wiping.
  6. SPECIAL TESTS:
    • Straight Leg Raise (SLR) / Lasègue’s Test (Patient supine): For L4, L5, S1 nerve root irritation. Passively lift extended leg. Positive if reproduces radicular pain down leg (not just back/hamstring tightness) between 30-70 degrees. Note angle. Dorsiflexion of foot may exacerbate pain (Bragard’s sign). Crossed SLR (pain in affected leg when contralateral leg raised) is more specific for disc prolapse.
    • Femoral Nerve Stretch Test (Patient prone): For L2, L3, L4 nerve root irritation. Flex knee and extend hip. Positive if pain in anterior thigh.
    • Schober’s Test (Patient standing): For lumbar flexion. Mark 5cm below and 10cm above dimples of Venus (or PSIS level). Measure distance. Ask patient to flex fully. Increase <5cm is abnormal (suggests restricted flexion, e.g., in AS).
  7. GAIT: Observe walking, heel walking (L5), toe walking (S1). Antalgic gait? Foot drop? Broad-based?
  8. PERIPHERAL VASCULAR EXAM: Check peripheral pulses if vascular claudication is a differential for leg pain.
  9. ABDOMINAL EXAM: If referred pain suspected (e.g., AAA, pancreatitis, renal colic).
  10. RECTAL EXAM (if CES suspected): Assess anal tone, perianal sensation, voluntary squeeze. (Usually state you would do this with a chaperone rather than perform in OSCE without specific instruction).
  11. Summarise Findings.

Investigations for Back Pain

Most acute non-specific low back pain does NOT require imaging. Investigations are primarily indicated if red flags are present, or if pain persists despite conservative management and a specific diagnosis is sought.

  • Blood Tests (if red flags for infection, malignancy, or inflammatory cause):
    • FBC (WCC, Hb)
    • ESR, CRP (inflammatory markers)
    • Serum Calcium, Alkaline Phosphatase, PSA (for bony mets, myeloma screen)
    • Protein Electrophoresis / Urine Bence Jones Protein (myeloma screen)
    • HLA-B27 (if inflammatory spondyloarthropathy suspected)
    • Blood cultures (if infection suspected)
  • Imaging:
    • X-rays (Lumbar Spine – AP, Lateral, +/- Obliques):
      • Limited utility in acute non-specific LBP.
      • Indications: Trauma, suspected fracture (especially in elderly/osteoporotic), suspected spondylolisthesis, monitoring known deformity, initial screen if malignancy/infection suspected (but MRI much better).
      • Can show: Fractures, spondylolisthesis, degenerative changes (disc space narrowing, osteophytes), lytic/sclerotic lesions, sacroiliitis (less sensitive than MRI).
    • MRI Scan (Lumbar Spine):
      • Investigation of choice for suspected serious pathology: CES, spinal cord/nerve root compression, tumour, infection, significant neurological deficit, inflammatory spondyloarthropathy (sacroiliitis, vertebral inflammation).
      • Provides detailed images of soft tissues (discs, nerves, ligaments, cord) and bone marrow.
      • Often shows degenerative changes in asymptomatic individuals, so findings must be correlated clinically.
    • CT Scan (Lumbar Spine):
      • Better for bony detail than MRI (e.g., complex fractures, bony stenosis, post-op assessment of fusion).
      • CT Myelogram (CT after contrast into a dural sac) if MRI contraindicated or inconclusive.
    • Bone Scan (Radionuclide Scintigraphy): Sensitive for detecting areas of increased bone turnover (mets, infection, fracture, Paget’s). Non-specific. Often followed by MRI/CT for characterisation.
  • Nerve Conduction Studies / Electromyography (NCS/EMG):
    • To assess nerve function and differentiate radiculopathy from peripheral neuropathy or myopathy.
    • Not usually first-line for back pain.

OSCE Tips for Back Pain Station

  • Safety Netting is Key: If no red flags, explain natural history of non-specific LBP, advise on self-management (stay active, analgesia), and crucially, provide clear “red flag” advice (what symptoms warrant urgent re-assessment – especially CES symptoms).
  • Red Flag Focus: Systematically ask about red flags. If any are positive, state your concern clearly and your plan for urgent investigation/referral. “Given the new onset bladder dysfunction and saddle anaesthesia, I am concerned about Cauda Equina Syndrome, which is a surgical emergency. I would urgently refer this patient to the on-call orthopaedic/neurosurgical team for an MRI and assessment.”
  • Neurological Examination: Perform a competent and structured lower limb neuro exam if indicated. Verbalize what you are testing (myotomes, dermatomes, reflexes).
  • Straight Leg Raise: Explain what constitutes a positive test (reproduction of radicular leg pain, not just hamstring tightness).
  • Be Empathetic: Back pain can be very distressing and disabling.
  • Psychosocial Factors: Briefly acknowledge or ask about “yellow flags” if appropriate, as they influence prognosis and management.
  • Management for Non-Specific LBP: Reassurance, encourage activity, simple analgesia (paracetamol, NSAIDs if appropriate), consider physiotherapy referral. Avoid prolonged bed rest.
  • Key Phrases:
    • “I need to ask you some important questions to ensure there’s nothing serious causing your back pain.” (Then go through red flags).
    • “Are you having any problems with your waterworks, like difficulty passing urine or any numbness when you wipe yourself?”
    • “I’m now going to test the power in your legs. Can you try to… against my resistance?”
    • “This is called the straight leg raise test. Please tell me if you feel your usual leg pain as I lift your leg.”
    • “Based on my assessment, your back pain appears to be mechanical in nature, and I don’t see any red flag signs at present. However, if you develop [list key CES symptoms], you must seek urgent medical attention.”
  • Common Pitfalls:
    • Missing or not adequately exploring red flags.
    • Performing an incomplete or disorganized neurological examination.
    • Not providing adequate safety netting advice.
    • Over-investigating non-specific low back pain.
    • Failing to consider psychosocial factors in chronic pain.

Flashcards: Back Pain Assessment

Click on each card to reveal the answer.

List 5 “Red Flag” symptoms for Cauda Equina Syndrome.

(Click to flip)

Answer

1. Bilateral sciatica/leg weakness.

2. Saddle anaesthesia/paraesthesia (S2-S5 dermatomes).

3. Bladder dysfunction (retention, incontinence, loss of sensation).

4. Bowel dysfunction (incontinence, loss of tone).

5. Severe or progressive neurological deficit in lower limbs.

What are the key features differentiating inflammatory back pain from mechanical back pain?

(Click to flip)

Answer

Inflammatory: Insidious onset, age <40, morning stiffness >30min, improves with exercise but NOT rest, nocturnal pain (wakes in 2nd half of night).

Mechanical: Often acute onset, variable age, morning stiffness <30min, improves with rest, worse with activity/certain movements.

What does a positive Straight Leg Raise (SLR) test indicate?

(Click to flip)

Answer

A positive SLR indicates irritation or compression of the L4, L5, or S1 nerve roots (sciatic nerve).

It is positive if it reproduces the patient’s radicular leg pain (not just back or hamstring pain) between 30-70 degrees of hip flexion.

Name three red flags that might suggest spinal malignancy as a cause of back pain.

(Click to flip)

Answer

1. History of cancer.

2. Unexplained weight loss.

3. Pain worse at night or at rest, non-mechanical pain, thoracic pain, age >50.

(Also: no improvement after 4-6 weeks conservative therapy, fever).

Which nerve root is primarily tested by asking a patient to extend their great toe against resistance?

(Click to flip)

Answer

L5 nerve root (Extensor Hallucis Longus muscle).

What is “neurogenic claudication” and what condition is it typically associated with?

(Click to flip)

Answer

Neurogenic claudication is pain, numbness, or weakness in the legs (often buttocks/thighs) brought on by walking or standing, and relieved by sitting or leaning forward (flexing the spine).

It is typically associated with Lumbar Spinal Stenosis.

What is Schober’s test used to measure?

(Click to flip)

Answer

Schober’s test is used to measure the range of lumbar spine flexion.

An increase of <5cm between the two skin marks upon full flexion is considered abnormal and indicative of restricted lumbar flexion (e.g., as seen in Ankylosing Spondylitis).

What is the initial imaging modality of choice if Cauda Equina Syndrome is suspected?

(Click to flip)

Answer

Urgent MRI of the lumbosacral spine.

Back Pain Assessment Quiz

Test your knowledge with these OSCE/PLAB2/MLA-style questions.

1. A 65-year-old man presents with new onset low back pain and bilateral leg weakness. He reports difficulty passing urine and numbness around his perineum. What is the most important immediate step?

A. Prescribe strong analgesia and review in 1 week.
B. Urgent referral to hospital for MRI and neurosurgical/orthopaedic assessment.
C. Order an urgent lumbar spine X-ray.
D. Advise physiotherapy and gentle exercises.
Explanation: These are red flag symptoms highly suggestive of Cauda Equina Syndrome, which is a surgical emergency. Urgent hospital admission for MRI and specialist assessment is required.

2. A 30-year-old woman complains of low back pain that is worse in the morning with stiffness lasting over an hour. The pain improves with exercise and worsens with rest. What type of back pain is this most characteristic of?

A. Mechanical low back pain
B. Inflammatory back pain (e.g., spondyloarthropathy)
C. Nerve root pain (radiculopathy)
D. Pain due to spinal malignancy
Explanation: The features described (morning stiffness >30min, improvement with exercise, worsening with rest, typical age of onset) are classic for inflammatory back pain, such as that seen in ankylosing spondylitis or other spondyloarthropathies.

3. A positive Straight Leg Raise (SLR) test, causing radicular pain down the leg between 30-70 degrees, typically indicates irritation of which nerve roots?

A. L1, L2
B. L2, L3
C. L4, L5, S1
D. S2, S3, S4
Explanation: The SLR test primarily stretches the sciatic nerve and its roots, which are L4, L5, and S1. Pain reproduction in a dermatomal pattern associated with these roots indicates irritation.

4. Which of the following is considered a “yellow flag” in back pain assessment?

A. Fever and unexplained weight loss.
B. Fear avoidance behaviour and catastrophizing.
C. History of previous cancer.
D. Saddle anaesthesia.
Explanation: Yellow flags are psychosocial factors that predict poor prognosis and increased risk of chronic disability from back pain. Fear avoidance behaviour (avoiding movement/activity due to fear of pain/harm) and catastrophizing (negative cognitive-emotional response to pain) are key yellow flags. Options A, C, D are red flags.

5. Testing a patient’s ability to walk on their heels primarily assesses the integrity of which nerve root?

A. L4
B. L5
C. S1
D. S2
Explanation: Heel walking requires ankle dorsiflexion, primarily mediated by the tibialis anterior muscle, innervated by the L4 and L5 nerve roots, with L5 being the major contributor for this specific action (and great toe extension). Weakness in heel walking strongly suggests L5 radiculopathy.

6. Which of these is NOT a red flag for serious spinal pathology?

A. Unexplained weight loss.
B. Thoracic pain.
C. Pain worse with activity, relieved by rest.
D. History of intravenous drug use.
Explanation: Pain that is worse with activity and relieved by rest is typical of mechanical back pain, which is generally not a red flag. The other options (unexplained weight loss, thoracic pain, history of IVDU) are all red flags suggesting potential malignancy or infection.

7. The ankle jerk reflex corresponds to which nerve root level?

A. L4
B. L5
C. S1
D. S2
Explanation: The ankle jerk reflex (Achilles reflex) primarily tests the integrity of the S1 nerve root.

8. A patient describes back pain that radiates to the buttock and posterior thigh but not below the knee. There are no neurological symptoms. This is best described as:

A. True sciatica (radiculopathy).
B. Referred somatic pain.
C. Neurogenic claudication.
D. Inflammatory back pain.
Explanation: Pain radiating to the buttock and posterior thigh without extending below the knee in a dermatomal pattern, and without neurological signs, is often referred somatic pain from structures like facet joints or ligaments, rather than true nerve root compression (sciatica).

9. An elderly patient with known osteoporosis sustains a fall and complains of acute, severe mid-thoracic back pain with point tenderness over T8. What is the most likely diagnosis?

A. Disc herniation.
B. Spinal infection (discitis).
C. Vertebral compression fracture.
D. Metastatic spinal cord compression.
Explanation: In an elderly patient with osteoporosis and a history of a fall (even minor), acute severe back pain with focal tenderness is highly suggestive of a vertebral compression fracture. Thoracic pain is also a red flag.

10. For acute non-specific low back pain (<6 weeks) with no red flags, what is the most appropriate initial management advice?

A. Strict bed rest for 1 week and strong opioid analgesia.
B. Urgent MRI scan to identify the cause.
C. Reassurance, encourage staying active, simple analgesia (e.g., paracetamol/NSAIDs), and safety netting.
D. Referral for spinal injections.
Explanation: For acute non-specific LBP without red flags, guidelines recommend reassurance about the favorable prognosis, advice to remain active as tolerated (avoid bed rest), simple analgesia, and clear safety netting advice (when to seek further help if symptoms worsen or red flags develop). Imaging and invasive treatments are not indicated initially.

Trauma & Orthopaedics: Hip Fracture Management

Hip fractures (proximal femoral fractures) are common and serious injuries, particularly in older adults, associated with significant morbidity and mortality. Prompt diagnosis, appropriate surgical management, and comprehensive multidisciplinary care are crucial for optimising outcomes.

Overview & Epidemiology of Hip Fractures

A hip fracture is a break in the upper quarter of the femur (thigh bone). It is a major public health issue due to its frequency, impact on individuals, and healthcare costs.

Key Facts:

  • Incidence: Increases sharply with age, most common in those >65 years. More common in women than men (due to higher rates of osteoporosis).
  • Mechanism of Injury:
    • Older adults: Usually low-energy trauma (e.g., fall from standing height) in the context of osteoporosis.
    • Younger adults: Typically high-energy trauma (e.g., road traffic accident, fall from height).
  • Impact: Associated with significant pain, loss of mobility and independence, increased risk of complications, and substantial mortality (up to 10% at 30 days, 20-30% at 1 year).
  • National Hip Fracture Database (NHFD – UK): Collects data to monitor and improve care quality. Emphasises key performance indicators like prompt surgery, orthogeriatric review.

Risk Factors for Hip Fracture:

  • Older age
  • Female sex
  • Osteoporosis (low bone mineral density)
  • Previous fragility fracture
  • History of falls / increased falls risk
  • Low body weight / malnutrition
  • Smoking, excessive alcohol intake
  • Certain medications (e.g., corticosteroids, psychotropics increasing fall risk)
  • Medical conditions increasing fall risk or affecting bone health (e.g., dementia, Parkinson’s, stroke, rheumatoid arthritis, CKD).
💡 OSCE/PLAB Tip

Hip fracture is a “tracer condition” for frailty and osteoporosis. Management should not just focus on the fracture itself but also on the patient’s overall health, falls risk, bone health, and rehabilitation needs. Think “bone, stone, groan, moan” for osteoporosis risk factors and symptoms if relevant.

Relevant Anatomy & Classification of Hip Fractures

Understanding the anatomy of the proximal femur and fracture classification is key to determining management.

Proximal Femur Anatomy:
  • Femoral Head: Articulates with the acetabulum of the pelvis.
  • Femoral Neck: Connects the head to the shaft. Vulnerable to fracture.
  • Trochanters:
    • Greater Trochanter: Lateral prominence, attachment site for abductor muscles.
    • Lesser Trochanter: Medial prominence, attachment site for iliopsoas muscle.
  • Intertrochanteric Line/Crest: Connects the trochanters.
  • Subtrochanteric Region: Area below the lesser trochanter.
Blood Supply to Femoral Head:
  • Mainly via retinacular arteries (branches of medial and lateral circumflex femoral arteries) which run along the femoral neck.
  • Small contribution from artery of ligamentum teres (more important in children).
  • Significance: Intracapsular fractures (especially displaced femoral neck fractures) can disrupt this blood supply, leading to avascular necrosis (AVN) of the femoral head.
Classification of Hip Fractures:

Broadly classified based on anatomical location relative to the hip joint capsule:

  • 1. Intracapsular Fractures (Femoral Neck Fractures):
    • Occur within the hip joint capsule.
    • Risk of AVN and non-union is higher.
    • Garden Classification (for displacement):
      • Type I: Incomplete, undisplaced, impacted (valgus).
      • Type II: Complete, undisplaced.
      • Type III: Complete, partially displaced (varus tilt, but some contact remains).
      • Type IV: Complete, fully displaced (no contact between fracture fragments).
      (Simplified: Undisplaced = Garden I & II; Displaced = Garden III & IV)
  • 2. Extracapsular Fractures:
    • Occur outside the hip joint capsule. Better blood supply, lower risk of AVN/non-union, but can be unstable and associated with more blood loss.
    • Intertrochanteric Fractures: Occur between the greater and lesser trochanters. Classified by stability (number of fragments, comminution).
    • Subtrochanteric Fractures: Occur below the lesser trochanter, extending down the femoral shaft. Often require different fixation methods. Can be associated with pathological fractures or bisphosphonate use (atypical femoral fractures).
Visual Classification Aid (Simplified)
Hip Fracture Classification

Image depicting common hip fracture types: Subcapital (intracapsular), Transcervical (intracapsular), Basicervical (intracapsular/extracapsular border), Intertrochanteric (extracapsular), Subtrochanteric (extracapsular). Source: Orthobullets.com (Illustrative)

Clinical Assessment & Diagnosis

Prompt recognition and diagnosis are vital.

History:
  • Mechanism of Injury: Usually a fall in older adults. Enquire about circumstances (trip, slip, collapse, “found on floor”). Consider syncope/pre-syncope.
  • Symptoms: Hip/groin/thigh/knee pain, inability to weight-bear.
  • Past Medical History: Osteoporosis, previous fractures, comorbidities (cardiac, respiratory, renal, diabetes, cognitive impairment), medications (anticoagulants, steroids).
  • Social History: Baseline mobility and function (ADLs, IADLs), living situation, social support, smoking/alcohol. This informs rehabilitation potential and discharge planning.
  • Pre-fall functional status is crucial.
Examination (ABCDE approach initially if trauma/unwell):
  • General: Pain, distress. Signs of shock if significant blood loss (more common with extracapsular).
  • Look: Affected leg often shortened and externally rotated (classic sign, especially with displaced fractures). Swelling, bruising (may be delayed).
  • Feel: Tenderness over hip region (femoral neck, greater trochanter).
  • Move: Pain on any attempted movement of the hip (active or passive). Log-rolling the leg is painful. Inability to straight leg raise. Avoid excessive movement if fracture strongly suspected to prevent further displacement or pain.
  • Neurovascular Assessment: Check distal pulses, capillary refill, sensation, and motor function in the affected leg. (Sciatic nerve or femoral nerve injury is rare but possible).
  • Assess for other injuries from the fall (e.g., head injury, wrist fracture, rib fractures).
Investigations:
  • Imaging:
    • X-ray: AP pelvis and lateral hip views of the affected side are standard.
      • Look for fracture line, displacement, angulation. Shenton’s line disruption.
      • If X-ray is negative but high clinical suspicion (e.g., patient unable to weight-bear, significant pain): Further imaging is indicated.
    • MRI: Gold standard for occult hip fractures (not visible on X-ray). Should be performed within 24 hours if X-ray negative and suspicion remains.
    • CT Scan: Alternative if MRI contraindicated or unavailable quickly. Less sensitive than MRI for occult fractures but good for characterising complex fractures or pre-operative planning.
    • Bone Scan: Rarely used acutely for diagnosis now due to delay; MRI/CT preferred.
  • Blood Tests (Pre-operative Workup):
    • FBC (anaemia, platelets), U&Es (renal function, electrolytes), LFTs.
    • Coagulation screen (PT/INR, APTT) – especially if on anticoagulants.
    • Group & Save (Crossmatch if high risk of bleeding or surgery imminent).
    • Glucose, Calcium, Vitamin D, TFTs (may be part of orthogeriatric assessment).
  • ECG: Pre-operative assessment, check for arrhythmias that may have caused fall.
  • Chest X-ray: If respiratory symptoms or for pre-operative assessment.

Initial Management (Pre-operative Care)

Focus on pain relief, medical stabilisation, and preparation for surgery. NICE NG124 provides detailed guidance.

Key Principles (often in A&E and Orthopaedic Ward):
  • Analgesia:
    • Offer regular paracetamol and opioids (e.g., morphine) as needed. Titrate to effect.
    • Consider regional anaesthesia (fascia iliaca compartment block or femoral nerve block) if available and expertise exists – provides excellent analgesia and reduces opioid requirements. Should be performed by trained personnel.
    • Avoid NSAIDs in older adults due to renal, GI, and cardiac risks.
  • IV Fluids: If dehydrated or hypotensive. Monitor fluid balance.
  • Oxygen: If hypoxic.
  • Nil By Mouth (NBM): In anticipation of surgery. Check local NBM guidelines.
  • VTE Prophylaxis: Pharmacological prophylaxis (e.g., LMWH) should be started as soon as possible unless contraindicated or surgery imminent (within hours). Mechanical prophylaxis (anti-embolism stockings) may also be used, but ensure correct fitting and no contraindications (e.g., PVD).
  • Pressure Area Care: Assess risk, use pressure-relieving mattress. Regular repositioning (as pain allows).
  • Medical Optimisation:
    • Identify and manage acute medical problems (e.g., infection, electrolyte imbalance, cardiac issues).
    • Review and manage anticoagulation/antiplatelet therapy in discussion with orthopaedics/anaesthetics/haematology. Bridging may be needed.
    • Optimise chronic conditions (e.g., diabetes, heart failure).
  • Orthogeriatric Liaison: Early involvement of geriatricians is crucial for comprehensive medical assessment and co-management, especially for frail older patients. This improves outcomes.
  • Fasting & Theatre Planning: Aim for surgery on the day of, or day after, admission (ideally within 36 hours of presentation if medically fit). Delays increase complications.
  • Catheterisation: Consider if urinary retention or for accurate fluid balance monitoring, but avoid routine catheterisation.
  • Cognitive Assessment: Screen for delirium and dementia (e.g., AMTS, 4AT).
⚠️ Fascia Iliaca Block

A fascia iliaca compartment block (FICB) is an effective regional anaesthetic technique for hip fractures. It blocks the femoral, lateral femoral cutaneous, and obturator nerves. It can significantly reduce pain and opioid consumption. If available, it should be considered early.

Surgical Management Options

The aim of surgery is to achieve stable fracture fixation or replacement, allowing early mobilisation and pain relief. Choice of surgery depends on fracture type, displacement, patient age, pre-fracture mobility, cognitive status, and comorbidities.

1. Intracapsular (Femoral Neck) Fractures:
  • Undisplaced (Garden I & II):
    • Internal Fixation: Usually with cannulated screws or dynamic hip screw (DHS) with anti-rotation screw. Aims to preserve the native femoral head.
  • Displaced (Garden III & IV):
    • Younger, fit patients (<60-65 years, physiologically young): Internal fixation may be attempted if fracture pattern suitable, but high risk of AVN/non-union. Requires informed discussion.
    • Older patients (typically >65-70 years, or less mobile/cognitively impaired younger patients):
      • Hemiarthroplasty: Replacement of the femoral head with a prosthesis. Articulates with native acetabulum. (e.g., Austin Moore (monopolar, older design), Thompson (monopolar), or more commonly now, Unipolar/Bipolar cemented or uncemented stems). Quicker surgery, less dislocation risk than THR.
      • Total Hip Replacement (THR): Replacement of both femoral head and acetabulum. Considered for patients who were independently mobile outdoors, cognitively intact, medically fit for longer surgery, and not needing walking aids before fracture. Better long-term functional outcomes and less pain/re-operation than hemiarthroplasty if criteria met. Higher dislocation risk.
2. Extracapsular Fractures:
  • Intertrochanteric Fractures:
    • Dynamic Hip Screw (DHS) / Sliding Hip Screw (SHS): Plate and screw construct allowing controlled collapse/impaction at fracture site. Common for stable patterns.
    • Cephalomedullary Nail (Intramedullary Nail): Rod inserted into femoral canal with screws into head/neck. Preferred for unstable patterns, reverse oblique fractures, or subtrochanteric extension. Allows earlier weight-bearing.
  • Subtrochanteric Fractures:
    • Usually require Intramedullary Nailing (long nail). Plate fixation is an alternative in some cases.
Anaesthesia:
  • Choice between general anaesthesia (GA) and regional anaesthesia (e.g., spinal, epidural) depends on patient factors, anaesthetist/surgeon preference. Regional often preferred in frail elderly if suitable.
💡 Cemented vs. Uncemented Arthroplasty

In hemiarthroplasty or THR for hip fracture, cemented fixation of the femoral stem is generally recommended by NICE for most older patients as it allows earlier weight-bearing and may reduce post-operative pain and risk of periprosthetic fracture. Uncemented may be used in younger patients with good bone stock.

Post-operative Care & Rehabilitation

Multidisciplinary team (MDT) approach is essential for recovery.

Immediate Post-operative Care:
  • Pain Management: Continue regular analgesia, adjust based on need. Consider patient-controlled analgesia (PCA) initially if appropriate. Transition to oral.
  • Fluid Balance & Nutrition: Monitor intake/output. Encourage oral intake as tolerated. Nutritional supplements if at risk of malnutrition.
  • VTE Prophylaxis: Continue pharmacological and/or mechanical prophylaxis as per local protocol (usually for ~28-35 days post-op).
  • Wound Care: Monitor for signs of infection, haematoma.
  • Prevention of Complications: E.g., pressure sores, chest infection (encourage deep breathing, mobilisation).
  • Delirium Prevention & Management: Maintain orientation, hydration, avoid problematic medications.
Mobilisation & Rehabilitation:
  • Early Mobilisation: Aim for mobilisation (sit out of bed, stand, walk with aid) on the day after surgery, guided by physiotherapists. Weight-bearing status depends on fracture type and fixation (often full weight-bearing as tolerated for arthroplasty and most internal fixations).
  • Physiotherapy: Crucial for restoring mobility, strength, balance, and confidence. Individualised exercise programme. Gait re-education with appropriate walking aids.
  • Occupational Therapy: Assess ADL function, provide equipment, plan for safe discharge (e.g., home assessment, adaptations).
  • Orthogeriatric Review: Continued medical co-management, addressing comorbidities, optimising bone health, falls prevention.
Discharge Planning:
  • Starts early. MDT discussion involving patient and family/carers.
  • Aim for discharge to usual place of residence if possible.
  • May require intermediate care, community rehabilitation, or package of care at home.
  • Clear information on follow-up, exercises, wound care, pain relief, and when to seek help.

Complications of Hip Fracture (Medical & Surgical)

Hip fractures are associated with a high rate of complications.

Medical Complications (Common in frail elderly):
  • Delirium (Post-operative Confusion): Very common.
  • Infections: Chest infection (pneumonia), Urinary Tract Infection (UTI), wound infection.
  • Venous Thromboembolism (VTE): Deep Vein Thrombosis (DVT), Pulmonary Embolism (PE).
  • Pressure Sores.
  • Cardiovascular: Myocardial Infarction (MI), heart failure, arrhythmias.
  • Anaemia: Due to blood loss from fracture/surgery, or pre-existing. May require transfusion.
  • Electrolyte Imbalance, Renal Impairment.
  • Malnutrition, Dehydration.
  • Constipation (opioid-related, immobility).
  • Functional Decline, Loss of Independence.
  • Depression.
Surgical Complications (Specific to fracture type/procedure):
  • General Surgical Risks: Infection (superficial or deep prosthetic joint infection), bleeding/haematoma, nerve/vessel damage, anaesthetic risks.
  • Complications of Internal Fixation (e.g., for femoral neck or intertrochanteric fractures):
    • Non-union: Fracture fails to heal.
    • Malunion: Fracture heals in an abnormal position.
    • Avascular Necrosis (AVN) of Femoral Head: Especially with displaced intracapsular fractures.
    • Fixation Failure / Cut-out of screws: Implant loosens or cuts through bone.
    • Implant-related pain.
  • Complications of Arthroplasty (Hemiarthroplasty or THR):
    • Dislocation: Higher risk with THR than hemiarthroplasty. Specific precautions taught.
    • Prosthetic Joint Infection (PJI): Serious, may require revision surgery.
    • Periprosthetic Fracture: Fracture around the implant.
    • Loosening of Prosthesis (aseptic or septic).
    • Leg length discrepancy.
    • Heterotopic ossification.
  • Mortality: Significant risk, especially in the first year post-fracture.

Secondary Prevention of Future Fractures

A hip fracture is often a sentinel event indicating underlying osteoporosis and high risk of future fractures. All patients with a hip fracture should be assessed and managed for osteoporosis and falls risk.

Osteoporosis Assessment & Management:
  • All patients with a fragility hip fracture should be considered to have osteoporosis and usually offered treatment without necessarily needing a DEXA scan first (though DEXA may be done for baseline or if uncertainty).
  • Lifestyle Advice:
    • Adequate dietary calcium intake.
    • Sufficient Vitamin D (sun exposure, diet, consider supplements for all).
    • Regular weight-bearing exercise (as able post-rehab).
    • Smoking cessation, sensible alcohol intake.
  • Pharmacological Treatment for Osteoporosis (usually initiated in hospital or by GP post-discharge):
    • Calcium and Vitamin D Supplementation: Often co-prescribed with bone-sparing agents, especially if dietary intake is insufficient or for housebound individuals. (e.g., Adcal-D3®, Calcichew D3 Forte®).
    • Bisphosphonates (First-line):
      • Oral: Alendronate (weekly), Risedronate (weekly/monthly). Counsel on administration: take on empty stomach with plenty of water, remain upright for 30-60 mins. Side effects: Oesophagitis, GI upset. Contraindications: oesophageal abnormalities, hypocalcaemia, severe renal impairment (eGFR <30-35).
      • IV: Zoledronic acid (annual infusion). Option if oral bisphosphonates contraindicated/not tolerated, or for adherence. Requires adequate renal function. Acute phase reaction common (flu-like symptoms).
      • Rare risks: Osteonecrosis of the jaw (ONJ), atypical femoral fractures (AFFs) with long-term use. Review treatment duration (e.g., after 3-5 years).
    • Denosumab: Monoclonal antibody (subcutaneous injection every 6 months). Alternative if bisphosphonates not suitable. Risk of ONJ/AFFs. Rapid bone loss on cessation if not followed by another agent.
    • Other options (specialist use): Teriparatide (PTH analogue), Raloxifene (SERM), Romosozumab.
Falls Prevention:
  • All hip fracture patients should have a multifactorial falls risk assessment and interventions implemented (as discussed in Falls section).
  • This is a crucial part of secondary fracture prevention.
💡 Fracture Liaison Services (FLS)

Many hospitals have FLS, which systematically identify, investigate, and initiate treatment for osteoporosis in patients with fragility fractures. This improves secondary prevention rates.

Flashcards: Hip Fracture Management

Click on each card to reveal the answer.

What is the classic clinical presentation of a displaced hip fracture on examination?

(Click to flip)

Answer

The affected leg is shortened and externally rotated, with inability to weight-bear and pain on hip movement.

What is the main concern with displaced intracapsular (femoral neck) fractures regarding blood supply?

(Click to flip)

Answer

Disruption of the retinacular arteries, leading to a high risk of Avascular Necrosis (AVN) of the femoral head.

What imaging is indicated if X-rays are negative for a hip fracture but clinical suspicion remains high?

(Click to flip)

Answer

MRI scan (or CT if MRI contraindicated/unavailable quickly).

Name two common surgical options for a displaced intracapsular femoral neck fracture in an older, less mobile patient.

(Click to flip)

Answer

1. Hemiarthroplasty
2. Total Hip Replacement (THR – if more active/fit)

What type of fixation is commonly used for stable intertrochanteric hip fractures?

(Click to flip)

Answer

Dynamic Hip Screw (DHS) / Sliding Hip Screw (SHS).

What is a fascia iliaca compartment block (FICB) used for in hip fracture management?

(Click to flip)

Answer

To provide regional anaesthesia/analgesia by blocking the femoral, lateral femoral cutaneous, and obturator nerves, reducing pain and opioid requirements.

Name three common medical complications following hip fracture surgery in older adults.

(Click to flip)

Answer

Delirium, Chest infection (pneumonia), UTI, VTE (DVT/PE), Pressure sores, Anaemia.

What is the first-line pharmacological treatment for osteoporosis in most patients following a hip fracture?

(Click to flip)

Answer

Oral bisphosphonates (e.g., Alendronate, Risedronate), usually with Calcium and Vitamin D supplementation.

Hip Fracture Management Quiz

Test your knowledge with these Orthopaedic questions.

1. An 80-year-old woman falls at home and presents with a shortened and externally rotated right leg, unable to weight-bear. X-rays confirm a displaced intracapsular femoral neck fracture. She was independently mobile before the fall. What is the most likely definitive surgical management?

A. Internal fixation with cannulated screws.
B. Hemiarthroplasty or Total Hip Replacement.
C. Dynamic Hip Screw (DHS).
D. Conservative management with bed rest.
Explanation: For a displaced intracapsular fracture in an older adult, arthroplasty (hemiarthroplasty or THR if fit and active) is generally preferred due to the high risk of AVN and non-union with internal fixation. DHS is for extracapsular fractures. Conservative management is rarely appropriate.

2. A 75-year-old man sustains an intertrochanteric hip fracture. Which surgical implant is commonly used for this type of fracture?

A. Cannulated screws.
B. Hemiarthroplasty.
C. Dynamic Hip Screw (DHS) or Cephalomedullary Nail.
D. Total Hip Replacement (THR).
Explanation: Intertrochanteric fractures are extracapsular and are typically treated with internal fixation using a Dynamic Hip Screw (DHS) for stable patterns or a Cephalomedullary Nail for unstable patterns. Arthroplasty is for intracapsular fractures.

3. A patient with a suspected hip fracture has a negative initial X-ray but is unable to weight-bear and has significant hip pain. What is the most appropriate next diagnostic step?

A. Repeat X-ray in 24 hours.
B. MRI scan of the hip.
C. Bone scan.
D. Discharge with analgesia and advise rest.
Explanation: If clinical suspicion for a hip fracture is high despite negative X-rays (occult fracture), an MRI is the gold standard for diagnosis and should be performed promptly (ideally within 24 hours).

4. Which of the following is a key component of initial pre-operative management for a patient with a hip fracture?

A. Immediate full weight-bearing mobilisation.
B. Adequate analgesia, including consideration of regional nerve blocks.
C. Withholding VTE prophylaxis until after surgery.
D. Routine administration of broad-spectrum antibiotics.
Explanation: Effective pain relief is paramount. Regional blocks like fascia iliaca block are very effective. VTE prophylaxis should be started pre-operatively if no contraindication. Antibiotics are given prophylactically just before surgery, not routinely on admission. Mobilisation is post-op.

5. Avascular necrosis (AVN) of the femoral head is a significant complication primarily associated with which type of hip fracture?

B. Displaced intracapsular (femoral neck) fractures.
A. Undisplaced intracapsular fractures.
C. Intertrochanteric fractures.
D. Subtrochanteric fractures.
Explanation: Displaced intracapsular fractures have a high risk of disrupting the blood supply (retinacular arteries) to the femoral head, leading to AVN. Extracapsular fractures have a much lower risk.

6. According to NICE guidelines, when should surgery ideally be performed for a hip fracture patient who is medically fit?

A. Within 72 hours of admission.
B. On the day of, or the day after, admission (ideally within 36 hours).
C. Within 1 week of admission.
D. After 48 hours to allow for full medical optimisation.
Explanation: Prompt surgery (within 36 hours of presentation, or on day of/day after admission) is associated with better outcomes and reduced complications. Unnecessary delays should be avoided.

7. Which of the following is a common and serious MEDICAL complication in elderly patients following hip fracture surgery?

A. Prosthetic dislocation.
B. Delirium.
C. Non-union of the fracture.
D. Avascular necrosis.
Explanation: Delirium is a very common medical complication. Dislocation, non-union, and AVN are primarily surgical/fracture-related complications.

8. What is the primary aim of prescribing bisphosphonates to a patient after a hip fracture?

A. To accelerate fracture healing.
B. To reduce the risk of future fragility fractures by treating osteoporosis.
C. To provide pain relief.
D. To prevent prosthetic joint infection.
Explanation: Bisphosphonates are anti-resorptive agents used to treat osteoporosis and reduce the risk of subsequent fragility fractures. They do not primarily accelerate healing or relieve acute pain.

9. The Garden classification is used for which type of hip fracture?

A. Intertrochanteric fractures.
B. Intracapsular (femoral neck) fractures.
C. Subtrochanteric fractures.
D. Acetabular fractures.
Explanation: The Garden classification describes the degree of displacement of intracapsular (femoral neck) fractures.

10. Early mobilisation after hip fracture surgery is crucial primarily to:

A. Ensure the surgical implant beds in correctly.
B. Prevent complications such as VTE, chest infections, and pressure sores, and to restore function.
C. Reduce post-operative pain significantly.
D. Allow for earlier discharge regardless of functional status.
Explanation: Early mobilisation helps prevent numerous immobility-related complications and is key to functional recovery. While it can help with pain over time, its primary role is complication prevention and functional restoration.

Knee Pain Assessment (GP Focus)

Knee pain is a frequent complaint in primary care, affecting individuals of all ages. A systematic approach to history taking and clinical examination is essential to formulate a differential diagnosis, identify red flags, and guide appropriate management or referral.

Overview & Basic Knee Anatomy

The knee is a complex hinge joint comprising bones, cartilage, ligaments, and tendons, all of which can be sources of pain.

Key Anatomical Structures:

  • Bones: Femur (distal), Tibia (proximal), Patella (kneecap). Fibula is lateral but not part of the true knee joint.
  • Articular Cartilage: Smooth lining covering bone ends, allows low-friction movement. Degeneration leads to osteoarthritis.
  • Menisci (Medial and Lateral): C-shaped fibrocartilage discs between femur and tibia. Act as shock absorbers, improve joint congruity, aid lubrication. Prone to tears.
  • Ligaments (Provide Stability):
    • Cruciate Ligaments: Anterior Cruciate Ligament (ACL – prevents anterior tibial translation), Posterior Cruciate Ligament (PCL – prevents posterior tibial translation).
    • Collateral Ligaments: Medial Collateral Ligament (MCL – resists valgus stress), Lateral Collateral Ligament (LCL – resists varus stress).
  • Tendons:
    • Quadriceps Tendon: Connects quadriceps muscle to patella.
    • Patellar Tendon (Ligament): Connects patella to tibial tuberosity.
    • Hamstring tendons, Iliotibial band (ITB).
  • Bursae: Fluid-filled sacs reducing friction (e.g., prepatellar, infrapatellar, anserine). Can become inflamed (bursitis).
  • Synovium: Lining of the joint capsule, produces synovial fluid. Inflammation (synovitis) common in arthritis.
Placeholder for Basic Knee Anatomy Diagram
(Consider adding a simple diagram here)
💡 PLAB/MSRA Focus

Focus on recognizing patterns of common knee conditions (OA, meniscal tears, ligament sprains, patellofemoral pain), performing key examination manoeuvres (e.g., for effusion, ligament stability, meniscal tears), and identifying red flags requiring urgent action.

History Taking for Knee Pain

A detailed history is crucial for narrowing down the differential diagnosis.

Using “SOCRATES” Framework:

  • Site:
    • Anterior: Patellofemoral pain, patellar tendinopathy, Osgood-Schlatter, prepatellar bursitis.
    • Medial: Medial meniscus tear, MCL sprain, medial compartment OA, anserine bursitis.
    • Lateral: Lateral meniscus tear, LCL sprain, lateral compartment OA, ITB syndrome.
    • Posterior: Baker’s cyst, hamstring tendinopathy, PCL injury, referred pain.
    • Diffuse/generalized: OA, inflammatory arthritis, septic arthritis.
  • Onset:
    • Acute/Traumatic: Mechanism of injury is vital (e.g., twisting, direct blow, valgus/varus force, hyperextension). Did they hear a “pop” or “snap” (suggests ACL tear, fracture)? Immediate swelling (haemarthrosis) vs. delayed swelling (effusion)? Ability to weight-bear immediately?
    • Gradual/Insidious: Overuse injury, degenerative condition (OA), inflammatory process.
  • Character: Sharp, dull, aching, burning, stabbing.
  • Radiation: Any radiation to thigh or shin? (May suggest referred pain from hip/back, or nerve involvement).
  • Associated Symptoms:
    • Swelling: Timing (immediate suggests haemarthrosis – ACL tear, fracture; delayed suggests synovial effusion – meniscal tear, OA flare), location.
    • Stiffness: Morning stiffness (prolonged in inflammatory arthritis, brief in OA), stiffness after inactivity.
    • Mechanical Symptoms:
      • Locking: True locking (inability to fully extend/flex) suggests meniscal tear, loose body. Pseudo-locking due to pain/effusion.
      • Giving Way / Instability: Sensation of knee “buckling” – suggests ligamentous laxity (ACL, PCL, collaterals) or quadriceps weakness/inhibition.
      • Clicking/Popping/Grinding (Crepitus): Common, can be meniscal tear, OA, patellofemoral issues.
    • Weakness, numbness, paraesthesia in leg/foot.
    • Fever, malaise, warmth, erythema (suggests infection, gout).
  • Timing / Pattern: Worse with specific activities (stairs, squatting, running, walking), worse at night (inflammatory, tumour, severe OA), better/worse with rest.
  • Exacerbating / Relieving Factors: Activity, rest, analgesia, ice/heat.
  • Severity: Pain score (0-10), impact on function (walking distance, ADLs, work, sport).

Other Important History:

  • Past Medical History: Previous knee injuries/surgery, OA, gout, pseudogout, inflammatory arthritis (RA, PsA, SpA), IBD, psoriasis, bleeding disorders, DVT/PE risk factors (for Baker’s cyst complications).
  • Drug History: Analgesics used, steroids, anticoagulants, fluoroquinolones.
  • Family History: OA, inflammatory arthritis.
  • Social History: Occupation, sports/hobbies, impact on life.
  • Red Flags & Yellow Flags (see dedicated sections).

Knee Examination (“Look, Feel, Move, Special Tests”)

A systematic examination comparing both knees is essential.

General:

  • Observe patient walking (gait assessment – antalgic, Trendelenburg, varus/valgus thrust).
  • Patient standing: Alignment (varus/valgus), muscle bulk, fixed flexion deformity.

LOOK (Patient supine, knee exposed from mid-thigh to ankle):

  • Skin: Scars, erythema, bruising, rashes (e.g., psoriasis), swelling (localised, diffuse).
  • Swelling:
    • Effusion: Diffuse swelling, loss of normal hollows around patella. Test for effusion (patellar tap, bulge/sweep test).
    • Localised swelling: Prepatellar bursitis (anterior to patella), Baker’s cyst (popliteal fossa).
  • Muscle Wasting: Quadriceps (especially VMO – vastus medialis obliquus).
  • Deformity: Varus (bow-legged), valgus (knock-kneed), fixed flexion deformity. Patellar position.

FEEL (Palpation):

  • Temperature: Compare knees. Warmth suggests inflammation/infection.
  • Tenderness: Systematically palpate:
    • Joint lines (medial and lateral – for meniscal tears, OA).
    • Patellar facets, tibial tuberosity, quadriceps/patellar tendons.
    • Collateral ligaments (MCL, LCL insertions).
    • Popliteal fossa (Baker’s cyst).
    • Around bursae (prepatellar, anserine).
  • Effusion Tests:
    • Patellar Tap (Ballottement): For moderate/large effusion. Push fluid from suprapatellar pouch downwards, then sharply tap patella. Positive if patella “bounces” or clicks on femur.
    • Bulge Test / Sweep Test / Wipe Test: For small effusion. Stroke medial side of knee upwards to “milk” fluid into suprapatellar pouch, then stroke lateral side downwards. Positive if fluid bulge seen on medial side.
  • Crepitus: Palpable grinding with movement (common in OA, patellofemoral issues).

MOVE (Range of Motion – Active and Passive):

  • Flexion: Normal ~0-135/140 degrees. (Heel to buttock).
  • Extension: Normal 0 degrees (straight). Hyperextension up to 5-10 degrees can be normal. Note any fixed flexion deformity (inability to fully extend).
  • Assess for pain, crepitus, or restriction during movement. Compare sides.

SPECIAL TESTS (Perform selectively based on history. A few key tests for GP):

Note: Sensitivity/specificity of these tests vary. Often used in combination.

  • Meniscal Tear Tests:
    • McMurray’s Test: Flex knee, externally/internally rotate tibia, then extend knee while applying varus/valgus stress and palpating joint line. Pain or click suggests meniscal tear. (Often difficult to perform reliably in primary care).
    • Thessaly Test (Standing): Patient stands on affected leg with knee flexed at 5° and 20°, then rotates body internally/externally. Pain/locking/catching suggests meniscal tear. (Requires good balance).
    • Joint Line Tenderness: Good sensitivity but lower specificity.
  • Ligament Stability Tests:
    • Anterior Cruciate Ligament (ACL):
      • Anterior Drawer Test: Knee flexed 90°, sit on foot, pull tibia anteriorly. Increased anterior translation suggests ACL tear.
      • Lachman’s Test: Knee flexed 20-30°, stabilise femur, pull tibia anteriorly. More sensitive than anterior drawer. Look for laxity and quality of end-feel (soft/mushy in ACL tear).
    • Posterior Cruciate Ligament (PCL):
      • Posterior Drawer Test: Knee flexed 90°, push tibia posteriorly. Increased posterior translation suggests PCL tear. Sag sign (tibia drops back) may be visible.
    • Collateral Ligaments (MCL & LCL):
      • Valgus Stress Test (for MCL): Knee slightly flexed (20-30°), apply valgus force to knee while stabilising ankle. Pain or increased medial joint line opening suggests MCL injury. Repeat in full extension (if lax in extension, suggests more severe injury involving ACL/PCL/capsule).
      • Varus Stress Test (for LCL): Knee slightly flexed, apply varus force. Pain or increased lateral joint line opening suggests LCL injury.
  • Patellofemoral Joint Tests:
    • Patellar Grind Test / Clarke’s Sign: Press patella distally, ask patient to contract quadriceps. Pain suggests patellofemoral chondromalacia/pain syndrome. (Low sensitivity/specificity).
    • Patellar Apprehension Test: Gently push patella laterally. Apprehension or pain suggests patellar instability.

Also examine hip and lumbar spine if referred pain is suspected.

Red Flags in Knee Pain

These indicate potentially serious underlying pathology requiring urgent investigation or referral.

  • Suspected Septic Arthritis: (MEDICAL EMERGENCY – A&E)
    • Hot, swollen, erythematous joint.
    • Severe pain, especially on minimal movement.
    • Restricted range of motion (active and passive).
    • Systemic signs: Fever, rigors, malaise, tachycardia.
    • Recent infection, joint surgery/injection, immunosuppression, IV drug use are risk factors.
  • Suspected Fracture: (A&E or Ortho)
    • Significant trauma.
    • Inability to weight-bear immediately after injury or for 4 steps (Ottawa Knee Rules apply here).
    • Obvious deformity.
    • Bony tenderness (patella, fibular head).
    • Consider fragility fracture in elderly/osteoporotic with less trauma.
  • Acute Ligament Rupture with Significant Instability / Haemarthrosis: (Urgent Ortho/A&E)
    • Audible “pop” at time of injury (e.g., ACL tear).
    • Rapid onset large effusion (haemarthrosis) within hours.
    • Gross instability or giving way.
    • Inability to continue activity.
  • Locked Knee (True Mechanical Lock): (Urgent Ortho)
    • Inability to fully extend or flex the knee, often due to displaced meniscal tear or loose body.
  • Neurovascular Compromise: (EMERGENCY – A&E)
    • Signs after trauma/dislocation: Pale/cold foot, absent pulses, paraesthesia, weakness (foot drop). Suspect popliteal artery injury or peroneal nerve injury.
  • Suspected Malignancy (Bone Tumour): (Urgent Ortho/Oncology via 2WW)
    • Persistent, progressive bone pain, especially night pain unrelated to movement.
    • Unexplained swelling or mass.
    • History of cancer.
    • Unexplained weight loss, fatigue.
    • Pathological fracture.
  • Deep Vein Thrombosis (DVT) / Complicated Baker’s Cyst: (Urgent Assessment/USS)
    • Calf swelling, tenderness, warmth, erythema (can mimic cellulitis or Baker’s cyst rupture). Consider DVT risk factors (Wells’ score). Baker’s cyst can dissect or rupture causing calf pain/swelling.
  • Compartment Syndrome (Acute): (EMERGENCY – A&E/Ortho)
    • Severe pain out of proportion to injury, tense swollen compartment, pain on passive stretch of muscles in compartment, paraesthesia, pulselessness (late sign). Often after significant trauma/fracture.

Always consider the patient’s age, mechanism of injury, and overall clinical picture when assessing for red flags.

Common Causes of Anterior Knee Pain

Pain felt at the front of the knee.

  • Patellofemoral Pain Syndrome (PFPS) / “Runner’s Knee”:
    • Common in young, active individuals, often females.
    • Diffuse ache around or behind patella, worse with stairs (especially going down), squatting, prolonged sitting (“cinema sign”), running, jumping.
    • May have crepitus, mild swelling. Often related to maltracking, muscle imbalance (weak VMO/glutes, tight ITB/hamstrings).
    • Management: Physio (VMO strengthening, gluteal activation, stretching), activity modification, taping, orthotics if foot mechanics contribute.
  • Patellar Tendinopathy / “Jumper’s Knee”:
    • Pain localised to inferior pole of patella or along patellar tendon.
    • Overuse injury, common in jumping sports. Pain worse with activity, tender on palpation.
    • Management: Activity modification, eccentric exercises, physio, analgesia.
  • Osgood-Schlatter Disease: (Adolescents)
    • Pain, tenderness, and swelling over tibial tuberosity.
    • Apophysitis (inflammation of growth plate) due to repetitive traction from patellar tendon during growth spurts. Common in sporty adolescents.
    • Management: Activity modification, ice, analgesia, stretching. Usually self-limiting once growth ceases.
  • Sinding-Larsen-Johansson Syndrome: (Adolescents)
    • Similar to Osgood-Schlatter but pain is at inferior pole of patella. Apophysitis at patellar origin.
  • Prepatellar Bursitis / “Housemaid’s Knee”:
    • Swelling, tenderness, +/- warmth directly over anterior patella. Pain with kneeling or direct pressure.
    • Caused by repetitive friction/pressure or direct trauma. Consider infection if very red/hot/systemic symptoms.
    • Management: Avoid aggravating activities, ice, NSAIDs. Aspiration +/- steroid injection if persistent (ensure no infection). Antibiotics if septic.
  • Infrapatellar Bursitis / “Clergyman’s Knee”:
    • Swelling and tenderness over infrapatellar region, either side of patellar tendon.
  • Chondromalacia Patellae:
    • Softening/degeneration of articular cartilage on undersurface of patella. Often part of PFPS spectrum. Retropatellar pain, crepitus.
  • Patellar Instability / Subluxation / Dislocation:
    • Often traumatic. Sensation of patella “slipping out,” acute pain, swelling. Apprehension test positive. Refer if recurrent or significant first episode.

Common Causes of Medial & Lateral Knee Pain

Medial Knee Pain (Pain on inner side):

  • Medial Meniscus Tear:
    • Often twisting injury on a flexed, weight-bearing knee. Can be degenerative in older adults with minimal trauma.
    • Medial joint line tenderness, effusion (may be delayed), pain with twisting/squatting. May have locking or catching.
    • McMurray’s/Thessaly test may be positive.
    • Management: RICE, physio. Refer if mechanical symptoms or persistent pain despite conservative care.
  • Medial Collateral Ligament (MCL) Sprain:
    • Valgus stress injury (force to outer knee pushing it inwards).
    • Medial knee pain, tenderness over MCL, +/- instability, local swelling/bruising. Pain on valgus stress test.
    • Management: RICE, analgesia, hinged knee brace (for Grade II/III), physio. Most heal non-operatively.
  • Medial Compartment Osteoarthritis (OA):
    • Gradual onset pain, stiffness <30 mins, worse with activity. Medial joint line tenderness, crepitus, possible varus deformity.
    • See Osteoarthritis section for management.
  • Pes Anserine Bursitis/Tendinopathy:
    • Pain and tenderness over medial aspect of proximal tibia, inferior to medial joint line (insertion of sartorius, gracilis, semitendinosus).
    • Common in runners, OA patients.
    • Management: Activity modification, ice, NSAIDs, physio, steroid injection.

Lateral Knee Pain (Pain on outer side):

  • Lateral Meniscus Tear:
    • Similar mechanism and symptoms to medial tear but pain/tenderness on lateral joint line.
    • Management similar to medial meniscal tears.
  • Lateral Collateral Ligament (LCL) Sprain:
    • Varus stress injury (force to inner knee pushing it outwards). Less common than MCL sprain.
    • Lateral knee pain, tenderness over LCL, +/- instability. Pain on varus stress test.
    • Management: Similar to MCL sprains. LCL injuries can be associated with other posterolateral corner injuries requiring specialist input.
  • Lateral Compartment Osteoarthritis (OA):
    • Gradual onset pain, stiffness, worse with activity. Lateral joint line tenderness, crepitus, possible valgus deformity.
  • Iliotibial Band (ITB) Syndrome / “Runner’s Knee”:
    • Common overuse injury in runners/cyclists.
    • Pain over lateral femoral epicondyle or lateral tibial condyle, worse with repetitive knee flexion/extension (e.g., running downhill). Tender over ITB insertion. Noble’s compression test positive.
    • Management: Activity modification, physio (stretching ITB, gluteal strengthening), foam rolling, NSAIDs.

Common Causes of Posterior Knee Pain & Swelling

Pain or swelling at the back of theknee.

  • Baker’s Cyst (Popliteal Cyst):
    • Swelling in popliteal fossa, often causing tightness or ache. Usually a communication with knee joint, fluid accumulates due to underlying intra-articular pathology (e.g., OA, meniscal tear, inflammatory arthritis).
    • Can rupture or dissect into calf, causing acute pain, swelling, erythema (mimicking DVT – “pseudothrombophlebitis”).
    • Management: Treat underlying knee pathology. Aspiration +/- steroid injection (of cyst or joint) may provide temporary relief. Reassure if asymptomatic/mild. USS can confirm and rule out DVT if ruptured.
  • Hamstring Tendinopathy/Injury:
    • Pain at insertion of hamstring tendons (semimembranosus, semitendinosus, biceps femoris) on posterior aspect of knee/proximal calf.
    • Often overuse or acute strain. Pain with resisted knee flexion or stretching hamstrings.
    • Management: RICE, physio, eccentric exercises.
  • Posterior Cruciate Ligament (PCL) Injury:
    • Less common than ACL. “Dashboard injury” (direct blow to anterior tibia with knee flexed) or fall onto flexed knee.
    • Posterior knee pain, instability (especially downhill/stairs), positive posterior drawer/sag sign.
    • Management: Often conservative (physio) for isolated Grade I/II. Refer for Grade III or combined injuries.
  • Gastrocnemius Strain / “Tennis Leg”:
    • Acute pain in medial head of gastrocnemius (upper calf), often during forceful push-off. May feel a “pop”. Bruising, tenderness.
  • Referred Pain:
    • From lumbar spine (e.g., S1/S2 radiculopathy) or hip.
  • Deep Vein Thrombosis (DVT):
    • Although typically calf pain, DVT in popliteal vein can cause posterior knee pain/swelling. Consider risk factors, Wells’ score. Urgent USS if suspected.
  • Popliteal Artery Aneurysm (Rare):
    • Pulsatile mass in popliteal fossa. Can cause claudication or distal embolisation. Refer to vascular surgery.

Common Causes of Knee Pain in Children & Adolescents

Knee pain in this age group has some specific considerations.

  • Osgood-Schlatter Disease: (See Anterior Knee Pain) – Apophysitis of tibial tuberosity. Common in sporty adolescents during growth spurts.
  • Sinding-Larsen-Johansson Syndrome: (See Anterior Knee Pain) – Apophysitis of inferior pole of patella.
  • Patellofemoral Pain Syndrome (PFPS): Common, especially in adolescent girls. (See Anterior Knee Pain).
  • Patellar Tendinopathy.
  • Juvenile Idiopathic Arthritis (JIA):
    • Persistent joint swelling, pain, stiffness >6 weeks, onset <16 years. Knee is commonly affected.
    • Morning stiffness, limp, systemic features (fever, rash in systemic JIA).
    • Refer to paediatric rheumatology if suspected.
  • Traumatic Injuries (Sprains, Strains, Meniscal/Ligament Tears):
    • Increasingly common with sports participation. Mechanism of injury important. Physeal (growth plate) fractures can occur.
    • ACL tears are seen in adolescents.
  • Osteochondritis Dissecans (OCD):
    • Localized area of bone and overlying cartilage separates from articular surface, often on medial femoral condyle.
    • Vague, activity-related pain, swelling, catching, locking if fragment loose.
    • More common in adolescent males. Refer to orthopaedics.
  • Slipped Capital Femoral Epiphysis (SCFE): (EMERGENCY/URGENT ORTHO REFERRAL)
    • Displacement of femoral head epiphysis. Typically occurs in overweight adolescent males during growth spurt (age 10-16).
    • Presents with hip, groin, thigh, OR referred knee pain. Limp, externally rotated leg.
    • Always examine the hip in a child presenting with knee pain. X-ray hips (AP and frog-leg lateral).
  • Perthes Disease (Legg-Calvé-Perthes):
    • Avascular necrosis of femoral head. Typically younger children (age 4-8). Presents with hip/groin/thigh or referred knee pain, limp. Hip examination crucial.
  • Transient Synovitis of the Hip (“Irritable Hip”):
    • Common cause of acute hip pain/limp in children (age 3-10), often post-viral. Can present with referred knee pain. Diagnosis of exclusion. Hip examination.
  • Growing Pains:
    • Diagnosis of exclusion. Bilateral, intermittent, non-articular pain (shins, calves, thighs), often at night, resolves by morning. Child well between episodes, normal examination.
  • Septic Arthritis / Osteomyelitis: (EMERGENCY – A&E/Ortho)
    • As in adults, but child may present with pseudoparalysis (refusal to move limb), fever, irritability. Knee is a common site.
  • Benign or Malignant Bone Tumours (Rare):
    • Persistent, progressive pain, night pain, swelling, pathological fracture. E.g., Osteosarcoma, Ewing’s sarcoma often around knee. Red flag.
Hip Pathology in Children

Remember that hip pathology (e.g., SCFE, Perthes, transient synovitis) can commonly present with referred pain to the knee in children. Always examine the hip joint in a child complaining of knee pain.

Investigations for Knee Pain in Primary Care

Many knee conditions are diagnosed clinically. Investigations are used selectively.

Blood Tests:

  • Generally NOT required for most acute traumatic or mechanical knee pain.
  • Consider if:
    • Suspected Septic Arthritis: FBC, CRP, ESR, Blood Cultures (before antibiotics if possible). Joint aspiration for MC&S and crystals by specialist is diagnostic.
    • Suspected Inflammatory Arthritis (e.g., RA, JIA, PsA, Gout, Pseudogout): FBC, ESR, CRP, U&Es, LFTs.
      • If RA suspected: Rheumatoid Factor (RF), Anti-CCP antibodies.
      • If Gout suspected: Serum Urate (can be normal during acute attack, better checked 4-6 weeks later).
      • If other connective tissue disease features: ANA.
    • Suspected Malignancy: FBC, ESR, CRP, Calcium, Alk Phos.
    • If considering NSAIDs long-term: Baseline U&Es, LFTs.

Imaging:

  • X-rays (Weight-bearing AP, lateral, skyline/patellar views often requested):
    • Indicated if:
      • Suspected fracture (Ottawa Knee Rules can guide necessity after acute injury).
      • Suspected Osteoarthritis (to confirm diagnosis if atypical, assess severity, or pre-referral). Look for joint space narrowing, osteophytes, subchondral sclerosis/cysts.
      • Suspected bone tumour or other bony lesion.
      • Persistent pain unresponsive to conservative treatment.
      • In children: Osgood-Schlatter (may show fragmentation of tibial tuberosity but often clinical diagnosis), SCFE/Perthes (hip X-rays crucial).
    • Often normal in soft tissue injuries (ligament, meniscus, tendinopathy).
  • Ultrasound (USS):
    • Useful for:
      • Detecting effusions, synovitis.
      • Assessing tendons (tendinopathy, tears – e.g. patellar, quadriceps).
      • Assessing bursae (bursitis).
      • Diagnosing Baker’s cyst and assessing for rupture/DVT.
      • Guiding aspirations or injections.
      • Assessing some superficial ligament injuries.
    • Not good for intra-articular structures like menisci or cruciate ligaments.
  • Magnetic Resonance Imaging (MRI):
    • Gold standard for soft tissue pathology (menisci, ligaments, cartilage, tendons, bone marrow oedema).
    • Not routinely available/indicated from primary care for most knee pain. Usually requested by specialists or MSK interface services.
    • Consider requesting (or referring for consideration of MRI) if:
      • Red flags for serious pathology (e.g., tumour, infection, osteonecrosis) not clear on X-ray.
      • Persistent mechanical symptoms (locking, instability) suggestive of significant meniscal tear or ligament rupture, where surgical intervention is being considered.
      • Diagnostic uncertainty despite clinical assessment and other imaging.
      • Failure of conservative management for significant symptoms.

Ottawa Knee Rules (To decide if X-ray needed after acute knee injury):

An X-ray of the knee is required if ANY of the following are present:

  1. Age ≥55 years.
  2. Isolated tenderness of the patella (no bone tenderness of knee other than patella).
  3. Tenderness at the head of the fibula.
  4. Inability to flex knee to 90 degrees.
  5. Inability to weight-bear both immediately after injury AND in the emergency department/clinic (for 4 steps – unable to transfer weight twice onto each leg regardless of limping).

These rules have high sensitivity for detecting clinically significant fractures.

Initial Management Principles & Referral Criteria for Knee Pain

General Initial Management (Non-Red Flag, Mechanical Pain):

Based on the “PEACE & LOVE” principle for soft tissue injuries, or general MSK advice.

  • Protection: Avoid aggravating activities for first few days.
  • Elevation: Raise the knee above heart level to reduce swelling.
  • Avoid Anti-inflammatories (initially): Some debate, but avoiding NSAIDs in first few days may allow natural healing. Consider paracetamol.
  • Compression: Elastic bandage can help control swelling and provide support.
  • Education: About the condition, expected recovery, self-management.

  • Load: Gradual introduction of mechanical stress once symptoms allow, to promote healing and strength.
  • Optimism: Encourage positive outlook, address yellow flags.
  • Vascularisation: Pain-free cardiovascular activity to improve blood flow.
  • Exercise: Restore mobility, strength, and proprioception.

Specific Initial Advice:

  • Analgesia:
    • Paracetamol (regularly).
    • Topical NSAIDs (e.g., for OA, localised soft tissue pain).
    • Oral NSAIDs (short course, if appropriate, consider PPI cover) for inflammatory component or more severe pain.
    • Weak opioids for short-term use in severe acute pain if other options insufficient/contraindicated.
  • Relative Rest / Activity Modification: Avoid activities that clearly worsen pain.
  • Ice/Heat: Ice for acute swelling/pain (15-20 mins, wrapped). Heat for muscle stiffness.
  • Simple Exercises: Gentle range of motion, isometric quadriceps exercises.
  • Weight Management advice if overweight/obese (especially for OA, PFPS).
  • Support/Bracing: Simple neoprene sleeve or hinged brace may provide comfort/support for some conditions but avoid over-reliance.

Referral Criteria (General – specific conditions may have own pathways):

  • Emergency (A&E / Same-day Ortho):
    • Suspected septic arthritis.
    • Acute fracture or dislocation.
    • Neurovascular compromise.
    • Acute compartment syndrome.
    • Irreducible locked knee.
    • SCFE in children (urgent ortho).
  • Urgent Referral (e.g., within days to 2 weeks, depending on local pathways):
    • Suspected malignancy (2WW pathway).
    • Significant acute ligamentous injury (e.g., ACL tear with haemarthrosis/instability) in active individual.
    • Persistent true locking.
    • Inflammatory arthritis with acute flare or diagnostic uncertainty requiring rapid specialist input.
  • Routine Referral (to Orthopaedics, Rheumatology, Physiotherapy, MSK Interface Clinic, Pain Clinic):
    • Orthopaedics:
      • Severe OA unresponsive to conservative measures, for consideration of joint replacement.
      • Persistent mechanical symptoms (locking, giving way) from meniscal tear or ligament instability not improving with physio.
      • Specific conditions requiring surgical opinion (e.g., recurrent patellar dislocation, OCD).
    • Rheumatology:
      • Suspected inflammatory arthritis (RA, PsA, SpA, crystal arthropathy).
      • Diagnostic uncertainty for widespread MSK pain (e.g., ?fibromyalgia vs other).
    • Physiotherapy / MSK Interface Clinic:
      • Most non-red flag MSK knee pain not resolving with initial self-management/GP advice (e.g., PFPS, tendinopathies, mild/moderate OA, stable meniscal/ligament injuries).
      • For rehabilitation, exercise prescription, manual therapy, advice.
    • Pain Management Clinic:
      • Chronic knee pain (>3-6 months) with significant functional impact, especially if complex psychosocial factors or failure of other treatments.

Always provide safety netting advice: what to do if symptoms worsen, new red flags develop, or no improvement within expected timeframe.

Flashcards: Knee Pain Assessment

Click on each card to reveal the answer.

A patient reports an audible “pop” during a twisting knee injury, followed by rapid large swelling. What ligament is commonly injured?

(Click to flip)

Answer

Anterior Cruciate Ligament (ACL). Rapid large swelling suggests haemarthrosis.

What are the Ottawa Knee Rules used for?

(Click to flip)

Answer

To determine the need for an X-ray after an acute knee injury to rule out a fracture.

Pain and tenderness localised to the tibial tuberosity in an active adolescent is characteristic of what condition?

(Click to flip)

Answer

Osgood-Schlatter Disease.

A hot, swollen, erythematous knee with severe pain on minimal movement and fever is highly suggestive of what emergency condition?

(Click to flip)

Answer

Septic Arthritis. (Requires immediate A&E referral).

What is “true locking” of the knee and what might it suggest?

(Click to flip)

Answer

Inability to fully extend or flex the knee. Suggests a displaced meniscal tear or a loose body within the joint.

A child presents with knee pain and a limp. Hip examination reveals pain and limited internal rotation. What serious hip condition must be excluded?

(Click to flip)

Answer

Slipped Capital Femoral Epiphysis (SCFE). (Also Perthes disease in younger children).

What type of exercises are particularly beneficial for patellofemoral pain syndrome?

(Click to flip)

Answer

Quadriceps strengthening (especially VMO) and gluteal muscle strengthening exercises.

What are the characteristic X-ray findings of osteoarthritis (LOSS)?

(Click to flip)

Answer

Loss of joint space, Osteophytes, Subchondral sclerosis, Subchondral cysts.

Knee Pain Assessment Quiz (GP Focus)

Test your knowledge with these GP-focused questions.

1. A 25-year-old footballer sustained a twisting injury to his right knee. He heard a “pop” and developed immediate significant swelling. He feels his knee is unstable. Which ligament is most likely torn?

A. Medial Collateral Ligament (MCL).
B. Anterior Cruciate Ligament (ACL).
C. Posterior Cruciate Ligament (PCL).
D. Lateral Collateral Ligament (LCL).
Explanation: A “pop,” immediate haemarthrosis (significant swelling), and instability following a non-contact twisting injury are classic features of an ACL tear.

2. A 65-year-old woman complains of gradual onset medial knee pain, worse with walking and stairs, and brief morning stiffness (<30 mins). Examination reveals crepitus and mild bony tenderness over the medial joint line. What is the most likely diagnosis?

A. Rheumatoid Arthritis.
B. Gout.
C. Osteoarthritis.
D. Septic Arthritis.
Explanation: Gradual onset, activity-related pain, brief morning stiffness, crepitus, and typical age group are consistent with Osteoarthritis.

3. A 14-year-old boy, who plays a lot of basketball, presents with pain and a tender lump over the tibial tuberosity of his left knee. The pain is worse with running and jumping. What is the most likely diagnosis?

A. Patellar tendinopathy.
B. Slipped Capital Femoral Epiphysis.
C. Osgood-Schlatter Disease.
D. Osteochondritis Dissecans.
Explanation: Pain, tenderness, and swelling at the tibial tuberosity in an active adolescent, related to overuse, is characteristic of Osgood-Schlatter Disease (apophysitis).

4. Which of the following is a RED FLAG symptom for knee pain requiring urgent referral to A&E?

A. Clicking sensation with movement.
B. Mild swelling after exercise.
C. Hot, swollen, erythematous joint with inability to weight-bear and fever.
D. Pain worse going downstairs.
Explanation: A hot, swollen, erythematous joint with severe pain, inability to weight-bear, and systemic symptoms like fever is highly suspicious of septic arthritis, a medical emergency.

5. A patient describes anterior knee pain that is worse when going down stairs and after prolonged sitting with the knee bent (“cinema sign”). Which condition is most likely?

A. Medial meniscus tear.
B. Patellofemoral Pain Syndrome (PFPS).
C. Iliotibial Band Syndrome.
D. Prepatellar bursitis.
Explanation: Anterior knee pain exacerbated by activities like descending stairs, squatting, and prolonged sitting (flexed knee) is typical of Patellofemoral Pain Syndrome.

6. Which of the Ottawa Knee Rules criteria would indicate the need for an X-ray after an acute knee injury?

A. Presence of any bruising around the knee.
B. Age 55 years or older.
C. Hearing a “click” at the time of injury.
D. Mild effusion developing 24 hours after injury.
Explanation: Age ≥55 years is one of the Ottawa Knee Rules criteria. Other criteria include isolated patellar tenderness, fibular head tenderness, inability to flex to 90°, or inability to weight-bear for 4 steps.

7. A patient presents with a tense swelling in the popliteal fossa. This is most likely a:

A. Prepatellar bursitis.
B. Femoral artery aneurysm.
C. Baker’s cyst (Popliteal cyst).
D. Quadriceps tendon rupture.
Explanation: A Baker’s cyst (popliteal cyst) presents as a swelling in the popliteal fossa, often due to an underlying intra-articular knee problem.

8. In a child presenting with a limp and knee pain, which joint MUST also be thoroughly examined to exclude referred pain from serious pathology?

A. Ankle.
B. Spine.
C. Hip.
D. Shoulder.
Explanation: Hip pathology in children (e.g., Slipped Capital Femoral Epiphysis, Perthes disease, transient synovitis) frequently presents with referred pain to the knee. Always examine the hip in a child with knee pain.

9. McMurray’s test is used to assess for injury to which structure(s) in the knee?

A. Cruciate ligaments (ACL/PCL).
B. Menisci (medial and lateral).
C. Collateral ligaments (MCL/LCL).
D. Patellar tendon.
Explanation: McMurray’s test is a provocative manoeuvre designed to detect tears of the medial or lateral meniscus by eliciting pain or a click along the joint line.

10. What is the primary indication for an MRI scan of the knee in a primary care setting?

A. All cases of acute knee pain lasting more than one week.
B. To confirm a clinical diagnosis of osteoarthritis.
C. Suspected significant soft tissue injury (e.g., meniscal tear with locking, ACL rupture) where surgery is contemplated, or if red flags for serious pathology are present.
D. Patient request for detailed imaging.
Explanation: MRI is not a routine investigation for most knee pain in GP. It’s reserved for cases with suspected significant internal derangement where surgical intervention is considered, or if there’s suspicion of serious pathology (e.g., tumour, infection) not clarified by other means. OA is primarily a clinical diagnosis; X-rays may be used.

Ankle & Foot Injuries

Ankle and foot injuries are extremely common presentations in primary care, urgent care, and emergency departments. Accurate assessment, including the application of clinical decision rules like the Ottawa Ankle Rules, appropriate imaging, and initial management are crucial for optimal outcomes and to avoid missing significant injuries.

Overview & Relevant Anatomy

The ankle and foot form a complex structure responsible for weight-bearing, shock absorption, and propulsion.

Ankle Joint (Talocrural Joint):

  • A synovial hinge joint formed by the articulation of the tibia, fibula, and talus.
  • Bones:
    • Tibia: Forms the medial malleolus (medial side) and the roof (plafond) of the ankle joint.
    • Fibula: Forms the lateral malleolus (lateral side).
    • Talus: Fits into the mortise created by the tibia and fibula.
  • Movements: Primarily dorsiflexion and plantarflexion.
  • Ligaments (Key for stability):
    • Lateral Ligament Complex (most commonly injured in sprains):
      • Anterior Talofibular Ligament (ATFL) – resists inversion and plantarflexion. MOST COMMON.
      • Calcaneofibular Ligament (CFL) – resists inversion.
      • Posterior Talofibular Ligament (PTFL) – resists posterior displacement of talus.
    • Medial Ligament Complex (Deltoid Ligament): Strong, fan-shaped ligament resisting eversion. Less commonly sprained but injuries can be severe.
    • Syndesmosis (High Ankle Sprain): Ligaments connecting distal tibia and fibula (Anterior Inferior Tibiofibular Ligament – AITFL, Posterior Inferior Tibiofibular Ligament – PITFL, Interosseous ligament). Injured with external rotation or severe dorsiflexion.

Foot:

Composed of 26 bones, divided into:

  • Hindfoot: Talus and Calcaneus.
  • Midfoot: Navicular, Cuboid, and three Cuneiforms (medial, intermediate, lateral). Forms the arches of the foot. Lisfranc joint complex is between midfoot and forefoot.
  • Forefoot: Metatarsals (5) and Phalanges (14 – proximal, middle, distal, except great toe which has 2).

Numerous ligaments and tendons provide stability and allow complex movements.

💡 Ankle Mortise

The “ankle mortise” refers to the socket formed by the distal tibia and fibula, into which the talus fits. Stability of the mortise is crucial for normal ankle function. Disruption (e.g., fracture with ligamentous injury) can lead to instability and arthritis.

History & Examination of Ankle & Foot Injuries

History:
  • Mechanism of Injury (Crucial):
    • How did it happen? (e.g., twisting, direct blow, fall from height).
    • Inversion injury (foot turns inwards): Suggests lateral ligament sprain, lateral malleolus fracture, 5th metatarsal base fracture.
    • Eversion injury (foot turns outwards): Suggests deltoid ligament sprain, medial malleolus fracture, fibula fracture (Maisonneuve).
    • Forced dorsiflexion/external rotation: Suggests syndesmotic injury (“high ankle sprain”).
    • Axial load (fall from height): Consider calcaneal, talar, tibial pilon, or spinal fractures.
  • Sound at time of injury: “Pop” or “snap” may suggest ligament rupture or fracture.
  • Immediate ability to weight-bear: Key component of Ottawa Ankle Rules.
  • Pain: Onset (immediate/delayed), location (point tenderness?), character, severity (VAS score).
  • Swelling & Bruising: Onset, location, extent.
  • Previous injuries to ankle/foot.
  • Past Medical History: Diabetes (neuropathy, healing), peripheral vascular disease, osteoporosis, anticoagulants.
  • Occupation/Activities: Impact on management and rehabilitation.
  • Tetanus status (if open wound).
Examination (“Look, Feel, Move” + Special Tests):
  • General: Assess gait (if able), use of aids.
  • Look:
    • Swelling: Localised or diffuse.
    • Bruising (Ecchymosis): Pattern can indicate site of injury.
    • Deformity: Obvious angulation, malalignment. Compare to other side.
    • Skin: Open wounds, abrasions, blisters, signs of compartment syndrome (tense, shiny skin).
  • Feel:
    • Temperature: Warmth (inflammation) vs. coolness (vascular compromise).
    • Pulses: Dorsalis pedis and posterior tibial arteries. Capillary refill.
    • Sensation: Assess dermatomes for nerve injury.
    • Palpation for Tenderness (Systematic Approach):
      • Bony tenderness: Medial malleolus (posterior edge/tip), lateral malleolus (posterior edge/tip), navicular bone, base of 5th metatarsal (key points for Ottawa Rules). Also palpate along tibia, fibula, talus, calcaneus, other metatarsals/phalanges.
      • Ligamentous tenderness: ATFL, CFL, deltoid ligament, syndesmosis.
      • Tendon tenderness: Achilles tendon, peroneal tendons, tibialis posterior tendon.
  • Move (Active and Passive, if pain allows):
    • Ankle: Dorsiflexion, plantarflexion, inversion, eversion.
    • Subtalar joint: Inversion, eversion (stabilise ankle).
    • Midfoot & Forefoot joints.
    • Compare range of motion to unaffected side. Assess for pain or crepitus on movement.
  • Special Tests (Perform cautiously if fracture suspected):
    • Anterior Drawer Test (for ATFL integrity): Stabilise tibia, pull heel anteriorly. Increased laxity/soft end-feel suggests ATFL rupture.
    • Talar Tilt Test (for CFL integrity): Stabilise tibia, invert calcaneus. Increased laxity suggests CFL rupture.
    • Squeeze Test / External Rotation Stress Test (for Syndesmosis injury): Squeeze tibia and fibula together at mid-calf (squeeze test) or externally rotate foot with ankle in dorsiflexion (stress test). Pain over syndesmosis suggests injury.
    • Thompson’s Test (for Achilles tendon rupture): Patient prone with feet off edge of couch. Squeeze calf muscle. Absence of plantarflexion is a positive test.
  • Neurovascular Assessment: Essential, especially with significant swelling or suspected fracture/dislocation. Check pulses, capillary refill, sensation, motor function distally.

Ottawa Ankle & Foot Rules

The Ottawa Ankle Rules (OAR) and Ottawa Foot Rules are validated clinical decision tools designed to help clinicians decide whether an X-ray is necessary for acute ankle or midfoot injuries. Their use reduces unnecessary radiographs, costs, and radiation exposure.

Ankle X-ray series is required ONLY IF there is any pain in the malleolar zone AND any of these findings:

  1. Bone tenderness along the distal 6 cm of the posterior edge of the tibia or tip of the medial malleolus.
    (Point A on diagram)
  2. Bone tenderness along the distal 6 cm of the posterior edge of the fibula or tip of the lateral malleolus.
    (Point B on diagram)
  3. An inability to bear weight both immediately after injury AND for 4 steps in the emergency department or clinic (unable to transfer weight twice onto each leg).

Foot X-ray series is required ONLY IF there is any pain in the midfoot zone AND any of these findings:

  1. Bone tenderness at the base of the fifth metatarsal (lateral side).
    (Point C on diagram)
  2. Bone tenderness at the navicular bone (medial side).
    (Point D on diagram)
  3. An inability to bear weight both immediately after injury AND for 4 steps in the emergency department or clinic.

Imagine a diagram of the ankle and foot here:

A: Posterior edge/tip of Medial Malleolus

B: Posterior edge/tip of Lateral Malleolus

C: Base of 5th Metatarsal

D: Navicular Bone

Important Considerations for Ottawa Rules:
  • Apply to patients >18 years (some evidence supports use in older children, but caution). Not validated for children <5 years typically.
  • Apply for acute injuries (within 10 days).
  • High sensitivity (approaching 100%) for clinically significant fractures, meaning if rules are negative, fracture is very unlikely.
  • Moderate specificity, meaning some patients who are positive by the rules will not have a fracture.
  • Do not apply if: patient intoxicated, distracting painful injury elsewhere, diminished sensation in legs, gross swelling preventing palpation.
  • Not for assessing chronic pain or re-injury.
Using the Rules Effectively

Systematically palpate the four specific bony points and assess weight-bearing ability. If ALL criteria for X-ray are NEGATIVE for either ankle or foot series, then X-ray is NOT indicated for that series. Document your application of the rules clearly.

Ankle Sprains (Ligament Injuries)

Ankle sprains are one of the most common musculoskeletal injuries. They involve stretching or tearing of ligaments.

Mechanism:
  • Lateral Ankle Sprains (Most Common – ~85%):
    • Caused by an inversion injury (foot rolls inwards) with plantarflexion.
    • Typically affects the lateral ligament complex:
      • Anterior Talofibular Ligament (ATFL) is the most frequently injured.
      • Calcaneofibular Ligament (CFL) may be involved in more severe sprains.
      • Posterior Talofibular Ligament (PTFL) injured only in severe sprains, often with dislocation.
  • Medial Ankle Sprains (Deltoid Ligament):
    • Caused by an eversion injury (foot rolls outwards).
    • Less common due to strength of deltoid ligament. Often associated with fractures (e.g., fibula) due to the force required.
  • Syndesmotic Sprain (“High Ankle Sprain”):
    • Injury to the ligaments connecting distal tibia and fibula (syndesmosis).
    • Caused by forced external rotation of the foot or excessive dorsiflexion.
    • Takes longer to heal than typical lateral sprains.
Grading of Ligament Sprains (General):
  • Grade I (Mild): Stretching or microscopic tearing of ligament fibres. Mild pain, swelling, minimal loss of function, able to bear weight. No instability.
  • Grade II (Moderate): Partial tearing of ligament fibres. Moderate pain, swelling, bruising. Some loss of function, pain with weight-bearing. Mild to moderate instability.
  • Grade III (Severe): Complete rupture of ligament fibres. Severe pain, swelling, bruising. Significant loss of function, unable to bear weight. Gross instability.
Clinical Features:
  • Pain, swelling, bruising, tenderness over affected ligaments.
  • Limited range of motion.
  • Instability or “giving way” sensation (especially with Grade II/III).
  • Positive special tests (Anterior Drawer, Talar Tilt, Squeeze test for syndesmosis) if performed, but may be difficult in acute phase due to pain/swelling.
Management (See also “General Management Principles”):
  • Confirm no fracture (Ottawa Ankle Rules).
  • Initial (PRICE/POLICE): Protection, Optimal Loading, Ice, Compression, Elevation.
  • Analgesia: Paracetamol, NSAIDs (oral or topical).
  • Support/Immobilisation:
    • Grade I: Minimal support, early mobilisation.
    • Grade II: Functional support (e.g., ankle brace, strapping) for 2-6 weeks.
    • Grade III: May require short period of immobilisation (e.g., walker boot, cast) followed by functional support.
  • Early Mobilisation & Rehabilitation: Crucial to prevent chronic instability and re-injury.
    • Range of motion exercises.
    • Strengthening exercises (especially peroneal muscles for lateral sprains).
    • Proprioception and balance training (e.g., wobble board).
    • Gradual return to activity.
  • Physiotherapy referral is often beneficial, especially for Grade II/III sprains or persistent symptoms.
  • Surgery: Rarely indicated for acute lateral ankle sprains. Considered for chronic instability despite adequate conservative management, or for some severe syndesmotic injuries.

Most ankle sprains heal well with appropriate conservative management, but recurrent sprains and chronic instability can occur if not rehabilitated properly.

Ankle Fractures

Ankle fractures involve a break in one or more of the malleoli (medial, lateral, posterior – which is part of the distal tibia).

Classification (Simplified – based on malleoli involved):
  • Unimalleolar Fracture:
    • Isolated lateral malleolus fracture (most common).
    • Isolated medial malleolus fracture.
  • Bimalleolar Fracture: Involves both medial and lateral malleoli. Often indicates an unstable ankle joint.
  • Trimalleolar Fracture: Involves medial malleolus, lateral malleolus, AND posterior malleolus (fracture of posterior aspect of distal tibia). Highly unstable.

More complex classifications (e.g., Weber, Lauge-Hansen) are used by orthopaedic specialists to guide treatment based on fracture pattern and mechanism, predicting ligamentous injury and stability.

Mechanism:
  • Similar to ankle sprains (inversion, eversion, rotation), but with greater force.
  • Direct blows.
Clinical Features:
  • Severe pain, often immediate inability to weight-bear.
  • Significant swelling and bruising.
  • Deformity may be present, especially with displaced fractures or dislocations.
  • Point tenderness over the fractured malleolus/malleoli.
  • Crepitus may be felt.
Investigations:
  • X-rays: Standard ankle series (Anterior-Posterior, Lateral, Mortise views).
    • Mortise view is crucial for assessing congruity of the ankle joint and syndesmotic integrity (widening suggests instability).
    • Look for fracture lines, displacement, angulation, talar shift.
  • CT scan may be used for complex fractures, pre-operative planning, or if posterior malleolus fracture extent is unclear.
Management:

Goal: Achieve a stable, congruent joint to allow healing and restore function.

  • Undisplaced, Stable Fractures (e.g., some isolated lateral malleolus fractures – Weber A):
    • Conservative management: Immobilisation in a below-knee cast or walker boot for 4-6 weeks.
    • Non-weight bearing or partial weight-bearing initially, progressing as tolerated.
    • Follow-up X-rays to ensure no displacement.
    • Rehabilitation with physiotherapy after immobilisation.
  • Displaced or Unstable Fractures (e.g., most bimalleolar, trimalleolar, displaced unimalleolar, Weber B/C with talar shift or syndesmotic injury):
    • Require Orthopaedic Referral URGENTLY.
    • Initial Management in ED/Urgent Care:
      • Analgesia.
      • Neurovascular assessment.
      • Attempt closed reduction if significantly displaced or dislocated (often under sedation). Check post-reduction X-ray and neurovascular status.
      • Immobilise in a well-padded backslab or cast.
      • Elevate limb.
    • Surgical Management (Open Reduction and Internal Fixation – ORIF): Most common treatment for unstable fractures. Involves plates and screws to restore anatomical alignment.
  • Open Fractures: Surgical emergency. IV antibiotics, tetanus prophylaxis, urgent debridement and fixation.
Specific Types to Note:
  • Maisonneuve Fracture: Spiral fracture of the proximal fibula associated with a tear of the deltoid ligament and/or syndesmosis, and often a medial malleolus fracture. Caused by eversion/external rotation. Ankle X-ray may seem “normal” or show only medial injury; requires palpation of entire fibula and potentially full-length tib/fib X-rays if high index of suspicion. Unstable, usually requires surgery.

Common Foot Fractures

Fractures can occur in any of the 26 bones of the foot. Some common or significant ones include:

Metatarsal Fractures:
  • Stress Fractures: Common in 2nd and 3rd metatarsals (march fracture) due to repetitive stress (e.g., runners, military recruits). Insidious onset of pain, worse with activity. X-rays may be normal initially; bone scan or MRI more sensitive. Management: rest, activity modification, supportive footwear, sometimes boot.
  • Acute Fractures (e.g., shaft, neck): Usually due to direct trauma or twisting. Management depends on displacement and location. Most can be treated conservatively with boot/stiff-soled shoe and protected weight-bearing. Displaced fractures may need ORIF.
  • Fifth Metatarsal Fractures:
    • Tuberosity Avulsion Fracture (Pseudo-Jones / Dancer’s fracture): Avulsion at base by peroneus brevis tendon due to inversion injury. Usually heals well with symptomatic treatment (boot/stiff shoe).
    • Jones Fracture: Transverse fracture at the metaphyseal-diaphyseal junction (Zone 2). Higher risk of non-union due to tenuous blood supply. Often requires non-weight bearing cast for 6-8 weeks; surgery (screw fixation) considered for athletes or displaced fractures.
    • Diaphyseal Stress Fracture (Zone 3): Also prone to delayed union/non-union.
Phalangeal Fractures (Toes):
  • Very common, usually due to direct trauma (stubbing toe).
  • Most are undisplaced or minimally displaced.
  • Management: Buddy strapping to adjacent toe, stiff-soled shoe, analgesia.
  • Displaced fractures of great toe or intra-articular fractures may need reduction or surgical referral.
Calcaneal Fractures:
  • Usually due to high-energy trauma (fall from height, MVA). Often severe and debilitating.
  • Associated with lumbar spine fractures (check spine!).
  • Pain, swelling, inability to weight-bear, heel bruising/widening.
  • X-rays (lateral, axial views). CT scan essential for characterising fracture pattern and intra-articular involvement (subtalar joint).
  • Many require ORIF, but outcomes can be variable. High risk of post-traumatic arthritis.
  • URGENT ORTHOPAEDIC REFERRAL.
Talar Fractures:
  • Relatively uncommon, often due to high-energy trauma (forced dorsiflexion).
  • Risk of avascular necrosis (AVN) of talar body, especially with displaced neck fractures, due to tenuous blood supply.
  • Pain, swelling, inability to weight-bear.
  • URGENT ORTHOPAEDIC REFERRAL. Many require surgical fixation.
Navicular Fractures:
  • Can be acute (often body fractures) or stress fractures (common in athletes).
  • Tenderness over dorsal navicular (“N-spot”).
  • Stress fractures may have vague midfoot pain. Risk of AVN/non-union.
  • Displaced fractures or some stress fractures may require surgery.
Lisfranc Injury (Tarsometatarsal Fracture-Dislocation):
  • Injury to the Lisfranc joint complex (articulation between midfoot and forefoot). Can be purely ligamentous or involve fractures.
  • Often subtle on X-ray, easily missed. Look for widening between 1st and 2nd metatarsal bases, or dorsal displacement.
  • Mechanism: Axial load on plantarflexed foot, or severe twisting.
  • Pain, swelling in midfoot, inability to bear weight, bruising on sole of foot (plantar ecchymosis) is a key sign.
  • High index of suspicion needed. Weight-bearing X-rays or CT/MRI may be required.
  • URGENT ORTHOPAEDIC REFERRAL. Most require surgical stabilisation to prevent long-term pain and arthritis.

Achilles Tendon Injuries

The Achilles tendon is the largest and strongest tendon in the body, connecting calf muscles (gastrocnemius, soleus) to the calcaneus. Injuries include tendinopathy and rupture.

Achilles Tendinopathy:
  • Overuse injury causing pain, swelling, and stiffness of the Achilles tendon.
  • Types:
    • Non-insertional: Affects mid-portion of tendon (2-6 cm proximal to insertion).
    • Insertional: Affects tendon at its insertion onto calcaneus. Often associated with bony spurs (Haglund’s deformity).
  • Risk Factors: Sudden increase in activity, improper footwear, tight calf muscles, foot malalignment (e.g., overpronation).
  • Clinical Features: Gradual onset pain, worse with activity (especially at start), morning stiffness. Palpable thickening or nodule in tendon.
  • Management:
    • Activity modification, relative rest.
    • Ice, NSAIDs for pain.
    • Eccentric strengthening exercises (e.g., heel drops) are the cornerstone of treatment.
    • Stretching of calf muscles.
    • Heel lifts/orthotics may provide temporary relief.
    • Avoid corticosteroid injections directly into tendon (risk of rupture).
    • Refer to physiotherapy. Surgery rarely needed.
Achilles Tendon Rupture:
  • Complete tear of the Achilles tendon.
  • Mechanism: Sudden, forceful plantarflexion or dorsiflexion (e.g., pushing off to sprint, landing from jump, sudden stumble). Often in “weekend warriors” (middle-aged recreational athletes). Risk increased by fluoroquinolone antibiotics and corticosteroids.
  • Clinical Features:
    • Sudden, sharp pain in back of ankle/lower calf, often described as being “kicked” or “shot”.
    • Audible “pop” or “snap” may be reported.
    • Difficulty weight-bearing, weak plantarflexion.
    • Palpable gap in the tendon (may be obscured by swelling later).
    • Positive Thompson’s Test (Simmonds’ Triad): Squeeze calf muscle while patient is prone with feet off couch edge. Absence of passive plantarflexion indicates rupture. Highly sensitive and specific.
    • Increased passive dorsiflexion compared to other side.
  • Diagnosis: Usually clinical. Ultrasound or MRI can confirm diagnosis if uncertain or to assess extent of tear/gap. X-rays usually normal.
  • Management: URGENT ORTHOPAEDIC REFERRAL (same/next day).
    • Non-operative: Functional bracing or casting with foot in equinus (plantarflexion), gradually brought to neutral over several weeks. Requires strict adherence. Similar functional outcomes to surgery for many, but potentially higher re-rupture rate.
    • Operative (Surgical Repair): Open or percutaneous repair of the tendon. May allow earlier mobilisation and potentially lower re-rupture rate, but carries surgical risks (infection, wound problems, nerve injury).
    • Choice of treatment depends on patient factors (age, activity level, comorbidities), surgeon preference, and patient preference after discussion.
    • Both require prolonged rehabilitation.

Red Flags & Urgent Orthopaedic Referrals (Ankle & Foot)

Recognizing conditions that require immediate or urgent specialist orthopaedic input is vital to prevent long-term disability or limb-threatening complications.

IMMEDIATE / EMERGENCY ORTHOPAEDIC REFERRAL (Often via ED):
  • Open Fractures or Dislocations: Bone exposed, high risk of infection. Requires IV antibiotics, tetanus, urgent surgical debridement and stabilisation.
  • Irreducible Dislocation: Ankle or foot joint remains dislocated after attempts at closed reduction.
  • Neurovascular Compromise: Absent pulses, cold/pale foot, paraesthesia, paralysis distal to injury. Suggests arterial injury or compartment syndrome. Requires immediate attempts to restore circulation (e.g., reducing dislocation/fracture).
  • Compartment Syndrome: Increased pressure within a fascial compartment, compromising circulation and function.
    • Symptoms (6 P’s): Pain out of proportion (early & key), Pallor, Pulselessness (late), Paraesthesia, Paralysis (late), Poikilothermia (coolness). Pain on passive stretch of muscles in compartment is a key sign.
    • Clinical diagnosis, compartment pressures can be measured. Requires URGENT fasciotomy.
  • Suspected Septic Arthritis or Osteomyelitis: Hot, swollen, erythematous joint/bone with severe pain on movement, fever, systemic upset. Requires joint aspiration/blood cultures, IV antibiotics, possible surgical washout.
  • Significant, Unstable Fractures requiring immediate reduction/stabilisation: e.g., grossly displaced ankle fracture-dislocation.
  • Suspected Calcaneal or Talar neck fractures (displaced): High risk of complications.
  • Suspected Lisfranc Fracture-Dislocation: Easily missed, high morbidity if untreated.
URGENT ORTHOPAEDIC REFERRAL (e.g., within 24-72 hours, via fracture clinic pathway):
  • Achilles Tendon Rupture: For discussion of operative vs. non-operative management.
  • Most displaced or unstable ankle fractures that have been temporarily stabilised.
  • Significant foot fractures requiring specialist assessment/management (e.g., Jones fracture, displaced metatarsal fractures, some navicular fractures).
  • Syndesmotic injuries (“High ankle sprains”) that are unstable or not improving.
  • Persistent severe pain or inability to weight-bear despite negative X-rays, where significant ligamentous injury or occult fracture is suspected.
  • Children with suspected physeal (growth plate) fractures.
Considerations for Routine Orthopaedic Referral (Non-urgent):
  • Chronic ankle instability despite physiotherapy.
  • Persistent pain from conditions like Achilles tendinopathy or plantar fasciitis unresponsive to conservative measures.
  • Degenerative conditions (e.g., ankle/foot arthritis) significantly impacting quality of life.
  • Foot deformities causing problems (e.g., severe bunions, hammer toes).

Always document neurovascular status thoroughly. If in doubt about urgency, discuss with orthopaedic registrar on call or local fracture clinic.

General Management Principles for Acute Ankle & Foot Injuries

Many acute soft tissue injuries and some minor fractures can be managed with similar initial principles.

PRICE / POLICE Acronyms:

These guide initial self-care and professional advice for acute soft tissue injuries (e.g., sprains, strains).

  • P – Protection: Protect the injured area from further damage. This might involve crutches, splints, braces, or avoiding certain activities.
  • R – Rest (Relative Rest): Avoid activities that cause pain. Complete rest is often not ideal; “Optimal Loading” is now preferred for many injuries.
    OR
    OL – Optimal Loading: Gradually reintroduce movement and stress to the injured tissue as tolerated, to promote healing and prevent deconditioning. Pain should guide progression. Avoid complete immobilisation unless necessary.
  • I – Ice: Apply ice packs (wrapped in a damp cloth) for 15-20 minutes every 2-3 hours during the first 24-48 hours to reduce pain and swelling. Avoid direct contact with skin.
  • C – Compression: Apply a compression bandage (e.g., elasticated bandage, tubigrip) to help reduce swelling. Ensure it’s not too tight to impede circulation.
  • E – Elevation: Elevate the injured limb above the level of the heart as much as possible, especially in the first 24-48 hours, to reduce swelling.
Analgesia:
  • Paracetamol: Regular dosing for pain relief.
  • Non-Steroidal Anti-Inflammatory Drugs (NSAIDs):
    • e.g., Ibuprofen, Naproxen (oral or topical).
    • Can help reduce pain and inflammation.
    • Consider starting 48 hours after injury for soft tissue injuries to avoid potentially interfering with early healing phases (though evidence is debated).
    • Use with caution in patients with asthma, renal impairment, GI issues, cardiovascular disease.
  • Opioid analgesia (e.g., codeine) may be needed for short periods for severe pain (e.g., fractures) but use sparingly due to side effects.
Immobilisation & Support:
  • Splints/Casts: For fractures requiring immobilisation or severe ligament injuries. Backslabs are often used initially to allow for swelling.
  • Walker Boots: Provide support and allow for some controlled weight-bearing or non-weight bearing. Useful for stable fractures and severe sprains.
  • Ankle Braces/Supports: For moderate sprains or as a step-down from more rigid immobilisation. Provide stability during rehabilitation.
  • Crutches: To allow non-weight bearing or partial weight-bearing. Ensure correct fitting and patient education on use.
Rehabilitation:
  • Crucial for restoring function, strength, range of motion, and proprioception.
  • Should be started as soon as appropriate, guided by pain and type of injury.
  • Involves:
    • Range of motion exercises.
    • Strengthening exercises.
    • Balance and proprioception exercises.
    • Gradual return to functional activities and sports.
  • Physiotherapy referral is often very beneficial.
Patient Education & Safety Netting:
  • Explain the likely diagnosis and expected recovery timeframe.
  • Clear instructions on weight-bearing status, use of supports, exercises.
  • Advice on when to seek further medical attention (e.g., worsening pain, swelling, numbness/tingling, signs of infection, DVT symptoms).
  • Arrange appropriate follow-up (e.g., fracture clinic, GP review, physiotherapy).

Flashcards: Ankle & Foot Injuries

Click on each card to reveal the answer.

According to the Ottawa Ankle Rules, an ankle X-ray is needed if there is pain in the malleolar zone AND what other findings? (Name one bony point and the weight-bearing criterion).

(Click to flip)

Answer

Any ONE of:
1. Bone tenderness at posterior edge/tip of lateral malleolus.
2. Bone tenderness at posterior edge/tip of medial malleolus.
3. Inability to bear weight for 4 steps (immediately AND in clinic/ED).

What is the most commonly injured ligament in a typical inversion ankle sprain?

(Click to flip)

Answer

Anterior Talofibular Ligament (ATFL).

What is Thompson’s Test used to assess, and what constitutes a positive test?

(Click to flip)

Answer

Used to assess for Achilles tendon rupture.
A positive test is absence of passive plantarflexion of the foot when the calf muscle is squeezed.

A fracture involving the medial malleolus, lateral malleolus, and posterior malleolus of the tibia is called what?

(Click to flip)

Answer

Trimalleolar fracture. (It is highly unstable).

A Jones fracture involves which bone and what is a key concern with this fracture?

(Click to flip)

Answer

Bone: Fifth metatarsal (at the metaphyseal-diaphyseal junction).
Concern: High risk of non-union or delayed union due to tenuous blood supply.

What are the “6 P’s” that might indicate Compartment Syndrome?

(Click to flip)

Answer

Pain (out of proportion, on passive stretch – early & key)
Pallor
Pulselessness (late)
Paraesthesia
Paralysis (late)
Poikilothermia (coolness).

What is a Maisonneuve fracture?

(Click to flip)

Answer

A spiral fracture of the proximal fibula associated with injury to the medial ankle structures (deltoid ligament/medial malleolus fracture) and disruption of the ankle syndesmosis. It’s an unstable ankle injury.

Plantar ecchymosis (bruising on the sole of the foot) can be a sign of what significant midfoot injury?

(Click to flip)

Answer

Lisfranc injury (tarsometatarsal fracture-dislocation).

Ankle & Foot Injuries Quiz

Test your knowledge with these Orthopaedic scenarios.

1. A 25-year-old footballer “rolled” his ankle inwards during a match. He has pain and swelling over the lateral aspect of his ankle. He is tender over the tip of his lateral malleolus but can bear weight for more than 4 steps. According to Ottawa Ankle Rules, is an X-ray indicated?

A. Yes, due to bone tenderness at the lateral malleolus.
B. No, because he can bear weight.
C. Yes, because it’s a sporting injury.
D. Only if the anterior drawer test is positive.
Explanation: The Ottawa Ankle Rules state an ankle X-ray is needed if there’s pain in the malleolar zone AND EITHER bony tenderness at specific points (posterior edge/tip of lateral or medial malleolus) OR inability to bear weight for 4 steps. He has tenderness at the lateral malleolus tip, so an X-ray is indicated.

2. A patient presents after an inversion injury to the ankle. Examination reveals a positive anterior drawer test. This suggests instability primarily due to injury of which ligament?

A. Anterior Talofibular Ligament (ATFL).
B. Calcaneofibular Ligament (CFL).
C. Deltoid Ligament.
D. Anterior Inferior Tibiofibular Ligament (AITFL).
Explanation: The anterior drawer test specifically assesses the integrity of the Anterior Talofibular Ligament (ATFL), which resists anterior displacement of the talus on the tibia.

3. A 45-year-old man felt a sudden “pop” in his lower calf while playing squash and now has difficulty pushing off with his foot. Thompson’s test is positive. What is the most likely diagnosis?

A. Calf muscle strain.
B. Ankle sprain.
C. Achilles tendon rupture.
D. Plantar fasciitis.
Explanation: The history of a “pop,” sudden pain, difficulty with plantarflexion, and a positive Thompson’s test are classic for an Achilles tendon rupture.

4. A patient falls from a height and lands on their feet. They have severe heel pain and swelling. What specific fracture should be highly suspected and investigated, often with a CT scan?

A. Talar neck fracture.
B. Calcaneal fracture.
C. Navicular fracture.
D. Fifth metatarsal fracture.
Explanation: A fall from height with axial loading is a classic mechanism for a calcaneal fracture. These are often complex and intra-articular, requiring CT for full assessment. Always check for associated lumbar spine fractures.

5. The acronym PRICE (or POLICE) is used for initial management of acute soft tissue injuries. What does the ‘P’ and ‘OL’ in POLICE stand for?

A. Painkiller, Operation.
B. Protection, Optimal Loading.
C. Physiotherapy, On-going care.
D. Pressure, Overlook.
Explanation: In the POLICE acronym for managing acute soft tissue injuries, ‘P’ stands for Protection and ‘OL’ stands for Optimal Loading, which emphasizes gradual reintroduction of stress to promote healing over complete rest.

6. A fracture of the base of the 5th metatarsal at the metaphyseal-diaphyseal junction, known for its risk of non-union, is called a:

A. Dancer’s fracture (Tuberosity avulsion).
B. Jones fracture.
C. March fracture.
D. Lisfranc fracture.
Explanation: A Jones fracture is a transverse fracture of the fifth metatarsal at the metaphyseal-diaphyseal junction (Zone 2), notorious for its higher risk of non-union due to a watershed area of blood supply.

7. Which of the following signs is most indicative of a syndesmotic injury (high ankle sprain)?

A. Positive anterior drawer test.
B. Tenderness over the ATFL.
C. Pain on squeezing the tibia and fibula together at mid-calf (Squeeze Test).
D. Inability to perform a single heel raise.
Explanation: The Squeeze Test (compressing tibia and fibula proximally) and the External Rotation Stress Test elicit pain at the syndesmosis if it’s injured. Anterior drawer tests ATFL. Heel raise difficulty can be non-specific or Achilles related.

8. A patient presents with severe ankle pain and deformity after a twisting injury. The foot is pale and cool, and you cannot palpate the dorsalis pedis pulse. What is the most critical immediate concern?

A. Ligament rupture.
B. Neurovascular compromise.
C. Occult fracture.
D. Compartment syndrome (though it’s a consideration).
Explanation: A pale, cool foot with absent pulses signifies acute neurovascular compromise, likely due to arterial injury or compression from a severe fracture-dislocation. This is a limb-threatening emergency requiring immediate attempts to restore circulation (e.g., reduction) and urgent orthopaedic intervention.

9. Plantar ecchymosis (bruising on the sole of the foot) after a midfoot injury is a classic sign highly suggestive of:

A. Calcaneal fracture.
B. Achilles tendon rupture.
C. Lisfranc injury.
D. Severe plantar fasciitis.
Explanation: Plantar ecchymosis is a key clinical sign that should raise suspicion for a Lisfranc (tarsometatarsal) injury, which can be easily missed and has high morbidity if not treated appropriately.

10. What is the cornerstone of conservative management for Achilles tendinopathy?

A. Corticosteroid injections into the tendon.
B. Eccentric strengthening exercises (e.g., heel drops).
C. Complete immobilisation in a cast.
D. Regular high-impact activities to “strengthen” it.
Explanation: Eccentric loading exercises, such as controlled heel drops, are the most evidence-based conservative treatment for Achilles tendinopathy. Corticosteroid injections are generally avoided due to rupture risk.

Compartment Syndrome (T&O Focus)

Compartment syndrome is an orthopaedic emergency characterized by increased pressure within a closed osteofascial compartment, leading to impaired tissue perfusion, ischaemia, and potentially irreversible muscle and nerve damage if not promptly diagnosed and treated. A high index of suspicion is paramount.

Overview & Pathophysiology of Compartment Syndrome

Compartment syndrome occurs when the pressure within a non-expansile osteofascial compartment rises to a level that compromises circulation and function of the tissues within that space.

Pathophysiological Cascade:

  1. Increased Intracompartmental Pressure: Due to increased volume within the compartment (e.g., bleeding, oedema) or decreased compartment size (e.g., tight cast, circumferential burn).
  2. Venous Outflow Obstruction: As intracompartmental pressure (ICP) approaches venous pressure, veins collapse, increasing venous resistance and further raising ICP.
  3. Impaired Capillary Perfusion: When ICP exceeds capillary perfusion pressure (CPP = Mean Arterial Pressure – ICP), capillary blood flow diminishes, leading to tissue hypoxia.
  4. Tissue Ischaemia: Reduced oxygen and nutrient delivery causes ischaemia of muscles and nerves within the compartment.
  5. Cellular Damage & Necrosis: Prolonged ischaemia leads to irreversible cell damage, muscle necrosis, and nerve dysfunction. Muscle necrosis typically occurs after 4-8 hours of ischaemia; nerve damage can occur earlier.
  6. Release of Myoglobin & Potassium: Necrotic muscle releases myoglobin (risking rhabdomyolysis and acute kidney injury) and potassium (risking hyperkalaemia and arrhythmias).

This creates a vicious cycle: ischaemia → oedema → further increased ICP → worsening ischaemia.

Key Concepts:

  • Osteofascial Compartments: Enclosed spaces bounded by bone and inelastic fascia, common in limbs (e.g., anterior compartment of leg).
  • Perfusion Pressure: Delta P (ΔP) = Diastolic Blood Pressure (DBP) – Intracompartmental Pressure (ICP). A ΔP <30 mmHg is highly suggestive of compartment syndrome (some use <20 mmHg). Another measure is ICP >30-45 mmHg absolute, but relative to DBP is more reliable.
Time is Muscle (and Nerve)!

Irreversible damage can occur within hours. Early diagnosis and prompt surgical decompression (fasciotomy) are critical to prevent permanent disability.

Causes & Risk Factors for Acute Compartment Syndrome

Any condition that increases the volume within a compartment or decreases the size of the compartment can cause ACS.

Increased Compartment Volume:

  • Fractures (Most Common Cause): Especially tibial shaft fractures (up to 10% risk), forearm fractures (both bones), supracondylar humerus fractures (children). Bleeding and oedema from the fracture site.
  • Soft Tissue Injury / Contusion: Severe crush injuries, muscle trauma, vigorous exercise (especially in unconditioned individuals).
  • Bleeding / Haemorrhage:
    • Coagulopathy (e.g., haemophilia, anticoagulation therapy).
    • Vascular injury (arterial or venous).
    • Post-operative haematoma.
  • Reperfusion Injury: After prolonged ischaemia (e.g., revascularisation of an ischaemic limb), leading to oedema.
  • Burns: Oedema formation. Circumferential burns can also decrease compartment size.
  • Intravenous or Intra-arterial Infiltration/Extravasation: Large volumes of fluid or irritant substances.
  • Snake Bites / Envenomation.
  • Nephrotic Syndrome (rarely, due to severe oedema).

Decreased Compartment Size:

  • Tight Casts or Dressings: Especially circumferential casts applied too early after injury before swelling peaks.
  • Circumferential Burns: Constricting eschar.
  • External Compression / Lying on Limb: Prolonged immobilisation or unconsciousness with limb compression (e.g., drug overdose, prolonged surgery in lithotomy).
  • Surgical Closure under Tension.

Patient-Related Risk Factors:

  • Young age (especially young males, due to larger muscle mass and tighter fascia).
  • High-energy trauma.
  • Altered mental status (e.g., head injury, intoxication) making clinical assessment difficult.
  • Patients on anticoagulation.

Common Anatomical Locations for Compartment Syndrome

ACS can occur in any closed fascial compartment, but some locations are more common.

Lower Limb:

  • Anterior Compartment of the Leg (Most Common Site Overall):
    • Contents: Tibialis anterior, extensor hallucis longus (EHL), extensor digitorum longus (EDL), peroneus tertius muscles; deep peroneal nerve; anterior tibial artery & vein.
    • Clinical Signs of Dysfunction: Weakness of ankle/toe dorsiflexion (foot drop), paraesthesia/numbness in 1st web space (deep peroneal nerve).
  • Lateral Compartment of the Leg:
    • Contents: Peroneus longus, peroneus brevis muscles; superficial peroneal nerve.
    • Clinical Signs of Dysfunction: Weakness of foot eversion, paraesthesia/numbness on dorsum of foot (superficial peroneal nerve).
  • Deep Posterior Compartment of the Leg:
    • Contents: Tibialis posterior, flexor hallucis longus (FHL), flexor digitorum longus (FDL) muscles; tibial nerve; posterior tibial artery & vein, peroneal artery & vein.
    • Clinical Signs of Dysfunction: Weakness of ankle/toe plantarflexion, paraesthesia/numbness on sole of foot (tibial nerve). Pain on passive dorsiflexion of toes/ankle.
  • Superficial Posterior Compartment of the Leg:
    • Contents: Gastrocnemius, soleus, plantaris muscles; sural nerve.
    • Clinical Signs of Dysfunction: Weakness of ankle plantarflexion (less pronounced than deep posterior). Pain on passive dorsiflexion of ankle. Sural nerve sensation may be affected.
  • Thigh Compartments (Anterior, Posterior, Medial): Less common than leg, usually after high-energy trauma or femoral fracture. Can be life-threatening due to large muscle volume and potential for massive fluid shifts/rhabdomyolysis.
  • Foot Compartments: Rare, often after severe crush injuries. Multiple small compartments.

Upper Limb:

  • Volar (Anterior) Compartment of the Forearm (Second Most Common Site):
    • Contents: Flexor muscles of wrist/fingers; median nerve, ulnar nerve; radial artery, ulnar artery.
    • Clinical Signs of Dysfunction: Pain on passive extension of fingers/wrist, weakness of finger/wrist flexion, paraesthesia/numbness in median/ulnar nerve distributions. Late: Volkmann’s ischaemic contracture.
  • Dorsal (Posterior) Compartment of the Forearm:
    • Contents: Extensor muscles of wrist/fingers; posterior interosseous nerve (branch of radial).
    • Clinical Signs of Dysfunction: Pain on passive flexion of fingers/wrist, weakness of finger/wrist extension.
  • Hand Compartments (Interossei, Thenar, Hypothenar, Adductor Pollicis): Often after crush injuries or high-pressure injection.
  • Arm Compartments (Anterior, Posterior): Rare, usually with humeral fractures or severe trauma.

Understanding the contents of each compartment is key to recognizing specific neurological and motor deficits associated with ACS in that location.

Clinical Features of Acute Compartment Syndrome

The “classic” signs are often taught as the “6 P’s”, but some are late and unreliable findings. Pain out of proportion is the most important early symptom.

Key Symptoms & Signs (In approximate order of appearance/reliability):

  1. Pain out of Proportion to Injury (Earliest & Most Sensitive Symptom):
    • Severe, deep, throbbing, burning pain that is poorly localized within the affected compartment.
    • Often described as “bursting” or “tense.”
    • Pain is typically not relieved by immobilization, elevation, or standard analgesia (including opioids).
    • Pain on Passive Stretch of the muscles within the affected compartment is a very important early sign (e.g., passive toe dorsiflexion for deep posterior leg compartment, passive finger extension for volar forearm compartment).
  2. Palpably Tense (Firm, “Woody”) Compartment:
    • Affected compartment feels very firm, tense, and tender to palpation compared to the contralateral limb or unaffected compartments.
  3. Paraesthesia and/or Numbness (Altered Sensation):
    • Indicates nerve ischaemia. Often follows specific nerve distributions within the affected compartment (e.g., deep peroneal nerve in anterior leg compartment – 1st web space numbness).
    • May start as tingling or burning.
  4. Paresis / Muscle Weakness (Motor Deficit):
    • Weakness of muscles within the affected compartment. A later sign than sensory changes.
    • E.g., foot drop with anterior leg compartment syndrome.
  5. Pallor:
    • Uncommon and a very late sign, indicating severe arterial insufficiency. Not reliable for diagnosis.
  6. Pulselessness:
    • A very late and ominous sign, indicating that intracompartmental pressure exceeds arterial pressure. Irreversible damage is likely by this stage. Peripheral pulses are often INTACT in early compartment syndrome. Absence of a pulse is NOT required for diagnosis.

Other Important Considerations:

  • Agitation and Anxiety: Patients are often very distressed due to severe pain.
  • Symptoms may develop insidiously over hours or rapidly after acute injury.
  • Clinical assessment can be difficult in:
    • Unconscious or sedated patients (e.g., head injury, ITU).
    • Patients with nerve blocks or epidural analgesia.
    • Young children who cannot articulate symptoms.
    • Polytrauma patients with distracting injuries.
    In these situations, a lower threshold for objective pressure measurement is required.
⚠️ Do NOT Rely on Late Signs!

Waiting for pallor, pulselessness, or significant motor deficit will result in delayed diagnosis and poor outcomes. Pain out of proportion and pain on passive stretch are the most reliable early clinical indicators. A high index of suspicion is crucial.

Diagnosis & Intracompartmental Pressure (ICP) Measurement

Diagnosis of ACS is primarily clinical, supported by ICP measurement in equivocal cases or when clinical assessment is unreliable.

Clinical Diagnosis:

  • Based on high index of suspicion in at-risk patients, combined with characteristic symptoms and signs (especially pain out of proportion and pain on passive stretch).
  • Serial examinations are vital if ACS is suspected but not definitive. Document findings meticulously.
  • If clinical diagnosis is clear (e.g., conscious patient with tibial fracture, severe pain unrelieved by opioids, tense anterior compartment, pain on passive toe dorsiflexion, and 1st web space paraesthesia), proceed directly to fasciotomy without necessarily needing ICP measurement, as this can delay treatment.

Indications for Intracompartmental Pressure (ICP) Measurement:

  • Equivocal clinical findings.
  • Patient unable to cooperate with clinical examination (e.g., unconscious, sedated, intoxicated, distracting injuries, regional anaesthesia).
  • Young children.
  • To confirm diagnosis before fasciotomy if any doubt exists.
  • Monitoring high-risk patients prophylactically (less common).

ICP Measurement Techniques:

  • Various devices available:
    • Stryker ICP Monitor (or similar handheld devices): Commonly used. Needle connected to a pressure transducer.
    • Arterial line transducer system.
    • Slit catheter / Wick catheter (for continuous monitoring, less common for acute diagnosis).
  • Procedure (General Principles):
    • Sterile technique.
    • Needle inserted perpendicular to skin into the suspected compartment(s). Ensure correct anatomical landmarking for each compartment.
    • Needle tip should be at the level of the muscle belly, avoiding proximity to fracture site (which can give falsely high readings).
    • Inject small amount of sterile saline to prime system (if required by device).
    • Measure pressure in all compartments of the affected limb if suspicion is high, as multiple compartments can be involved.
    • Compare with diastolic blood pressure (DBP) to calculate Delta P (ΔP).

Interpretation of ICP Values:

  • Normal ICP: 0-10 mmHg.
  • Absolute ICP concerning for ACS:
    • Often quoted as >30 mmHg.
    • Some use >40-45 mmHg as a stronger indication.
  • Delta P (ΔP = DBP – ICP):
    • ΔP <30 mmHg is highly suggestive of ACS.
    • Some use ΔP <20 mmHg as a more specific threshold.
    • This relative measure is generally considered more reliable than absolute ICP, especially in hypotensive patients (where a lower absolute ICP might still compromise perfusion).
  • Continuous monitoring may show rising pressures.
Clinical Judgement is Key

ICP measurement is an adjunct to, not a replacement for, clinical judgement. A “normal” pressure reading does not exclude ACS if clinical signs are compelling, as pressures can fluctuate or be measured inaccurately. If in doubt, err on the side of caution and discuss with a senior/proceed with fasciotomy based on strong clinical suspicion.

Management: Urgent Fasciotomy

Once ACS is diagnosed or strongly suspected, urgent surgical decompression via fasciotomy is the definitive treatment. This is a surgical emergency.

Immediate Pre-Fasciotomy Measures:

  • Summon Senior Orthopaedic Help IMMEDIATELY.
  • Nil By Mouth (NBM): In preparation for surgery.
  • Remove All Constricting Dressings/Casts: Bivalve casts and loosen all bandages down to skin. This may reduce ICP somewhat but is rarely sufficient alone.
  • Place Limb at Heart Level: Avoid elevation above heart level, as this can further reduce arterial inflow and worsen perfusion pressure. Avoid dependency if it increases swelling/pain.
  • Analgesia: Opioids may be needed but often provide poor relief. Avoid NSAIDs if surgery imminent (bleeding risk).
  • Oxygen: If hypoxic.
  • IV Fluids: For resuscitation if hypotensive or signs of rhabdomyolysis.
  • Correct Hypotension: Maintain adequate systemic blood pressure to optimize perfusion pressure (CPP = MAP – ICP).
  • Consent for surgery (if patient able). Explain risks/benefits/alternatives.
  • Pre-operative bloods (FBC, U&Es, Coagulation screen, Group & Save/Crossmatch).

Surgical Fasciotomy:

  • Goal: To decompress all involved fascial compartments by making long incisions through skin and fascia.
  • Timing: URGENT. Aim for surgery within hours of diagnosis (ideally <6 hours from onset of symptoms if possible, but still beneficial even if delayed, though prognosis worsens).
  • Technique:
    • Performed under general or regional anaesthesia.
    • Leg Fasciotomies:
      • Anterolateral Incision (Single Incision Technique – less common now for full decompression): For anterior and lateral compartments. Risk to superficial peroneal nerve.
      • Dual Incision Technique (Most Common for Leg):
        • Anterolateral Incision: Decompresses anterior and lateral compartments. Made midway between tibial crest and fibula.
        • Posteromedial Incision: Decompresses superficial and deep posterior compartments. Made ~2cm posterior to posteromedial border of tibia. Care to protect saphenous nerve and vein.
      • Incisions must be long enough to decompress entire length of compartments. Fascia incised fully.
    • Forearm Fasciotomies: Volar (Henry) approach for volar compartment, often extending to release carpal tunnel. Dorsal incision for dorsal compartment.
    • Thigh/Arm: Long incisions over respective compartments.
  • Muscle Viability Assessment: After decompression, muscle viability is assessed (color, consistency, contractility, bleeding). Any necrotic muscle must be debrided.
  • Wound Management:
    • Fasciotomy wounds are typically left open initially due to swelling. Covered with sterile, non-adherent dressings or a vacuum-assisted closure (VAC) device.
    • Serial debridement and “second look” operations may be needed every 24-72 hours.
    • Delayed primary closure or skin grafting is performed once swelling subsides and muscle viability is confirmed (usually 5-7 days later).

Post-Operative Care:

  • Analgesia, limb elevation (once stable), monitor neurovascular status.
  • Monitor for rhabdomyolysis (CK, urine myoglobin, renal function, electrolytes) and manage accordingly (IV fluids, alkalinisation of urine – controversial).
  • Physiotherapy for range of motion and rehabilitation.
Missed Compartment Syndrome is Catastrophic!

Failure to diagnose and treat ACS promptly can lead to devastating consequences including muscle necrosis, Volkmann’s contracture, nerve damage, limb amputation, rhabdomyolysis, AKI, and even death. Medico-legally, it is a high-risk area.

Complications & Sequelae of Acute Compartment Syndrome

Complications depend on the severity and duration of ischaemia before decompression.

Early Complications:

  • Rhabdomyolysis: Release of myoglobin from necrotic muscle, leading to myoglobinuria and potentially Acute Kidney Injury (AKI). Monitor CK, U&Es, urine. Requires aggressive IV hydration.
  • Hyperkalaemia: Release of potassium from damaged cells, can cause cardiac arrhythmias.
  • Metabolic Acidosis.
  • Infection: Of fasciotomy wounds or underlying devitalised tissue.
  • Need for Amputation: If extensive muscle necrosis or limb non-viable.

Late Sequelae (Often Permanent):

  • Muscle Necrosis & Fibrosis: Leads to loss of muscle function and contractures.
  • Volkmann’s Ischaemic Contracture: Classic deformity of forearm/hand due to untreated or late-treated volar forearm compartment syndrome. Characterized by flexion contracture of wrist and fingers, muscle atrophy, and paralysis.
  • Nerve Damage / Neuropathy:
    • Persistent pain (neuropathic pain, complex regional pain syndrome – CRPS).
    • Sensory loss (numbness, paraesthesia).
    • Motor weakness or paralysis (e.g., foot drop from peroneal nerve damage).
  • Chronic Pain.
  • Joint Stiffness & Contractures.
  • Cosmetic Deformity: From scarring of fasciotomy wounds, muscle loss.
  • Non-union or Malunion of associated fractures.
  • Psychological impact.

The goal of prompt fasciotomy is to minimize these devastating long-term complications.

Special Considerations

1. Chronic Exertional Compartment Syndrome (CECS):

  • Not an acute emergency like ACS, but a cause of activity-related limb pain, often in athletes (runners, military recruits).
  • Pathophysiology: Repetitive muscle expansion during exercise within a non-compliant fascial compartment leads to transiently increased ICP and ischaemic pain. Pain resolves with rest.
  • Common Sites: Anterior and deep posterior compartments of the leg.
  • Symptoms: Cramping, aching, or burning pain that occurs at a predictable point during exercise, forcing cessation of activity. Resolves within minutes to hours of stopping. May have associated paraesthesia or weakness during episodes. Examination between episodes is often normal.
  • Diagnosis: Clinical history is key. Dynamic ICP measurement (pre- and post-exercise) can confirm (e.g., resting pressure >15 mmHg, 1-min post-exercise >30 mmHg, or 5-mins post-exercise >20 mmHg – criteria vary). MRI may show muscle oedema after exercise.
  • Management:
    • Conservative: Activity modification, physio (stretching, gait analysis, orthotics).
    • Surgical: Elective fasciotomy of involved compartments if conservative measures fail and diagnosis confirmed. Good outcomes in selected patients.

2. Abdominal Compartment Syndrome (ACS):

  • Life-threatening condition where increased intra-abdominal pressure (IAP) leads to organ dysfunction. Seen in critically ill patients (e.g., severe trauma, pancreatitis, major abdominal surgery, massive fluid resuscitation).
  • Pathophysiology: Increased IAP impairs perfusion to abdominal organs (kidneys, gut, liver) and has systemic effects (reduced venous return, increased intrathoracic pressure, respiratory compromise, intracranial hypertension).
  • Diagnosis: Clinical suspicion + measurement of intra-abdominal pressure (IAP) via bladder catheter transducer (IAP >20 mmHg with new organ dysfunction).
  • Management: Medical (sedation, paralysis, diuresis, NG/rectal tube decompression, body positioning) and surgical (decompressive laparotomy) if IAP remains high and organ dysfunction progresses. Managed in ITU setting.

While distinct from limb ACS, understanding the general principle of pressure impairing perfusion is common.

Flashcards: Compartment Syndrome

Click on each card to reveal the answer.

What is the earliest and most reliable clinical symptom of acute compartment syndrome?

(Click to flip)

Answer

Pain out of proportion to the injury, often not relieved by opioids.

What is a critical early physical sign highly suggestive of compartment syndrome?

(Click to flip)

Answer

Pain on passive stretch of the muscles within the affected compartment.

A Delta P (Diastolic BP – Intracompartmental Pressure) of what value is highly suggestive of compartment syndrome?

(Click to flip)

Answer

<30 mmHg (some use <20 mmHg).

What is the definitive treatment for acute compartment syndrome?

(Click to flip)

Answer

Urgent surgical fasciotomy of all involved compartments.

Which common fracture is most frequently associated with acute compartment syndrome of the leg?

(Click to flip)

Answer

Tibial shaft fractures.

What is Volkmann’s ischaemic contracture?

(Click to flip)

Answer

A permanent flexion contracture of the hand and wrist resulting from untreated or late-treated acute compartment syndrome of the forearm (volar compartment).

Are peripheral pulses typically absent in early acute compartment syndrome?

(Click to flip)

Answer

No. Pulselessness is a very late and ominous sign. Peripheral pulses are often intact in early compartment syndrome.

How should a limb with suspected acute compartment syndrome be positioned while awaiting surgery?

(Click to flip)

Answer

At the level of the heart. Elevation above the heart can decrease arterial inflow and worsen perfusion.

Compartment Syndrome Quiz (T&O Focus)

Test your knowledge on this orthopaedic emergency.

1. A 25-year-old male sustains a tibial shaft fracture. Hours later, he complains of severe, worsening pain in his lower leg despite morphine. His anterior compartment is extremely tense and exquisitely tender. Passive plantarflexion of his toes elicits severe pain. What is the most likely diagnosis?

A. Deep Vein Thrombosis (DVT).
B. Acute Compartment Syndrome (ACS) of the anterior compartment.
C. Peroneal nerve palsy.
D. Fat embolism syndrome.
Explanation: Pain out of proportion, tense compartment, and pain on passive stretch (plantarflexion stretches anterior compartment muscles) are classic for ACS. DVT pain is different. Nerve palsy is a sign, not the primary issue. Fat embolism has systemic features.

2. Which of the “6 P’s” of compartment syndrome is the LEAST reliable and latest sign to appear?

A. Pain out of proportion.
B. Paraesthesia.
C. Pulselessness.
D. Pain on passive stretch.
Explanation: Pulselessness is a very late sign, indicating intracompartmental pressure exceeds arterial pressure, by which time irreversible damage is highly likely. Pain out of proportion and pain on passive stretch are early signs.

3. A patient has a diastolic blood pressure of 70 mmHg. Intracompartmental pressure measurement of their anterior leg compartment is 45 mmHg. What is the Delta P (ΔP) and is it concerning for compartment syndrome?

A. ΔP = 115 mmHg, not concerning.
B. ΔP = 25 mmHg, highly concerning.
C. ΔP = 45 mmHg, not concerning based on this alone.
D. ΔP cannot be calculated with this information.
Explanation: Delta P (ΔP) = Diastolic BP – Intracompartmental Pressure. So, ΔP = 70 mmHg – 45 mmHg = 25 mmHg. A ΔP <30 mmHg is highly suggestive/concerning for compartment syndrome.

4. What is the definitive surgical treatment for established acute compartment syndrome?

A. Application of a tighter cast to reduce swelling.
B. Intracompartmental steroid injection.
C. Urgent fasciotomy of all affected compartments.
D. Limb elevation and ice.
Explanation: Urgent surgical fasciotomy is the only definitive treatment to decompress the involved compartments and restore perfusion. Other measures are temporizing or supportive.

5. Numbness and tingling in the first web space of the foot (between the big toe and second toe) is often an early sign of ischaemia to which nerve, commonly affected in anterior compartment syndrome of the leg?

A. Tibial nerve.
B. Superficial peroneal nerve.
C. Deep peroneal nerve.
D. Sural nerve.
Explanation: The deep peroneal nerve runs in the anterior compartment of the leg and provides sensory innervation to the first web space. Its compression or ischaemia is an early sign of anterior compartment syndrome.

6. After performing a fasciotomy for acute compartment syndrome, how are the wounds typically managed initially?

A. Immediate primary closure with sutures.
B. Left open and covered with sterile dressings or a VAC device.
C. Closed with skin staples under tension.
D. Immediately skin grafted.
Explanation: Fasciotomy wounds are left open initially because of significant swelling. They are covered with sterile dressings or a VAC device, and delayed primary closure or skin grafting is performed once swelling subsides (typically 5-7 days later after “second look” operations).

7. Volkmann’s ischaemic contracture is a severe, late complication of untreated compartment syndrome in which location?

A. Anterior compartment of the thigh.
B. Deep posterior compartment of the leg.
C. Volar (anterior) compartment of the forearm.
D. Lateral compartment of the leg.
Explanation: Volkmann’s contracture is the classic flexion deformity of the wrist and fingers resulting from ischaemic necrosis and fibrosis of the muscles in the volar compartment of the forearm.

8. A young runner presents with recurrent, predictable pain in the anterior aspect of both lower legs during exercise, which resolves quickly with rest. Examination between episodes is normal. What is the most likely diagnosis?

A. Acute compartment syndrome.
B. Medial tibial stress syndrome (shin splints).
C. Chronic Exertional Compartment Syndrome (CECS).
D. Stress fracture of the tibia.
Explanation: Activity-induced pain that occurs at a predictable point during exercise and resolves with rest, with normal examination findings between episodes, is characteristic of Chronic Exertional Compartment Syndrome.

9. What is the primary pathophysiological event that initiates the vicious cycle in compartment syndrome?

A. Arterial spasm.
B. Increased intracompartmental pressure leading to reduced capillary perfusion.
C. Primary nerve damage.
D. Lymphatic obstruction.
Explanation: The cycle begins with increased pressure within the compartment. As this pressure rises above capillary perfusion pressure, blood flow to tissues is compromised, leading to ischaemia, oedema, and further pressure increases.

10. Which of these is NOT a common cause of acute compartment syndrome?

A. Tibial shaft fracture.
B. Severe crush injury to a limb.
C. Tight circumferential cast.
D. Osteoarthritis flare-up.
Explanation: Osteoarthritis flare-ups cause pain and swelling due to inflammation within the joint capsule, not typically leading to increased pressure within an osteofascial muscle compartment to the extent of causing ACS. The other options are well-recognized causes.

Trauma & Orthopaedics: Hand & Wrist Injuries

Hand and wrist injuries are extremely common, accounting for a significant proportion of emergency department visits. Accurate assessment, diagnosis, and appropriate management are crucial to preserve the complex function of the hand, which is vital for daily activities, work, and quality of life. This guide covers common injuries encountered at an ST1/ST2 level.

Overview & Importance of Hand & Wrist Injuries

The hand and wrist form a complex anatomical unit responsible for a vast range of movements and functions. Injuries can result from various mechanisms, including falls onto an outstretched hand (FOOSH), direct blows, crush injuries, or penetrating trauma.

Key Considerations:

  • Functional Impact: Even seemingly minor injuries can lead to significant functional impairment, affecting a patient’s ability to perform activities of daily living (ADLs), work, and hobbies.
  • Economic Impact: Hand injuries can result in considerable time off work and long-term disability.
  • High Prevalence: These injuries are frequently seen in emergency settings and primary care.
  • Need for Precision: The intricate anatomy demands careful assessment and management to optimize outcomes and prevent long-term complications like stiffness, pain, or deformity.

Common Mechanisms of Injury (MOI):

  • Fall On Outstretched Hand (FOOSH): Classic mechanism for distal radius, scaphoid, and other carpal fractures/dislocations.
  • Direct Blow: Can cause metacarpal, phalangeal fractures, or contusions.
  • Crush Injury: Often results in complex fractures, soft tissue damage, and potential compartment syndrome.
  • Twisting/Rotational Forces: Can lead to ligamentous injuries or spiral fractures.
  • Penetrating Trauma: Risk of neurovascular and tendon injury.
💡 OSCE/PLAB Tip

Always ask about hand dominance, occupation, and hobbies. This information is crucial for understanding the functional impact of the injury and guiding management decisions, especially regarding return to work/activity.

🧠 Describing a Fracture: “LOST CABS”
L Location (which bone, which part – e.g., distal radius, scaphoid waist)
O Open or Closed
S Simple (2 parts) or Comminuted (>2 parts)
T Type of fracture line (Transverse, Oblique, Spiral, Avulsion, Impacted, Greenstick)
C Angulation (direction and degrees) / Displacement (direction and amount/%) / Rotation
A Articular involvement (intra-articular vs. extra-articular, step/gap)
B Bone quality (e.g., osteoporotic – often inferred)
S Soft tissue injury (swelling, associated wounds, neurovascular status)

Applied Anatomy of the Hand & Wrist

A fundamental understanding of hand and wrist anatomy is essential for diagnosing and managing injuries.

Bones:

  • Distal Radius & Ulna: Form the proximal articulation of the wrist. Key landmarks: radial styloid, ulnar styloid, Lister’s tubercle.
  • Carpal Bones (8): Arranged in two rows.
    • Proximal Row (Radial to Ulnar): Scaphoid, Lunate, Triquetrum, Pisiform.
    • Distal Row (Radial to Ulnar): Trapezium, Trapezoid, Capitate, Hamate.
    • Mnemonic (Proximal to Distal, Radial to Ulnar): “Some Lovers Try Positions That They Can’t Handle”
  • Metacarpals (5): Numbered 1 (thumb) to 5 (little finger). Each has a base, shaft, neck, and head.
  • Phalanges (14): Thumb has 2 (proximal, distal). Fingers 2-5 have 3 (proximal, middle, distal).

Joints:

  • Radiocarpal Joint: Distal radius and scaphoid/lunate.
  • Midcarpal Joint: Between proximal and distal carpal rows.
  • Carpometacarpal (CMC) Joints: Distal carpal row and metacarpal bases. Thumb CMC is a saddle joint.
  • Metacarpophalangeal (MCP) Joints: Metacarpal heads and proximal phalanges.
  • Interphalangeal (IP) Joints: Proximal (PIP) and Distal (DIP) joints.

Key Ligaments:

  • Scapholunate Ligament: Critical for wrist stability; injury leads to scapholunate dissociation.
  • Lunotriquetral Ligament: Stabilizes ulnar side of the wrist.
  • Triangular Fibrocartilage Complex (TFCC): Major stabilizer of the distal radioulnar joint (DRUJ) and ulnar carpus.
  • Collateral Ligaments: Stabilize MCP and IP joints (e.g., Ulnar Collateral Ligament of thumb MCPJ).

Nerves (Major branches in hand/wrist):

  • Median Nerve:
    • Course: Passes through carpal tunnel.
    • Sensory: Volar aspect of thumb, index, middle, and radial half of ring finger. Dorsal tips of these fingers.
    • Motor: Thenar muscles (LOAF: Lumbricals 1&2, Opponens pollicis, Abductor pollicis brevis, Flexor pollicis brevis – superficial head). Prone to compression in Carpal Tunnel Syndrome.
  • Ulnar Nerve:
    • Course: Passes through Guyon’s canal at the wrist.
    • Sensory: Little finger and ulnar half of ring finger (volar and dorsal).
    • Motor: Hypothenar muscles, interossei, adductor pollicis, lumbricals 3&4, flexor pollicis brevis (deep head).
  • Radial Nerve:
    • Course: Superficial branch provides sensation to dorsum of hand. Deep branch (Posterior Interosseous Nerve – PIN) is motor.
    • Sensory: Dorsal aspect of thumb, index, middle fingers proximal to DIPJs, and radial side of dorsum of hand (1st dorsal webspace).
    • Motor (via PIN): Wrist extensors, finger extensors, thumb extensors (supinator).

Tendons:

  • Flexor Tendons: Pass through carpal tunnel (except FCR) and digital flexor sheaths. Flexor Digitorum Superficialis (FDS – flexes PIPJ), Flexor Digitorum Profundus (FDP – flexes DIPJ), Flexor Pollicis Longus (FPL – flexes thumb IPJ).
  • Extensor Tendons: Organized into 6 dorsal compartments at the wrist. Responsible for wrist, finger, and thumb extension.

Vascular Supply:

  • Mainly from Radial and Ulnar arteries, forming superficial and deep palmar arches.

Clinical Assessment of Hand & Wrist Injuries

A systematic approach (History and Examination) is crucial.

History:

  • Patient Demographics: Age, occupation, hand dominance, hobbies.
  • Mechanism of Injury (MOI): How did it happen? (e.g., FOOSH, direct blow, twisting). Force involved?
  • Presenting Complaint:
    • Pain: SOCRATES (Site, Onset, Character, Radiation, Associated symptoms, Timing, Exacerbating/Relieving factors, Severity).
    • Swelling, Deformity, Bruising.
    • Loss of Function: Inability to move wrist/fingers, weakness, difficulty gripping.
    • Neurological Symptoms: Numbness, tingling (paresthesia), weakness. Which digits/area?
    • Sounds at time of injury: Pop, snap, crack.
  • Past Medical History: Previous injuries to hand/wrist, arthritis, diabetes, osteoporosis, smoking, medications (e.g., steroids, anticoagulants).
  • Allergies, Tetanus status.

Examination: (Always compare with the uninjured side)

“Look, Feel, Move, Special Tests, Neurovascular Assessment”

  • General: If polytrauma, follow ATLS guidelines first. Expose the entire limb adequately.
  • Look (Inspect):
    • Attitude of the hand/fingers at rest.
    • Swelling, bruising (ecchymosis), erythema.
    • Deformity: Angulation (e.g., “dinner fork” in Colles’), rotation, shortening.
    • Wounds: Open vs. closed fracture. Note size, contamination. Check for “fight bites” over MCPJs.
    • Skin integrity, scars, muscle wasting (e.g., thenar/hypothenar).
  • Feel (Palpate):
    • Temperature (warmth may suggest inflammation/infection).
    • Tenderness: Systematically palpate bony landmarks and soft tissues.
      • Distal radius, ulnar styloid, Lister’s tubercle.
      • Anatomical snuffbox, scaphoid tubercle, hook of hamate.
      • Metacarpal shafts and heads, phalanges.
      • Ligaments (e.g., UCL of thumb), tendon sheaths.
    • Crepitus, swelling (soft, boggy, tense).
    • Assess for fluctuance (abscess/haematoma).
  • Move (Assess Range of Motion – ROM):
    • Active ROM first, then passive ROM if active is limited and tolerated.
    • Wrist: Flexion, extension, radial deviation, ulnar deviation, supination, pronation.
    • Fingers: MCPJ, PIPJ, DIPJ flexion/extension. Abduction/adduction.
    • Thumb: Flexion/extension (MCPJ, IPJ), abduction/adduction, opposition.
    • Note pain, crepitus, or block to movement.
  • Neurovascular Assessment (CRITICAL – perform before and after any manipulation/reduction):
    • Vascular:
      • Capillary refill time (CRT <2 seconds).
      • Radial and Ulnar pulses.
      • Skin colour and temperature.
      • Allen’s Test (if considering radial artery cannulation or harvest).
    • Neurological (Motor & Sensory for each nerve):
      • Median Nerve:
        • Sensory: Volar tip of index finger. Two-point discrimination (2PD).
        • Motor: “OK” sign (FPL and FDP to index finger), or thumb abduction against resistance (Abductor Pollicis Brevis).
      • Ulnar Nerve:
        • Sensory: Volar tip of little finger. 2PD.
        • Motor: Finger abduction/adduction (interossei – “starfish”), or Froment’s sign (tests Adductor Pollicis – patient grips paper, if weak, IPJ of thumb flexes due to FPL compensating).
      • Radial Nerve (Superficial and PIN):
        • Sensory: Dorsum of 1st web space.
        • Motor: Wrist extension, finger MCPJ extension, thumb extension (“hitchhiker’s sign”).
  • Special Tests (Perform as indicated by history and initial findings):
    • Scaphoid: Snuffbox tenderness, scaphoid tubercle tenderness, axial compression of thumb.
    • Ligamentous instability: Watson’s test (scapholunate), UCL stress test (thumb MCPJ).
    • Tendon integrity: Test FDS/FDP isolation, Elson’s test (central slip), test for Mallet/Jersey finger.
    • Nerve compression: Tinel’s test, Phalen’s test (Carpal Tunnel Syndrome).
    • Tenosynovitis: Finkelstein’s test (De Quervain’s).
⚠️ Red Flag in Examination

Any rotational deformity of the fingers (e.g., fingers overlapping or “scissoring” when making a fist) is abnormal and usually requires surgical correction, especially in metacarpal or phalangeal fractures. Document this clearly!

Imaging & Investigations for Hand & Wrist Injuries

1. X-rays (Radiographs):

  • Indications: Suspected fracture, dislocation, foreign body, or bony abnormality. Follow Ottawa Ankle/Knee rules equivalent for wrist if local guidelines exist, but generally lower threshold for X-raying wrist/hand injuries.
  • Standard Views:
    • Wrist: Posteroanterior (PA), Lateral, Oblique.
    • Hand: PA, Lateral, Oblique.
    • Fingers: PA, Lateral.
  • Specific Views:
    • Scaphoid Series: PA with ulnar deviation, lateral, oblique (often supinated and pronated obliques), and a coned PA view of the scaphoid.
    • Clenched Fist View (PA): For suspected scapholunate instability (shows widening of scapholunate interval – “Terry Thomas sign”).
    • Carpal Tunnel View: Axial view to assess carpal tunnel space and hook of hamate.
    • Stress Views: To assess ligamentous instability (less common in acute setting).
  • Systematic Interpretation (ABCDE’S or similar):
    • Alignment & Articular congruity: Check carpal arcs (Gilula’s lines), joint spaces, displacement, angulation.
    • Bones: Trace cortices for breaks/fracture lines. Note bone density, lytic/sclerotic lesions.
    • Cartilage: Inferred from joint space width.
    • Distribution: Is the injury pattern typical for a certain mechanism?
    • Soft Tissues: Swelling, effusions, foreign bodies, gas (infection).
  • Describing a Fracture: Use a systematic approach (e.g., “LOST CABS” mnemonic – see Overview section).

2. Computed Tomography (CT) Scan:

  • Indications:
    • Detailed assessment of complex intra-articular fractures (e.g., comminuted distal radius, Rolando’s).
    • Pre-operative planning.
    • Suspected occult fractures not clearly visible on X-ray (e.g., scaphoid, hook of hamate, carpal boss).
    • Assessment of fracture healing (non-union/malunion).

3. Magnetic Resonance Imaging (MRI):

  • Indications:
    • Suspected significant soft tissue injuries: Ligaments (scapholunate, TFCC), tendons (ruptures, tendinopathy), cartilage.
    • Early detection of Avascular Necrosis (AVN) (e.g., scaphoid, lunate – Kienbock’s disease).
    • Occult fractures or bone bruising if X-ray/CT inconclusive.
    • Nerve compression or injury assessment.
    • Infection (osteomyelitis, abscess).

4. Ultrasound (USS):

  • Indications:
    • Dynamic assessment of tendons (ruptures, subluxation, tenosynovitis – e.g., De Quervain’s).
    • Assessment of ligaments, ganglion cysts, foreign bodies (especially radiolucent).
    • Guiding aspirations or injections.
    • Assessment of nerve morphology (e.g., median nerve in CTS).

5. Nerve Conduction Studies (NCS) / Electromyography (EMG):

  • Indications:
    • To confirm and quantify nerve entrapment syndromes (e.g., Carpal Tunnel Syndrome, Cubital Tunnel Syndrome).
    • To assess severity and prognosis of nerve injuries.
    • Differentiating radiculopathy from peripheral neuropathy.
💡 OSCE/PLAB Tip: Scaphoid Fractures

If a patient has clinical signs of a scaphoid fracture (snuffbox tenderness after FOOSH) but initial X-rays are normal, treat as a scaphoid fracture (thumb spica cast/splint) and arrange repeat X-rays in 10-14 days or consider early MRI/CT if available and indicated. Missing a scaphoid fracture can lead to non-union or AVN.

Common Hand & Wrist Fractures

1. Distal Radius Fractures:

  • Epidemiology: Most common fracture in adults, especially elderly women (osteoporosis) and young active individuals. MOI: FOOSH.
  • Key Types:
    • Colles’ Fracture: Extra-articular or intra-articular, dorsal displacement and angulation, radial shortening. “Dinner fork” deformity. MOI: FOOSH with wrist dorsiflexion.
    • Smith’s Fracture (“Reverse Colles’”): Volar displacement and angulation. “Garden spade” deformity. MOI: Fall on back of hand (wrist flexed).
    • Barton’s Fracture: Intra-articular fracture with associated radiocarpal subluxation/dislocation. Can be dorsal or volar. Unstable.
    • Chauffeur’s (Hutchinson’s) Fracture: Intra-articular fracture of the radial styloid.
  • Assessment: Deformity, swelling, pain. Crucially, assess median nerve function and vascular status.
  • Management Principles:
    • Undisplaced/Minimally displaced stable #: Cast immobilisation (e.g., below-elbow backslab initially, then full cast).
    • Displaced #: Closed reduction (under haematoma block, Bier’s block, or sedation/GA) followed by cast. Check post-reduction X-rays and neurovascular status.
    • Surgical Indications (ORIF with volar plate common, K-wires, Ex-fix): Open #, irreducible #, unstable # (e.g., significant comminution, Barton’s), severe displacement/angulation/shortening despite reduction, intra-articular step >2mm, associated neurovascular injury requiring repair.
  • Complications: Median nerve injury (acute or carpal tunnel syndrome), malunion, stiffness, CRPS, EPL tendon rupture (late).

2. Scaphoid Fractures:

  • Epidemiology: Most common carpal bone fracture. MOI: FOOSH with wrist hyperextension and radial deviation.
  • Clinical Features: Pain and tenderness in anatomical snuffbox, pain over scaphoid tubercle (volar), pain with axial compression of thumb. Swelling may be minimal.
  • Anatomical Significance: Precarious retrograde blood supply (mainly from dorsal carpal branch of radial artery), especially to proximal pole, leading to high risk of Avascular Necrosis (AVN) and non-union.
  • Investigations: Scaphoid X-ray series. If negative but high clinical suspicion: immobilise in thumb spica and repeat X-ray in 10-14 days, OR early MRI/CT.
  • Management:
    • Undisplaced waist/distal pole #: Thumb spica cast for 6-12 weeks (longer for proximal pole).
    • Displaced (>1mm), proximal pole #, unstable #, or established non-union: Surgical fixation (e.g., percutaneous or open headless compression screw).
  • Complications: Non-union (can lead to SNAC wrist – Scaphoid Non-union Advanced Collapse), AVN, osteoarthritis.

3. Metacarpal Fractures:

  • Commonly due to direct trauma or axial loading (punching).
  • Types by Location:
    • Metacarpal Neck (e.g., Boxer’s Fracture): Fracture of the 5th (or 4th) metacarpal neck, typically from punching a solid object. Apex dorsal angulation. Check for rotational deformity (“scissoring” of fingers) – this is unacceptable.
      • Management: Acceptable angulation varies (more tolerated in 4th/5th MCs, e.g., up to 30-40° for 5th MC). Ulnar gutter splint/cast. Surgical fixation (K-wires, plate) for excessive angulation, rotation, or open fracture.
    • Metacarpal Shaft: Transverse, oblique, spiral, or comminuted. Rotational alignment is key.
    • Metacarpal Base (e.g., Bennett’s, Rolando’s):
      • Bennett’s Fracture: Intra-articular fracture-dislocation at the base of the 1st metacarpal (thumb). Small ulnar fragment held by ligaments, while the larger fragment is pulled radially and dorsally by Abductor Pollicis Longus (APL). Inherently unstable. Often requires CRPP or ORIF.
      • Rolando’s Fracture: Comminuted intra-articular fracture at the base of the 1st metacarpal (T or Y-shaped). Difficult to manage, often poor prognosis. Usually requires surgical fixation.

4. Phalangeal Fractures:

  • Common, result from direct blows, crush injuries, or sporting injuries.
  • Types:
    • Tuft Fractures (Distal Phalanx): Crush injury (e.g., door slam). Often associated with subungual haematoma or nail bed injury. Management: Symptomatic, protective splint. Drain large/painful subungual haematoma by trephination.
    • Shaft/Neck/Base Fractures (Proximal and Middle Phalanges): Rotational and angular deformities are poorly tolerated.
      • Management: Stable, undisplaced fractures can be managed with buddy taping to adjacent finger and early mobilisation. Unstable, displaced, or intra-articular fractures often require closed reduction and splinting, or surgical fixation (K-wires, mini-screws/plates).
  • Complications: Stiffness, malunion (especially rotational), non-union, tendon adhesions.

Common Soft Tissue & Nerve Injuries

Tendon Injuries:

  • Mallet Finger (Baseball Finger):
    • Mechanism: Forced flexion of an actively extended distal interphalangeal joint (DIPJ), e.g., ball hitting tip of finger. Results in rupture or avulsion of the extensor tendon from its insertion on the distal phalanx (± small bony fragment).
    • Clinical: Inability to actively extend the DIPJ. The DIPJ rests in a flexed position (~30-40°). Passive extension is possible.
    • Management: Continuous extension splinting of the DIPJ (e.g., Stack splint, custom thermoplastic splint) for 6-8 weeks, followed by night splinting. Surgical intervention if large bony fragment (>30% articular surface) or joint subluxation.
  • Jersey Finger:
    • Mechanism: Forced hyperextension of an actively flexed DIPJ, e.g., player grabs opponent’s jersey. Avulsion of the Flexor Digitorum Profundus (FDP) tendon from its insertion on the volar base of the distal phalanx. Ring finger is most commonly affected.
    • Clinical: Inability to actively flex the DIPJ of the affected finger. Tenderness over volar aspect. The avulsed tendon may retract into palm (Leddy and Packer classification based on retraction level).
    • Management: Surgical repair is usually required, ideally within 7-10 days to prevent tendon retraction and scarring.
  • Boutonnière Deformity:
    • Mechanism: Disruption of the central slip of the extensor tendon at its insertion on the base of the middle phalanx, often from forced flexion of PIPJ or volar dislocation. Lateral bands displace volarly.
    • Clinical: PIPJ flexion and DIPJ hyperextension. Deformity may not be apparent immediately. Elson’s test can be diagnostic.
    • Management: Splinting of the PIPJ in full extension for 4-6 weeks, allowing DIPJ active flexion. Surgery for failed conservative treatment or open injuries.
  • Flexor/Extensor Tendon Lacerations:
    • Mechanism: Usually penetrating trauma (e.g., knife, glass).
    • Assessment: Carefully assess individual tendon function (e.g., FDS and FDP isolation). Note zone of injury (Verdan’s zones for flexors, Doyle’s zones for extensors).
    • Management: Requires specialist hand surgical repair, followed by complex rehabilitation protocol.

Ligament Injuries:

  • Skier’s Thumb (Gamekeeper’s Thumb):
    • Mechanism: Forced abduction and hyperextension of the thumb MCPJ, leading to injury (sprain or rupture) of the Ulnar Collateral Ligament (UCL).
    • Clinical: Pain, swelling, and tenderness on the ulnar aspect of the thumb MCPJ. Instability with valgus stress testing (compare to contralateral side).
    • Stener Lesion: If UCL is completely torn, the adductor aponeurosis can become interposed between the ruptured ends of the ligament, preventing healing. This usually requires surgical repair.
    • Management: Partial tears without instability: Thumb spica cast/splint for 4-6 weeks. Complete tears or suspected Stener lesion: Surgical repair.
  • Scapholunate Ligament Injury:
    • Mechanism: FOOSH, often with wrist hyperextension and ulnar deviation. Most common carpal instability.
    • Clinical: Dorsal wrist pain (especially over SL interval), clicking, weakness, reduced grip strength. Watson’s test (scaphoid shift test) may be positive.
    • X-ray: May show scapholunate diastasis (>3mm, “Terry Thomas sign” or “David Letterman sign”), especially on clenched fist PA view. Dorsal Intercalated Segment Instability (DISI) on lateral view in chronic cases.
    • Management: Acute partial tears: Immobilisation. Complete tears or chronic instability: Arthroscopic debridement, repair, or reconstruction.

Nerve Entrapment Syndromes:

  • Carpal Tunnel Syndrome (CTS):
    • Pathophysiology: Compression of the median nerve within the carpal tunnel.
    • Causes: Idiopathic (most common), pregnancy, hypothyroidism, rheumatoid arthritis, diabetes, previous wrist fracture, repetitive motions.
    • Symptoms: Pain, numbness, and tingling (paresthesia) in the median nerve distribution (thumb, index, middle, radial half of ring finger). Often worse at night, relieved by shaking hand. Weakness or atrophy of thenar muscles (Abductor Pollicis Brevis) in late stages.
    • Signs: Positive Tinel’s test (tapping over carpal tunnel), Phalen’s test (wrist flexion for 60s), Durkan’s test (carpal compression test). Thenar wasting.
    • Investigations: Nerve Conduction Studies (NCS)/EMG to confirm diagnosis and severity.
    • Management: Conservative (night splints in neutral, activity modification, NSAIDs, steroid injections). Surgical (carpal tunnel decompression/release) for persistent/severe symptoms or thenar atrophy.
  • De Quervain’s Tenosynovitis:
    • Pathophysiology: Stenosing tenosynovitis of the first dorsal compartment of the wrist, affecting Abductor Pollicis Longus (APL) and Extensor Pollicis Brevis (EPB) tendons.
    • Causes: Repetitive thumb abduction and wrist ulnar deviation (e.g., new mothers lifting baby, repetitive gripping).
    • Symptoms: Pain and tenderness over the radial styloid, radiating up forearm or down to thumb. Swelling may be present.
    • Signs: Positive Finkelstein’s test (patient makes fist with thumb inside, then ulnarly deviates wrist, causing pain).
    • Management: Conservative (thumb spica splint, NSAIDs, activity modification, steroid injection into 1st dorsal compartment). Surgical release for refractory cases.

General Management Principles for Hand & Wrist Injuries

The goals of treatment are to restore anatomy, achieve stable fixation, allow early mobilisation where possible, and ultimately restore optimal function while minimising pain.

Initial Management (ED / Acute Setting):

  • DRSABCD/ATLS: If part of major trauma.
  • Control Haemorrhage: Direct pressure for bleeding.
  • Analgesia: Provide adequate pain relief (e.g., paracetamol, NSAIDs, opioids if severe). Consider regional blocks (e.g., haematoma block for distal radius reduction).
  • Neurovascular Assessment: Document pre- and post-intervention status.
  • Reduction (if indicated): For displaced fractures or dislocations. Aim for anatomical alignment. Requires appropriate analgesia/anaesthesia.
  • Immobilisation: Splint suspected or confirmed fractures/unstable injuries to provide pain relief, prevent further displacement, and protect soft tissues. Common splints: backslab, volar slab, U-slab, thumb spica.
  • Elevation: To reduce swelling.
  • Ice: Intermittent application to reduce swelling and pain.
  • Wound Care: For open fractures/wounds: Clean, sterile dressing. Tetanus prophylaxis. Antibiotics (e.g., IV co-amoxiclav for open fractures).
  • Referral: Refer to Orthopaedics/Hand Surgery as appropriate (e.g., open fractures, irreducible dislocations, neurovascular compromise, complex fractures needing surgery).

Non-Operative Management:

  • Indications: Many stable, undisplaced or minimally displaced fractures, some ligament sprains, tendinopathies.
  • Methods:
    • Casting/Splinting: For a defined period. Educate on cast care, red flag symptoms (e.g., tight cast, severe pain, numbness).
    • Buddy Taping: For stable phalangeal fractures.
    • Analgesia and Anti-inflammatories.
    • Physiotherapy/Occupational Therapy: For rehabilitation after immobilisation (ROM, strengthening, functional exercises).

Operative Management:

  • General Indications:
    • Open fractures (require debridement and washout).
    • Unstable fractures (likely to displace in cast).
    • Irreducible fractures/dislocations.
    • Significantly displaced intra-articular fractures (e.g., >2mm step or gap).
    • Associated neurovascular injury requiring repair.
    • Certain fracture patterns known for poor outcomes with non-operative treatment (e.g., displaced scaphoid proximal pole, Bennett’s fracture).
    • Failed non-operative management (e.g., non-union, symptomatic malunion).
    • Tendon ruptures (e.g., FDP avulsion, some extensor tendon lacerations).
    • Significant ligamentous instability requiring repair/reconstruction.
  • Common Surgical Techniques:
    • Open Reduction and Internal Fixation (ORIF): Using plates, screws, wires (e.g., volar plating for distal radius fractures, screw fixation for scaphoid).
    • Closed Reduction and Percutaneous Pinning (CRPP): Using Kirschner wires (K-wires) inserted through the skin to hold fracture fragments after closed reduction.
    • External Fixation: For severe open fractures, extensive soft tissue damage, or temporary stabilisation.
    • Tendon Repair/Reconstruction.
    • Ligament Repair/Reconstruction.
    • Nerve Repair/Decompression.
    • Arthrodesis (fusion) or Arthroplasty (joint replacement) for severe arthritis or unsalvageable joints.

Rehabilitation:

  • Crucial for all significant hand/wrist injuries, whether managed operatively or non-operatively.
  • Aims: Control pain and oedema, restore ROM, improve strength, restore function.
  • Involves physiotherapists and occupational therapists (hand therapists).
  • May include splinting, scar management, oedema control, specific exercises, and functional retraining.
💡 OSCE/PLAB Tip: Consent for Reduction

When consenting for a closed reduction (e.g., of a Colles’ fracture), explain the procedure, risks (pain, swelling, bruising, nerve/vessel injury, stiffness, failure of reduction, need for surgery, CRPS), benefits (pain relief, improved alignment/function), and alternatives (e.g., no reduction, surgery). Document this discussion.

Complications & Red Flags in Hand & Wrist Injuries

Red Flags (Require Urgent Senior Review / Intervention):

  • Open Fracture:
    • Any fracture communicating with the external environment. High risk of infection (osteomyelitis).
    • Management: Urgent IV antibiotics (e.g., co-amoxiclav + gentamicin depending on severity/contamination), tetanus prophylaxis, surgical debridement and washout (usually within 6-24 hours). (Gustilo-Anderson classification for severity).
  • Acute Neurovascular Compromise:
    • Signs: Pallor, pulselessness (late sign!), paraesthesia, paralysis, poikilothermia (coldness). Rapidly progressing neurological deficit.
    • Management: Immediate attempt at gentle realignment/reduction. If unsuccessful or deficit persists, urgent surgical exploration/decompression is required.
  • Compartment Syndrome:
    • Increased pressure within a closed fascial compartment, compromising circulation and function of tissues within that space. Most common in forearm after supracondylar or distal radius fractures, or crush injuries. Can occur in hand intrinsic compartments.
    • Key Symptoms/Signs (The 6 P’s – but pain is earliest and most reliable):
      • Pain out of proportion to the injury, especially on passive stretch of muscles in the compartment (e.g., passive finger extension for forearm flexor compartment).
      • Paraesthesia (early sign of nerve ischaemia).
      • Pallor (late).
      • Pulselessness (very late and ominous sign).
      • Paralysis/Paresis (late).
      • Pressure (tense, swollen, tender compartment).
    • Diagnosis: Primarily clinical. Compartment pressure measurement can confirm if >30 mmHg or delta pressure (Diastolic BP – Compartment Pressure) <20-30 mmHg.
    • Management: Surgical emergency! Immediate removal of all constricting dressings/casts. Urgent fasciotomy to decompress all affected compartments. Delay can lead to Volkmann’s ischaemic contracture (irreversible muscle and nerve damage).
  • Irreducible Dislocation / Gross Instability.
  • “Fight Bite” (Human Bite over MCPJ):
    • Often appears innocuous but has very high risk of septic arthritis due to direct inoculation of oral flora (e.g., Eikenella corrodens) into joint.
    • Management: Thorough exploration (often in theatre), washout, debridement, IV antibiotics (e.g., co-amoxiclav). Do not close wound primarily.

Early Complications (Days to Weeks):

  • Infection: Wound infection, pin-site infection, osteomyelitis, septic arthritis.
  • Haematoma / Seroma.
  • Loss of Reduction / Fixation Failure.
  • Pressure Sores / Blisters: From poorly applied cast/splint.
  • Thromboembolism (DVT/PE): Less common for isolated upper limb but risk increases with prolonged immobility, surgery, or polytrauma.

Late Complications (Weeks to Months/Years):

  • Malunion: Fracture heals in a non-anatomical (angulated, rotated, shortened) position. Can cause pain, deformity, functional limitation, and secondary arthritis.
  • Non-union: Fracture fails to heal after an adequate period (typically >6 months). Causes: poor blood supply (e.g., scaphoid proximal pole), instability, infection, smoking, systemic factors. X-ray: persistent fracture line, sclerotic bone ends.
  • Avascular Necrosis (AVN): Bone death due to compromised blood supply. Common sites: scaphoid (proximal pole), lunate (Kienbock’s disease).
  • Stiffness / Contractures: Very common. Due to immobilisation, oedema, adhesions, scarring. Requires intensive physiotherapy/hand therapy.
  • Post-Traumatic Osteoarthritis: Degenerative joint changes following injury, especially intra-articular fractures or malunion.
  • Complex Regional Pain Syndrome (CRPS):
    • Type 1 (formerly Reflex Sympathetic Dystrophy – RSD): No identifiable nerve injury.
    • Type 2 (formerly Causalgia): Identifiable nerve injury.
    • Features: Severe pain (burning, disproportionate), allodynia, hyperalgesia, swelling, stiffness, skin changes (colour, temperature, sweating abnormalities – “Sudeck’s atrophy”), trophic changes (hair, nails).
    • Management: Early recognition, multidisciplinary approach (pain team, physio/OT, psychology).
  • Tendon Rupture (e.g., EPL rupture after distal radius #) or Adhesions.
  • Nerve Damage Sequelae: Persistent numbness, weakness, chronic neuropathic pain.
  • Implant-related Problems: Loosening, breakage, irritation, infection.
⚠️ Compartment Syndrome is a Clinical Diagnosis!

Do not wait for all 6 P’s to manifest or for compartment pressure measurements if clinical suspicion is high, especially pain on passive stretch. Delay in performing fasciotomy can have devastating consequences. If in doubt, escalate to a senior immediately.

Flashcards: Hand & Wrist Injuries

Click on each card to reveal the answer.

What is the classic deformity seen in a Colles’ fracture?

(Click to flip)

Answer

“Dinner fork” deformity (dorsal displacement and angulation of the distal radius).

Which carpal bone is most commonly fractured and what is a major complication?

(Click to flip)

Answer

Scaphoid bone.
Major complications: Avascular Necrosis (AVN) and Non-union, especially of the proximal pole.

A patient punches a wall and now has pain and swelling over the 5th MCPJ. What is the likely diagnosis?

(Click to flip)

Answer

Boxer’s fracture (fracture of the 5th metacarpal neck).

What are the key motor and sensory functions to test for the Median nerve in the hand?

(Click to flip)

Answer

Sensory: Volar tip of index finger.
Motor: Thumb abduction (Abductor Pollicis Brevis) or “OK” sign (FPL/FDP index).

What is Mallet Finger and how is it typically managed?

(Click to flip)

Answer

Rupture/avulsion of extensor tendon at DIPJ, causing inability to actively extend DIPJ.
Managed with continuous extension splinting of DIPJ for 6-8 weeks.

What is the most sensitive early sign of compartment syndrome?

(Click to flip)

Answer

Pain out of proportion to the injury, especially pain on passive stretch of muscles in the affected compartment.

Name two specific X-ray views for a suspected scaphoid fracture.

(Click to flip)

Answer

1. PA view with ulnar deviation.
2. Oblique views (supinated/pronated).
(Also: coned PA, lateral).

What is a “Stener lesion” in the context of a Skier’s thumb?

(Click to flip)

Answer

Interposition of the adductor aponeurosis between the ruptured ends of the Ulnar Collateral Ligament (UCL) of the thumb MCPJ, preventing healing. Usually requires surgical repair.

Hand & Wrist Injuries Quiz

Test your knowledge with these Orthopaedics-focused questions.

1. A 65-year-old woman falls on her outstretched hand. She presents with a “dinner fork” deformity of her wrist. Which nerve is most commonly at risk of injury with this fracture type?

A. Ulnar nerve.
B. Median nerve.
C. Radial nerve.
D. Axillary nerve.
Explanation: The “dinner fork” deformity is characteristic of a Colles’ fracture (dorsally displaced distal radius fracture). The Median nerve is most commonly affected, either acutely by the fracture fragments or later by swelling causing carpal tunnel syndrome.

2. A young man presents with pain in the anatomical snuffbox after a fall onto his outstretched hand during football. Initial X-rays are reported as normal. What is the most appropriate next step in management?

A. Discharge with analgesia and advise to return if pain persists.
B. Immobilise in a thumb spica cast/splint and arrange for repeat X-rays or further imaging (MRI/CT) in 10-14 days.
C. Refer for urgent surgical exploration.
D. Prescribe a course of physiotherapy.
Explanation: Clinical suspicion of a scaphoid fracture (snuffbox tenderness post-FOOSH) warrants treatment as such, even with normal initial X-rays, due to the risk of non-union/AVN if missed. Immobilisation and follow-up imaging are crucial.

3. Which of the following is the most reliable early clinical sign of acute compartment syndrome in the forearm?

A. Absent radial pulse.
B. Pain on passive extension of the fingers.
C. Pallor of the hand.
D. Paralysis of finger movement.
Explanation: Pain out of proportion to the injury, especially exacerbated by passive stretching of muscles within the affected compartment (e.g., passive finger extension stretching forearm flexors), is the earliest and most sensitive sign of compartment syndrome. Pulselessness, pallor, and paralysis are late signs.

4. A patient is unable to actively extend the DIP joint of their ring finger after it was forcibly flexed by a ball. What is the most likely diagnosis?

A. Jersey finger.
B. Boutonnière deformity.
C. Mallet finger.
D. Trigger finger.
Explanation: Mallet finger results from injury to the extensor tendon at its insertion on the distal phalanx, leading to an inability to actively extend the DIP joint. Jersey finger is an FDP avulsion (inability to flex DIPJ). Boutonnière affects PIPJ. Trigger finger is stenosing tenosynovitis.

5. What is the primary concern with a “fight bite” (human bite over an MCP joint)?

A. Risk of rabies transmission.
B. Associated fracture of the metacarpal head.
C. High risk of septic arthritis due to inoculation of oral flora.
D. Extensive soft tissue loss.
Explanation: Human bites, especially over joints like the MCPJ (“fight bite”), have a very high risk of causing septic arthritis due to direct inoculation of virulent oral bacteria (e.g., Eikenella corrodens). They require aggressive management with washout, debridement, and antibiotics.

6. Which test involves asking the patient to make a fist with their thumb tucked inside, followed by ulnar deviation of the wrist, to diagnose De Quervain’s tenosynovitis?

A. Phalen’s test.
B. Tinel’s test.
C. Finkelstein’s test.
D. Watson’s test.
Explanation: Finkelstein’s test is used to diagnose De Quervain’s tenosynovitis (inflammation of APL and EPB tendons). Phalen’s and Tinel’s are for carpal tunnel syndrome. Watson’s test is for scapholunate instability.

7. A Bennett’s fracture involves which bone?

A. Distal phalanx of the thumb.
B. Base of the first metacarpal.
C. Scaphoid waist.
D. Hook of hamate.
Explanation: A Bennett’s fracture is an intra-articular fracture-dislocation of the base of the first metacarpal (thumb). It is inherently unstable due to the pull of the Abductor Pollicis Longus tendon.

8. What is the “Terry Thomas sign” (or David Letterman sign) on a wrist X-ray indicative of?

A. Scaphoid fracture.
B. Scapholunate ligament dissociation.
C. Kienbock’s disease (AVN of lunate).
D. Distal radioulnar joint (DRUJ) instability.
Explanation: The “Terry Thomas sign” refers to a gap of >3mm between the scaphoid and lunate bones on a PA wrist X-ray, indicative of scapholunate ligament injury/dissociation.

9. When assessing motor function of the ulnar nerve in the hand, which of these actions is most specific?

A. Wrist flexion.
B. Finger abduction (spreading fingers apart).
C. Thumb opposition.
D. Wrist extension.
Explanation: Finger abduction is primarily performed by the dorsal interossei muscles, which are innervated by the ulnar nerve. Froment’s sign (testing adductor pollicis) is also specific. Wrist flexion has dual innervation (median/ulnar). Thumb opposition is median. Wrist extension is radial.

10. A patient has a displaced intra-articular fracture of the distal radius. What is generally considered the maximum acceptable articular step or gap before surgical intervention is strongly considered?

A. 0.5 mm
B. 2 mm
C. 5 mm
D. 10 mm
Explanation: For intra-articular fractures, an articular step-off or gap of more than 2mm is generally considered an indication for surgical reduction and fixation to restore joint congruity and reduce the risk of post-traumatic arthritis.

Trauma & Orthopaedics: Shoulder Pain Assessment

Shoulder pain is a frequent presentation in primary care and musculoskeletal clinics. A systematic approach to history taking and physical examination is essential to formulate a differential diagnosis and guide appropriate management or referral. This guide focuses on the clinical assessment of the painful shoulder.

Overview & Relevant Anatomy

The shoulder is a highly mobile complex of joints, allowing a wide range of motion but also making it susceptible to injury and degenerative conditions. Understanding the basic anatomy is key to interpreting clinical findings.

Key Anatomical Structures:

  • Bones:
    • Humerus: Head articulates with glenoid. Landmarks: Greater tuberosity, lesser tuberosity, bicipital groove.
    • Scapula (Shoulder Blade): Glenoid fossa, acromion, coracoid process, spine.
    • Clavicle (Collarbone): Articulates with acromion (ACJ) and sternum (SCJ).
  • Joints:
    • Glenohumeral Joint (GHJ): True “ball and socket” shoulder joint. Highly mobile, inherently unstable. Stabilised by rotator cuff, labrum, capsule, ligaments.
    • Acromioclavicular Joint (ACJ): Between acromion and clavicle.
    • Sternoclavicular Joint (SCJ): Between sternum and clavicle.
    • Scapulothoracic Articulation: “Physiological” joint between scapula and chest wall, crucial for overall shoulder movement.
  • Key Muscles & Tendons:
    • Rotator Cuff (SITS muscles): Primary dynamic stabilisers and rotators of GHJ.
      • Supraspinatus: Abduction (initiates), external rotation.
      • Infraspinatus: External rotation.
      • Teres Minor: External rotation.
      • Subscapularis: Internal rotation.
    • Deltoid: Powerful abductor (after supraspinatus initiation), flexor, extensor.
    • Biceps Brachii (Long Head): Runs in bicipital groove, attaches to superior labrum. Assists flexion, supination. Can be a pain source.
    • Trapezius, Serratus Anterior, Rhomboids, Pectoralis Major: Also contribute to shoulder girdle movement and stability.
  • Other Structures:
    • Subacromial Bursa: Lies between acromion and supraspinatus tendon. Reduces friction. Can become inflamed (bursitis).
    • Glenoid Labrum: Fibrocartilaginous rim around glenoid, deepens socket, attachment site for ligaments and long head of biceps.
    • Brachial Plexus & Axillary Artery/Vein: Pass near the shoulder, important to consider in trauma or with neurological/vascular symptoms.
💡 Clinical Relevance

Pain location often gives clues: Anterior pain (biceps, subscapularis), Anterosuperior/Lateral pain (ACJ, subacromial space – supraspinatus/bursa), Posterior pain (infraspinatus, teres minor, posterior capsule). However, pain can be diffuse or referred.

🧠 Rotator Cuff Muscles: “SITS”
S Supraspinatus (Abduction)
I Infraspinatus (External Rotation)
T Teres Minor (External Rotation)
S Subscapularis (Internal Rotation)

History Taking for Shoulder Pain

A detailed history is crucial for narrowing down the differential diagnosis.

Key Areas to Cover (SOCRATES + Specifics):
  • Site: Precise location of pain. Ask patient to point (“One finger test”). Is it localised or diffuse? Does it radiate?
    • e.g., Lateral deltoid area (rotator cuff, subacromial), superior (ACJ), anterior (biceps).
  • Onset: Sudden (trauma, acute tear) or gradual/insidious (degenerative, overuse, inflammatory)?
  • Character: Sharp, dull, aching, burning, throbbing?
  • Radiation:
    • Down lateral arm (C5 dermatome – common in rotator cuff/subacromial pain, referred pain from neck).
    • To neck, scapula, or chest.
  • Associated Symptoms:
    • Weakness: True weakness (tear, nerve issue) vs. pain inhibition.
    • Stiffness: Difficulty moving (e.g., frozen shoulder, OA). Morning stiffness?
    • Clicking, Clunking, Popping: Instability, labral tear, OA.
    • Locking/Catching: Loose body, labral tear.
    • Numbness/Paraesthesia: Cervical radiculopathy, brachial plexus issue, peripheral nerve entrapment.
    • Swelling, Bruising, Deformity.
    • Systemic symptoms: Fever, weight loss, malaise (red flags).
  • Timing/Pattern:
    • Night Pain: Common in rotator cuff pathology, frozen shoulder, OA, malignancy. Difficulty lying on affected side?
    • Worse with specific activities (e.g., overhead, reaching behind back)?
    • Constant or intermittent?
  • Exacerbating/Relieving Factors: What makes it worse/better? Rest vs. activity. Specific movements. Analgesia.
  • Severity: Pain score (0-10). Impact on daily activities (ADLs), work, sleep, sport.
  • Mechanism of Injury (if traumatic): Fall (FOOSH?), direct blow, forceful movement (e.g., throwing, lifting). Position of arm during injury.
  • Past Medical History: Previous shoulder problems/surgery. Diabetes, thyroid disease (risk for frozen shoulder). Inflammatory conditions (RA, polymyalgia rheumatica). History of cancer.
  • Drug History: Analgesics, steroids (oral/injected), anticoagulants.
  • Occupation & Hobbies/Sports: Repetitive overhead activities, heavy lifting, throwing sports. Hand dominance.
  • Patient’s Ideas, Concerns, and Expectations (ICE).
Don’t Forget the Neck!

Cervical spine pathology (e.g., radiculopathy, referred pain from facet joints) is a common mimic of shoulder pain. Always ask about neck pain, stiffness, and radicular arm symptoms. Consider a brief neck screen if suspicion high.

Physical Examination: Look, Feel, Move

A systematic examination is essential. Always compare with the contralateral (unaffected) side. Ensure adequate exposure of both shoulders (patient ideally topless or in a vest top).

1. Look (Inspection):

Inspect from anterior, lateral, and posterior aspects. Observe patient walking in and undressing.

  • Symmetry: Compare both shoulders.
  • Swelling: Localised (e.g., ACJ, bursa) or diffuse.
  • Erythema, Rashes, Skin Changes.
  • Scars: Previous surgery or trauma.
  • Deformity:
    • Clavicle fracture, ACJ separation (“step deformity”).
    • Anterior shoulder dislocation (squared-off appearance, prominent acromion, anterior bulge).
    • Scapular position and symmetry.
  • Muscle Wasting:
    • Supraspinatus/Infraspinatus: Wasting in fossae above/below scapular spine (suprascapular nerve palsy, chronic rotator cuff tear).
    • Deltoid: Axillary nerve injury.
  • “Popeye” Sign: Bulge in lower anterior arm (rupture of long head of biceps).
  • Scapular Winging: Medial (serratus anterior/long thoracic nerve) or lateral (trapezius/spinal accessory nerve). Ask patient to push against wall or do press-up.
2. Feel (Palpation):

Palpate systematically for tenderness, warmth, swelling, crepitus.

  • Bony Landmarks:
    • Sternoclavicular joint (SCJ).
    • Clavicle (full length).
    • Acromioclavicular joint (ACJ) – often tender in ACJ pathology.
    • Acromion (anterolateral edge – subacromial tenderness).
    • Coracoid process.
    • Spine of scapula.
    • Greater tuberosity of humerus (attachment of supraspinatus, infraspinatus, teres minor).
    • Lesser tuberosity (attachment of subscapularis).
    • Bicipital groove (between tuberosities, for long head of biceps tendon).
  • Soft Tissues:
    • Rotator cuff tendons (especially supraspinatus insertion).
    • Biceps tendon in its groove.
    • Deltoid muscle.
    • Trapezius and periscapular muscles.
    • Axilla (for lymph nodes if relevant).
  • Assess temperature.
3. Move (Range of Motion – ROM):

Assess active ROM first, then passive ROM if active is restricted. Note pain, crepitus, and quality of movement. Isolate glenohumeral movement by stabilising scapula if needed.

  • Active ROM:
    • Forward Flexion: (Normal ~180°) “Raise your arms forward as high as you can.”
    • Extension: (Normal ~50-60°) “Reach your arms backwards.”
    • Abduction: (Normal ~180°) “Raise your arms out to the sides.” Observe for painful arc (typically 60-120° in subacromial impingement/RC pathology). Note scapulohumeral rhythm.
    • Adduction: (Normal ~30-45°) “Bring your arm across your body.”
    • External Rotation (ER):
      • Arms by side, elbows flexed to 90°: “Turn your forearms outwards.” (Normal ~70-90°)
      • Arms abducted to 90° (if possible): “Keep your elbows high and rotate your hands backwards.” (Normal ~90°)
    • Internal Rotation (IR):
      • “Reach up your back as far as you can.” Record highest vertebral level reached (e.g., T7, L3, buttock). (Combined adduction, IR, extension). This is Apley’s Scratch Test component.
      • Can also test with arms abducted to 90° (if possible): “Rotate your hands forwards/downwards.”
  • Passive ROM:
    • Gently take the patient’s arm through the same movements if active ROM is limited.
    • Assess if passive ROM is greater than active (suggests weakness or pain inhibition if full passive; suggests true stiffness/contracture if passive also limited e.g. frozen shoulder, OA).
    • Note end-feel (e.g., bony block, capsular tightness, muscle spasm).
  • Resisted ROM (Isometric Muscle Testing):
    • Assess power (MRC grade 0-5) and pain for key muscle groups. Stabilise proximally.
    • Supraspinatus (Abduction): Jobe’s test position (see Special Tests).
    • Infraspinatus/Teres Minor (External Rotation): Patient’s arm by side, elbow 90° flexed, resist ER.
    • Subscapularis (Internal Rotation): Gerber’s lift-off or Belly press (see Special Tests).
    • Deltoid (Abduction): Resist abduction with arm at 90°.
    • Biceps (Flexion/Supination): Resist elbow flexion and forearm supination.
  • Scapular Dyskinesis Assessment: Observe scapular movement during arm elevation/lowering. Look for abnormal winging, shrugging, or lack of smooth coordinated motion.
📐 Painful Arc

Pain experienced during active abduction, typically between 60° and 120°, is suggestive of subacromial impingement or rotator cuff tendinopathy/tear. Pain at the very end of abduction (>160°) may suggest ACJ pathology.

Special Tests: Impingement & Rotator Cuff

These provocative tests aim to reproduce the patient’s pain and help identify specific pathologies. Sensitivity and specificity vary; interpret in context of full assessment. Perform gently and warn patient about potential discomfort.

Subacromial Impingement Tests:
  • Neer’s Test:
    • Procedure: Examiner passively flexes the patient’s internally rotated arm maximally (stabilise scapula).
    • Positive: Pain reproduced in anterior/lateral shoulder (impingement of supraspinatus tendon/bursa under acromion).
  • Hawkins-Kennedy Test:
    • Procedure: Patient’s arm flexed to 90°, elbow flexed to 90°. Examiner passively internally rotates the arm.
    • Positive: Pain reproduced (impingement of supraspinatus tendon against coracoacromial ligament).
Rotator Cuff Integrity Tests:
  • Supraspinatus Tests:
    • Jobe’s Test (Empty Can Test):
      • Procedure: Arm abducted to 90° in scapular plane (~30° anterior to coronal), thumb pointing down (full internal rotation). Examiner applies downward pressure.
      • Positive: Pain and/or weakness (supraspinatus tendinopathy/tear).
    • Drop Arm Test:
      • Procedure: Passively abduct patient’s arm to 90°. Ask them to slowly lower it.
      • Positive: Arm drops uncontrollably or patient has significant difficulty lowering it slowly (suggests large supraspinatus tear).
  • Infraspinatus & Teres Minor Tests (External Rotation):
    • Resisted External Rotation:
      • Procedure: Arm by side, elbow 90° flexed. Patient attempts to externally rotate against resistance.
      • Positive: Pain and/or weakness.
    • External Rotation Lag Sign:
      • Procedure: Passively place arm in near maximal external rotation (elbow 90° flexed, by side or 90° abducted). Ask patient to hold.
      • Positive: Arm “lags” or drops into internal rotation (significant infraspinatus/teres minor tear).
  • Subscapularis Tests (Internal Rotation):
    • Gerber’s Lift-Off Test:
      • Procedure: Patient places dorsum of hand on their lower back. Asks to lift hand away from back.
      • Positive: Inability to lift hand off back (subscapularis tear). If cannot get hand there, proceed to belly press.
    • Belly Press Test (Napoleon Sign):
      • Procedure: Patient places hand on their abdomen and presses inwards. Elbow should remain anterior.
      • Positive: Weakness, or if elbow drops backwards/wrist flexes to compensate (subscapularis tear).
    • Internal Rotation Lag Sign:
      • Procedure: Examiner passively lifts hand off back (as in Gerber’s) and asks patient to hold it there.
      • Positive: Hand drops back onto back (subscapularis tear).
⚠️ Interpreting Special Tests

No single special test is perfectly diagnostic. Cluster tests for a particular pathology. Positive tests increase likelihood, negative tests decrease it, but always correlate with history and other examination findings.

Special Tests: ACJ, Biceps Tendon & Instability

Further tests to assess other common sources of shoulder pain and dysfunction.

Acromioclavicular Joint (ACJ) Tests:
  • Direct Palpation of ACJ: Tenderness directly over the ACJ is a key sign.
  • Scarf Test (Cross-Body Adduction Test):
    • Procedure: Patient’s arm flexed to 90°, elbow flexed. Examiner passively adducts the arm across the patient’s chest.
    • Positive: Pain localised to the ACJ.
  • O’Brien’s Test (Active Compression Test – can also indicate SLAP lesion):
    • Part 1: Arm flexed to 90°, adducted 10-15°, fully internally rotated (thumb down). Examiner applies downward pressure.
    • Part 2: Arm in same position but fully externally rotated (thumb up). Examiner applies downward pressure.
    • Positive for ACJ: Pain localised to ACJ in Part 1, reduced/absent in Part 2. (If pain is deep/diffuse, more suggestive of labral issue).
Biceps Tendon Tests (Long Head):
  • Speed’s Test:
    • Procedure: Patient’s arm flexed forward to ~60-90°, elbow extended, forearm supinated. Examiner resists further forward flexion.
    • Positive: Pain in the bicipital groove (biceps tendinopathy or SLAP lesion).
  • Yergason’s Test:
    • Procedure: Patient’s elbow flexed to 90°, forearm pronated. Examiner holds patient’s wrist and resists active supination and external rotation of the shoulder.
    • Positive: Pain in bicipital groove (biceps tendinopathy, instability of biceps tendon in groove). Some find this difficult to elicit.
  • Palpation of bicipital groove for tenderness.
Glenohumeral Instability Tests:

Perform with caution, especially if history of dislocation. Stop if patient is apprehensive or pain is severe.

  • Anterior Apprehension Test:
    • Procedure: Patient supine or sitting. Shoulder abducted to 90°, elbow flexed 90°. Examiner gently externally rotates the arm.
    • Positive: Patient expresses apprehension (fear of dislocation) or pain. (Tests anterior instability).
  • Relocation Test (Jobe’s Relocation Test):
    • Procedure: If apprehension test is positive, apply posteriorly directed pressure to the anterior humeral head while maintaining external rotation.
    • Positive: Relief of apprehension or pain (confirms anterior instability).
  • Sulcus Sign:
    • Procedure: Patient sitting or standing with arm relaxed by side. Examiner applies downward traction on the arm.
    • Positive: Appearance of a sulcus (indentation) below the acromion (indicates inferior instability / multidirectional instability). Grade by size of sulcus.
  • Posterior instability tests (e.g., Posterior Apprehension Test) are less commonly performed in primary care screening.
Labral Tear Tests (e.g., SLAP Lesion – Superior Labrum Anterior to Posterior):
  • O’Brien’s Test (Active Compression Test) – as above:
    • Positive for SLAP lesion: Deep, diffuse pain within the shoulder in Part 1 (thumb down), reduced/absent in Part 2 (thumb up). Clicking may be present.
  • Other labral tests (e.g., Crank test, Biceps Load test) are more for specialist settings.

Common Shoulder Conditions & Differential Diagnosis

Based on history and examination, a working diagnosis can often be formed.

Common Shoulder Pathologies:
Condition
Key Features & Typical Exam Findings
Rotator Cuff Tendinopathy / Subacromial Pain Syndrome (Impingement)
Gradual onset, lateral deltoid pain, painful arc (60-120°), pain on overhead activities, night pain. Positive Neer’s/Hawkins-Kennedy. Weakness/pain on resisted RC testing (esp. supraspinatus/Jobe’s).
Rotator Cuff Tear (Partial or Full Thickness)
Can be acute (trauma) or chronic (degenerative). Similar to tendinopathy but often more significant weakness (esp. on Jobe’s, Drop Arm, lag signs for ER/IR). Night pain common. May have history of recurrent tendinopathy.
Adhesive Capsulitis (Frozen Shoulder)
Insidious onset. Three phases: Freezing (painful, progressive stiffness), Frozen (stiffness predominates, less pain), Thawing (gradual return of motion). Global restriction of BOTH active AND passive ROM, especially external rotation and abduction. Often associated with diabetes, thyroid disease.
Acromioclavicular Joint (ACJ) Pathology (OA, Sprain/Separation)
Pain localised to top of shoulder (ACJ). Tenderness on ACJ palpation. Positive Scarf test. Pain at end-range abduction. History of fall onto shoulder (sprain/separation). Step deformity if significant separation.
Glenohumeral Joint Osteoarthritis (OA)
Gradual onset, deep aching pain, stiffness, crepitus. Global reduction in ROM (passive often similarly restricted to active, but ER may not be as bad as frozen shoulder initially). Pain worse with activity, may have night pain. X-ray shows joint space narrowing, osteophytes.
Biceps Tendinopathy / Rupture (Long Head)
Anterior shoulder pain, tenderness in bicipital groove. Positive Speed’s/Yergason’s. If rupture: sudden pain, “Popeye” deformity, often relief of chronic tendinopathy pain.
Shoulder Instability / Dislocation
History of dislocation or subluxation. Feeling of shoulder “giving way.” Apprehension with certain movements. Positive apprehension/relocation tests, sulcus sign. Young, active individuals common.
SLAP Lesion (Superior Labrum Anterior to Posterior)
Often history of trauma (FOOSH, traction) or repetitive overhead activity. Deep, vague shoulder pain, clicking, catching. Pain with overhead movements. Positive O’Brien’s (deep pain), Speed’s. Diagnosis often needs MRI arthrogram.
Calcific Tendinitis
Acute, severe pain, often sudden onset, may awaken from sleep. Markedly restricted ROM due to pain. X-ray may show calcium deposits (often supraspinatus). Chronic form can mimic impingement.
Important Differential Diagnoses (Pain Referred to Shoulder):
  • Cervical Spine Disease: Radiculopathy (dermatomal pain, paraesthesia, weakness, reflex changes), facet joint arthropathy, myofascial neck pain. Always consider C-spine screen.
  • Myocardial Ischaemia/Infarction: Especially left shoulder/arm pain. Consider with cardiac risk factors, exertional pain, associated symptoms (chest pain, SOB, sweating).
  • Diaphragmatic Irritation: (e.g., gallbladder disease, subphrenic abscess) – pain referred to shoulder tip (Kehr’s sign).
  • Polymyalgia Rheumatica (PMR): Bilateral shoulder/hip girdle pain and stiffness, morning stiffness >45 mins, age >50, raised ESR/CRP. Responds dramatically to steroids.
  • Malignancy: (e.g., Pancoast tumour, bony metastases) – consider with red flags.
  • Peripheral nerve entrapment (e.g., suprascapular nerve).

Red Flags & Investigations in Shoulder Pain

Recognizing red flags is crucial to identify serious underlying pathology. Investigations are guided by clinical suspicion.

Red Flags in Shoulder Pain:
  • History of Significant Trauma: Suspect fracture (humerus, clavicle, scapula) or acute large rotator cuff tear / dislocation. Urgent assessment/imaging.
  • Unexplained Deformity or Swelling.
  • Suspected Infection (Septic Arthritis/Osteomyelitis): Fever, chills, systemic illness, hot/red/swollen joint, severe pain with minimal movement. Urgent hospital admission.
  • Suspected Malignancy (Primary or Metastatic):
    • History of cancer.
    • Unexplained weight loss, night sweats, persistent severe night pain not relieved by position change.
    • Age >50 with new onset, unremitting pain.
    • Palpable mass.
  • Unexplained Neurological Deficit: Significant, progressive weakness or sensory loss not fitting a simple nerve root or peripheral nerve pattern, or suggesting brachial plexopathy / cord lesion.
  • Vascular Compromise: Absent pulses, pallor, coldness of limb (rare, but consider in trauma or thoracic outlet syndrome).
  • Systemic Inflammatory Condition: e.g., suspicion of PMR, RA with acute flare.
  • Inability to Actively Move Arm (especially after trauma).
  • Pain that is Unremitting, Severe, and Worsening Despite Conservative Measures.
Investigations:

Not always needed for common conditions if diagnosis clear clinically. Indicated if red flags, diagnostic uncertainty, failure to respond to conservative treatment, or pre-operative planning.

  • Bloods:
    • FBC, ESR, CRP: If infection or inflammatory condition (e.g., PMR, RA) suspected.
    • U&E, LFTs, Bone profile (Calcium, ALP): If malignancy or metabolic bone disease suspected.
    • Rheumatoid Factor, Anti-CCP: If RA suspected.
  • X-rays:
    • Standard views: AP (anteroposterior), axillary lateral, and/or Grashey view (true AP of glenohumeral joint). Outlet view (Y-scapular view) can show acromial morphology.
    • Indications: Trauma (fracture/dislocation), suspected OA, calcific tendinitis, instability (e.g., Hill-Sachs, Bankart lesions), assessing acromial morphology, ruling out gross bony pathology. Often first-line imaging.
  • Ultrasound (USS):
    • Indications: Suspected rotator cuff tears (good for full-thickness, less for partial), tendinopathy, bursitis, biceps tendon pathology, dynamic assessment of impingement.
    • Advantages: Non-invasive, no radiation, dynamic, can guide injections.
    • Disadvantages: Operator dependent, limited view of intra-articular structures (e.g., labrum) and bone.
  • Magnetic Resonance Imaging (MRI):
    • Indications: Detailed assessment of soft tissues (rotator cuff, labrum, ligaments, cartilage, capsule), bone marrow oedema, occult fractures, tumours. Used when diagnosis unclear after X-ray/USS, or for pre-operative planning.
    • MRI Arthrography (MRA): MRI with intra-articular contrast. Best for labral tears (e.g., SLAP lesions) and subtle instability.
  • CT Scan: Less common for routine shoulder pain. Used for complex fractures, bony detail, pre-op planning for joint replacement if MRI contraindicated.
  • Nerve Conduction Studies / EMG: If neurological cause suspected (e.g., peripheral nerve entrapment, radiculopathy, plexopathy).
Act on Red Flags!

If red flags are present, ensure timely investigation and/or referral to the appropriate specialist (e.g., Orthopaedics, Rheumatology, Oncology, A&E if acute emergency like suspected septic arthritis or major trauma).

OSCE Tips & Examination Summary for Shoulder Pain

General OSCE Approach (WIPER + Structured Exam):
  • Wash hands, Introduce self, Patient ID & consent, Expose adequately (both shoulders, remove top), Reposition (standing/sitting as needed).
  • “I’ve been asked to examine your shoulder today. This will involve me looking at your shoulder, feeling around the area, and asking you to do some movements. I’ll also do some special tests. Please let me know if you feel any pain at any point.”
  • Always compare with the asymptomatic side.
  • Start with observation while patient walks in and undresses if possible.
  • A logical sequence: Look → Feel → Move (Active, then Passive, then Resisted) → Special Tests.
  • Don’t forget to screen the neck and elbow if relevant/time permits.
  • Verbalise your findings clearly.
  • Summarise positive findings and offer a differential diagnosis at the end if asked.
Key Steps in Shoulder Examination Sequence:
  1. Inspection (Anterior, Lateral, Posterior): Scars, swelling, erythema, deformity, muscle wasting (supraspinatus, infraspinatus, deltoid), symmetry, scapular position. Popeye sign.
  2. Palpation:
    • Bony: SCJ, clavicle, ACJ, acromion, coracoid, greater/lesser tuberosities, bicipital groove.
    • Soft tissue: Rotator cuff insertion points, biceps tendon.
  3. Active Range of Motion:
    • Forward Flexion, Extension.
    • Abduction (note painful arc), Adduction.
    • External Rotation (arm by side, and 90° abduction).
    • Internal Rotation (hand up back – Apley’s scratch test component).
  4. Passive Range of Motion (if active limited):
    • Gently assess same movements, note end-feel and if greater than active.
  5. Resisted Range of Motion (Isometric):
    • Supraspinatus (Jobe’s position).
    • Infraspinatus/Teres Minor (ER by side).
    • Subscapularis (Gerber’s/Belly Press).
    • Deltoid (Abduction at 90°).
    • Biceps (Elbow flexion/supination).
  6. Special Tests (Select based on history and findings):
    • Impingement: Neer’s, Hawkins-Kennedy.
    • Rotator Cuff: Jobe’s, Drop Arm, Resisted ER, ER Lag, Gerber’s/Belly Press, IR Lag.
    • ACJ: Scarf test, O’Brien’s (for ACJ).
    • Biceps: Speed’s, Yergason’s.
    • Instability: Apprehension, Relocation, Sulcus sign.
    • (SLAP: O’Brien’s – deep pain)
  7. Neurovascular Assessment (if indicated): Peripheral pulses, sensation (axillary nerve for deltoid patch), motor function.
  8. Cervical Spine Screen (if indicated): ROM, Spurling’s test.
Summarising Findings:

“On examination of Mr./Ms. [Patient Name]’s [left/right] shoulder, key findings include [e.g., wasting of the supraspinatus fossa, tenderness over the ACJ, a painful arc on abduction, positive Jobe’s test with weakness, restricted external rotation both actively and passively…]. These findings are suggestive of [top differential, e.g., rotator cuff pathology, likely a supraspinatus tear, or adhesive capsulitis]. I would also consider [other differentials]. To complete my assessment, I would [e.g., perform a neurovascular assessment of the limb, examine the cervical spine, consider appropriate investigations like an X-ray or ultrasound].”

🎯 Focus on Positive Findings

While it’s good to mention normal findings briefly, spend more time highlighting and interpreting the abnormal findings and how they lead to your differential diagnoses. Practice your summary to be concise and structured.

Flashcards: Shoulder Pain Assessment

Click on each card to reveal the answer.

What are the 4 muscles of the rotator cuff (SITS)?

(Click to flip)

Answer

Supraspinatus
Infraspinatus
Teres Minor
Subscapularis

Name two special tests for subacromial impingement.

(Click to flip)

Answer

1. Neer’s Test
2. Hawkins-Kennedy Test

A “painful arc” during abduction, typically between 60-120°, suggests what pathology?

(Click to flip)

Answer

Subacromial impingement or rotator cuff tendinopathy/tear.

What is the Jobe’s Test (Empty Can Test) primarily assessing?

(Click to flip)

Answer

Supraspinatus muscle strength and integrity.

Global restriction of BOTH active and passive range of motion, especially external rotation, is characteristic of what condition?

(Click to flip)

Answer

Adhesive Capsulitis (Frozen Shoulder).

What does a positive “Popeye” sign indicate?

(Click to flip)

Answer

Rupture of the long head of the biceps tendon.

Name a red flag symptom in a patient presenting with shoulder pain that might suggest malignancy.

(Click to flip)

Answer

Unexplained weight loss, persistent severe night pain not relieved by position, history of cancer. (Any one of these or similar)

What is the Scarf Test (Cross-Body Adduction Test) used to assess?

(Click to flip)

Answer

Acromioclavicular Joint (ACJ) pathology.

Shoulder Pain Assessment Quiz

Test your knowledge on assessing shoulder pain.

1. A 55-year-old patient describes pain in the lateral deltoid region, worse with overhead activities and at night, particularly when lying on the affected side. They also report a painful arc between 70-120° of abduction. Which of the following is the most likely diagnosis?

A. ACJ Osteoarthritis.
B. Rotator Cuff Tendinopathy / Subacromial Pain Syndrome.
C. Adhesive Capsulitis.
D. Biceps Tendinopathy.
Explanation: Lateral deltoid pain, pain with overhead activities, night pain, and a painful arc are classic features of rotator cuff tendinopathy or subacromial pain syndrome (impingement).

2. Which special test involves passively flexing the patient’s internally rotated arm and is positive if it reproduces anterior/lateral shoulder pain?

A. Neer’s Test.
B. Hawkins-Kennedy Test.
C. Jobe’s Test.
D. Speed’s Test.
Explanation: Neer’s test is performed by passively flexing the internally rotated arm to assess for subacromial impingement.

3. A patient is unable to actively lift their hand off their lower back during Gerber’s Lift-Off test. This suggests a potential tear or dysfunction of which rotator cuff muscle?

A. Supraspinatus.
B. Infraspinatus.
C. Subscapularis.
D. Teres Minor.
Explanation: Gerber’s Lift-Off test specifically assesses the integrity and function of the Subscapularis muscle, which is responsible for internal rotation.

4. A 60-year-old diabetic patient presents with a 3-month history of insidious onset shoulder pain and progressive stiffness. Examination reveals significantly restricted active and passive external rotation and abduction. What is the most likely diagnosis?

A. Glenohumeral Osteoarthritis.
B. Rotator Cuff Tear.
C. Adhesive Capsulitis (Frozen Shoulder).
D. Calcific Tendinitis.
Explanation: Global restriction of both active and passive ROM, particularly external rotation, in a diabetic patient is highly characteristic of adhesive capsulitis.

5. Which of the following is a RED FLAG symptom when assessing shoulder pain?

A. Clicking sensation with movement.
B. Pain radiating to the lateral deltoid.
C. Unexplained weight loss and persistent night pain not relieved by position.
D. Morning stiffness lasting 15 minutes.
Explanation: Unexplained weight loss and persistent, severe night pain are red flags that could indicate malignancy or other serious systemic pathology. The other options are common symptoms of benign shoulder conditions.

6. The “Scarf test” or “Cross-body adduction test” is primarily used to assess pathology in which joint?

A. Glenohumeral joint.
B. Acromioclavicular joint (ACJ).
C. Sternoclavicular joint (SCJ).
D. Scapulothoracic articulation.
Explanation: The Scarf test stresses the ACJ, and pain localised to this joint on the manoeuvre is indicative of ACJ pathology.

7. A young athlete reports a feeling of their shoulder “giving way” or “slipping out” during throwing motions, and they are apprehensive when their arm is placed in abduction and external rotation. Which test would be positive?

A. Sulcus Sign.
B. Yergason’s Test.
C. Anterior Apprehension Test.
D. Drop Arm Test.
Explanation: The history and apprehension in abduction/external rotation are classic for anterior glenohumeral instability, which is assessed by the Anterior Apprehension Test.

8. Wasting in the supraspinatus and infraspinatus fossae of the scapula might suggest injury to which nerve, or a chronic tear of which muscles?

A. Axillary nerve.
B. Suprascapular nerve (or chronic supraspinatus/infraspinatus tears).
C. Long thoracic nerve.
D. Musculocutaneous nerve.
Explanation: The suprascapular nerve innervates both the supraspinatus and infraspinatus muscles. Chronic, large tears of these muscles can also lead to disuse atrophy.

9. What is the first-line imaging modality for suspected rotator cuff tears in many pathways, offering dynamic assessment without radiation?

A. X-ray.
B. MRI.
C. Ultrasound (USS).
D. CT Scan.
Explanation: Ultrasound is often used as a first-line investigation for suspected rotator cuff pathology due to its accessibility, dynamic capability, and lack of radiation. X-rays are good for bone/OA, MRI for detailed soft tissue/pre-op.

10. Referred pain to the shoulder tip (Kehr’s sign) can be caused by irritation of the:

A. Cervical spine nerve roots C5/C6.
B. Diaphragm.
C. Brachial plexus.
D. Heart.
Explanation: Kehr’s sign refers to acute pain in the tip of the shoulder due to the presence of blood or other irritants in the peritoneal cavity when a person is lying down and the legs are elevated, irritating the phrenic nerve which innervates the diaphragm (and shares nerve roots C3, C4, C5 with shoulder sensation).

Trauma & Orthopaedics: Open Fractures Management

An open fracture (or compound fracture) is defined as a fracture that communicates with the external environment, or a fracture with an overlying skin wound where the bone has been exposed. They are orthopaedic emergencies due to the high risk of infection and complications. Prompt and systematic management is crucial to optimise outcomes.

Overview & Key Definitions

Open fractures are serious injuries with significant implications for limb viability and patient morbidity. The break in the skin allows bacterial contamination of the fracture site, leading to a high risk of infection.

Key Definitions:

  • Open Fracture: A fracture in which there is a break in the overlying skin and soft tissues, leading to direct communication between the fracture/fracture haematoma and the external environment.
  • Contamination: All open fractures are considered contaminated. The degree of contamination influences management and prognosis.
  • Debridement: Surgical removal of all contaminated, devitalized, and foreign material from the wound and fracture site. This is the most crucial step in preventing infection.
  • Gustilo-Anderson Classification: The most widely used system for classifying open fractures, based on mechanism, soft tissue damage, contamination, and fracture pattern. Guides treatment and prognosis.

Significance:

  • Orthopaedic emergency requiring urgent intervention.
  • High risk of infection (superficial, deep, osteomyelitis).
  • Potential for neurovascular injury.
  • Increased risk of non-union, malunion, and delayed union.
  • May require multiple surgeries and prolonged rehabilitation.
  • Risk of amputation in severe cases.
💡 OSCE/PLAB Tip

In an open fracture scenario, immediate priorities are ATLS (if major trauma), haemorrhage control, gross alignment/splinting, sterile dressing, IV antibiotics, and tetanus prophylaxis. Early surgical debridement is critical (“Time is tissue”).

🧠 Open Fracture Management Principles: “WASHED B”
W Wound care (photos, sterile dressing)
A Antibiotics (IV, broad-spectrum, early)
S Stabilization (splint initially, then surgical)
H Haemorrhage control
E Early surgical debridement & irrigation
D Definitive soft tissue & bone reconstruction
B BOAST guidelines / BAHT guidelines (UK specific – refers to national standards) / Tetanus Booster

Pathophysiology & Key Risks

The communication with the external environment introduces several critical challenges.

Pathophysiological Consequences:
  • Bacterial Contamination: The skin barrier is breached, allowing bacteria from the environment, skin flora, or penetrating object to access the fracture site and bone. This is the primary driver of infection risk.
  • Soft Tissue Damage: The energy required to cause an open fracture often results in significant damage to surrounding muscles, tendons, nerves, and blood vessels. This devitalized tissue provides a nidus for infection and impairs healing.
  • Disruption of Blood Supply: Both to the bone fragments (periosteal stripping, endosteal damage) and the overlying soft tissues. This compromises healing potential and immune response.
  • Inflammatory Response: Trauma and contamination trigger a significant local and systemic inflammatory response.
  • Fracture Haematoma Exposure: The fracture haematoma, rich in growth factors essential for healing, is exposed and potentially lost or contaminated.
Key Risks:
  • Infection:
    • Superficial wound infection.
    • Deep infection / Osteomyelitis: Infection of the bone itself. Can be very difficult to eradicate and may lead to chronic problems, non-union, and need for amputation. Risk increases with severity of soft tissue injury (Gustilo grade).
    • Gas gangrene (Clostridial myonecrosis): Rare but life-threatening, associated with severe contamination (e.g., agricultural injuries).
  • Neurovascular Injury: Direct trauma to nerves and vessels, or secondary compromise from swelling/compartment syndrome. Requires careful assessment and prompt management. Gustilo IIIC fractures involve arterial injury requiring repair.
  • Compartment Syndrome: Increased pressure within a fascial compartment, compromising circulation and function. High risk in tibial and forearm fractures. Clinical diagnosis (pain out of proportion, pain on passive stretch, tense compartment).
  • Delayed Union / Non-Union / Malunion: Impaired healing due to infection, poor blood supply, instability, or significant bone loss.
  • Soft Tissue Complications: Wound breakdown, need for complex soft tissue reconstruction (flaps).
  • Systemic Complications: Sepsis, DVT/PE, fat embolism (especially with long bone fractures).
  • Chronic Pain & Functional Impairment.
  • Amputation: May be necessary for severe, unsalvageable injuries or intractable infection.

Initial Assessment (ATLS Principles)

Open fractures, especially from high-energy trauma, often occur in polytrauma patients. A systematic ATLS approach is paramount.

Primary Survey (ABCDE):
  • A – Airway with C-spine protection: Ensure patent airway. If trauma, maintain c-spine immobilisation until cleared.
  • B – Breathing and Ventilation: Assess respiratory rate, effort, oxygen saturation, chest expansion, air entry. Administer high-flow oxygen. Manage life-threatening chest injuries (e.g., tension pneumothorax).
  • C – Circulation with Haemorrhage Control:
    • Assess pulse, blood pressure, capillary refill, level of consciousness. Look for signs of shock.
    • Control external haemorrhage from the open fracture: Direct pressure, elevation. Tourniquets may be needed for life-threatening bleeding (note time of application). Avoid blind clamping.
    • Establish IV access (2 large-bore cannulae). Take bloods (FBC, U&E, Coag, Group & Save/Crossmatch). Start IV fluids if shocked.
  • D – Disability (Neurological Status): Glasgow Coma Scale (GCS), pupil size/reactivity, gross motor/sensory function in all limbs.
  • E – Exposure and Environment: Fully expose patient to assess for other injuries, preventing hypothermia.
Secondary Survey (after primary survey and resuscitation):

Focus on the injured limb once life-threatening conditions are addressed/managed.

  • History (AMPLE): Allergies, Medications, Past medical history, Last meal, Events/Environment related to injury. Mechanism of injury is crucial (high vs. low energy, type of force).
  • Detailed Examination of the Injured Limb:
    • Look: Wound size, location, contamination (dirt, foreign bodies), skin viability, deformity, swelling, bruising. Take photographs if possible before dressing, for documentation and communication.
    • Feel: Palpate for tenderness, crepitus, compartment tension. Assess peripheral pulses, capillary refill, skin temperature.
    • Move: Assess active/passive movement if possible (often limited by pain/instability). Document if movement not assessed due to pain.
    • Neurovascular Assessment: CRITICAL. Document pre-reduction and post-reduction/splinting.
      • Vascular: Pulses (distal to injury), capillary refill time, skin colour, temperature. Doppler USS if pulses impalpable. Consider Ankle-Brachial Pressure Index (ABPI).
      • Neurological: Sensation (light touch, pinprick in specific nerve distributions), motor function (specific muscle groups distal to injury). Identify specific nerve at risk based on fracture location.
Do Not Probe Wound in ED!

Avoid probing the wound or removing embedded foreign bodies in the Emergency Department unless there is ongoing vascular compromise directly related to it. This should be done in the controlled environment of the operating theatre during formal debridement. Do not attempt to close the wound in ED if grossly contaminated or if there’s significant soft tissue damage.

Emergency Management (Pre-hospital / Emergency Department)

Time-critical interventions to reduce complications and prepare for surgery.

Immediate Actions:
  1. ABCDE Assessment and Resuscitation: (As per previous section). Life-saving measures take priority.
  2. Control Haemorrhage:
    • Direct pressure is first-line.
    • Elevation of the limb.
    • If severe arterial bleeding unresponsive to direct pressure, a tourniquet may be applied (proximal to wound). Note time of application. Ensure it’s tight enough to stop arterial flow.
  3. Remove Gross Contaminants & Cover Wound:
    • Gently remove any obvious gross debris (e.g., large pieces of dirt, grass) from the surface of the wound without deep probing.
    • Cover the wound with a sterile, saline-soaked gauze dressing, then an occlusive dressing. This helps prevent further contamination and desiccation of tissues.
  4. Analgesia: Provide adequate pain relief (e.g., IV opioids like Morphine or Fentanyl). Consider regional nerve blocks if appropriate and expertise available.
  5. Realign & Splint:
    • If significant deformity or evidence of neurovascular compromise, attempt gentle realignment of the limb to a more anatomical position. Document neurovascular status before and after.
    • Immobilise the injured limb using an appropriate splint (e.g., backslab, traction splint for femur, collar & cuff for upper limb) to reduce pain, prevent further soft tissue damage, and protect neurovascular structures. Ensure splint allows for neurovascular checks.
  6. Photographs: Take clinical photographs of the wound (after gross decontamination, before dressing) if possible. Essential for documentation, communication with specialist teams, and medico-legal purposes.
  7. IV Antibiotics: Administer broad-spectrum IV antibiotics as soon as possible (ideally within 1 hour of injury or presentation). Choice depends on local guidelines and Gustilo-Anderson grade (see specific section).
  8. Tetanus Prophylaxis: Assess tetanus immunization status and administer booster and/or immunoglobulin as indicated (see specific section).
  9. Imaging:
    • X-rays of the entire affected bone (AP and lateral views), including joints above and below the fracture.
    • Consider imaging of other areas if polytrauma.
    • CT scan may be needed for complex intra-articular fractures or to assess vascular injury (CT angiogram).
  10. NBM (Nil By Mouth): In anticipation of urgent surgery.
  11. Urgent Orthopaedic Referral: All open fractures require urgent discussion with the on-call orthopaedic team for surgical debridement and management. Plastic surgery involvement may also be needed for severe soft tissue injuries.
“The Golden Hours”

While the concept of a strict “6-hour rule” for debridement has been debated, early and thorough surgical debridement, along with prompt antibiotic administration, remains paramount in reducing infection rates. Current BOAST guidelines recommend surgery within 12-24 hours, depending on severity and other patient factors, but as soon as feasible.

Gustilo-Anderson Classification of Open Fractures

This is the most widely used classification system, guiding treatment and predicting prognosis (especially infection risk). It’s based on wound size, degree of soft tissue injury, contamination, and fracture pattern.

Type
Wound Size
Soft Tissue Injury
Contamination
Fracture Pattern
Infection Risk (approx.)
Type I
< 1 cm
Clean, minimal muscle contusion
Minimal
Simple (transverse, short oblique)
0-2%
Type II
> 1 cm
Moderate, some muscle damage, no extensive stripping of periosteum, no flaps/avulsion
Moderate
Moderate comminution
2-7%
Type IIIA
> 10 cm (often) or high energy regardless of size
Extensive soft tissue laceration/damage, muscle devitalization, comminution. Adequate soft tissue coverage of bone is possible.
Severe (often high velocity trauma)
Severe comminution, segmental fracture
10-25%
Type IIIB
> 10 cm (often) or high energy regardless of size
Extensive soft tissue loss with periosteal stripping and bone exposure. Requires soft tissue flap coverage (plastic surgery). Massive contamination common.
Severe
Severe comminution, segmental fracture, often bone loss
25-50% (or higher)
Type IIIC
Any size
As above, PLUS major arterial injury requiring repair for limb salvage, regardless of degree of soft tissue injury.
Variable
Variable
High (often requires amputation if revascularisation fails or infection uncontrolled)
Important Considerations:
  • Classification is primarily made intra-operatively by the surgeon after debridement, as the full extent of injury may not be apparent externally.
  • High-energy injuries (e.g., gunshot, farm injuries, crush) are typically at least Type IIIA, even if the skin wound appears small.
  • Contamination with farm soil, sewage, or stagnant water significantly increases infection risk (consider Gram-negative and anaerobic cover).
  • The higher the grade, the higher the risk of infection, non-union, and amputation.

Surgical Principles: Debridement, Irrigation, Stabilisation, & Coverage

The goals of surgery are to prevent infection, achieve bone union, and restore function. This involves several key steps, often summarised by the “6 D’s” or similar frameworks.

The “6 D’s” of Open Fracture Management (common framework):
  1. Debridement (Excision):
    • This is the single most important step in preventing infection.
    • Involves meticulous surgical removal of all non-viable skin, subcutaneous tissue, muscle, and bone fragments. “Excise until it bleeds.”
    • Foreign material (dirt, clothing, debris) must be completely removed.
    • Wound edges are often excised back to healthy bleeding tissue.
    • Small, completely detached bone fragments without soft tissue attachment are usually removed as they are avascular and can act as a nidus for infection. Larger structural fragments may be retained if clean and viable.
  2. Decontamination (Irrigation):
    • Copious irrigation of the wound and fracture site with large volumes of sterile fluid (typically normal saline).
    • Aims to mechanically remove bacteria, debris, and reduce contaminant load.
    • Historically, high-pressure pulsatile lavage was used, but evidence now suggests low-pressure lavage is equally effective and potentially less damaging to tissues.
    • Volume of irrigation fluid often cited: e.g., 3L for Type I, 6L for Type II, 9L+ for Type III (though evidence for specific volumes is debated; thoroughness is key).
  3. Decision on Bone (Stabilisation):
    • Fracture reduction and stabilisation are essential for soft tissue healing, bone union, patient comfort, and mobilisation.
    • External Fixation: Often used for initial stabilisation in severe (Type IIIA/B/C) open fractures, especially with gross contamination or significant soft tissue injury. Allows access for wound care and further debridements. Can be converted to internal fixation later.
    • Internal Fixation:
      • Intramedullary Nailing (IMN): Preferred for many diaphyseal long bone fractures (e.g., tibia, femur) if soft tissue allows and contamination is manageable. Can be done at initial surgery or delayed.
      • Plates and Screws: Used for periarticular fractures or fractures not amenable to nailing. Risk of infection is higher if placed under compromised soft tissue.
    • The choice and timing of fixation depend on fracture type, severity, contamination, soft tissue status, and patient factors.
  4. Dead Space Management:
    • Large soft tissue or bone defects (“dead space”) can fill with haematoma and become a focus for infection.
    • May require antibiotic-impregnated cement beads/spacers, bone grafting (later), or muscle flaps to obliterate dead space.
  5. Definitive Soft Tissue Coverage (Closure):
    • Goal is to achieve durable soft tissue coverage over the fracture site as soon as possible after adequate debridement (usually within 5-7 days if staged).
    • Delayed Primary Closure: For less severe wounds (Type I, some Type II) if clean after initial debridement.
    • Skin Grafts (Split or Full Thickness): Require a vascularised bed.
    • Local or Free Flaps (Plastic Surgery): For larger defects or exposed bone/tendon/implants (Type IIIB). Involves transferring skin, subcutaneous tissue, fascia, and/or muscle with its blood supply.
    • Negative Pressure Wound Therapy (NPWT / VAC dressing): Can be used as a temporary measure to manage wounds, promote granulation tissue, and reduce oedema while awaiting definitive closure.
  6. Delayed Reconstruction / Rehabilitation: Bone grafting for defects, management of non-union, and intensive physiotherapy are often part of the longer-term plan.

Note: Some frameworks use slightly different “D” terms (e.g., Drugs for antibiotics), but the core principles of debridement, irrigation, stabilisation, and soft tissue coverage remain central.

Antibiotics & Tetanus Prophylaxis

These are critical adjuncts to surgical management in preventing infection.

Antibiotic Therapy:
  • Timing: Administer IV antibiotics ASAP after injury, ideally within 1 hour (or 3 hours at latest from injury to first dose). Pre-operative administration is crucial.
  • Duration:
    • Typically continued for 24-72 hours post-operatively for uncomplicated Type I and II fractures after adequate debridement and closure.
    • For Type III fractures, or if gross contamination, delayed closure, or multiple debridements, antibiotics are often continued for longer (e.g., 5-7 days or until definitive soft tissue coverage). Duration guided by microbiology, clinical picture, and local policies.
  • Choice of Antibiotics (General Guidelines – Always follow local hospital policy/microbiology advice):
    Gustilo-Anderson Type
    Recommended IV Antibiotic Cover
    Type I & II
    First-generation Cephalosporin (e.g., Cefazolin/Cefuroxime). If penicillin allergic: Clindamycin or Vancomycin.
    Type IIIA, IIIB, IIIC
    First-generation Cephalosporin PLUS an Aminoglycoside (e.g., Gentamicin) for Gram-negative cover.
    OR Broad-spectrum Beta-Lactam/Beta-Lactamase inhibitor (e.g. Piperacillin-Tazobactam, Co-amoxiclav).
    Farm/Agricultural Injuries or Gross Contamination with Soil/Sewage
    As for Type III, PLUS consider adding Penicillin G (for Clostridia) and/or Metronidazole (for anaerobes). Some guidelines recommend Piperacillin-Tazobactam + Gentamicin.
    Marine Contamination (salt water)
    Cover for Vibrio species (e.g., Doxycycline or Ciprofloxacin) in addition to standard cover.
  • Intra-operative wound cultures are often taken, especially in Type III injuries, to guide later antibiotic therapy if infection develops, but initial empirical therapy should not be delayed.
  • Local antibiotic delivery (e.g., antibiotic-loaded cement beads) may be used in high-risk cases.
Tetanus Prophylaxis:
  • All open fractures are considered tetanus-prone wounds.
  • Assess patient’s immunisation history.
  • General Guidelines (UK – Green Book):
    • Fully immunised (≥3 doses of tetanus toxoid, last dose <10 years ago): No booster needed generally for clean wounds. For tetanus-prone wounds (like open fractures), give a reinforcing dose of tetanus-containing vaccine if >5 years since last dose, or if high-risk wound.
    • Fully immunised (last dose >10 years ago): Give reinforcing dose of tetanus-containing vaccine.
    • Incompletely immunised (<3 doses or unknown status): Give a dose of tetanus-containing vaccine immediately. Arrange completion of full course. Human Tetanus Immunoglobulin (HTIG) should also be given for tetanus-prone wounds if significant contamination or devitalised tissue.
    • For heavily contaminated wounds (e.g., soil, manure) in incompletely immunised individuals, HTIG is usually recommended in addition to vaccine.
  • Always consult local guidelines and patient’s vaccination record.

Post-Operative & Definitive Care Considerations

Management extends well beyond the initial surgery, often involving staged procedures and long-term rehabilitation.

Early Post-Operative Period:
  • Wound Monitoring: Regular checks for signs of infection (erythema, swelling, discharge, increasing pain), wound breakdown, or flap necrosis.
  • Neurovascular Observations: Continue frequent checks, especially if external fixator in situ or risk of compartment syndrome.
  • Pain Management: Adequate multimodal analgesia.
  • Antibiotics: Continue IV antibiotics as planned, then consider switch to oral based on clinical progress and microbiology (if available).
  • DVT Prophylaxis: Mechanical (e.g., anti-embolic stockings, calf pumps) and/or pharmacological (e.g., LMWH) as per local policy, especially for lower limb fractures or reduced mobility.
  • Nutritional Support: Important for wound and bone healing, especially in polytrauma or malnourished patients.
  • Elevation of the Limb: To reduce swelling.
Staged Management & Definitive Reconstruction:
  • Second Look / Further Debridements: Often planned 24-72 hours after initial surgery, especially for severe injuries, to reassess tissue viability and perform further debridement if needed.
  • Definitive Soft Tissue Coverage: If not done at initial surgery, planned within 5-7 days once wound is clean (e.g., delayed primary closure, skin graft, flap).
  • Conversion of Fixation: External fixators may be converted to internal fixation (e.g., IM nail, plate) once soft tissues have settled and infection risk is reduced, if appropriate for fracture pattern.
  • Bone Grafting: May be required for significant bone loss or to promote union in non-healing fractures. Can be autograft (from patient, e.g., iliac crest) or allograft (donor bone) or synthetic substitutes. Usually performed as a delayed procedure once soft tissues stable and infection controlled.
Rehabilitation:
  • Early Mobilisation: As tolerated and permitted by fracture stability and soft tissue status. Crucial to prevent stiffness, muscle atrophy, and DVT.
  • Physiotherapy: Essential for restoring range of motion, strength, and function. Starts early post-op.
  • Occupational Therapy: To assist with activities of daily living and return to work/hobbies.
  • Weight-bearing status will be determined by the orthopaedic surgeon based on fracture type, stability, and fixation.
Long-Term Follow-Up:
  • Regular outpatient clinic reviews with serial X-rays to monitor fracture healing and detect complications.
  • Address psychological impact (e.g., PTSD, depression, anxiety related to trauma and prolonged recovery).
  • Management of chronic pain if it develops.
  • Vocational rehabilitation if needed.

Complications of Open Fractures

Open fractures are prone to a range of early and late complications.

Early Complications (Days to Weeks):
  • Infection:
    • Superficial Wound Infection: Cellulitis around the wound.
    • Deep Wound Infection / Abscess: May involve muscle, fascia.
    • Osteomyelitis: Bone infection. Can become chronic and very difficult to treat.
    • Gas Gangrene (Clostridial Myonecrosis): Rare but life-threatening. Rapid onset of severe pain, swelling, crepitus, systemic toxicity. Requires aggressive surgical debridement and antibiotics.
    • Sepsis: Systemic response to infection.
  • Compartment Syndrome: Increased pressure within a fascial compartment impairing perfusion. Clinical diagnosis: pain out of proportion (especially on passive stretch), tense compartment, paraesthesia, weakness. Pulselessness is a LATE sign. Requires urgent fasciotomy.
  • Neurovascular Injury: Damage to nerves or blood vessels from initial trauma or subsequent swelling/surgery. May require vascular repair or nerve exploration/grafting.
  • Haemorrhage / Haematoma Formation.
  • Fat Embolism Syndrome: Rare, typically with long bone fractures. Respiratory distress, neurological signs, petechial rash.
  • DVT/PE: Due to immobility and trauma.
  • Wound Breakdown / Flap Failure.
  • Fixation Failure: e.g., pin site infection with external fixators, implant loosening.
Late Complications (Months to Years):
  • Delayed Union: Fracture healing takes longer than expected (typically >3-6 months).
  • Non-Union: Fracture fails to heal (typically >6-9 months with no progress). Can be hypertrophic (callus present but not bridging) or atrophic (no callus). Often requires further surgery (e.g., bone grafting, revision fixation). Infection is a major cause of non-union.
  • Malunion: Fracture heals in an incorrect or deformed position, leading to functional impairment or cosmetic issues.
  • Chronic Osteomyelitis: Persistent bone infection, often with sinus tracts, recurrent flare-ups. May require multiple debridements, long-term antibiotics, and sometimes amputation.
  • Joint Stiffness / Contractures: Common after prolonged immobilisation or intra-articular injury.
  • Post-Traumatic Osteoarthritis: Especially with intra-articular fractures.
  • Chronic Pain Syndromes (e.g., Complex Regional Pain Syndrome – CRPS).
  • Psychological Sequelae: PTSD, depression, anxiety, body image issues.
  • Amputation: May be a late consequence of intractable infection, severe non-union, or chronic pain where limb salvage is not feasible or desired by patient.
  • Muscle Weakness and Atrophy.
🚨 Vigilance for Compartment Syndrome!

Maintain a high index of suspicion for compartment syndrome, especially in tibial and forearm fractures. The “6 Ps” (Pain, Pallor, Paraesthesia, Pulselessness, Paralysis, Perishingly cold) are classic but pulselessness and paralysis are late signs. Pain out of proportion and pain on passive stretch of muscles in the compartment are key early indicators. Urgent fasciotomy is needed.

MDT Approach & Long-Term Management (GP Role)

Successful management of open fractures relies on a coordinated multidisciplinary team (MDT) approach. GPs play a key role in long-term follow-up and support.

The Multidisciplinary Team (MDT):
  • Orthopaedic Surgeons: Lead surgical management (debridement, fracture stabilisation).
  • Plastic Surgeons: Involved in complex soft tissue reconstruction (flaps, grafts), especially for Type IIIB/C injuries.
  • Emergency Department Physicians & Nurses: Initial assessment, resuscitation, and emergency interventions.
  • Anaesthetists & Critical Care Physicians: Peri-operative care, pain management, management of polytrauma.
  • Radiologists: Imaging and interpretation. Interventional radiologists for angiography/embolisation if needed.
  • Microbiologists & Infectious Disease Physicians: Guide antibiotic therapy, manage complex infections.
  • Physiotherapists: Crucial for rehabilitation, restoring mobility and strength.
  • Occupational Therapists: Assist with ADLs, functional adaptation, return to work.
  • Specialist Nurses (e.g., Trauma Nurse Coordinators, Tissue Viability Nurses): Coordinate care, wound management.
  • Dietitians: Nutritional support for healing.
  • Social Workers & Psychologists: Address psychosocial impact, support for patient and family.
  • Prosthetists/Orthotists: If amputation or specialised bracing needed.
Role of the General Practitioner (GP) in Long-Term Management:

Once discharged from hospital, the GP often becomes a key point of contact for ongoing care and support.

  • Wound Care Monitoring: Liaise with community nurses. Monitor for signs of superficial infection (erythema, warmth, discharge, increasing pain) or wound breakdown after discharge.
  • Recognising Signs of Deep Infection / Osteomyelitis: Persistent pain, recurrent discharge, sinus formation, fevers, general malaise. Prompt re-referral to orthopaedics if suspected.
  • Pain Management: Continue and adjust analgesia as needed, stepping down opioids where possible. Consider neuropathic pain agents if indicated.
  • Monitoring Fracture Healing (indirectly): Be aware of planned orthopaedic follow-up and expected healing timelines. Note persistent pain or instability.
  • DVT/PE Prophylaxis & Monitoring: Ensure appropriate duration of thromboprophylaxis. Be alert for symptoms of DVT/PE.
  • Psychological Support: Screen for and manage/refer for anxiety, depression, PTSD. These are common after severe trauma.
  • Supporting Rehabilitation: Encourage adherence to physiotherapy. Liaise with physio/OT if concerns.
  • Certification for Work / Benefits Advice: Fit notes, supporting applications for disability benefits.
  • Managing Comorbidities: Optimise management of existing conditions (e.g., diabetes) that can affect healing.
  • Communication Hub: Often the GP is central in coordinating communication between different services if issues arise.
  • Health Promotion: e.g., smoking cessation (smoking impairs healing).
🤝 Holistic Care

Open fractures have a profound impact on patients physically, psychologically, and socially. The GP’s role in providing holistic, person-centered care and long-term support is invaluable during the often lengthy recovery period.

Flashcards: Open Fractures Management

Click on each card to reveal the answer.

What is the definition of an open fracture?

(Click to flip)

Answer

A fracture that communicates with the external environment, or a fracture with an overlying skin wound where the bone has been exposed.

What is the most important initial surgical step in managing an open fracture to prevent infection?

(Click to flip)

Answer

Thorough surgical debridement of all contaminated and devitalized tissue.

According to the Gustilo-Anderson classification, what defines a Type IIIC open fracture?

(Click to flip)

Answer

An open fracture associated with a major arterial injury requiring repair for limb salvage, regardless of the degree of soft tissue injury.

When should IV antibiotics ideally be administered for an open fracture?

(Click to flip)

Answer

As soon as possible after injury, ideally within 1 hour of injury or presentation.

Name two key principles of initial emergency department management of an open fracture (after ATLS).

(Click to flip)

Answer

1. Control haemorrhage & Cover wound with sterile dressing.
2. Administer IV antibiotics & Tetanus prophylaxis.
(Also: Analgesia, Realign & Splint, Photos, NBM, Ortho referral)

What is compartment syndrome and name one early symptom?

(Click to flip)

Answer

Definition: Increased pressure within a fascial compartment compromising circulation and function.
Early symptom: Pain out of proportion to the injury, or pain on passive stretch of muscles in the affected compartment.

A Gustilo-Anderson Type IIIB open fracture typically requires what for soft tissue coverage?

(Click to flip)

Answer

Soft tissue flap coverage (often by plastic surgeons) due to extensive soft tissue loss and bone exposure.

What are the “6 D’s” (or similar principles) of surgical open fracture management?

(Click to flip)

Answer

Debridement, Decontamination (Irrigation), Decision on Bone (Stabilisation), Dead Space Management, Definitive Soft Tissue Coverage, Delayed Reconstruction. (Slight variations exist).

Open Fractures Management Quiz

Test your knowledge on managing open fractures.

1. A patient presents with a 0.5 cm wound overlying a tibial fracture, with minimal soft tissue damage and no visible contamination. According to Gustilo-Anderson, what type is this likely to be?

A. Type I.
B. Type II.
C. Type IIIA.
D. Type IIIB.
Explanation: A wound <1 cm, clean, with minimal soft tissue injury typically classifies as Gustilo-Anderson Type I.

2. What is the most critical initial non-surgical intervention in the ED for an open fracture to reduce infection risk, alongside surgical debridement?

A. Application of a tourniquet.
B. Immediate wound closure with sutures.
C. Prompt administration of IV broad-spectrum antibiotics.
D. Detailed photographic documentation.
Explanation: Prompt IV broad-spectrum antibiotics (ideally within 1 hour of injury) are crucial. Wound closure is usually delayed. Tourniquets are for haemorrhage control. Photos are important but antibiotics are a direct intervention against infection.

3. A patient with an open tibial fracture develops severe pain in the calf unresponsive to opioids, and pain on passive dorsiflexion of the toes. The calf feels very tense. What is the most likely diagnosis?

A. Deep Vein Thrombosis (DVT).
B. Compartment Syndrome.
C. Osteomyelitis.
D. Fat Embolism Syndrome.
Explanation: Pain out of proportion, pain on passive stretch, and a tense compartment are classic early signs of compartment syndrome, an orthopaedic emergency.

4. For a Gustilo-Anderson Type IIIB open fracture, what is typically required for definitive soft tissue coverage?

A. Delayed primary closure.
B. Split-thickness skin graft directly onto bone.
C. Vascularised soft tissue flap (e.g., local or free flap).
D. Negative pressure wound therapy (VAC) as definitive cover.
Explanation: Type IIIB involves extensive soft tissue loss with bone exposure, requiring a vascularised flap for durable coverage. Skin grafts need a vascular bed. VAC is a temporary measure.

5. Which of these is NOT a primary goal of the initial surgical debridement of an open fracture?

A. Removal of all non-viable tissue.
B. Removal of foreign material and gross contaminants.
C. Definitive internal fixation of the fracture.
D. Copious irrigation of the wound.
Explanation: While fracture stabilisation is important, definitive internal fixation may be delayed if there’s gross contamination or significant soft tissue compromise. The primary goals of initial debridement are to clean the wound and remove non-viable tissue to prevent infection. External fixation is often used initially.

6. A farm worker sustains an open tibial fracture heavily contaminated with soil. In addition to a cephalosporin and gentamicin, what antibiotic class should be strongly considered?

A. Penicillin G (or Metronidazole for anaerobes).
B. Vancomycin.
C. Doxycycline.
D. A macrolide (e.g., Clarithromycin).
Explanation: Farm injuries with soil contamination carry a high risk of Clostridial (e.g., gas gangrene) and other anaerobic infections. Adding Penicillin G or Metronidazole provides cover for these organisms.

7. What is the most common long-term complication that can significantly impair healing and function after an open fracture?

A. Fat embolism syndrome.
B. Chronic osteomyelitis / deep infection.
C. Acute compartment syndrome.
D. Malignant transformation.
Explanation: Chronic osteomyelitis is a devastating long-term complication that can lead to non-union, recurrent surgeries, and potential amputation. Compartment syndrome and fat embolism are acute.

8. In the ATLS primary survey for a patient with a suspected open femur fracture and signs of shock, what is the immediate priority after ensuring a patent airway and adequate breathing?

A. Detailed neurological assessment of the injured limb.
B. Application of a traction splint.
C. Controlling external haemorrhage and establishing IV access for fluid resuscitation.
D. Obtaining urgent X-rays of the femur.
Explanation: In the ABCDE approach, Circulation (including haemorrhage control and IV access/fluids for shock) comes before detailed limb assessment (part of secondary survey or E), splinting, or imaging, if life-threatening circulatory compromise exists.

9. A patient with an open fracture has an unknown tetanus immunization status. The wound is heavily contaminated. According to UK guidelines, what tetanus prophylaxis is typically recommended?

A. Tetanus toxoid vaccine only.
B. Human Tetanus Immunoglobulin (HTIG) only.
C. Both tetanus toxoid vaccine and Human Tetanus Immunoglobulin (HTIG).
D. No prophylaxis needed if antibiotics are given.
Explanation: For a tetanus-prone wound in an individual with unknown or incomplete immunisation, both active immunisation (vaccine) and passive immunisation (HTIG) are typically recommended.

10. Which member of the MDT is primarily responsible for complex soft tissue reconstruction (e.g., free flaps) in severe open fractures (e.g., Gustilo Type IIIB)?

A. Orthopaedic Surgeon.
B. Plastic Surgeon.
C. Vascular Surgeon.
D. General Surgeon.
Explanation: Plastic surgeons specialise in complex soft tissue reconstruction, including local and free flaps, which are often required for Gustilo Type IIIB injuries where there is significant soft tissue loss and exposed bone.

Trauma & Orthopaedics: Osteoarthritis Management

Osteoarthritis (OA) is the most common form of arthritis, characterized by joint pain, stiffness, and functional limitation due to degeneration of articular cartilage and changes in underlying bone. Management is primarily focused on symptom relief, improving function, and enhancing quality of life.

Overview & Pathophysiology of Osteoarthritis

OA is a degenerative joint disease involving the entire joint, including cartilage, subchondral bone, ligaments, synovium, and periarticular muscles. It is not simply “wear and tear.”

Pathophysiology:

  • Cartilage Degradation: Imbalance between synthesis and degradation of articular cartilage components (collagen, proteoglycans). Cartilage becomes thinner, fibrillated, and eventually wears away.
  • Subchondral Bone Changes: Sclerosis (thickening), cyst formation, and osteophyte (bony spur) development at joint margins.
  • Synovial Inflammation (Synovitis): Mild to moderate inflammation of the synovial membrane can occur, contributing to pain and stiffness.
  • Ligamentous Laxity & Muscle Weakness: Can contribute to joint instability and progression.

Key Characteristics:

  • Primarily affects weight-bearing joints and those with high use.
  • Prevalence increases with age.
  • A leading cause of pain and disability worldwide.
  • Management is typically stepwise, starting with conservative measures.
💡 OSCE/PLAB Tip

When assessing a patient with suspected OA, focus on the impact of symptoms on their daily life, function, and mood. Explore their understanding of the condition and their treatment goals. Differentiate from inflammatory arthritis (e.g., rheumatoid arthritis often has morning stiffness >30-60 mins, systemic features, symmetrical small joint involvement initially).

Risk Factors & Commonly Affected Joints

Risk Factors for Osteoarthritis:
  • Age: Strongest risk factor. Prevalence increases significantly after 45-50 years.
  • Female Sex: More common in women, especially knee and hand OA post-menopause.
  • Obesity: Major modifiable risk factor, especially for knee and hip OA (increased joint loading and metabolic factors).
  • Previous Joint Injury or Trauma: E.g., meniscal tears, ACL injury, fractures involving the joint.
  • Occupational Factors: Repetitive joint loading or stress (e.g., farming, construction, mining).
  • Genetics/Family History: Contributes to susceptibility, especially for hand and hip OA.
  • Joint Malalignment or Deformity: E.g., varus/valgus knee, hip dysplasia.
  • Muscle Weakness: Particularly quadriceps weakness for knee OA.
  • Certain metabolic diseases (e.g., haemochromatosis, acromegaly) and inflammatory arthritides (e.g., RA can lead to secondary OA).
Commonly Affected Joints:
  • Knees: Very common, often bilateral.
  • Hips: Common, can cause groin, buttock, or referred knee pain.
  • Hands:
    • Distal Interphalangeal (DIP) joints – Heberden’s nodes.
    • Proximal Interphalangeal (PIP) joints – Bouchard’s nodes.
    • First Carpometacarpal (CMC) joint (base of thumb) – squaring of thumb base.
    • Metacarpophalangeal (MCP) joints are typically spared (unlike RA).
  • Spine (Cervical & Lumbar): Spondylosis. Can cause neck/back pain, stiffness, radiculopathy.
  • Feet: First Metatarsophalangeal (MTP) joint (big toe – can mimic gout).
  • Shoulders (glenohumeral and acromioclavicular joints) and elbows are less commonly affected by primary OA unless previous injury.

Clinical Features & Diagnosis of Osteoarthritis

Diagnosis is often clinical, especially in older adults with typical symptoms and risk factors.

Key Symptoms:
  • Joint Pain:
    • Typically activity-related or mechanical pain (worse on use, relieved by rest initially).
    • Can become persistent and present at rest or at night in advanced stages.
    • Often described as a deep ache.
  • Joint Stiffness:
    • Morning stiffness usually lasts <30 minutes (cf. inflammatory arthritis >30-60 mins).
    • Stiffness after periods of inactivity (“gelling”).
  • Functional Limitation: Difficulty with activities like walking, climbing stairs, dressing, gripping objects.
  • Reduced Range of Motion.
  • Joint Instability or “Giving Way” (especially knee).
  • Crepitus: Grating, cracking, or popping sensation on joint movement.
Key Examination Signs:
  • Bony Enlargement/Swelling: Osteophytes (e.g., Heberden’s and Bouchard’s nodes in hands).
  • Joint Line Tenderness.
  • Crepitus on Movement.
  • Restricted Range of Motion (active and passive).
  • Joint Deformity: E.g., varus/valgus knee, fixed flexion deformity.
  • Muscle Wasting/Weakness around the affected joint (e.g., quadriceps wasting in knee OA).
  • Joint Effusion: Usually cool, non-inflammatory (can be mildly warm if synovitis present).
  • Gait Abnormalities: E.g., antalgic gait.
Clinical Diagnostic Criteria (NICE CG177 – for patients ≥45 years):

Diagnose OA clinically without investigations if a person is ≥45 years old AND has activity-related joint pain AND has either no morning joint-related stiffness or morning stiffness that lasts no longer than 30 minutes.

If atypical features or diagnostic uncertainty, consider investigations (see next section).

⚠️ Red Flags / Atypical Features

Consider alternative diagnoses or refer if:

  • Younger age of onset (<45 years).
  • Prolonged morning stiffness (>30-60 minutes).
  • Significant joint warmth, redness, or marked swelling (suggests inflammation/infection).
  • Systemic symptoms (fever, weight loss, fatigue).
  • Rapidly progressive symptoms.
  • History of trauma suggesting acute injury.
  • Neurological signs/symptoms.
  • Pain at rest disproportionate to activity-related pain.

Investigations in Osteoarthritis

Investigations are not always needed for diagnosis if clinical picture is typical, but can be useful to exclude other conditions or assess severity, especially if considering surgery.

1. X-rays:
  • Plain radiographs of affected joint(s) (weight-bearing views for knees/hips if possible).
  • Characteristic Features of OA on X-ray (LOSS):
    • Loss of joint space (joint space narrowing – JSN).
    • Osteophytes (bony spurs at joint margins).
    • Subchondral sclerosis (increased bone density beneath cartilage).
    • Subchondral cysts (fluid-filled cavities in bone).
  • Limitations:
    • Poor correlation between X-ray findings and symptoms (some have severe X-ray changes but few symptoms, and vice-versa).
    • Early OA may show normal X-rays.
    • Not routinely needed to make a clinical diagnosis in typical cases (NICE).
  • Indications for X-ray: Atypical presentation, diagnostic uncertainty, to rule out other pathology (e.g., fracture, tumour), pre-operative planning.
X-ray Example: Knee Osteoarthritis
Knee OA X-ray

X-ray showing features of knee OA: medial joint space narrowing, osteophytes, subchondral sclerosis. Source: Wikimedia Commons (Illustrative)

2. Blood Tests:
  • Generally NOT helpful for diagnosing primary OA. OA is not primarily an inflammatory systemic disease.
  • Inflammatory markers (ESR, CRP): Usually normal or only mildly elevated in OA (can be raised if significant synovitis or coexisting illness). If significantly elevated, consider inflammatory arthritis or other pathology.
  • Rheumatoid Factor (RF), Anti-CCP antibodies: To exclude Rheumatoid Arthritis if suspected.
  • Uric Acid: If gout is suspected (especially in first MTP joint OA).
  • Routine bloods (FBC, U&E, LFT) may be done as part of general workup or pre-operatively.
3. Other Imaging (Less Common for Routine Diagnosis):
  • MRI: More sensitive for early cartilage changes, meniscal tears, ligament injuries, bone marrow oedema, synovitis. Not routinely needed for OA diagnosis but can be useful if internal derangement suspected or diagnostic uncertainty.
  • Ultrasound: Can detect synovitis, effusions, osteophytes, and guide joint injections.
  • Joint Aspiration: If significant effusion, to exclude crystal arthropathy (gout, pseudogout) or septic arthritis. Synovial fluid in OA is typically non-inflammatory (clear, viscous, low white cell count).

Conservative Management: Core Treatments (NICE CG177)

NICE recommends these core treatments should be offered to all patients with OA, irrespective of age, comorbidity, pain, or disability.

1. Information & Education:
  • Provide accurate information about OA (nature of condition, prognosis, management options).
  • Address misconceptions (e.g., that OA is just “wear and tear” and nothing can be done).
  • Promote self-management strategies.
  • Signpost to reliable resources (e.g., Versus Arthritis, NHS website).
2. Exercise & Physical Activity:
  • Crucial for all patients. Should be encouraged regardless of pain severity or functional limitation.
  • Types of Exercise:
    • Local muscle strengthening exercises: E.g., quadriceps strengthening for knee OA.
    • General aerobic fitness exercises: E.g., walking, cycling, swimming.
  • Benefits: Reduces pain, improves muscle strength and joint stability, improves function, mood, and general health.
  • Advise that some initial discomfort with exercise is normal but should settle. “Start low, go slow.”
  • Referral to physiotherapy for tailored exercise programmes. Consider group exercise classes.
3. Weight Management (if overweight or obese):
  • Weight loss is highly effective for reducing pain and improving function, especially in knee and hip OA.
  • Even modest weight loss (5-10%) can be beneficial.
  • Offer advice on diet and physical activity for weight loss.
  • Referral to weight management services if appropriate.
💡 The “Big Three”

Remember these three core pillars of OA management:

  1. Education & Self-Management
  2. Exercise (Strengthening & Aerobic)
  3. Weight Loss (if applicable)
These should form the foundation of any OA management plan.

Conservative Management: Adjunctive Non-Pharmacological Treatments

These can be considered alongside core treatments, based on individual needs and preferences.

1. Aids & Adaptations:
  • Walking Aids: Canes, crutches, walkers. Can improve stability, reduce joint load, and increase confidence. Assessed by physiotherapist.
  • Footwear Assessment & Advice: Appropriate, supportive, shock-absorbing footwear. Insoles or orthotics may be helpful for some (e.g., medial wedge insole for medial knee OA – evidence mixed).
  • Supports & Braces: E.g., knee braces. May provide symptomatic relief and improve stability for some, but evidence is variable.
  • Home Adaptations: Handrails, raised toilet seats, grab bars. Assessed by occupational therapist.
2. Thermotherapy:
  • Heat: Hot packs, warm baths/showers. Can relieve pain and stiffness.
  • Cold: Ice packs. Can reduce pain and swelling, especially after activity or if acute flare.
3. Transcutaneous Electrical Nerve Stimulation (TENS):
  • May provide short-term pain relief for some individuals. Patient can be taught to use at home.
  • NICE suggests considering it as an adjunct to core treatments.
4. Manual Therapy:
  • Techniques like massage, stretching, joint mobilisation/manipulation by a physiotherapist.
  • May provide short-term pain relief and improve range of motion as part of an exercise programme.
5. Acupuncture:
  • NICE does NOT recommend acupuncture for OA management due to lack of strong evidence of benefit over placebo.
6. Glucosamine & Chondroitin:
  • NICE does NOT recommend these supplements for OA management due to lack of evidence of clinically significant benefit.
7. Psychological Support:
  • Address impact of chronic pain on mood, sleep, coping.
  • Consider CBT, mindfulness, pain management programmes for those with significant psychological distress or poor coping.

Pharmacological Management (Analgesia)

Pharmacological treatments are for symptomatic relief and should be used as an adjunct to core non-pharmacological treatments, not as a replacement. Use the lowest effective dose for the shortest possible time.

Stepwise Approach (NICE CG177):
  1. Paracetamol:
    • Can be tried for mild to moderate pain. Regular dosing (e.g., 1g QDS) may be more effective than PRN.
    • However, NICE notes that evidence for its benefit in OA is limited, and it should only be continued if effective.
    • Monitor for efficacy and stop if no benefit.
  2. Topical NSAIDs (Non-Steroidal Anti-Inflammatory Drugs):
    • Consider for knee or hand OA, particularly if localised pain.
    • E.g., ibuprofen gel, diclofenac gel, ketoprofen gel.
    • Fewer systemic side effects than oral NSAIDs. Can be used alongside paracetamol.
  3. Oral NSAIDs / COX-2 Inhibitors:
    • Consider if paracetamol and/or topical NSAIDs are insufficient.
    • Use the lowest effective dose for the shortest possible period.
    • Co-prescribe a Proton Pump Inhibitor (PPI) to reduce GI risk, especially in older adults or those with GI risk factors.
    • Assess cardiovascular, renal, and GI risk before prescribing. Caution/Contraindications: Peptic ulcer disease, GI bleed, renal impairment, heart failure, uncontrolled hypertension, asthma (can precipitate), concurrent anticoagulants/steroids.
    • COX-2 inhibitors (e.g., celecoxib, etoricoxib) have lower GI risk than non-selective NSAIDs but similar CV and renal risks.
  4. Capsaicin Cream (Topical):
    • Consider for knee or hand OA if other analgesics ineffective or contraindicated.
    • Derived from chilli peppers. Depletes substance P.
    • Can cause initial burning sensation. Requires regular application for several weeks to see benefit.
  5. Opioid Analgesics:
    • NICE recommends AVOIDING routine use of strong opioids (e.g., morphine, oxycodone, fentanyl) for OA due to limited evidence of long-term benefit and significant risks (dependence, tolerance, side effects like constipation, nausea, drowsiness, falls, confusion).
    • Weak opioids (e.g., codeine, tramadol – often in co-codamol, co-dydramol) may be considered for short-term use for severe pain if other options failed or contraindicated, but also have significant side effects and limited long-term efficacy.
    • If used, prescribe lowest effective dose, for shortest duration, with regular review and clear plan for stopping.
  6. Intra-articular Corticosteroid Injections:
    • Consider for moderate to severe OA pain with acute flare-ups or if awaiting surgery.
    • Provide short-term pain relief (weeks to few months).
    • Can facilitate engagement with exercise.
    • Performed by trained clinician, often under ultrasound guidance for hip.
    • Risks: Post-injection flare, infection (rare), skin atrophy, tendon rupture (rare), facial flushing, temporary glucose elevation in diabetics. Limit frequency (e.g., not more than every 3-4 months per joint).
  7. Intra-articular Hyaluronic Acid Injections:
    • NICE does NOT recommend these due to lack of strong evidence of clinically significant benefit.

Surgical Management & Referral Criteria

Surgery is considered for OA when conservative measures have failed to control symptoms and there is significant impact on quality of life and function.

Indications for Referral for Consideration of Joint Replacement Surgery (Arthroplasty):

(NICE CG177 – refer before severe functional limitation or established deformity)

  • Persistent, significant joint pain unresponsive to adequate trial of non-surgical treatments.
  • Substantial impact on quality of life (e.g., sleep disturbance, inability to perform daily activities, social isolation).
  • Functional limitation (e.g., reduced mobility, difficulty with self-care).
  • Radiographic evidence of OA (though severity of symptoms is more important than X-ray changes).
  • Patient is medically fit enough for surgery and anaesthesia (risks/benefits discussed).
  • Patient is motivated and understands the implications of surgery and rehabilitation.

There should be no arbitrary cut-offs for referral based on age, BMI, smoking status, or comorbidities, but these factors will be considered in the surgical decision-making process.

Common Surgical Procedures for OA:
  • Joint Replacement (Arthroplasty): Most common surgical treatment for end-stage OA.
    • Total Hip Replacement (THR): Replacement of femoral head and acetabulum. Highly successful for relieving pain and improving function in hip OA.
    • Total Knee Replacement (TKR): Replacement of femoral, tibial, and often patellar articular surfaces. Effective for severe knee OA.
    • Unicompartmental (Partial) Knee Replacement (UKR): Option if OA is confined to one compartment of the knee (medial or lateral) and ligaments are intact. Less invasive, quicker recovery than TKR for suitable patients.
    • Shoulder, elbow, ankle, and small joint (finger) replacements are also performed but are less common.
  • Arthroscopic Surgery (e.g., debridement, lavage, meniscectomy):
    • NICE recommends AGAINST arthroscopy for OA unless there is clear evidence of mechanical locking of the knee. Routine debridement or lavage has not shown significant benefit over placebo or conservative care for OA symptoms.
    • Meniscectomy may be considered if acute meniscal tear symptoms coexist with OA, but benefits specifically for OA pain are uncertain.
  • Osteotomy:
    • Surgical cutting and realignment of bone to redistribute weight-bearing forces across the joint.
    • More commonly performed in younger, active patients with unicompartmental knee OA and malalignment (e.g., high tibial osteotomy for varus knee). Aims to delay need for joint replacement.
  • Joint Fusion (Arthrodesis):
    • Surgical fusion of a joint to eliminate movement and pain.
    • Reserved for severe OA in certain joints where replacement is not suitable or has failed (e.g., ankle, wrist, some finger joints). Results in loss of motion but pain relief.
💡 Managing Expectations for Surgery

It’s crucial that patients have realistic expectations about joint replacement surgery. While highly effective for pain relief and function, it involves risks, a significant rehabilitation period, and the prosthesis has a limited lifespan (though modern implants often last 15-20+ years).

Flashcards: Osteoarthritis Management

Click on each card to reveal the answer.

What are the three core non-pharmacological treatments recommended by NICE for all patients with OA?

(Click to flip)

Answer

1. Information & Education
2. Exercise (local muscle strengthening and general aerobic)
3. Weight Management (if overweight/obese)

What are the characteristic X-ray features of Osteoarthritis (LOSS)?

(Click to flip)

Answer

Loss of joint space (narrowing)
Osteophytes
Subchondral sclerosis
Subchondral cysts

What is the typical duration of morning stiffness in Osteoarthritis compared to Rheumatoid Arthritis?

(Click to flip)

Answer

Osteoarthritis: Usually <30 minutes.
Rheumatoid Arthritis: Often >30-60 minutes.

Name two common sites for Heberden’s and Bouchard’s nodes in hand OA.

(Click to flip)

Answer

Heberden’s nodes: Distal Interphalangeal (DIP) joints.
Bouchard’s nodes: Proximal Interphalangeal (PIP) joints.

What important precaution should be taken when prescribing an oral NSAID for OA in an older adult?

(Click to flip)

Answer

Co-prescribe a Proton Pump Inhibitor (PPI) to reduce gastrointestinal risk. Use lowest effective dose for shortest duration. Assess CV/renal risk.

When might an intra-articular corticosteroid injection be considered for OA?

(Click to flip)

Answer

For short-term relief of moderate to severe pain, particularly during an acute flare-up, or if awaiting surgery, to facilitate engagement with exercise.

What is the NICE recommendation regarding routine arthroscopy for osteoarthritis of the knee?

(Click to flip)

Answer

NICE recommends AGAINST routine arthroscopic lavage and debridement for OA unless there is clear evidence of mechanical locking of the knee.

Name a major modifiable risk factor for knee osteoarthritis.

(Click to flip)

Answer

Obesity / Overweight.

Osteoarthritis Management Quiz

Test your knowledge with these Orthopaedic questions.

1. A 65-year-old woman presents with bilateral knee pain, worse on activity, and morning stiffness lasting 20 minutes. Examination reveals crepitus and mild bony enlargement. According to NICE, what is the most appropriate initial step?

A. Request knee X-rays and ESR/CRP.
B. Diagnose OA clinically and offer core treatments (education, exercise, weight management if needed).
C. Refer immediately for consideration of knee replacement.
D. Prescribe strong opioid analgesia.
Explanation: For a patient ≥45 years with typical activity-related pain and morning stiffness <30 mins, OA can be diagnosed clinically. Core treatments (education, exercise, weight management) are the first line. X-rays are not routinely needed initially.

2. Which of the following is NOT a characteristic radiographic feature of osteoarthritis?

A. Joint space narrowing.
B. Osteophytes.
C. Juxta-articular osteoporosis.
D. Subchondral sclerosis.
Explanation: Juxta-articular osteoporosis (bone thinning near the joint) is characteristic of inflammatory arthritis like Rheumatoid Arthritis. OA typically shows subchondral sclerosis (bone hardening).

3. A 58-year-old man with knee OA is overweight (BMI 32). Which core treatment is likely to provide the most significant benefit for his knee symptoms?

A. Regular paracetamol.
B. Weight loss combined with an exercise programme.
C. Intra-articular hyaluronic acid injections.
D. Using a TENS machine daily.
Explanation: For overweight/obese individuals with knee OA, weight loss is a highly effective core intervention, especially when combined with exercise, for reducing pain and improving function.

4. When prescribing an oral NSAID for a 70-year-old patient with osteoarthritis, what is an essential co-prescription to consider?

A. An opioid analgesic.
B. A Proton Pump Inhibitor (PPI).
C. A diuretic.
D. An anticoagulant.
Explanation: Oral NSAIDs increase the risk of gastrointestinal bleeding, especially in older adults. Co-prescription of a PPI is recommended to mitigate this risk.

5. Heberden’s nodes are bony enlargements typically found at which joints in hand osteoarthritis?

A. Metacarpophalangeal (MCP) joints.
B. Proximal Interphalangeal (PIP) joints.
C. Distal Interphalangeal (DIP) joints.
D. First Carpometacarpal (CMC) joint.
Explanation: Heberden’s nodes are characteristic osteophytes found at the DIP joints in hand OA. Bouchard’s nodes are at the PIP joints.

6. A patient with severe hip osteoarthritis has failed conservative management and has significant pain and functional limitation affecting their quality of life. What is the most appropriate next step?

A. Long-term strong opioid prescription.
B. Referral to an orthopaedic surgeon for consideration of total hip replacement.
C. Advise complete bed rest.
D. Start a course of intra-articular hyaluronic acid injections.
Explanation: If conservative measures fail and symptoms are severe with functional impact, referral for surgical consideration (e.g., THR) is appropriate. Long-term strong opioids are not recommended. Hyaluronic acid is not recommended by NICE.

7. Which of the following statements about arthroscopy for knee osteoarthritis is most accurate according to NICE guidelines?

A. It is recommended as a first-line treatment for all knee OA.
B. It should not be offered unless there is clear evidence of mechanical knee locking.
C. It is more effective than exercise for long-term pain relief.
D. It is primarily used to diagnose osteoarthritis.
Explanation: NICE guidelines state that arthroscopic lavage and debridement should not be offered as part of treatment for OA, unless the person has knee OA with clear evidence of mechanical locking.

8. A patient with knee OA asks about glucosamine supplements. What is the current NICE recommendation?

A. Recommend daily glucosamine sulphate for cartilage repair.
B. Do not recommend glucosamine or chondroitin due to lack of evidence of benefit.
C. Recommend a trial for 3 months to see if it helps.
D. Only recommend if X-rays show severe OA.
Explanation: NICE guidelines do not recommend glucosamine or chondroitin products for the management of osteoarthritis due to a lack of robust evidence demonstrating clinically significant benefit.

9. Which of these is a common feature of osteoarthritis pain?

A. Prominent morning stiffness lasting over an hour.
B. Pain typically worse with activity and relieved by rest (initially).
C. Symmetrical small joint swelling with systemic upset.
D. Pain that rapidly improves with low-dose corticosteroids.
Explanation: OA pain is characteristically mechanical, meaning it is often exacerbated by joint use and improves with rest, especially in the early stages. Prolonged morning stiffness and systemic features are more typical of inflammatory arthritis.

10. Topical NSAIDs are particularly recommended by NICE as a treatment option for osteoarthritis affecting which joints?

A. Hip and Spine.
B. Knee and Hand.
C. Shoulder and Ankle.
D. All joints equally.
Explanation: NICE recommends considering topical NSAIDs for osteoarthritis of the knee or hand, as these joints are more superficial, allowing for better local drug delivery and efficacy with fewer systemic side effects.

Rheumatoid Arthritis (RA)

Rheumatoid Arthritis (RA) is a chronic, systemic autoimmune inflammatory disease primarily affecting synovial joints, leading to progressive joint damage, disability, and reduced quality of life if untreated. Early diagnosis and aggressive management with disease-modifying anti-rheumatic drugs (DMARDs) are crucial to improve outcomes.

Overview & Definitions of Rheumatoid Arthritis

RA is characterized by persistent synovial inflammation, which can lead to cartilage destruction, bone erosions, and subsequent joint deformity. It typically affects small joints of the hands and feet in a symmetrical pattern.

Key Definitions:

  • Rheumatoid Arthritis (RA): A chronic systemic autoimmune disease causing inflammation of synovial joints, potentially leading to joint destruction and extra-articular manifestations.
  • Synovitis: Inflammation of the synovial membrane, the lining of diarthrodial joints. This is the hallmark of RA.
  • Pannus: An abnormal layer of fibrovascular tissue or granulation tissue that grows over the joint surface in RA, leading to erosion of cartilage and bone.
  • Seropositive RA: RA in patients who test positive for Rheumatoid Factor (RF) and/or Anti-Cyclic Citrullinated Peptide (Anti-CCP) antibodies. Often associated with more severe disease.
  • Seronegative RA: RA in patients who test negative for both RF and Anti-CCP.

Epidemiology & Significance:

  • Prevalence: Affects approximately 0.5-1% of the adult population worldwide.
  • Peak age of onset: 30-50 years, but can occur at any age.
  • Gender: More common in women (Female:Male ratio ~2-3:1).
  • Impact: Significant cause of disability, reduced work capacity, and increased mortality (primarily due to cardiovascular disease and infection).
  • Early and aggressive treatment (“treat-to-target”) significantly improves outcomes.
💡 OSCE/PLAB Tip

When suspecting RA, always ask about the duration of morning stiffness (typically >30-60 minutes in RA), symmetry of joint involvement, and impact on daily activities. Differentiate from osteoarthritis (OA), which typically has shorter morning stiffness (<30 mins) and affects different joints (DIPs, CMCs, large weight-bearing joints).

🧠 RA Features: “RHEUMATOID”
R Rheumatoid Factor (RF) / Anti-CCP positive (often)
H Hands (MCP, PIP) & Feet (MTP) typically involved
E Extra-articular manifestations
U Ulnar deviation / Deformities (late)
M Morning stiffness (>30-60 minutes)
A Arthritis (symmetrical, inflammatory polyarthritis)
T Tenosynovitis / Tendon involvement
O Osteopenia (juxta-articular) / Erosions (on X-ray)
I Insidious onset (usually)
D Disease activity scores used for monitoring (e.g., DAS28)

Pathophysiology of Rheumatoid Arthritis

RA is a complex autoimmune disease involving genetic predisposition, environmental triggers, and a dysregulated immune response.

  • Genetic Susceptibility:
    • Strongest association with HLA-DRB1 alleles (e.g., HLA-DR4, HLA-DR1) containing a “shared epitope.”
    • Other non-HLA genes also contribute.
  • Environmental Triggers (Hypothesized):
    • Smoking: Strongest known environmental risk factor, particularly for seropositive RA. May induce citrullination of proteins.
    • Infections (e.g., Porphyromonas gingivalis in periodontal disease, Epstein-Barr virus) have been implicated but causality is not definitively proven.
  • Autoimmune Process:
    1. Initiation: In genetically susceptible individuals, an environmental trigger may lead to modification of self-proteins (e.g., citrullination – conversion of arginine to citrulline). These modified proteins are recognized as foreign by the immune system.
    2. Activation of Immune Cells:
      • Antigen-presenting cells (APCs) present these autoantigens to T-cells (CD4+ helper T-cells).
      • Activated T-cells stimulate B-cells to produce autoantibodies:
        • Rheumatoid Factor (RF): Antibodies (usually IgM) against the Fc portion of IgG.
        • Anti-Cyclic Citrullinated Peptide (Anti-CCP) Antibodies (ACPA): More specific for RA than RF. Target citrullinated proteins.
      • T-cells and B-cells, along with macrophages, infiltrate the synovium.
    3. Synovial Inflammation (Synovitis):
      • Immune cells release pro-inflammatory cytokines (e.g., TNF-α, IL-1, IL-6), chemokines, and growth factors.
      • This leads to synovial cell proliferation (hyperplasia), increased vascularity (angiogenesis), and influx of more inflammatory cells into the synovium.
    4. Pannus Formation:
      • The inflamed, thickened synovium transforms into an aggressive, invasive tissue called pannus.
      • Pannus grows over the articular cartilage and invades underlying bone.
    5. Joint Destruction:
      • Pannus releases enzymes (e.g., matrix metalloproteinases – MMPs) that degrade cartilage.
      • Osteoclasts are activated (stimulated by RANKL produced by synovial cells and T-cells), leading to bone erosions.
      • This results in joint space narrowing, erosions, and eventually joint deformities and loss of function.
  • Systemic Inflammation: Pro-inflammatory cytokines enter circulation, contributing to extra-articular manifestations and systemic symptoms like fatigue and malaise.

Clinical Features of Rheumatoid Arthritis

RA typically presents with insidious onset of joint pain and stiffness, but can occasionally have an acute onset.

Articular Manifestations:

  • Symmetrical Polyarthritis: Affects multiple joints, usually in a symmetrical pattern (both sides of the body).
  • Joints Commonly Affected (Early):
    • Hands: Metacarpophalangeal (MCP) joints, Proximal Interphalangeal (PIP) joints. Distal Interphalangeal (DIP) joints are typically spared (unlike osteoarthritis).
    • Wrists: Often involved early.
    • Feet: Metatarsophalangeal (MTP) joints (“walking on pebbles” sensation).
  • Other Joints Affected (Later or Less Commonly): Knees, elbows, shoulders, ankles, cervical spine (atlantoaxial subluxation is a serious complication). Temporomandibular joint (TMJ).
  • Symptoms:
    • Pain: Worse with rest, improves with activity.
    • Stiffness: Prominent morning stiffness lasting >30-60 minutes (often hours). Stiffness after periods of inactivity (“gelling”).
    • Swelling: Due to synovitis and effusion. Joints feel warm and tender (“boggy” swelling).
    • Reduced Range of Motion and Function.
  • Specific Hand Deformities (Usually Late Stage):
    • Ulnar Deviation: Fingers drift towards the ulnar side at the MCP joints.
    • Swan Neck Deformity: PIP hyperextension and DIP flexion.
    • Boutonnière (Buttonhole) Deformity: PIP flexion and DIP hyperextension.
    • Z-Thumb Deformity: Hyperextension of IP joint, flexion and subluxation of MCP joint.
    • Piano Key Sign: Dorsal subluxation of ulnar styloid, which can be depressed like a piano key.
    • Volar subluxation of MCP joints.
  • Other Articular Features:
    • Tenosynovitis (inflammation of tendon sheaths), carpal tunnel syndrome (median nerve compression at wrist).
    • Baker’s cyst (popliteal cyst) in the knee.

Extra-Articular Manifestations (EAMs):

Occur in ~40% of patients, more common in seropositive (RF/Anti-CCP +ve) and severe disease. Can affect almost any organ system.

  • Constitutional Symptoms: Fatigue, malaise, low-grade fever, weight loss, depression.
  • Rheumatoid Nodules:
    • Most common EAM (~20-30%). Firm, non-tender, subcutaneous nodules.
    • Typically found over extensor surfaces and pressure points (e.g., olecranon, fingers, Achilles tendon). Can also occur in lungs, heart, sclera.
  • Pulmonary:
    • Pleural effusion (exudative, low glucose).
    • Interstitial Lung Disease (ILD) – e.g., Usual Interstitial Pneumonia (UIP), Non-Specific Interstitial Pneumonia (NSIP).
    • Rheumatoid lung nodules (can cavitate).
    • Caplan’s Syndrome: Rheumatoid nodules + pneumoconiosis in coal miners.
    • Bronchiolitis obliterans.
  • Cardiac:
    • Pericarditis (most common cardiac manifestation).
    • Myocarditis, coronary artery disease (accelerated atherosclerosis), valvular disease, conduction abnormalities.
  • Haematological:
    • Anaemia of Chronic Disease (most common). Iron deficiency anaemia can also occur.
    • Thrombocytosis (reactive).
    • Felty’s Syndrome (Rare Triad): RA + Splenomegaly + Neutropenia. Increased risk of infections.
    • Large Granular Lymphocyte (LGL) Leukaemia.
  • Ocular:
    • Keratoconjunctivitis Sicca (Secondary Sjögren’s Syndrome): Dry eyes, dry mouth.
    • Episcleritis (usually benign, redness).
    • Scleritis (Serious): Severe deep eye pain, redness, photophobia, visual loss. Can lead to scleromalacia perforans (thinning and perforation of sclera).
  • Vasculitis:
    • Rheumatoid vasculitis. Can affect small to medium-sized vessels.
    • Manifestations: Skin (digital infarcts, ulcers, purpura), peripheral neuropathy (mononeuritis multiplex), visceral involvement (rare but serious).
  • Neurological:
    • Peripheral neuropathy (e.g., entrapment neuropathies like carpal tunnel syndrome, sensory neuropathy, mononeuritis multiplex due to vasculitis).
    • Cervical myelopathy (due to atlantoaxial subluxation compressing spinal cord – neck pain, upper motor neurone signs in limbs).
  • Renal: Rarely directly affected by RA itself, but can be due to drugs (NSAIDs, gold, penicillamine) or secondary amyloidosis (AA type).
  • Osteoporosis: Due to chronic inflammation, steroid use, and immobility.

Investigations & Diagnosis of Rheumatoid Arthritis

Diagnosis is based on a combination of clinical features, laboratory tests, and imaging. Early diagnosis is key.

Laboratory Tests:

  • Inflammatory Markers:
    • Erythrocyte Sedimentation Rate (ESR): Usually elevated.
    • C-Reactive Protein (CRP): Usually elevated. More responsive to changes in inflammation than ESR.
    • Used to assess disease activity and response to treatment.
  • Autoantibodies:
    • Rheumatoid Factor (RF):
      • Present in ~70-80% of RA patients (seropositive RA).
      • Not specific for RA: can be positive in other autoimmune diseases (e.g., Sjögren’s, SLE), chronic infections (e.g., hepatitis C, endocarditis), and in ~5-10% of healthy individuals (especially elderly).
      • Higher titres may correlate with more severe disease and EAMs.
    • Anti-Cyclic Citrullinated Peptide (Anti-CCP) Antibodies (ACPA):
      • Similar sensitivity to RF (~70%) but much higher specificity (>95%) for RA.
      • Can be present years before clinical onset of RA.
      • Strong predictor of erosive disease.
    • Note: Patients can be RF positive and Anti-CCP negative, RF negative and Anti-CCP positive, both positive, or both negative (seronegative RA, ~20-30%).
  • Full Blood Count (FBC):
    • Normocytic, normochromic anaemia (Anaemia of Chronic Disease) is common.
    • Thrombocytosis (reactive to inflammation).
    • Neutropenia (consider Felty’s syndrome if splenomegaly present).
    • Eosinophilia (can occur).
  • Liver Function Tests (LFTs) & Urea and Electrolytes (U&Es):
    • Important as baseline before starting DMARDs (many are hepatotoxic or renally cleared/toxic).
    • May show mild abnormalities due to systemic inflammation.
  • Synovial Fluid Analysis (Arthrocentesis):
    • If diagnosis uncertain, or to exclude septic arthritis or crystal arthropathy (e.g., gout).
    • RA synovial fluid: Inflammatory (Type II) – cloudy, yellow, reduced viscosity, high white cell count (typically 2,000-50,000/mm³, predominantly neutrophils), negative crystals, negative culture.

Imaging:

  • X-rays (Hands and Feet):
    • May be normal in early disease.
    • Early signs: Soft tissue swelling, juxta-articular osteopenia (bone thinning near joints).
    • Later signs: Joint space narrowing (due to cartilage loss), marginal erosions (bony defects at joint margins), subluxation, and deformities.
    • Used for baseline assessment and monitoring disease progression.
  • Ultrasound (USS) with Power Doppler:
    • More sensitive than X-ray for detecting early synovitis, tenosynovitis, and erosions.
    • Power Doppler can show increased vascularity (active inflammation).
    • Useful for guiding joint aspirations/injections.
  • Magnetic Resonance Imaging (MRI):
    • Most sensitive imaging for detecting early synovitis, bone marrow oedema (precedes erosions), and early erosions.
    • Not routinely used for diagnosis due to cost and accessibility, but can be helpful in equivocal cases or for assessing specific joints (e.g., cervical spine).

ACR/EULAR 2010 Classification Criteria for RA:

Used for classifying patients with new-onset inflammatory arthritis as having “definite RA” for research and guiding early treatment. A score of ≥6 out of 10 is needed for classification of definite RA.
(Patient must have at least 1 joint with definite clinical synovitis, with the synovitis not better explained by another disease).

  1. Joint Involvement (Score 0-5):
    • 1 large joint (e.g., shoulder, elbow, hip, knee, ankle): 0 points
    • 2-10 large joints: 1 point
    • 1-3 small joints (MCPs, PIPs, MTPs, thumb IPs, wrists – large joints excluded): 2 points
    • 4-10 small joints (large joints excluded): 3 points
    • >10 joints (at least 1 small joint): 5 points
  2. Serology (RF and Anti-CCP) (Score 0-3):
    • Negative RF AND Negative Anti-CCP: 0 points
    • Low-positive RF OR Low-positive Anti-CCP (<3x Upper Limit of Normal - ULN): 2 points
    • High-positive RF OR High-positive Anti-CCP (≥3x ULN): 3 points
  3. Acute Phase Reactants (ESR and CRP) (Score 0-1):
    • Normal CRP AND Normal ESR: 0 points
    • Abnormal CRP OR Abnormal ESR: 1 point
  4. Duration of Symptoms (Score 0-1):
    • <6 weeks: 0 points
    • ≥6 weeks: 1 point
⚠️ Early Referral is Key!

NICE guidelines recommend referring anyone with suspected persistent synovitis of undetermined cause to a rheumatologist urgently (within 3 working days of GP referral if small joints of hands/feet affected or >1 joint affected, or if delay would lead to worse outcomes). Early DMARD initiation (ideally within 6 weeks of referral) improves long-term outcomes.

Management of Rheumatoid Arthritis

Management is multidisciplinary (rheumatologist, specialist nurse, physiotherapist, occupational therapist, podiatrist, pharmacist, psychologist) and aims to:

  • Achieve remission or low disease activity (“Treat-to-Target”).
  • Control pain and inflammation.
  • Prevent joint damage and deformity.
  • Maintain/improve function and quality of life.
  • Manage extra-articular manifestations and comorbidities.

Non-Pharmacological Management:

  • Patient Education & Self-Management: Understanding RA, managing symptoms, pacing activities.
  • Physiotherapy: Exercises to maintain joint mobility, muscle strength, and overall fitness. Hydrotherapy.
  • Occupational Therapy (OT): Joint protection techniques, adaptive equipment, splints (e.g., resting splints for night, working splints for day), strategies for ADLs.
  • Podiatry: Foot care, orthotics for MTP involvement.
  • Lifestyle Modification:
    • Smoking Cessation: Crucial, as smoking worsens RA and reduces treatment efficacy.
    • Healthy diet, weight management.
  • Psychological Support: Coping strategies for chronic pain, disability, depression/anxiety.

Pharmacological Management:

Follows a stepwise approach, often starting DMARDs early.

1. Symptomatic Relief (Analgesia & Anti-inflammatories):
  • Paracetamol: For mild pain.
  • Non-Steroidal Anti-Inflammatory Drugs (NSAIDs): E.g., Ibuprofen, Naproxen, COX-2 inhibitors (Celecoxib).
    • Provide rapid relief of pain and stiffness but do not alter disease course or prevent joint damage.
    • Use lowest effective dose for shortest possible time due to GI, renal, and cardiovascular side effects.
    • Co-prescribe a Proton Pump Inhibitor (PPI) if regular use, especially in at-risk patients.
2. Corticosteroids:
  • E.g., Prednisolone (oral), Methylprednisolone (IM/IV/intra-articular).
  • Potent anti-inflammatory and immunosuppressive effects.
    • Bridging therapy: Used for rapid symptom control when starting DMARDs (which take weeks/months to work). Tapered off once DMARD is effective.
    • Flares: Short courses for acute exacerbations.
    • Low-dose maintenance: Sometimes used in severe RA if other treatments fail (aim for ≤7.5mg prednisolone daily), but long-term use has significant side effects (osteoporosis, diabetes, hypertension, infections, Cushingoid features).
    • Intra-articular injections: Useful for 1-2 actively inflamed joints.
  • Bone protection (e.g., bisphosphonates, calcium/Vit D) essential with long-term steroid use.
3. Disease-Modifying Anti-Rheumatic Drugs (DMARDs):

Cornerstone of RA treatment. Aim to induce/maintain remission and prevent joint damage. Start as soon as diagnosis is made.

a) Conventional Synthetic DMARDs (csDMARDs):
  • Methotrexate (MTX):
    • First-line DMARD for most RA patients. Given once weekly (oral or subcutaneous).
    • Mechanism: Folate antagonist, inhibits dihydrofolate reductase, anti-inflammatory effects.
    • Co-prescribe Folic Acid (5mg once weekly, taken on a different day to MTX) to reduce side effects.
    • Side Effects: Nausea, vomiting, mouth ulcers, hepatotoxicity (LFT derangement, fibrosis/cirrhosis rarely), bone marrow suppression (leukopenia, thrombocytopenia), pneumonitis (rare but serious – cough, dyspnoea). Teratogenic.
    • Monitoring: Regular FBC, LFTs, U&Es (e.g., every 1-2 weeks initially, then 1-3 monthly). Chest X-ray baseline.
    • Contraindications: Pregnancy, breastfeeding, severe liver/renal disease, active infection, significant alcohol excess.
  • Sulfasalazine (SSZ):
    • Often used in combination with MTX or as an alternative if MTX contraindicated/not tolerated.
    • Mechanism: Not fully understood, anti-inflammatory and immunomodulatory.
    • Side Effects: Nausea, rash, headache, oligospermia (reversible), bone marrow suppression, hepatotoxicity. Can cause yellow-orange discoloration of urine/skin.
    • Monitoring: FBC, LFTs.
  • Leflunomide:
    • Alternative to MTX or used in combination.
    • Mechanism: Inhibits pyrimidine synthesis (dihydroorotate dehydrogenase inhibitor).
    • Side Effects: Diarrhoea, hypertension, hepatotoxicity, rash, alopecia, bone marrow suppression. Teratogenic (long half-life, requires washout procedure if pregnancy planned).
    • Monitoring: FBC, LFTs, BP.
  • Hydroxychloroquine (HCQ):
    • Milder DMARD, often used for mild RA or in combination.
    • Mechanism: Not fully understood, interferes with lysosomal function and toll-like receptors.
    • Side Effects: Generally well-tolerated. Nausea, rash. Rare but serious: retinal toxicity (maculopathy – requires baseline and annual eye screening).
  • Combination therapy (e.g., MTX + SSZ + HCQ) is often more effective than monotherapy.
b) Biologic DMARDs (bDMARDs):

Genetically engineered proteins that target specific components of the immune system. Used for moderate-severe RA unresponsive to or intolerant of csDMARDs (usually after trial of ≥2 csDMARDs, including MTX, unless contraindicated).

  • TNF-α Inhibitors (Anti-TNF):
    • E.g., Adalimumab (SC), Etanercept (SC), Infliximab (IV – often with MTX), Golimumab (SC), Certolizumab pegol (SC).
    • Block the action of TNF-α, a key pro-inflammatory cytokine.
    • Side Effects/Risks: Increased risk of infections (especially TB – screen before starting), demyelinating disorders (rare), worsening heart failure, injection site/infusion reactions, drug-induced lupus, lymphoma (small risk).
    • Monitoring: Screen for latent TB, Hepatitis B/C before starting. Monitor for infections.
  • Other Biologics (Non-TNF):
    • Rituximab (Anti-CD20): Chimeric monoclonal antibody that depletes B-cells. Given as IV infusion.
      • Side Effects: Infusion reactions, infections, Progressive Multifocal Leukoencephalopathy (PML – very rare).
    • Tocilizumab / Sarilumab (Anti-IL-6 Receptor): Monoclonal antibodies blocking IL-6 receptor.
      • Side Effects: Infections, neutropenia, thrombocytopenia, dyslipidaemia, LFT elevation, GI perforation (rare).
    • Abatacept (CTLA4-Ig): Fusion protein that inhibits T-cell co-stimulation.
      • Side Effects: Infections (especially respiratory), headache, nausea.
c) Targeted Synthetic DMARDs (tsDMARDs) – JAK Inhibitors:
  • Oral small molecules that inhibit Janus Kinase enzymes, involved in cytokine signalling.
  • E.g., Tofacitinib, Baricitinib, Upadacitinib, Filgotinib.
  • Used for moderate-severe RA, similar indications to biologics. Can be used as monotherapy or with MTX.
  • Side Effects/Risks: Infections (including herpes zoster reactivation – consider Shingrix vaccine), dyslipidaemia, LFT elevation, cytopenias, increased risk of VTE (venous thromboembolism), MACE (major adverse cardiovascular events), and malignancy (particularly in certain risk groups – debated).
4. Surgical Management:
  • Considered for persistent pain, severe joint damage, instability, or functional limitation despite optimal medical therapy.
  • Synovectomy: Removal of inflamed synovium.
  • Tendon Repair/Release: For ruptured tendons or entrapment.
  • Arthrodesis: Joint fusion (e.g., wrist, ankle, thumb IPJ) for pain relief and stability, at cost of movement.
  • Arthroplasty: Joint replacement (e.g., MCPJs, PIPJs, wrist, elbow, shoulder, hip, knee) to relieve pain and improve function.

Monitoring Disease Activity & Prognosis in RA

Monitoring Disease Activity:

Regular monitoring is essential to guide treatment (“Treat-to-Target” strategy). Aim for remission or low disease activity.

  • Clinical Assessment:
    • Patient-reported symptoms (pain, stiffness, fatigue, function).
    • Joint examination: Tender Joint Count (TJC), Swollen Joint Count (SJC) – typically 28 joints assessed.
  • Composite Disease Activity Scores:
    • DAS28 (Disease Activity Score 28): Most commonly used.
      • Combines TJC28, SJC28, ESR or CRP, and patient global assessment of health (VAS 0-100mm).
      • Score interpretation:
        • Remission: <2.6
        • Low disease activity: 2.6 to <3.2
        • Moderate disease activity: 3.2 to ≤5.1
        • High disease activity: >5.1
    • Other scores: SDAI (Simplified Disease Activity Index), CDAI (Clinical Disease Activity Index).
  • Laboratory Tests: ESR, CRP to track inflammation.
  • Imaging: X-rays periodically (e.g., annually) to assess for new erosions or progression of joint damage, especially if disease activity not well controlled. Ultrasound can also be used.
  • Functional Assessment: E.g., Health Assessment Questionnaire (HAQ) to measure impact on ADLs.

Monitoring for Drug Toxicity:

  • Crucial for DMARDs (especially MTX, leflunomide, sulfasalazine, biologics, JAK inhibitors).
  • Regular blood tests (FBC, LFTs, U&Es) as per local guidelines (e.g., for MTX: every 1-2 weeks initially, then 1-3 monthly once stable).
  • Screening for infections (TB, hepatitis) before starting biologics/JAK inhibitors.
  • Monitoring for specific side effects (e.g., BP for leflunomide, eye checks for hydroxychloroquine, lipid profile for JAK inhibitors/tocilizumab).

Prognosis:

Variable, but significantly improved with early diagnosis and modern treatment strategies.

  • Poor Prognostic Factors:
    • High disease activity at presentation (high TJC/SJC, high ESR/CRP).
    • Early erosions on X-ray.
    • Positive RF and/or Anti-CCP (especially high titres).
    • Multiple joint involvement.
    • Extra-articular manifestations.
    • Female sex.
    • Older age at onset.
    • Smoking.
    • Lower socioeconomic status, poor functional status at baseline.
    • Certain genetic markers (e.g., HLA-DRB1 shared epitope).
  • Outcomes:
    • Without effective treatment, ~50% of patients become unable to work within 10 years.
    • With early and aggressive treatment, many patients can achieve remission or low disease activity, maintain function, and prevent significant joint damage.
    • Increased risk of cardiovascular disease (MI, stroke) is a major contributor to mortality. Effective control of RA inflammation may reduce this risk.
    • Life expectancy can be reduced by 3-10 years, mainly due to CVD, infection, and malignancy.

Complications of Rheumatoid Arthritis

RA can lead to various complications, both from the disease itself and its treatment.

Articular Complications:

  • Progressive joint damage, erosions, and deformities.
  • Joint instability and subluxation.
  • Loss of function and disability.
  • Secondary osteoarthritis in damaged joints.
  • Tendon ruptures (e.g., extensor tendons in hand, Achilles).
  • Carpal tunnel syndrome and other entrapment neuropathies.
  • Septic arthritis (increased risk due to joint damage and immunosuppression).
  • Atlantoaxial subluxation (C1-C2 instability): Can lead to neck pain, occipital headache, neurological deficits (weakness, sensory changes, UMN signs), and potentially spinal cord compression (a neurosurgical emergency).

Systemic & Extra-Articular Complications:

  • Cardiovascular Disease:
    • Accelerated atherosclerosis leading to increased risk of MI, stroke, peripheral vascular disease. This is a major cause of excess mortality in RA.
    • Pericarditis, myocarditis, heart failure.
  • Osteoporosis: Due to chronic inflammation, steroid use, immobility. Increased fracture risk.
  • Infections: Increased risk due to immune dysregulation and immunosuppressive therapies (DMARDs, biologics, steroids). Common sites: respiratory, skin, urinary tract. Opportunistic infections.
  • Malignancy:
    • Slightly increased risk of lymphoma (especially diffuse large B-cell lymphoma), possibly related to chronic inflammation/immune stimulation.
    • Lung cancer (especially in smokers with RA).
    • Skin cancer (non-melanoma) with some therapies.
  • Interstitial Lung Disease (ILD): Can lead to progressive respiratory failure.
  • Felty’s Syndrome: RA + splenomegaly + neutropenia. High risk of severe infections.
  • Amyloidosis (AA type): Rare, due to chronic inflammation. Can cause renal failure, proteinuria, cardiomyopathy.
  • Vasculitis: Can affect skin, nerves, and internal organs.
  • Depression and Anxiety: Common due to chronic pain, disability, and impact on quality of life.

Complications Related to Treatment:

  • Side effects of NSAIDs (GI bleeding, renal impairment, cardiovascular events).
  • Side effects of corticosteroids (osteoporosis, diabetes, hypertension, infections, Cushingoid features, skin thinning, cataracts).
  • Toxicity of DMARDs (e.g., MTX: hepatotoxicity, bone marrow suppression, pneumonitis; HCQ: retinopathy).
  • Increased infection risk with biologics and JAK inhibitors (including TB reactivation, herpes zoster).

Special Considerations in Rheumatoid Arthritis

Pregnancy and Breastfeeding:

  • Disease Activity: RA may improve in ~50-75% of women during pregnancy (especially 3rd trimester), but often flares postpartum.
  • Pre-conception Counselling: Essential. Aim for well-controlled disease on pregnancy-compatible medications before conception.
    • Compatible Medications:
      • Sulfasalazine, Hydroxychloroquine are generally considered safe.
      • Corticosteroids (e.g., prednisolone) at lowest effective dose if needed.
      • Some TNF inhibitors (e.g., certolizumab pegol has low placental transfer; etanercept, adalimumab may be continued until end of 2nd/early 3rd trimester after discussion).
    • Contraindicated Medications (Must be stopped pre-conception):
      • Methotrexate: Teratogenic. Stop ≥3 months before conception (men and women).
      • Leflunomide: Teratogenic (long half-life). Stop ≥2 years before conception or undergo washout procedure.
      • Most other biologics and JAK inhibitors have limited safety data or are contraindicated.
  • Breastfeeding: Sulfasalazine, hydroxychloroquine, prednisolone generally considered compatible. Data limited for many biologics. MTX contraindicated.

Surgery:

  • Patients with RA may require elective orthopaedic surgery (e.g., joint replacement) or other surgeries.
  • Peri-operative Management of Medications:
    • NSAIDs: Usually stopped a few days before major surgery (risk of bleeding, renal impairment).
    • Corticosteroids: Continue. May need stress dose cover if on long-term steroids (e.g., hydrocortisone IV) for major surgery.
    • csDMARDs:
      • Methotrexate: Can often be continued peri-operatively if renal function stable and no active infection (some centres stop 1-2 weeks prior).
      • Sulfasalazine, Hydroxychloroquine: Usually continued.
      • Leflunomide: Often stopped 1-2 weeks prior due to long half-life and wound healing concerns.
    • Biologics & JAK Inhibitors: Usually withheld for a period before and after surgery to reduce infection risk (timing depends on drug half-life, e.g., stop anti-TNF 2-4 weeks prior, restart once wound healed and no infection). Decision by rheumatologist and surgeon.
  • Cervical Spine Assessment: Pre-operative X-rays (flexion/extension views) of cervical spine may be needed before general anaesthesia if C-spine involvement suspected, to assess for atlantoaxial instability.
  • Increased risk of post-operative complications (infection, poor wound healing).

Vaccinations:

  • Patients with RA, especially those on immunosuppressants, are at increased risk of infections.
  • Recommended Vaccinations:
    • Annual influenza vaccine.
    • Pneumococcal vaccine (PPV23 and PCV13 as per guidelines).
    • Hepatitis B vaccine (if at risk or before some biologics).
    • Shingles (Shingrix – recombinant, non-live vaccine) recommended for patients ≥50 years, especially before or on JAK inhibitors/biologics.
  • Live vaccines (e.g., MMR, yellow fever, live shingles vaccine Zostavax) are generally contraindicated in patients on significant immunosuppression (biologics, JAK inhibitors, high-dose steroids, MTX >25mg/week). Should be given before starting such therapies if possible.

Flashcards: Rheumatoid Arthritis

Click on each card to reveal the answer.

Which autoantibody is highly specific for Rheumatoid Arthritis?

(Click to flip)

Answer

Anti-Cyclic Citrullinated Peptide (Anti-CCP) antibodies (also known as ACPA). Specificity >95%.

What is the first-line DMARD for most patients with RA? What supplement is co-prescribed?

(Click to flip)

Answer

Methotrexate (once weekly).
Folic Acid (5mg once weekly, on a different day) is co-prescribed to reduce side effects.

Name three common hand deformities seen in late-stage RA.

(Click to flip)

Answer

1. Ulnar deviation.
2. Swan neck deformity.
3. Boutonnière deformity.
(Also: Z-thumb, volar subluxation of MCPJs).

What are the components of the DAS28 score?

(Click to flip)

Answer

1. Tender Joint Count (out of 28).
2. Swollen Joint Count (out of 28).
3. ESR or CRP.
4. Patient Global Assessment of health (0-100mm VAS).

What is Felty’s Syndrome?

(Click to flip)

Answer

A rare complication of RA characterized by the triad of:
1. Rheumatoid Arthritis
2. Splenomegaly
3. Neutropenia

Which joints in the hand are typically spared in RA but commonly affected in Osteoarthritis?

(Click to flip)

Answer

Distal Interphalangeal (DIP) joints. (RA typically affects MCPs and PIPs).

What is the primary mechanism of action of TNF-alpha inhibitors (e.g., Adalimumab, Etanercept)?

(Click to flip)

Answer

They block the action of Tumor Necrosis Factor-alpha (TNF-α), a key pro-inflammatory cytokine involved in the pathogenesis of RA.

What serious ocular side effect is associated with Hydroxychloroquine?

(Click to flip)

Answer

Retinal toxicity (maculopathy). Requires baseline and regular ophthalmological screening.

Rheumatoid Arthritis Quiz

Test your knowledge with these Rheumatology-focused questions.

1. A 45-year-old woman presents with a 3-month history of symmetrical pain and swelling in her MCP and PIP joints, and morning stiffness lasting 2 hours. Which blood test is most specific for diagnosing Rheumatoid Arthritis?

A. Erythrocyte Sedimentation Rate (ESR).
B. Rheumatoid Factor (RF).
C. Anti-Cyclic Citrullinated Peptide (Anti-CCP) antibody.
D. Antinuclear Antibody (ANA).
Explanation: Anti-CCP antibodies have a much higher specificity for RA (>95%) compared to RF. ESR is a non-specific inflammatory marker. ANA is more associated with SLE and other connective tissue diseases.

2. A patient newly diagnosed with RA is started on weekly Methotrexate. Which of the following is a common side effect that co-prescription of folic acid aims to reduce?

A. Hypertension.
B. Mouth ulcers and nausea.
C. Retinal toxicity.
D. Skin hyperpigmentation.
Explanation: Folic acid supplementation helps reduce methotrexate-induced side effects such as mucositis (mouth ulcers), nausea, and bone marrow suppression. Retinal toxicity is associated with hydroxychloroquine.

3. Which of the following is a characteristic X-ray finding in established Rheumatoid Arthritis?

A. Osteophytes.
B. Subchondral sclerosis.
C. Marginal erosions.
D. Preservation of joint space.
Explanation: Marginal erosions are a hallmark of RA, caused by pannus invading bone. Osteophytes and subchondral sclerosis are typical of osteoarthritis. Joint space narrowing (not preservation) occurs in RA due to cartilage loss.

4. A patient with long-standing RA develops severe deep eye pain, redness, and photophobia. What is the most likely serious ocular complication?

A. Episcleritis.
B. Scleritis.
C. Keratoconjunctivitis sicca.
D. Cataract.
Explanation: Scleritis is a serious, painful inflammation of the sclera associated with RA, which can threaten vision. Episcleritis is more superficial and less painful. Keratoconjunctivitis sicca causes dry eyes. Cataracts can be steroid-induced.

5. Before starting a TNF-alpha inhibitor (e.g., Adalimumab) for RA, screening for which infection is mandatory?

A. HIV.
B. Latent Tuberculosis (TB).
C. Cytomegalovirus (CMV).
D. Influenza.
Explanation: TNF-alpha inhibitors can reactivate latent TB. Therefore, screening (e.g., chest X-ray and IGRA/Mantoux test) and treatment for latent TB if positive are required before initiation. Hepatitis B/C screening is also important.

6. Which of these medications used in RA is strictly contraindicated in pregnancy due to teratogenicity and requires stopping at least 3 months prior to conception by both men and women?

A. Sulfasalazine.
B. Hydroxychloroquine.
C. Methotrexate.
D. Prednisolone.
Explanation: Methotrexate is a potent teratogen and must be discontinued at least 3 months (some guidelines say up to 6 months) before attempting conception by both partners. Leflunomide is also highly teratogenic.

7. A “Swan Neck” deformity in RA involves:

A. Flexion at PIP joint and hyperextension at DIP joint.
B. Hyperextension at PIP joint and flexion at DIP joint.
C. Ulnar deviation at MCP joint and radial deviation at wrist.
D. Flexion at MCP joint and hyperextension at PIP joint.
Explanation: Swan neck deformity is characterized by hyperextension of the proximal interphalangeal (PIP) joint and flexion of the distal interphalangeal (DIP) joint. Flexion at PIP and hyperextension at DIP is a Boutonnière deformity.

8. What is the primary aim of the “Treat-to-Target” strategy in RA management?

A. Solely to alleviate pain using NSAIDs.
B. To achieve clinical remission or low disease activity by adjusting therapy based on regular monitoring.
C. To replace all damaged joints surgically as soon as possible.
D. To rely only on non-pharmacological interventions.
Explanation: The “Treat-to-Target” strategy involves regular assessment of disease activity (e.g., using DAS28) and adjusting DMARD therapy accordingly, with the goal of achieving clinical remission or at least low disease activity to prevent joint damage and improve long-term outcomes.

9. Which of the following extra-articular manifestations is NOT typically associated with Rheumatoid Arthritis?

A. Rheumatoid nodules.
B. Interstitial lung disease.
C. Malar rash.
D. Pericarditis.
Explanation: A malar (butterfly) rash is characteristic of Systemic Lupus Erythematosus (SLE), not typically RA. Rheumatoid nodules, ILD, and pericarditis are well-recognized extra-articular features of RA.

10. A patient with RA on long-term methotrexate presents with increasing fatigue. FBC shows Hb 9.5 g/dL, MCV 85 fL, normal WBC and platelets. ESR is 60 mm/hr. What is the most likely type of anaemia?

A. Iron deficiency anaemia.
B. Anaemia of chronic disease.
C. Megaloblastic anaemia due to folate deficiency.
D. Autoimmune haemolytic anaemia.
Explanation: Anaemia of chronic disease is the most common type of anaemia in RA, characterized by normocytic (or mildly microcytic) red cells and related to chronic inflammation (high ESR). While MTX can cause folate deficiency leading to megaloblastic anaemia (high MCV), this patient’s MCV is normal. Iron deficiency can also occur but ACD is more typical with active RA.

Septic Arthritis (T&O Focus)

Septic arthritis is an infection within a joint space, most commonly bacterial. It is an orthopaedic emergency requiring prompt diagnosis and aggressive treatment (joint aspiration, surgical washout, and antibiotics) to prevent rapid cartilage destruction, joint damage, osteomyelitis, and systemic sepsis.

Overview & Pathophysiology of Septic Arthritis

Septic arthritis (or infectious arthritis) is an infection of a synovial joint, leading to inflammation and potential destruction of articular cartilage and subchondral bone.

Routes of Infection:

  • Haematogenous Spread (Most Common): Bacteria from a distant site of infection (e.g., skin, respiratory tract, urinary tract) travel via bloodstream to seed the highly vascular synovium.
  • Direct Inoculation:
    • Trauma (penetrating injury to the joint).
    • Iatrogenic (e.g., during joint aspiration, intra-articular injection, arthroscopy, or other joint surgery).
  • Contiguous Spread: From an adjacent osteomyelitis or soft tissue infection.

Pathophysiological Cascade:

  1. Bacterial Seeding & Synovial Inflammation: Bacteria colonise the synovium, triggering an intense acute inflammatory response. Neutrophils are recruited.
  2. Release of Inflammatory Mediators: Cytokines (TNF-α, IL-1, IL-6), prostaglandins, and leukotrienes are released, causing synovial oedema, increased vascular permeability, and pain.
  3. Enzyme Release & Cartilage Degradation: Neutrophils and synovial cells release proteolytic enzymes (e.g., collagenases, metalloproteinases) that degrade the articular cartilage matrix (proteoglycans and collagen). This process can be very rapid (within hours to days).
  4. Pus Formation & Increased Intra-articular Pressure: Accumulation of inflammatory exudate, bacteria, and cellular debris forms pus. Increased pressure can further compromise blood supply to cartilage and synovium.
  5. Subchondral Bone Involvement: If untreated, infection can spread to subchondral bone, leading to osteomyelitis and further joint destruction.
  6. Systemic Effects: Release of inflammatory mediators can cause systemic sepsis.

The avascular nature of adult articular cartilage makes it vulnerable to enzymatic degradation and unable to repair itself effectively once damaged.

Orthopaedic Emergency!

Septic arthritis can destroy a joint within 24-48 hours if untreated. Prompt diagnosis and urgent surgical drainage combined with appropriate antibiotics are crucial to preserve joint function and prevent long-term disability and systemic complications.

Epidemiology & Risk Factors for Septic Arthritis

Epidemiology:

  • Incidence: Approximately 2-10 cases per 100,000 person-years in the general population. Higher in specific risk groups.
  • Can affect any age, but more common in very young, elderly, and immunocompromised.
  • Most commonly affects large, weight-bearing joints:
    • Knee (most common, ~50% of cases).
    • Hip.
    • Shoulder, ankle, wrist, elbow.
    • Sternoclavicular and sacroiliac joints (especially in IV drug users).
  • Usually monoarticular (80-90% of cases). Polyarticular septic arthritis is less common but seen in patients with rheumatoid arthritis, immunosuppression, or disseminated gonococcal infection.

Risk Factors:

  • Pre-existing Joint Disease:
    • Rheumatoid arthritis (highest risk).
    • Osteoarthritis.
    • Gout / Pseudogout (crystal arthropathy).
    • Previous joint surgery or trauma.
  • Prosthetic Joint (Periprosthetic Joint Infection – PJI): Foreign material acts as a nidus for infection. (See Special Populations).
  • Increasing Age (>80 years).
  • Immunocompromised State:
    • Diabetes mellitus.
    • Chronic kidney disease / dialysis.
    • Alcoholism / Liver cirrhosis.
    • Malignancy (especially haematological).
    • HIV infection.
    • Immunosuppressive therapy (e.g., corticosteroids, biologics, chemotherapy).
  • Intravenous (IV) Drug Use: Often atypical joints (e.g., sternoclavicular, sacroiliac) and unusual organisms (e.g., Pseudomonas, Candida).
  • Recent Joint Aspiration or Injection.
  • Skin Infections / Ulcers: Allowing bacterial entry.
  • Bacteraemia from other sources (e.g., endocarditis, UTI, pneumonia).
  • Sickle Cell Disease (Salmonella is a notable pathogen).

Common Causative Organisms in Septic Arthritis

The most common pathogen varies by age group and risk factors.

Overall (Native Joints):

  • Staphylococcus aureus (Most Common Overall): Accounts for ~40-60% of cases. Includes Methicillin-Resistant S. aureus (MRSA).
  • Streptococcal species:
    • Streptococcus pyogenes (Group A Strep).
    • Streptococcus agalactiae (Group B Strep – neonates, elderly, diabetics).
    • Streptococcus pneumoniae (often polyarticular, associated with systemic infection).
    • Viridans group streptococci.
  • Gram-negative bacilli: More common in elderly, immunocompromised, IV drug users, or following GU/GI procedures.
    • Escherichia coli.
    • Pseudomonas aeruginosa (IV drug users, puncture wounds through footwear).
    • Klebsiella spp., Proteus spp.
  • Neisseria gonorrhoeae (Gonococcal Arthritis):
    • Common in sexually active young adults. Often presents as migratory polyarthralgia, tenosynovitis, and dermatitis, which can then localize to one or few joints (suppurative arthritis).
    • Synovial fluid cultures may be negative; culture from cervix, urethra, rectum, pharynx. NAAT testing.

Specific Age Groups / Situations:

  • Neonates (<2 months): Group B Streptococcus, S. aureus, Gram-negative bacilli (e.g., E. coli).
  • Infants & Young Children (<5 years): S. aureus, Streptococcus pneumoniae, Group A Streptococcus. Kingella kingae is increasingly recognized (requires specific culture media). Haemophilus influenzae type b (Hib) now rare due to vaccination.
  • Older Children & Adolescents: S. aureus. Consider N. gonorrhoeae if sexually active.
  • IV Drug Users: S. aureus (often MRSA), Pseudomonas aeruginosa, other Gram-negatives, Candida.
  • Prosthetic Joint Infections (PJI): Coagulase-negative staphylococci (e.g., Staphylococcus epidermidis), S. aureus, Streptococci, Gram-negatives, anaerobes, Cutibacterium (Propionibacterium) acnes (shoulder PJI). Often low-virulence organisms in delayed/late PJI.
  • Human or Animal Bites: Polymicrobial (aerobes and anaerobes), e.g., Pasteurella multocida (cat/dog bites), Eikenella corrodens (human bites).

Viral, fungal, and mycobacterial arthritis are much less common but can occur, especially in immunocompromised individuals.

Often, no organism is isolated (“culture-negative septic arthritis”) in up to 30-50% of cases, especially if antibiotics given prior to aspiration.

Clinical Features & Presentation of Septic Arthritis

Classic presentation is an acute monoarthritis, but this can vary.

Typical Symptoms & Signs (Often Rapid Onset):

  • Severe Joint Pain: Often described as intense, constant, and deep.
  • Swelling (Effusion): Joint appears visibly swollen.
  • Warmth and Erythema over the affected joint.
  • Markedly Restricted Range of Motion: Both active and passive movements are severely limited due to pain (“pseudoparalysis”). Patient often holds limb in position of maximal joint capacity (e.g., slight flexion).
  • Pain on Minimal Movement: Exquisite tenderness even with slight attempted movement or palpation.
  • Inability to Weight-Bear (if lower limb joint affected).
  • Systemic Symptoms:
    • Fever and Chills/Rigors: Common but may be absent, especially in elderly or immunocompromised patients. Absence of fever does NOT exclude septic arthritis.
    • Malaise, tachycardia, general unwellness.

Kocher Criteria for Septic Arthritis of the Hip in Children:

(Primarily for hip, but principles can be considered. Presence of more criteria increases likelihood).

  1. Non-weight-bearing on affected side.
  2. ESR >40 mm/hr.
  3. Fever >38.5°C.
  4. WBC count >12 x 109/L.

Probability of septic arthritis: 0 criteria = <0.2%; 1 = 3%; 2 = 40%; 3 = 93%; 4 = 99%.

Atypical Presentations:

  • Elderly/Immunocompromised: May have blunted inflammatory response, less fever, less pain, or more insidious onset. High index of suspicion needed.
  • Prosthetic Joint Infection (PJI): Can be acute (within weeks of surgery, similar to native joint SA) or chronic/delayed (insidious onset pain, loosening, sinus tract – often low-virulence organisms).
  • Polyarticular Septic Arthritis: Affects multiple joints. More common in RA, immunosuppression, gonococcal infection.
  • Deep Joints (Hip, Shoulder, SI joint): Swelling and erythema may be less obvious. Pain and restricted movement are key. Hip often held in flexion, abduction, external rotation.
  • Neonates/Infants: Non-specific signs – irritability, poor feeding, pseudoparalysis (not moving limb), fever or hypothermia.
⚠️ Key Red Flag Triad

A patient presenting with a hot, swollen, tender joint with severely restricted movement should be considered to have septic arthritis until proven otherwise. This is an orthopaedic emergency.

Diagnosis: Joint Aspiration & Synovial Fluid Analysis

Joint aspiration (arthrocentesis) is the cornerstone of diagnosis for suspected septic arthritis. It should be performed urgently, ideally BEFORE starting antibiotics, unless the patient is critically ill with sepsis.

Indications for Joint Aspiration:

  • Any acutely swollen, painful, warm joint with restricted movement, where septic arthritis is suspected.
  • To differentiate septic arthritis from other causes of acute monoarthritis (e.g., crystal arthropathy, haemarthrosis, reactive arthritis).

Procedure:

  • Aseptic technique is paramount.
  • Performed by trained personnel (often T&O or rheumatology). Ultrasound guidance can be helpful for deep joints (hip, shoulder) or small effusions.
  • Appropriate skin preparation (e.g., chlorhexidine or iodine).
  • Local anaesthesia (e.g., lignocaine) to skin and subcutaneous tissue, avoiding injection into the joint space if possible.
  • Use appropriate needle size (e.g., 19-21G for knee, larger joints; smaller for smaller joints).
  • Aspirate as much fluid as possible.

Synovial Fluid Analysis – Key Tests:

TestTypical Findings in Septic ArthritisNotes
Appearance Turbid, opaque, purulent (pus-like), yellowish-green. Low viscosity. Normal fluid is clear, straw-coloured, viscous. Inflammatory (non-septic) can be cloudy but less opaque.
White Blood Cell (WBC) Count & Differential Markedly elevated: Often >50,000 cells/µL (can be >100,000). Predominance of Neutrophils (>75-90%). WBC count can be lower in early infection, immunocompromised, or with certain organisms (e.g., gonococcal). Crystal arthropathy: WBC usually 10,000-50,000, neutrophils predominate. RA: WBC often 5,000-25,000.
Gram Stain May identify bacteria (positive in ~30-70% of bacterial SA). Guides initial antibiotic choice. Negative Gram stain does NOT exclude septic arthritis. Depends on bacterial load, prior antibiotics.
Culture & Sensitivity (Aerobic & Anaerobic) Gold standard for confirming bacterial SA and guiding definitive antibiotic therapy. Inoculate into blood culture bottles if available for better yield. May be negative if antibiotics given beforehand or with fastidious organisms (e.g., Kingella, gonococcus).
Crystal Microscopy (Polarised Light) Absence of crystals. (Presence of crystals does not exclude coexistent septic arthritis – “septic gout” can occur). Monosodium urate (MSU) crystals (needle-shaped, negatively birefringent) = Gout. Calcium pyrophosphate dihydrate (CPPD) crystals (rhomboid, weakly positively birefringent) = Pseudogout.
Glucose Low (often <50% of serum glucose) - bacteria consume glucose. Less specific.
Lactate / LDH May be elevated. Less specific. Lactate >10 mmol/L is suggestive.

Other Important Investigations:

  • Blood Cultures: Positive in up to 50% of cases of septic arthritis (especially S. aureus, Streptococci). Take BEFORE antibiotics.
  • Full Blood Count (FBC): Often shows leucocytosis with neutrophilia.
  • Inflammatory Markers: ESR and CRP usually markedly elevated. CRP rises and falls more rapidly than ESR, useful for monitoring response to treatment.
  • U&Es, Creatinine: Assess renal function (for antibiotic dosing, rhabdomyolysis risk).
  • Coagulation Screen: If invasive procedures planned or underlying coagulopathy.
  • Urate: If gout suspected (may be normal in acute attack).
  • Cultures from other suspected primary sites of infection (e.g., urine, throat, skin lesions, cervix/urethra for gonococcus).

Imaging in Suspected Septic Arthritis

Imaging is primarily adjunctive to clinical assessment and synovial fluid analysis. It should NOT delay urgent aspiration or treatment if septic arthritis is strongly suspected.

Plain X-rays (Affected Joint & Contralateral for Comparison):

  • Often normal in early septic arthritis. Main initial role is to exclude other pathology (e.g., fracture, established OA, bone tumour, chronic osteomyelitis).
  • Early signs (subtle): Soft tissue swelling, joint effusion (widening of joint space), periarticular osteopenia (if longstanding inflammation).
  • Late signs (indicate joint destruction, usually after >1-2 weeks):
    • Joint space narrowing (cartilage loss).
    • Subchondral bone erosions.
    • Periosteal reaction.
    • Signs of osteomyelitis.
    • Joint subluxation/dislocation.

Ultrasound (USS):

  • Highly sensitive for detecting joint effusion.
  • Can guide joint aspiration, especially for deep or difficult-to-access joints (e.g., hip, shoulder).
  • May show synovial thickening, increased vascularity on Doppler (suggests inflammation).
  • Cannot definitively differentiate septic from non-septic effusions based on appearance alone.
  • Can assess adjacent soft tissues for abscesses or cellulitis.

Magnetic Resonance Imaging (MRI):

  • Most sensitive imaging modality for detecting early changes of septic arthritis and associated complications.
  • Findings: Joint effusion, synovial enhancement (inflammation), cartilage damage, bone marrow oedema (early osteomyelitis), adjacent soft tissue inflammation/abscesses.
  • Indications:
    • Suspected septic arthritis of deep joints (hip, SI joint, sternoclavicular) if diagnosis uncertain or aspiration difficult/non-diagnostic.
    • To rule out or confirm adjacent osteomyelitis.
    • When clinical suspicion is high but aspiration is negative or equivocal.
    • Pre-operative planning if complex infection.
  • Not routinely required for straightforward cases where aspiration is diagnostic. Can delay treatment if used inappropriately.

Computed Tomography (CT):

  • Less useful for soft tissue detail than MRI.
  • May be used to assess bony destruction, sequestra (in chronic osteomyelitis), or guide aspiration/biopsy in some cases.
  • CT arthrogram can show cartilage defects.

Radionuclide Bone Scan (e.g., Technetium-99m MDP) / Labelled WBC Scan:

  • Historically used, less common now with availability of MRI.
  • Bone scan shows increased uptake (non-specific inflammation/turnover). Labelled WBC scan more specific for infection.
  • May be useful if multiple sites of infection suspected or MRI contraindicated.

Management of Septic Arthritis: Antibiotics & Surgical Drainage

Management involves two key principles: 1) Eradication of infection with appropriate antibiotics, and 2) Removal of pus and infected material from the joint via drainage.

1. Empirical Intravenous (IV) Antibiotics:

  • Start URGENTLY after joint aspiration and blood cultures have been taken (unless patient critically septic, then start immediately after cultures and aspirate concurrently if possible). Do not delay if aspiration difficult/delayed and strong clinical suspicion.
  • Initial choice is empirical, based on likely pathogens (age, risk factors, Gram stain result if available). Cover Staphylococcus aureus and Streptococci as a minimum.
    • Common empirical regimen (Native Joint, otherwise healthy adult): Flucloxacillin IV.
      • Alternatives if penicillin allergic: Clindamycin IV or Vancomycin IV (especially if MRSA suspected/prevalent).
    • Add Gram-negative cover (e.g., Ceftriaxone IV or Gentamicin IV) if:
      • Elderly, immunocompromised.
      • Recent GU/GI procedure or infection.
      • IV drug use (consider anti-pseudomonal like Ceftazidime or Piperacillin-Tazobactam).
      • Gram stain shows Gram-negative organisms.
    • Consider adding Rifampicin if prosthetic joint infection (PJI) or osteomyelitis suspected (aids biofilm penetration – specialist advice).
  • Gonococcal Arthritis: Ceftriaxone IV (or IM). Treat for chlamydia too (azithromycin or doxycycline).
  • Tailor antibiotics once culture and sensitivity results are available.
  • Duration of IV therapy: Typically at least 1-2 weeks, guided by clinical response, inflammatory markers, and organism. Then switch to appropriate oral antibiotics.
  • Total duration of antibiotics (IV + oral): Usually 4-6 weeks for native joint septic arthritis. Longer if osteomyelitis or PJI.

2. Joint Drainage / Debridement (Surgical Emergency):

Essential to remove pus, infected debris, and reduce intra-articular pressure to prevent cartilage damage and allow antibiotics to penetrate effectively.

  • Needle Aspiration (Arthrocentesis):
    • Can be diagnostic and therapeutic initially for easily accessible joints (e.g., knee).
    • Daily or frequent aspirations may be needed if significant re-accumulation.
    • May be sufficient for some early infections or certain organisms (e.g., gonococcal) if rapid clinical improvement.
  • Arthroscopic Lavage and Debridement:
    • Preferred method for most large joints (knee, shoulder, ankle, elbow).
    • Allows direct visualisation, thorough washout of the joint with large volumes of saline, removal of fibrin/pus, synovial biopsy, and debridement of infected/necrotic tissue.
    • Less invasive than open arthrotomy.
  • Open Arthrotomy and Washout:
    • Indicated for:
      • Hip joint infection (especially in children, though arthroscopy is increasingly used).
      • Joints not easily accessible by arthroscopy or if arthroscopy fails.
      • Presence of extensive necrotic tissue or loculated pus.
      • Prosthetic joint infections (often requiring implant removal/exchange).
    • Allows more extensive debridement and drainage.

Other Management Principles:

  • Analgesia: Adequate pain relief is crucial (opioids often required initially).
  • Splinting/Immobilisation: Initially for pain relief (e.g., backslab, sling).
  • Early Mobilisation & Physiotherapy: Once infection controlled and pain allows, to prevent stiffness, contractures, and muscle atrophy. Crucial for functional recovery.
  • Monitor Clinical Response: Pain, swelling, range of motion, temperature, systemic signs.
  • Monitor Inflammatory Markers: CRP should start to fall with effective treatment (within days). ESR falls more slowly.
  • Address and manage any underlying risk factors.
  • Consider DVT prophylaxis in immobilised patients.
“The Solution to Pollution is Dilution (and Debridement)!”

This orthopaedic adage emphasizes the importance of copious joint lavage/washout to remove infected material in septic arthritis.

Septic Arthritis in Special Populations

1. Prosthetic Joint Infection (PJI):

  • A devastating complication of arthroplasty. Infection involving the implant and surrounding tissues.
  • Classification by Onset:
    • Early (<3 months post-op): Often due to intraoperative contamination or wound healing issues. High-virulence organisms (S. aureus, Gram-negatives). Acute presentation.
    • Delayed (3-12/24 months post-op): Often low-virulence organisms (Coagulase-negative staphylococci, C. acnes). More insidious onset.
    • Late (>12/24 months post-op): Usually haematogenous spread from distant infection. Acute presentation.
  • Diagnosis: Complex. Combination of clinical features (pain, swelling, sinus tract, fever), inflammatory markers (ESR, CRP), imaging (X-ray for loosening, bone scan/WBC scan), multiple synovial fluid/tissue cultures. MSIS criteria often used.
  • Management: Requires specialist multidisciplinary input (ortho, micro, ID).
    • DAIR (Debridement, Antibiotics, and Implant Retention): For acute early PJI or acute late haematogenous PJI with stable implant and susceptible organism, if symptoms <3-4 weeks. Involves surgical debridement, exchange of modular components (e.g., liner), and long-term antibiotics (often with biofilm-active agents like rifampicin).
    • One-Stage Exchange Arthroplasty: Removal of infected implant, thorough debridement, and reimplantation of new prosthesis in single operation. For selected cases.
    • Two-Stage Exchange Arthroplasty (Most Common for Chronic PJI):
      • Stage 1: Removal of infected implant, radical debridement, insertion of antibiotic-loaded cement spacer. Followed by 4-8 weeks of IV/oral antibiotics.
      • Stage 2: After infection eradicated (normal inflammatory markers, negative cultures from spacer aspiration), reimplantation of new prosthesis.
    • Permanent Resection Arthroplasty (Girdlestone – hip) or Arthrodesis (fusion).
    • Amputation (rare, for limb salvage failure).
    • Long-term suppressive antibiotics may be used if surgery not feasible.

2. Septic Arthritis in Children:

  • Knee and hip are common sites. Hip SA in neonates/infants is an emergency to prevent femoral head necrosis.
  • Presentation: Fever, irritability, pseudoparalysis (refusal to move limb/bear weight), localized tenderness, swelling, warmth. Kocher criteria useful for hip.
  • Organisms: S. aureus, Streptococci. Kingella kingae important in <4 yrs (requires specific culture). Hib rare with vaccination.
  • Diagnosis: High index of suspicion. USS useful for hip effusion. Joint aspiration essential.
  • Management: Urgent surgical drainage (arthrotomy for hip, arthroscopy/arthrotomy for knee) and IV antibiotics. Risk of growth plate damage (physeal arrest) if infection involves epiphysis/physis.

Complications & Prognosis of Septic Arthritis

Complications:

  • Rapid Joint Destruction: Cartilage loss, erosions, leading to secondary osteoarthritis.
  • Osteomyelitis: Spread of infection to adjacent bone.
  • Joint Stiffness & Contractures: Due to pain, inflammation, fibrosis.
  • Chronic Pain.
  • Joint Instability or Subluxation/Dislocation.
  • Limb Length Discrepancy or Angular Deformity (in children if growth plate affected).
  • Avascular Necrosis (AVN): Especially of femoral head in paediatric hip septic arthritis.
  • Sinus Tract Formation (especially in chronic or PJI).
  • Systemic Sepsis & Septic Shock.
  • Metastatic Infection to other sites.
  • Mortality: ~5-15% for native joint septic arthritis, higher in elderly/comorbid patients.

Prognosis:

Depends on several factors:

  • Time to Diagnosis and Treatment: Early intervention (within hours) is critical. Delays >24-48 hours significantly worsen outcomes.
  • Organism Virulence & Susceptibility: More aggressive organisms or resistant strains lead to poorer prognosis.
  • Host Factors: Age, comorbidities (diabetes, RA, immunosuppression), pre-existing joint damage.
  • Joint Involved: Hip and shoulder infections may have worse functional outcomes than knee.
  • Adequacy of Surgical Drainage and Antibiotic Therapy.

Even with optimal treatment, up to 25-50% of survivors may have some degree of permanent joint dysfunction or residual pain.

Flashcards: Septic Arthritis

Click on each card to reveal the answer.

What is the most common causative organism in native joint septic arthritis overall?

(Click to flip)

Answer

Staphylococcus aureus.

A patient presents with an acutely hot, swollen, tender knee with severely restricted movement and fever. What is the immediate diagnostic priority?

(Click to flip)

Answer

Urgent joint aspiration (arthrocentesis) for synovial fluid analysis (WBC count, Gram stain, culture, crystals).

What is a typical synovial fluid WBC count highly suggestive of septic arthritis?

(Click to flip)

Answer

Often >50,000 cells/µL with a predominance of neutrophils (>75-90%).

What are the two main pillars of treatment for septic arthritis?

(Click to flip)

Answer

1. Appropriate (initially empirical IV) antibiotics.
2. Urgent joint drainage/debridement (e.g., arthroscopic lavage or arthrotomy).

Which joint is most commonly affected by septic arthritis?

(Click to flip)

Answer

The knee.

What does DAIR stand for in the context of managing prosthetic joint infections?

(Click to flip)

Answer

Debridement, Antibiotics, and Implant Retention.

A sexually active young adult presents with migratory polyarthralgia, tenosynovitis, and a rash, followed by a monoarthritis. What organism should be suspected?

(Click to flip)

Answer

Neisseria gonorrhoeae (Gonococcal arthritis).

What are the Kocher criteria used for?

(Click to flip)

Answer

To help assess the probability of septic arthritis of the hip in children (non-weight-bearing, ESR >40, fever >38.5°C, WBC >12).

Septic Arthritis Quiz (T&O Focus)

Test your knowledge on this orthopaedic emergency.

1. A 65-year-old diabetic patient presents with a 2-day history of a severely painful, swollen, warm, and erythematous right knee. They are unable to weight-bear and have a fever of 38.7°C. What is the most crucial initial investigation?

A. Plain X-ray of the knee.
B. Urgent joint aspiration (arthrocentesis).
C. MRI scan of the knee.
D. Serum urate level.
Explanation: The clinical picture is highly suggestive of septic arthritis. Urgent joint aspiration for synovial fluid analysis (Gram stain, culture, WBC count, crystals) is paramount for diagnosis and guiding treatment.

2. Synovial fluid analysis from a suspected septic joint reveals a WBC count of 80,000 cells/µL with 90% neutrophils, and no crystals seen on polarised microscopy. Gram stain is pending. What is the most appropriate next step in management?

A. Wait for culture results before starting treatment.
B. Start oral amoxicillin and observe.
C. Start empirical IV antibiotics and arrange for urgent surgical joint drainage/washout.
D. Prescribe NSAIDs and colchicine for presumed gout.
Explanation: A synovial WBC count >50,000 with neutrophil predominance is highly indicative of septic arthritis. Urgent surgical drainage and empirical IV antibiotics are required.

3. Which of these organisms is the most common cause of septic arthritis in a sexually active young adult?

A. Staphylococcus aureus.
B. Streptococcus pneumoniae.
C. Neisseria gonorrhoeae.
D. Escherichia coli.
Explanation: Disseminated gonococcal infection is a common cause of septic arthritis (often preceded by migratory arthralgia/tenosynovitis/dermatitis) in this demographic.

4. What is the primary reason for performing urgent surgical drainage (e.g., arthroscopic lavage or arthrotomy) in septic arthritis?

A. To obtain a definitive microbiological diagnosis.
B. To remove pus, infected debris, and proteolytic enzymes to prevent cartilage destruction.
C. To primarily relieve pain.
D. To allow for earlier weight-bearing.
Explanation: Surgical drainage is critical to remove the destructive elements (pus, enzymes, bacteria) from the joint space, thereby reducing intra-articular pressure and preventing irreversible damage to the articular cartilage.

5. A patient with a prosthetic knee joint develops acute pain, swelling, and fever 2 years after surgery, following a dental infection. This is most likely:

A. Aseptic loosening of the prosthesis.
B. Early postoperative prosthetic joint infection (PJI).
C. Late (haematogenous) prosthetic joint infection (PJI).
D. Metallosis.
Explanation: Acute onset PJI occurring >1-2 years post-op, especially after a known bacteraemic event (like a dental infection), suggests late haematogenous seeding of the prosthesis.

6. Which of the following imaging modalities is most sensitive for detecting early osteomyelitis associated with septic arthritis?

A. Plain X-ray.
B. Ultrasound.
C. MRI scan.
D. CT scan.
Explanation: MRI is highly sensitive for detecting bone marrow oedema, an early sign of osteomyelitis, as well as assessing soft tissues and cartilage. X-rays are often normal in early OM.

7. What is the typical duration of total antibiotic therapy (IV followed by oral) for native joint septic arthritis?

A. 7-10 days.
B. 2 weeks.
C. 4-6 weeks.
D. 3-6 months.
Explanation: The standard duration of antibiotic therapy for uncomplicated native joint septic arthritis is typically 4-6 weeks, starting with IV and then switching to oral once clinically improving and inflammatory markers are settling.

8. A negative Gram stain of synovial fluid from a clinically suspected septic joint means:

A. Septic arthritis is definitively ruled out.
B. Septic arthritis cannot be ruled out, as Gram stain sensitivity is limited.
C. The infection is likely viral.
D. The patient has received effective antibiotic therapy.
Explanation: Gram stain has a sensitivity of only ~30-70% for septic arthritis. A negative Gram stain does not exclude the diagnosis, and treatment should be based on strong clinical suspicion and other synovial fluid parameters (WBC count) pending culture results.

9. In children, septic arthritis of which joint is considered a particular emergency due to the risk of avascular necrosis of the femoral head?

A. Knee.
B. Hip.
C. Ankle.
D. Shoulder.
Explanation: Septic arthritis of the hip in neonates and infants is a true emergency. The increased intra-articular pressure can compromise the precarious blood supply to the femoral head epiphysis, leading to AVN and long-term growth disturbance.

10. Which of the following is a major risk factor for developing septic arthritis?

A. Hypertension.
B. Pre-existing rheumatoid arthritis.
C. Hypercholesterolaemia.
D. Osteoporosis.
Explanation: Patients with rheumatoid arthritis have a significantly increased risk of septic arthritis due to underlying joint damage, immune dysregulation, and often immunosuppressive therapy.

Paediatric Orthopaedics: Clubfoot Assessment

Clubfoot (Congenital Talipes Equinovarus – CTEV) is a common congenital foot deformity characterized by a complex, three-dimensional malalignment of the foot. Accurate assessment is crucial for diagnosis, determining severity, guiding treatment (primarily the Ponseti method), and monitoring progress.

Overview & Definitions of Clubfoot

Clubfoot is a developmental deformity where the infant’s foot is twisted out of shape or position. The term “Talipes Equinovarus” describes the position:

  • Talipes: Referring to the talus (ankle bone) and pes (foot).
  • Equinus: Plantarflexion of the foot at the ankle (like a horse’s foot).
  • Varus: Inversion and adduction of the hindfoot/heel.

Key Characteristics:

  • Incidence: Approximately 1-2 per 1000 live births. More common in males (2:1).
  • Laterality: Bilateral in ~50% of cases.
  • Types:
    • Idiopathic Clubfoot (most common, ~80%): Occurs in otherwise healthy infants, cause unknown (likely multifactorial – genetic and environmental). Responds well to Ponseti method.
    • Syndromic/Secondary Clubfoot (~20%): Associated with underlying neuromuscular disorders (e.g., spina bifida, arthrogryposis, cerebral palsy), genetic syndromes (e.g., Larsen syndrome, Streeter’s dysplasia), or teratogenic factors. Often more rigid and resistant to treatment.
  • Importance of Early Assessment: Early diagnosis and initiation of treatment (ideally within first few weeks of life) lead to best outcomes, minimizing long-term disability.
💡 OSCE/PLAB Tip

When assessing an infant with clubfoot, always handle the foot gently. Reassure parents that idiopathic clubfoot is treatable, and the Ponseti method has high success rates. Remember to screen for other congenital anomalies, especially hip dysplasia and spinal abnormalities.

🧠 Clubfoot Deformity Components: “CAVE”
C Cavus (High arch, forefoot plantarflexion relative to hindfoot)
A Adductus (Forefoot deviated medially)
V Varus (Hindfoot inverted/tilted inwards)
E Equinus (Hindfoot/ankle plantarflexed)

Relevant Anatomy & Pathoanatomy in Clubfoot

Understanding the abnormal anatomy is key to understanding the deformity and its correction.

Bony Deformities:

  • Talus: Plantarflexed in the ankle mortise. The neck is medially and plantarly deviated. The head of the talus is prominent laterally.
  • Calcaneus: Inverted (varus), adducted, and plantarflexed (equinus) beneath the talus. It is medially rotated.
  • Navicular: Severely medially displaced and articulates with the medial aspect of the talar head (instead of anteriorly). Often subluxed dorsally.
  • Cuboid: Medially displaced relative to the calcaneus.
  • Metatarsals: Adducted and supinated, contributing to the cavus and adductus.

Soft Tissue Contractures:

The bony deformities are held in place by tight and contracted ligaments, tendons, and joint capsules, primarily on the postero-medial aspect of the foot and ankle.

  • Posteriorly: Contracted Achilles tendon, posterior capsule of ankle and subtalar joints, posterior talofibular and calcaneofibular ligaments.
  • Medially: Contracted tibialis posterior tendon (often the tightest), flexor hallucis longus, flexor digitorum longus tendons. Contracted deltoid ligament, talonavicular capsule, spring ligament, master knot of Henry.
  • Plantar: Contracted plantar fascia and intrinsic muscles.

Muscle Abnormalities:

  • Calf muscles (gastrocnemius, soleus) are often smaller/atrophied on the affected side, even after successful correction. This is usually a permanent finding.
  • Tibialis anterior tendon may be medially inserted.

The Ponseti method aims to gradually stretch these contracted soft tissues, allowing the bones to be realigned into a more anatomical position.

Clinical Assessment of Clubfoot

A thorough history and systematic examination are essential.

History:

  • Prenatal History:
    • Antenatal ultrasound findings (was clubfoot suspected?).
    • Oligohydramnios, polyhydramnios.
    • Maternal health (diabetes, infections, medications).
  • Birth History:
    • Gestational age (premature infants may have positional deformities).
    • Mode of delivery, any complications.
    • Apgar scores.
  • Family History:
    • Clubfoot in parents or siblings (increases risk).
    • Other congenital musculoskeletal deformities or neuromuscular conditions in the family.
  • Postnatal History:
    • Any other congenital anomalies identified.
    • Feeding, general health.

Examination:

Perform in a warm environment with the infant calm. Always compare with the contralateral foot if unilateral.

  • General Examination:
    • Overall appearance, symmetry, any dysmorphic features (to look for syndromic causes).
    • Assess for other MSK abnormalities:
      • Hips: Screen for Developmental Dysplasia of the Hip (DDH) using Ortolani and Barlow manoeuvres (though less reliable in first few days). Check for leg length discrepancy, asymmetrical skin creases.
      • Spine: Inspect and palpate for sacral dimples, hairy patches, lipomas, or other signs of spinal dysraphism (e.g., tethered cord).
      • Other limbs for contractures or deformities.
    • Brief neurological assessment (tone, reflexes).
  • Foot Examination (Look, Feel, Move):
    • Look (Inspect):
      • Describe the CAVE components systematically:
        • Cavus: Increased medial arch, forefoot plantarflexion.
        • Adductus: Medial deviation of the forefoot.
        • Varus: Heel inverted and tilted inwards.
        • Equinus: Ankle and hindfoot in plantarflexion.
      • Skin Creases: Deep posterior and medial skin creases are characteristic. Absence of normal lateral creases.
      • Calf Size: Calf on affected side may be smaller.
      • Foot Size: Affected foot may be shorter and wider.
      • Heel: Small, high-riding, and difficult to palpate. “Empty heel” sign (gap below lateral malleolus where calcaneus should be).
      • Lateral Border: Curved (convex).
      • Medial Border: Concave.
    • Feel (Palpate):
      • Palpate bony landmarks (often difficult due to deformity and soft tissue):
        • Medial malleolus.
        • Lateral malleolus.
        • Head of Talus: Often prominent on the dorsolateral aspect of the foot.
        • Calcaneus: Difficult to feel in its normal position.
      • Assess the rigidity/flexibility of the deformity.
      • Note any tenderness (unusual in neonates).
    • Move (Assess Correctability & Range of Motion – GENTLY):
      • This is crucial for assessing severity and for scoring systems like Pirani.
      • Hindfoot Equinus: Attempt to dorsiflex the ankle with the hindfoot held in a corrected (neutral/everted) position. Note the degree of dorsiflexion possible.
      • Hindfoot Varus: Attempt to evert the heel.
      • Forefoot Adductus: Attempt to abduct the forefoot relative to the hindfoot.
      • Forefoot Supination/Cavus: Attempt to pronate the forefoot.
      • Assess overall rigidity versus flexibility. A “postural” clubfoot will correct easily with gentle manipulation. True clubfoot is rigid.
  • Neurovascular Assessment: Check capillary refill and pulses (dorsalis pedis, posterior tibial – often difficult to palpate). Neurological deficits are rare in idiopathic clubfoot but common in syndromic/neuromuscular types.

Scoring Systems for Clubfoot Severity

Objective scoring systems are used to quantify the initial severity of clubfoot and to monitor response to treatment (e.g., Ponseti casting).

1. Pirani Scoring System:

  • Widely used, simple, and reliable, especially for monitoring Ponseti treatment.
  • Assesses 6 clinical signs of deformity, each scored 0, 0.5, or 1 (0 = normal, 0.5 = moderately abnormal, 1 = severely abnormal).
  • Total score ranges from 0 (normal foot) to 6 (most severe).
  • The 6 signs are divided into two groups:
    • Hindfoot Score (HFS) – Max 3 points:
      1. Posterior Crease (PC): Number and depth of posterior ankle creases. (0 = multiple fine creases; 0.5 = 1-2 moderate creases; 1 = single deep crease).
      2. Empty Heel (EH): Palpation of calcaneus in heel pad. (0 = calcaneus easily palpable; 0.5 = some soft tissue, calcaneus felt deep; 1 = calcaneus not palpable, “empty” feel). Assessed by attempting dorsiflexion and feeling if heel descends.
      3. Rigid Equinus (RE): Degree of dorsiflexion possible from maximum plantarflexion. (0 = >15° dorsiflexion; 0.5 = 0-15° dorsiflexion; 1 = cannot dorsiflex to neutral, fixed plantarflexion).
    • Midfoot Score (MFS) – Max 3 points:
      1. Medial Crease (MC): Depth of medial midfoot crease. (0 = multiple fine creases; 0.5 = 1-2 moderate creases; 1 = single deep crease indicating cavus).
      2. Curved Lateral Border (CLB): Shape of lateral border of foot. (0 = straight; 0.5 = gentle curve; 1 = severe curve/adduction). Assessed by looking at the foot from the sole.
      3. Lateral Head of Talus (LHT): Palpation of talar head coverage by navicular during abduction. (0 = talar head fully covered; 0.5 = talar head partially covered; 1 = talar head fully palpable/uncovered). Assessed by abducting forefoot and feeling if navicular covers talar head.
  • A higher score indicates greater severity and may predict the need for more casts or higher likelihood of Achilles tenotomy.

2. Dimeglio Scoring System:

  • More comprehensive but also more complex than Pirani.
  • Scores four parameters (0-4 points each based on reducibility): Equinus, Varus, Supination (of forefoot), Adduction (of forefoot relative to hindfoot).
  • Additional points (0-1 each) for: posterior crease, medial crease, cavus, poor muscle condition.
  • Total score ranges from 0 to 20.
    • Grade I (Benign): ≤5 points (postural)
    • Grade II (Moderate): 6-10 points (reducible/partially reducible)
    • Grade III (Severe): 11-15 points (resistant)
    • Grade IV (Very Severe): 16-20 points (very short, very rigid, often syndromic)
  • Less commonly used for routine Ponseti monitoring in many centres compared to Pirani due to its complexity.

Associated Conditions & Differential Diagnosis

Associated Conditions (especially in non-idiopathic clubfoot):

The presence of clubfoot warrants a search for other abnormalities, as it can be part of a wider syndrome or neuromuscular condition.

  • Neuromuscular Disorders:
    • Spinal Dysraphism: Myelomeningocele, tethered cord syndrome, diastematomyelia. (Always examine the spine).
    • Arthrogryposis Multiplex Congenita: Multiple congenital joint contractures. Clubfoot is common and often very rigid.
    • Cerebral Palsy (though clubfoot here is usually acquired/spastic, not congenital).
    • Muscular dystrophies.
  • Genetic Syndromes:
    • Larsen Syndrome (multiple joint dislocations, characteristic facies).
    • Streeter’s Dysplasia (Amniotic Band Syndrome – constriction bands, amputations).
    • Trisomy 18 (Edward’s Syndrome), Trisomy 13 (Patau Syndrome).
    • Diastrophic Dysplasia.
  • Other Musculoskeletal Anomalies:
    • Developmental Dysplasia of the Hip (DDH) – reported incidence higher in infants with clubfoot.
    • Congenital limb deficiencies.

Differential Diagnosis:

It’s important to differentiate true (structural) clubfoot from other foot deformities.

  • Postural Clubfoot (Positional Talipes Equinovarus):
    • Foot appears like clubfoot but is flexible and can be passively corrected to a normal position or beyond.
    • Thought to be due to intrauterine malpositioning.
    • Usually resolves spontaneously or with simple stretching exercises. Pirani score typically very low.
  • Metatarsus Adductus:
    • Forefoot is adducted (deviated medially) relative to a normal hindfoot. Heel is in neutral or valgus, not varus or equinus.
    • “Bean-shaped” foot. Lateral border is convex, medial border concave.
    • Usually flexible and resolves spontaneously.
  • Congenital Vertical Talus (Rocker-Bottom Foot):
    • A rigid flatfoot deformity.
    • Characterized by a dorsally dislocated talonavicular joint, plantarflexed talus, and dorsiflexed calcaneus (equinus of calcaneus).
    • Sole of the foot is convex (“rocker-bottom” appearance). Forefoot is dorsiflexed and everted.
    • Requires different management to clubfoot.
  • Calcaneovalgus Foot:
    • Foot is markedly dorsiflexed (top of foot may touch shin) and everted (valgus).
    • Usually flexible, due to intrauterine positioning. Resolves spontaneously.

Role of Imaging in Clubfoot Assessment

X-rays (Radiographs):

  • Generally NOT required for the initial diagnosis or routine monitoring of idiopathic clubfoot being treated with the Ponseti method. Diagnosis is clinical.
  • Challenges with X-rays in neonates:
    • Significant portions of the foot bones are cartilaginous and not visible on X-ray.
    • Difficult to position the small, deformed foot accurately for meaningful views.
    • Unnecessary radiation exposure.
  • Indications for X-rays might include:
    • Atypical or complex clubfoot.
    • Suspected syndromic clubfoot or association with other bony anomalies.
    • Pre-operative planning if extensive non-Ponseti surgical correction is contemplated (rare nowadays for primary treatment).
    • Assessment of recurrent deformity or failed Ponseti treatment.
    • Differentiating from other conditions like congenital vertical talus (specific views like forced plantarflexion lateral can be helpful).
  • If performed, specific views and angular measurements (e.g., talocalcaneal angle, talo-first metatarsal angle) can be used, but interpretation is specialist.

Ultrasound (USS):

  • Prenatal Ultrasound: Clubfoot can often be detected from around 18-20 weeks gestation, allowing for parental counselling. However, prenatal diagnosis can have false positives and doesn’t reliably predict severity or distinguish idiopathic from syndromic.
  • Postnatal Ultrasound:
    • Can visualize cartilaginous structures better than X-ray.
    • Can be used to assess talonavicular alignment and monitor correction dynamically.
    • Operator-dependent and not widely used for routine monitoring in most centres. Research tool.
    • May be useful for screening for DDH or spinal dysraphism if suspected.

MRI / CT:

  • Rarely indicated for routine clubfoot assessment.
  • May be used in complex, syndromic cases or for detailed pre-operative planning in revision surgery.
  • MRI can assess soft tissues and cartilage but requires sedation/GA in infants.
Clinical Assessment is Key

For idiopathic clubfoot, the diagnosis, severity assessment (e.g., Pirani score), and monitoring of Ponseti treatment are primarily based on meticulous clinical examination by an experienced practitioner. Imaging plays a secondary role.

Post-Assessment Pathway for Clubfoot

Once clubfoot is diagnosed and assessed, a clear management pathway should be followed, focusing on early intervention.

1. Referral:

  • Urgent referral to a Paediatric Orthopaedic Surgeon or a specialist Clubfoot Clinic with expertise in the Ponseti method.
  • Treatment should ideally commence within the first 1-2 weeks of life for optimal results.

2. Counselling and Education:

  • Provide parents/caregivers with clear information about clubfoot, the proposed treatment (Ponseti method), expected duration, importance of adherence (especially bracing), and potential outcomes.
  • Address parental concerns and anxieties. Provide resources for support groups.

3. Initiation of Ponseti Method (by trained practitioner):

This is the gold standard for idiopathic clubfoot.

  • Serial Manipulation and Casting:
    • Gentle manipulation of the foot to stretch contracted tissues, followed by application of a long-leg (above-knee) cast to hold the correction.
    • Casts are typically changed weekly. The CAVE components are corrected in a specific order (Cavus, then Adductus & Varus, then Equinus).
    • Usually requires 4-8 casts. Pirani score is used to monitor progress.
  • Percutaneous Achilles Tenotomy (PAT):
    • Performed in ~80-90% of cases once the cavus, adductus, and varus are corrected, but equinus persists (hindfoot cannot be dorsiflexed to 10-15°).
    • A minor outpatient procedure, often done under local anaesthesia, where the Achilles tendon is cut to allow full dorsiflexion.
    • A final cast is applied for ~3 weeks to allow the tendon to heal in a lengthened position.
  • Bracing (Foot Abduction Brace – FAB):
    • Crucial to prevent recurrence. Started immediately after the final cast is removed.
    • Consists of shoes attached to a bar, holding the feet in abduction (60-70° for affected foot, 30-40° for normal foot) and dorsiflexion.
    • Worn for 23 hours a day for the first 3 months, then reduced to night-time and nap-time wear until age 4-5 years.
    • Non-adherence to bracing is the most common cause of recurrence.

4. Long-Term Follow-Up:

  • Regular follow-up appointments to monitor for recurrence, ensure brace compliance, and address any issues.
  • Follow-up typically continues until skeletal maturity.

5. Management of Recurrence or Complex Cases:

  • Minor recurrences can often be managed with further casting.
  • Persistent or severe recurrences, or complex/syndromic clubfoot, may require further interventions such as tibialis anterior tendon transfer (TATT) or, rarely, more extensive soft tissue releases or bony procedures.

Red Flags & When to Escalate in Clubfoot Management

While idiopathic clubfoot managed by the Ponseti method has excellent outcomes, certain situations require vigilance and potential escalation to a senior orthopaedic colleague or specialist.

During Initial Assessment:

  • Suspicion of Syndromic/Neuromuscular Clubfoot:
    • Presence of other congenital anomalies (e.g., spinal defects, arthrogryposis, significant dysmorphism).
    • Very rigid, atypical foot deformity (e.g., vertical talus appearance, severe shortening).
    • Associated neurological deficits.
    • Action: Highlight to senior/refer for further multidisciplinary assessment (e.g., genetics, neurology).
  • Signs of Spinal Dysraphism: Deep sacral dimple above gluteal crease, hairy patch, skin tag, lipoma over the spine.
    • Action: Refer for spinal ultrasound (if <3-6 months old) or MRI, and neurosurgical opinion if indicated.
  • Associated DDH: Positive Ortolani/Barlow, significant asymmetry.
    • Action: Refer for hip ultrasound and orthopaedic management of DDH.

During Ponseti Casting:

  • Poor Response to Casting: Little or no improvement in Pirani score after 2-3 casts. Foot remains very rigid.
    • Action: Re-evaluate diagnosis (is it truly idiopathic?), casting technique. Discuss with senior.
  • Skin Problems: Significant cast sores, blisters, pressure areas, or skin breakdown.
    • Action: Remove cast, allow skin to heal, review casting technique. Ensure parents know cast care and warning signs.
  • Vascular Compromise: Persistent dusky or pale toes, prolonged capillary refill, excessive swelling despite elevation, inconsolable crying.
    • Action: Urgent cast removal. Escalate immediately.
  • Parental Concerns / Non-Adherence: Difficulty with cast care, repeated missed appointments.
    • Action: Explore reasons, provide support and education. Involve social work if necessary.

During Bracing Phase:

  • Recurrence of Deformity: Any reappearance of CAVE components despite bracing.
    • Action: Assess brace compliance, fit, and wear schedule. May require re-casting. Discuss with senior.
  • Persistent Brace Intolerance: Infant consistently distressed, skin breakdown from brace despite adjustments.
    • Action: Check brace fit, consider alternative brace types if available. Reinforce importance. Discuss with senior/orthotist.
  • Poor Brace Compliance: Parents not using the brace as prescribed.
    • Action: This is the BIGGEST risk for recurrence. Re-educate, explore barriers, emphasize importance.

Long-Term Follow-Up:

  • Late Recurrence: Deformity reappearing after initial successful bracing period.
  • Pain, functional limitations, or gait abnormalities as the child grows.
  • Action: Refer back to paediatric orthopaedic specialist for assessment and management (may need further intervention like tendon transfers or bony surgery).

Flashcards: Clubfoot Assessment

Click on each card to reveal the answer.

What does the acronym “CAVE” stand for in clubfoot deformity?

(Click to flip)

Answer

Cavus (high arch)
Adductus (forefoot deviated medially)
Varus (hindfoot inverted)
Equinus (hindfoot/ankle plantarflexed)

Name the 3 components of the Hindfoot Score in the Pirani scoring system.

(Click to flip)

Answer

1. Posterior Crease (PC)
2. Empty Heel (EH)
3. Rigid Equinus (RE)

What is the gold standard treatment for idiopathic clubfoot?

(Click to flip)

Answer

The Ponseti method (serial manipulation and casting, Achilles tenotomy, followed by bracing).

What is the most common reason for clubfoot recurrence after successful Ponseti treatment?

(Click to flip)

Answer

Non-adherence to the bracing protocol (foot abduction brace).

How does a postural clubfoot differ from a true (structural) clubfoot?

(Click to flip)

Answer

A postural clubfoot is flexible and can be passively corrected to a normal position or beyond. A true clubfoot is rigid and cannot be fully corrected passively.

What is an “empty heel” sign in clubfoot assessment?

(Click to flip)

Answer

The calcaneus is not palpable in its normal position in the heel pad; there’s a gap below the lateral malleolus where it should be. It indicates severity of hindfoot varus and equinus.

Clubfoot Assessment Quiz

Test your knowledge with these Paediatric Orthopaedics-focused questions.

1. A newborn is noted to have a left clubfoot. On examination, the heel is inverted and the forefoot is deviated medially. Which components of CAVE does this primarily describe?

A. Cavus and Equinus.
B. Adductus and Varus.
C. Cavus and Adductus.
D. Varus and Equinus.
Explanation: Heel inversion describes Varus. Medial deviation of the forefoot describes Adductus.

2. When using the Pirani scoring system for clubfoot, which of these signs is part of the Midfoot Score?

A. Posterior Crease.
B. Empty Heel.
C. Curved Lateral Border.
D. Rigid Equinus.
Explanation: The Midfoot Score in Pirani consists of Medial Crease, Curved Lateral Border, and Lateral Head of Talus. Posterior Crease, Empty Heel, and Rigid Equinus are part of the Hindfoot Score.

3. What is the primary purpose of the foot abduction brace (FAB) in the Ponseti method?

A. To actively correct the initial deformity.
B. To prevent recurrence of the deformity after casting and tenotomy.
C. To stretch the Achilles tendon before tenotomy.
D. To provide pain relief.
Explanation: The foot abduction brace is crucial for maintaining the correction achieved by casting and tenotomy, thereby preventing recurrence. It does not actively correct the initial severe deformity.

4. An infant presents with a foot deformity that is fully correctable with gentle passive manipulation. The foot can be easily dorsiflexed and everted. What is the most likely diagnosis?

A. Idiopathic clubfoot.
B. Syndromic clubfoot.
C. Postural clubfoot (Positional Talipes Equinovarus).
D. Congenital vertical talus.
Explanation: Full flexibility and passive correctability are characteristic of postural clubfoot, often due to intrauterine positioning. True idiopathic or syndromic clubfoot is rigid.

5. During the examination of an infant with clubfoot, you notice a deep sacral dimple with a small tuft of hair. What associated condition should be suspected and investigated?

A. Developmental Dysplasia of the Hip (DDH).
B. Spinal dysraphism (e.g., tethered cord).
C. Arthrogryposis Multiplex Congenita.
D. Larsen Syndrome.
Explanation: Cutaneous stigmata over the spine, such as deep dimples, hairy patches, or lipomas, can indicate underlying spinal dysraphism. This is important as it may suggest a non-idiopathic (neuromuscular) cause for the clubfoot.

6. The “equinus” component of clubfoot refers to:

A. Inversion of the heel.
B. Medial deviation of the forefoot.
C. Plantarflexion of the foot at the ankle.
D. A high medial longitudinal arch.
Explanation: Equinus specifically describes the plantarflexion deformity of the ankle and hindfoot, making the foot point downwards.

7. Which of the following imaging modalities is routinely required for the initial diagnosis of idiopathic clubfoot in a newborn?

A. X-ray of the foot.
B. MRI of the foot.
C. Ultrasound of the foot.
D. None; diagnosis is primarily clinical.
Explanation: The diagnosis of idiopathic clubfoot is made clinically based on history and physical examination. Routine imaging is not typically required for initial diagnosis or monitoring of Ponseti treatment.

8. A percutaneous Achilles tenotomy (PAT) is most commonly performed as part of the Ponseti method to correct which residual component of clubfoot?

A. Cavus.
B. Adductus.
C. Varus.
D. Equinus.
Explanation: The Achilles tenotomy is performed to correct residual equinus (plantarflexion) once the cavus, adductus, and varus components have been corrected by serial casting.

9. What is a key feature that helps differentiate Metatarsus Adductus from true Clubfoot?

A. Metatarsus Adductus always involves rigid equinus of the hindfoot.
B. In Metatarsus Adductus, the hindfoot is typically normal or in valgus, without significant varus or equinus.
C. Metatarsus Adductus is always bilateral.
D. Metatarsus Adductus requires surgical correction more often than clubfoot.
Explanation: In Metatarsus Adductus, the deformity is primarily in the forefoot (adduction), while the hindfoot is usually in a normal or even valgus position, and there is no significant equinus. Clubfoot involves all CAVE components, including hindfoot varus and equinus.

10. When assessing the “Lateral Head of Talus” for the Pirani score, what are you primarily evaluating?

A. The degree of hindfoot equinus.
B. The degree of correction of forefoot adductus and navicular subluxation.
C. The severity of the posterior ankle crease.
D. The rigidity of the cavus deformity.
Explanation: The Lateral Head of Talus sign assesses how well the navicular covers the head of the talus when the forefoot is abducted. A palpable talar head laterally indicates persistent adduction and navicular subluxation.

Trauma & Orthopaedics: Peripheral Nerve Injuries

Peripheral nerve injuries (PNIs) can result from trauma, compression, traction, or systemic disease, leading to sensory, motor, and autonomic deficits. Accurate diagnosis through careful history and examination, understanding the type and severity of injury, and timely management are crucial for optimising functional recovery.

Overview & Basic Peripheral Nerve Anatomy

Peripheral nerves transmit signals between the central nervous system (CNS) and the rest of the body. Damage to these nerves can disrupt motor control, sensation, and autonomic function.

Basic Structure of a Peripheral Nerve:

  • Neuron: The basic functional unit. Comprises cell body, dendrites, and an axon.
  • Axon: The long projection of a neuron that conducts electrical impulses away from the neuron’s cell body.
  • Myelin Sheath: A fatty insulating layer surrounding many axons, formed by Schwann cells in the PNS. Speeds up nerve impulse conduction (saltatory conduction).
  • Connective Tissue Layers (within a nerve trunk):
    • Endoneurium: Surrounds individual axons (and their myelin sheath).
    • Perineurium: Surrounds bundles of axons (fascicles). Forms the blood-nerve barrier.
    • Epineurium: Outermost layer, surrounds the entire nerve trunk and cushions fascicles. Contains blood vessels (vasa nervorum).

Types of Nerve Fibres:

  • Motor (Efferent): Carry signals from CNS to muscles, causing contraction.
  • Sensory (Afferent): Carry signals from sensory receptors (touch, pain, temperature, proprioception) to CNS.
  • Autonomic: Regulate involuntary functions (e.g., sweat glands, blood vessel diameter). Sympathetic and parasympathetic fibres.

Nerve Regeneration:

  • Peripheral nerves have some capacity for regeneration if the cell body is intact and conditions are favourable.
  • Wallerian Degeneration: Degeneration of the axon and myelin sheath distal to the site of injury. Occurs within days.
  • Axonal sprouting occurs from the proximal stump, guided by the remaining Schwann cells and endoneurial tubes (if intact). Regeneration is slow (~1-3 mm/day).
💡 Clinical Relevance

The specific deficits seen depend on which nerve is injured and the type/severity of fibres affected. Understanding the nerve’s course and innervation pattern is key to localising the lesion.

Classification of Nerve Injury (Seddon & Sunderland)

These classifications help describe the severity of nerve injury and predict the potential for recovery.

Seddon Classification (1943): Based on three types of injury.
Type
Pathology
Clinical Features
Recovery
Neurapraxia
Local myelin damage/conduction block (axon intact). Often due to compression or mild traction. No Wallerian degeneration.
Temporary loss of motor function +/- sensory function. Autonomic function usually preserved. No muscle atrophy initially. Tinel’s sign absent.
Complete recovery within days to weeks (typically <12 weeks) as myelin remakes. Excellent prognosis.
Axonotmesis
Axon disrupted, but endoneurial tube and connective tissue sheaths (perineurium, epineurium) remain intact. Wallerian degeneration occurs distal to injury.
Loss of motor, sensory, and autonomic function distal to injury. Muscle atrophy occurs. Tinel’s sign may be present at injury site and advance with regeneration.
Good to fair recovery. Axon regenerates along intact endoneurial tubes (~1mm/day). May take months. Quality of recovery depends on distance and accuracy of reinnervation.
Neurotmesis
Complete transection or severe disruption of the entire nerve trunk, including connective tissue sheaths (endoneurium, perineurium, epineurium). Wallerian degeneration occurs.
Complete loss of motor, sensory, and autonomic function. Muscle atrophy. Tinel’s sign present at injury site but does not advance without repair.
Poor prognosis without surgical repair. Regeneration is haphazard, often blocked by scar tissue, or misdirected. Surgery (nerve repair/graft) often required. Recovery is slow and often incomplete.
Sunderland Classification (1951): Expands on Axonotmesis and Neurotmesis (5 degrees).
  • Grade I: Equivalent to Seddon’s Neurapraxia. Conduction block.
  • Grade II: Axonal disruption, endoneurium intact. Equivalent to Seddon’s Axonotmesis. Good prognosis for spontaneous recovery.
  • Grade III: Axonal disruption, endoneurium disrupted, but perineurium intact. Scarring within fascicles can hinder regeneration. Recovery variable, may need surgery.
  • Grade IV: Axonal disruption, endoneurium and perineurium disrupted, but epineurium intact. Nerve in continuity but extensive internal scarring (neuroma-in-continuity). Spontaneous recovery unlikely; surgery usually needed.
  • Grade V: Complete nerve transection. Equivalent to Seddon’s Neurotmesis. Surgery essential.

Sunderland Grade VI (added later): Mixed injury pattern with different fascicles having different degrees of injury.

Mechanisms of Peripheral Nerve Injury

PNIs can occur through various mechanisms, influencing the type and severity of injury.

Common Mechanisms:
  • Laceration/Transection:
    • Caused by sharp objects (glass, knives), projectiles, or surgical incision.
    • Often results in neurotmesis (Sunderland Grade V).
    • Associated with open wounds.
  • Compression:
    • Acute: e.g., Saturday night palsy (radial nerve compression), tourniquet palsy, compartment syndrome. Often leads to neurapraxia or axonotmesis if prolonged/severe.
    • Chronic: e.g., Carpal Tunnel Syndrome (median nerve), Cubital Tunnel Syndrome (ulnar nerve), peroneal nerve at fibular head. Leads to progressive demyelination and axonal loss.
  • Traction/Stretch:
    • Nerves are stretched beyond their elastic limit. Common in high-energy trauma, dislocations (e.g., shoulder dislocation and axillary nerve), birth injuries (e.g., brachial plexus).
    • Can cause range from neurapraxia to neurotmesis depending on force and duration. Axons are more susceptible to stretch than connective tissue.
  • Crush Injury:
    • Direct blunt trauma or prolonged pressure. Combination of compression and local tissue damage.
    • Severity depends on force. Can cause extensive internal disorganisation.
  • Ischaemia:
    • Compromised blood supply to the nerve (vasa nervorum). Can occur in compartment syndrome, vascular disease, prolonged tourniquet use.
    • Nerves are relatively resistant to ischaemia, but prolonged ischaemia causes axonal damage.
  • Thermal Injury: Burns or frostbite can directly damage nerves.
  • Electrical Injury: Can cause extensive and deep nerve damage.
  • Injection Injury: Direct trauma from needle or toxicity of injected substance.
  • Iatrogenic Injury: During surgery (e.g., traction, transection, suture entrapment) or other medical procedures.
  • Entrapment Neuropathies: Chronic compression of a nerve at specific anatomical sites (e.g., tunnels, fascial bands).
  • Systemic Conditions:
    • Diabetes Mellitus: Peripheral neuropathy (often symmetrical, stocking-glove).
    • Vitamin Deficiencies: (e.g., B1, B6, B12).
    • Autoimmune Diseases: (e.g., Guillain-Barré syndrome, vasculitis).
    • Infections: (e.g., Leprosy, Lyme disease, HIV).
    • Toxins/Medications: (e.g., alcohol, chemotherapy agents, heavy metals).

Clinical Assessment: History & Examination

A meticulous history and detailed neurological examination are paramount for diagnosing PNI, localising the lesion, and determining severity.

History Taking:
  • Presenting Complaint: Weakness, sensory loss (numbness, tingling, pain), autonomic changes.
  • Onset: Acute (trauma) or gradual (entrapment, systemic).
  • Mechanism of Injury (if traumatic): Crucial for understanding potential nerve involvement and severity. Type of force, position of limb.
  • Progression of Symptoms: Improving, worsening, static?
  • Specific Symptoms:
    • Motor: Weakness (specific movements), clumsiness, dropping objects, foot drop, muscle wasting (ask about changes in muscle bulk).
    • Sensory:
      • Negative: Numbness, loss of sensation.
      • Positive: Paraesthesia (pins and needles, tingling), dysaesthesia (unpleasant abnormal sensation), allodynia (pain from non-painful stimulus), hyperalgesia (increased pain from painful stimulus). Neuropathic pain (burning, shooting, electric shock-like).
      Distribution of sensory changes is key.
    • Autonomic: Changes in sweating (anhidrosis/hyperhidrosis), skin colour/temperature, nail/hair changes (trophic changes).
  • Functional Impact: How symptoms affect ADLs, work, hobbies.
  • Pain: Character, severity, location, exacerbating/relieving factors.
  • Past Medical History: Diabetes, thyroid disease, autoimmune conditions, previous injuries/surgeries, history of cancer (for paraneoplastic syndromes or direct infiltration).
  • Drug History & Toxin Exposure: Chemotherapy, alcohol, heavy metals.
  • Family History: Hereditary neuropathies (e.g., Charcot-Marie-Tooth).
  • Social History: Occupation (repetitive tasks), hobbies.
Physical Examination:

Always compare with the contralateral side. Follow a “Look, Feel, Move, Special Tests” approach, focusing on neurological components.

  • Look:
    • Muscle Wasting (Atrophy): Compare muscle bulk. Indicates LMN lesion and chronicity.
    • Deformity: e.g., claw hand (ulnar/median), wrist drop (radial), foot drop (common peroneal).
    • Skin Changes: Colour, temperature, texture (dry, shiny, hair loss – autonomic dysfunction), ulcers, calluses.
    • Scars: Previous trauma or surgery.
    • Fasciculations: Suggests LMN lesion.
  • Feel:
    • Palpate nerve trunks for tenderness or thickening (e.g., ulnar nerve at elbow).
    • Assess muscle tone.
  • Move (Motor Examination):
    • Power: Test individual muscles or muscle groups supplied by specific nerves/nerve roots. Grade using MRC scale (0-5).
      • 0: No contraction.
      • 1: Flicker or trace of contraction.
      • 2: Active movement, with gravity eliminated.
      • 3: Active movement against gravity.
      • 4: Active movement against gravity and resistance.
      • 5: Normal power.
    • Reflexes: May be diminished or absent if nerve supplying muscle or sensory arc is affected (LMN sign).
    • Coordination.
  • Sensation: Test in dermatomal and peripheral nerve distributions.
    • Light Touch: Cotton wool.
    • Pinprick (Pain): Neurotip or sterile needle.
    • Temperature: Cold/warm objects (often lost early in neuropathy).
    • Vibration: Tuning fork (128 Hz) over bony prominences.
    • Proprioception (Joint Position Sense).
    • Two-Point Discrimination: Especially in fingertips.
  • Special Tests / Provocative Manoeuvres:
    • Tinel’s Sign: Gently tap over the suspected site of nerve injury/entrapment. Positive if it reproduces paraesthesia or pain in the nerve’s distribution (suggests nerve irritability or regeneration).
    • Specific tests for entrapment syndromes (e.g., Phalen’s/Tinel’s at wrist for CTS, elbow flexion test for cubital tunnel).
  • Autonomic Examination (if indicated): Observe sweating, skin temperature/colour.
🗺️ Mapping Deficits

Carefully map out the motor weakness and sensory loss. Does it fit a specific peripheral nerve pattern, a nerve root pattern (dermatome/myotome), or a more diffuse “stocking-glove” pattern (polyneuropathy)? This is key to localisation.

Common Upper Limb Nerve Injuries

Focus on key nerves, sites of injury, motor/sensory deficits, and characteristic signs.

Nerve (Root Values)
Common Sites/Mechanisms of Injury
Motor Deficit (Key Muscles & Actions)
Sensory Deficit (Autonomous Zone)
Characteristic Sign / Deformity
Axillary Nerve (C5, C6)
Anterior shoulder dislocation, fracture of surgical neck of humerus, intramuscular injection in deltoid.
Deltoid (abduction of arm beyond initial 15°).
Teres Minor (external rotation).
“Regimental badge” area (lateral aspect of upper arm over deltoid).
Wasting of deltoid, inability to abduct arm fully.
Musculocutaneous Nerve (C5, C6, C7)
Rarely injured in isolation. Trauma to axilla/upper arm, strenuous exercise.
Biceps Brachii, Brachialis, Coracobrachialis (elbow flexion, forearm supination).
Lateral aspect of forearm.
Weak elbow flexion (especially with supinated forearm), weak supination.
Radial Nerve (C5, C6, C7, C8, T1)
Fracture of humeral shaft (spiral groove), “Saturday night palsy” (compression in axilla), crutch palsy, proximal radius fracture.
Triceps (elbow extension – if lesion high).
Wrist extensors (ECRL, ECRB, ECU).
Finger extensors (EDC, EPL, EIP, EDM).
Supinator, Brachioradialis.
Dorsal aspect of forearm and hand, especially 1st dorsal web space (between thumb and index finger).
Wrist Drop, Finger Drop.
Weakness of supination. Sensory loss variable.
Median Nerve (C5, C6, C7, C8, T1)
Carpal tunnel syndrome (compression at wrist), supracondylar fracture of humerus, pronator teres syndrome, wrist lacerations.
Pronators (pronator teres, pronator quadratus).
Wrist flexors (FCR, Palmaris longus).
Finger flexors (FDS, FDP to index/middle fingers, FPL).
Thenar muscles (APB, OP, FPB superficial head – “LOAF” muscles for thumb opposition/abduction).
Lumbricals to index/middle fingers.
Palmar aspect of thumb, index, middle, and radial half of ring finger. Dorsal tips of these fingers.
“Ape Hand” Deformity (wasting of thenar eminence, inability to abduct/oppose thumb – late sign).
“Hand of Benediction” / “Median Claw” (attempting to make fist – index/middle fingers remain extended due to FDP/FDS paralysis – seen in high lesions).
Weak wrist flexion/pronation. Paresthesia in CTS.
Ulnar Nerve (C8, T1)
Cubital tunnel syndrome (compression at elbow medial epicondyle), fracture of medial epicondyle, Guyon’s canal compression (wrist), wrist/forearm lacerations.
FCU (wrist flexion/ulnar deviation).
FDP to ring/little fingers.
Hypothenar muscles (ADM, ODM, FDM).
Interossei (finger abduction/adduction – DAB/PAD).
Lumbricals to ring/little fingers.
Adductor Pollicis.
Palmar and dorsal aspects of little finger and ulnar half of ring finger. Ulnar aspect of hand.
“Ulnar Claw Hand” (hyperextension at MCPJs, flexion at IPJs of ring/little fingers – due to lumbrical paralysis).
Wasting of hypothenar eminence and interossei (guttering).
Froment’s Sign (flexion of thumb IPJ when trying to grip paper due to adductor pollicis weakness).
Weak wrist flexion/ulnar deviation. Sensory loss. Wartenberg’s sign (abducted little finger).
Long Thoracic Nerve (C5, C6, C7 – from roots)
Trauma, surgery (mastectomy, axillary dissection), viral illness, heavy lifting.
Serratus Anterior (protraction and upward rotation of scapula, holds scapula to chest wall).
None (purely motor).
Winging of the Scapula (medial border protrudes, especially when pushing against wall or abducting arm). Pain, weakness with overhead activities.

Common Lower Limb Nerve Injuries

Focus on key nerves, sites of injury, motor/sensory deficits, and characteristic signs.

Nerve (Root Values)
Common Sites/Mechanisms of Injury
Motor Deficit (Key Muscles & Actions)
Sensory Deficit (Autonomous Zone)
Characteristic Sign / Deformity
Femoral Nerve (L2, L3, L4)
Pelvic fracture, hip surgery (anterior approach), retroperitoneal haematoma, diabetes, compression by mass in inguinal region.
Quadriceps Femoris (knee extension).
Iliopsoas (hip flexion – if lesion high in pelvis).
Sartorius, Pectineus.
Anterior thigh, medial aspect of lower leg and foot (via saphenous nerve branch).
Weak knee extension, difficulty climbing stairs/rising from chair. Absent/reduced knee jerk. Wasting of quadriceps.
Obturator Nerve (L2, L3, L4)
Pelvic fracture, hip surgery, obstetric injury, pelvic tumours.
Adductor muscles of thigh (adductor longus/brevis/magnus, gracilis, pectineus – part). Hip adduction.
Small area on medial aspect of mid-thigh.
Weak hip adduction. Difficulty crossing legs. Wide-based gait. Sensory loss often minimal.
Sciatic Nerve (L4, L5, S1, S2, S3)
Posterior hip dislocation, fracture of femur/pelvis, iatrogenic (IM injection in buttock, hip replacement), piriformis syndrome, tumours. Divides into Tibial and Common Peroneal nerves.
Hamstrings (knee flexion).
All muscles below the knee (via tibial and common peroneal branches).
Posterior thigh. Most of leg and foot (except area supplied by femoral/saphenous).
Weak knee flexion. Foot Drop (if common peroneal affected). Inability to plantarflex/dorsiflex/evert/invert foot. Absent ankle jerk. Widespread sensory loss below knee.
Tibial Nerve (L4, L5, S1, S2, S3 – branch of Sciatic)
Knee dislocation/trauma (popliteal fossa), tarsal tunnel syndrome (compression at ankle).
Plantarflexors (Gastrocnemius, Soleus, Plantaris).
Invertors (Tibialis Posterior).
Toe flexors (FDL, FHL).
Intrinsic foot muscles.
Sole of the foot.
Inability to plantarflex or stand on tiptoes. Weak inversion. Clawing of toes. Sensory loss on sole. Absent/reduced ankle jerk (if G/S affected).
Common Peroneal (Fibular) Nerve (L4, L5, S1, S2 – branch of Sciatic)
Compression at fibular head (crossing legs, plaster cast, prolonged bed rest, weight loss), direct trauma, knee dislocation, fibular fracture. Divides into Superficial and Deep Peroneal nerves.
Dorsiflexors (Tibialis Anterior, EHL, EDL – via Deep Peroneal).
Evertors (Peroneus Longus/Brevis – via Superficial Peroneal).
Deep Peroneal: 1st dorsal web space (between big toe and 2nd toe).
Superficial Peroneal: Dorsum of foot (except 1st web space) and anterolateral aspect of lower leg.
Foot Drop (inability to dorsiflex). High-stepping / slapping gait.
Weak eversion. Sensory loss in distributions.
Superior Gluteal Nerve (L4, L5, S1)
Iatrogenic (hip surgery, IM injection in superomedial buttock quadrant).
Gluteus Medius, Gluteus Minimus, Tensor Fascia Latae. (Hip abduction, internal rotation, pelvic stabilisation).
None (purely motor).
Trendelenburg Gait / Sign: Pelvis drops on contralateral (unsupported) side when standing on affected leg. Waddling gait if bilateral.
Inferior Gluteal Nerve (L5, S1, S2)
Posterior hip dislocation, iatrogenic.
Gluteus Maximus (Hip extension).
None (purely motor).
Difficulty rising from seated position, climbing stairs, or running (weak hip extension). Wasting of buttock.

Investigations for Peripheral Nerve Injuries

Investigations help confirm the diagnosis, localise the lesion, assess severity, and guide management. They are often used when clinical diagnosis is uncertain or for pre-operative planning.

Electrodiagnostic Studies (EDS):

Usually performed 3-4 weeks after acute injury (to allow for Wallerian degeneration if present) or for chronic conditions.

  • Nerve Conduction Studies (NCS):
    • Measure speed (conduction velocity) and amplitude of electrical impulses along a nerve.
    • Stimulating electrodes placed along nerve course, recording electrodes over muscle (motor NCS) or nerve (sensory NCS).
    • Findings:
      • Neurapraxia (Demyelination): Slowed conduction velocity across lesion, conduction block (drop in amplitude across lesion), normal conduction distal to block if tested soon after injury. Distal latency prolonged.
      • Axonotmesis/Neurotmesis (Axonal Loss): Reduced amplitude of Compound Muscle Action Potential (CMAP) or Sensory Nerve Action Potential (SNAP). Conduction velocity may be mildly slowed or normal in remaining fibres. Fibrillation potentials/positive sharp waves on EMG.
    • Help differentiate axonal vs. demyelinating injury, localise lesion, assess severity.
  • Electromyography (EMG):
    • Needle electrode inserted into muscle to record electrical activity at rest and during voluntary contraction.
    • Findings:
      • Normal Muscle: Electrically silent at rest. Motor Unit Action Potentials (MUAPs) appear on voluntary contraction.
      • Denervation (Acute/Subacute Axonal Loss): Spontaneous activity at rest: Fibrillation potentials, positive sharp waves (appear ~2-3 weeks post-injury). MUAPs reduced in number, may be polyphasic.
      • Reinnervation (Chronic Axonal Loss/Recovery): Large, polyphasic, long-duration MUAPs. Reduced recruitment.
      • Myopathy: Small, short-duration, polyphasic MUAPs with early recruitment.
    • Helps assess integrity of LMN and muscle. Differentiates neurogenic vs. myopathic weakness. Assesses for denervation/reinnervation.
Imaging:
  • Ultrasound (High-Resolution):
    • Can visualise nerves directly, assess for swelling, compression, discontinuity (transection), neuroma formation, surrounding pathology (cysts, tumours).
    • Dynamic assessment possible (e.g., nerve subluxation).
    • Useful for guiding injections.
    • Increasingly used for entrapment neuropathies (e.g., CTS).
  • Magnetic Resonance Imaging (MRI) / MR Neurography:
    • Provides detailed images of nerves and surrounding soft tissues.
    • Can show nerve oedema/inflammation, compression, discontinuity, neuroma, muscle denervation changes (oedema acutely, fatty atrophy chronically).
    • Useful for brachial/lumbosacral plexus injuries, deep-seated nerves, or when USS is inconclusive. MR Neurography uses specific sequences to highlight nerves.
  • X-rays: To rule out fracture, dislocation, bony abnormalities causing nerve compression.
  • CT Scan: For bony detail if fracture is suspected cause of nerve injury.
Other Investigations:
  • Bloods: For systemic causes (e.g., FBC, ESR/CRP, glucose/HbA1c, B12/folate, TFTs, autoimmune screen, ACE if sarcoid suspected).
  • Lumbar Puncture: If Guillain-Barré syndrome or CNS pathology suspected.
  • Nerve Biopsy: Rarely performed, usually in specialist centres for diagnosing certain neuropathies (e.g., vasculitis, amyloidosis, leprosy) if other tests inconclusive. Sural nerve is commonly biopsied.

Management Principles & Prognosis

Management depends on the type, severity, and cause of nerve injury. The goal is to maximise functional recovery.

General Principles:
  • Address the Cause:
    • Remove compression (e.g., carpal tunnel release, repositioning for pressure palsy).
    • Treat underlying systemic disease (e.g., diabetes control, vitamin supplementation).
    • Manage fractures/dislocations that may be causing nerve compromise.
  • Conservative Management (for Neurapraxia, mild Axonotmesis, or if recovery expected):
    • Observation & Monitoring: Regular clinical review to track recovery (motor, sensory, Tinel’s sign progression). Serial electrodiagnostic studies may be used.
    • Splinting/Bracing: To prevent contractures, protect joints, improve function (e.g., wrist drop splint for radial nerve palsy, AFO for foot drop).
    • Physiotherapy & Occupational Therapy:
      • Maintain range of motion in affected joints (passive/active-assisted exercises).
      • Strengthening exercises for paretic and compensatory muscles.
      • Sensory re-education.
      • Functional adaptation, assistive devices.
      • Pain management techniques.
    • Pain Management: For neuropathic pain (see below).
  • Surgical Management (for Neurotmesis, severe Axonotmesis not recovering, or specific indications):
    • Timing of Surgery:
      • Acute/Early Repair (within days/weeks): For sharp transections (e.g., lacerations) where nerve ends are clean. Better prognosis.
      • Delayed Repair (3-6 months or later): For crush/traction injuries where extent of damage isn’t immediately clear, or if conservative measures fail. Allows time for demarcation of injured segment and neuroma formation.
    • Surgical Techniques:
      • Neurolysis: Freeing the nerve from scar tissue or compression (external or internal).
      • Direct Nerve Repair (Neurorrhaphy): Suturing severed nerve ends together without tension. Epineurial or fascicular repair.
      • Nerve Grafting: Bridging a nerve gap with a segment of donor nerve (autograft, e.g., sural nerve, medial antebrachial cutaneous nerve) or a nerve conduit (synthetic tube).
      • Nerve Transfer (Neurotisation): Sacrificing a less important functioning nerve (or part of it) and connecting it to the distal stump of a more important injured nerve. Used when proximal stump is unavailable or gap too large.
      • Tendon Transfers: Rerouting functioning tendons to restore lost movement if nerve recovery is poor or not expected. A reconstructive/salvage procedure.
      • Joint Fusion (Arthrodesis) or Free Functioning Muscle Transfer: Salvage procedures for severe, irreparable injuries.
  • Management of Neuropathic Pain:
    • Pharmacological:
      • First-line: Amitriptyline, Duloxetine, Gabapentin, Pregabalin.
      • Second-line: Tramadol, Capsaicin cream, Lidocaine patches. Opioids generally have limited long-term efficacy.
    • Non-pharmacological: TENS machine, acupuncture, psychological therapies (CBT, ACT), pain management programmes.
Prognosis:
  • Neurapraxia: Excellent prognosis, full recovery expected.
  • Axonotmesis: Good to fair. Recovery depends on accuracy of axonal regeneration, distance to target organ, patient age, and absence of scarring. Regeneration rate ~1mm/day.
  • Neurotmesis: Poor without surgery. With surgery, recovery is variable, often incomplete, and depends on many factors (age, time to repair, type of repair, gap length, associated injuries). Sensory recovery often better than motor.
  • Factors Influencing Prognosis:
    • Age of patient (younger = better).
    • Type and severity of injury (Seddon/Sunderland grade).
    • Level of injury (proximal injuries have longer distance for regeneration, worse prognosis).
    • Mechanism of injury (sharp transection better than crush/traction).
    • Delay to treatment/repair.
    • Associated injuries (vascular, soft tissue).
    • Patient’s general health and motivation.
    • Accuracy of surgical repair.

Red Flags & Referral Criteria for Peripheral Nerve Injuries

Prompt referral to appropriate specialists is crucial in many PNI cases to optimise outcomes.

Red Flags Requiring Urgent Assessment/Referral (often to A&E or on-call Ortho/Plastics/Neurosurgery):
  • PNI associated with Open Fracture or Significant Trauma: Requires urgent surgical exploration, debridement, and potential nerve repair.
  • PNI with Vascular Compromise: (e.g., absent pulses, cold/pale limb). Urgent vascular assessment and intervention needed.
  • Suspected Compartment Syndrome causing Nerve Compression: Urgent fasciotomy required.
  • Rapidly Progressive Neurological Deficit: Suggests ongoing compression, ischaemia, or inflammatory process (e.g., Guillain-Barré).
  • PNI associated with Joint Dislocation that doesn’t resolve after reduction: Nerve may be entrapped.
  • Cauda Equina Syndrome: (Low back pain, bilateral sciatica, saddle anaesthesia, bladder/bowel dysfunction, leg weakness). Neurosurgical emergency. (Though technically CNS roots, important differential for leg symptoms).
Indications for Urgent/Semi-Urgent Referral to Specialist (e.g., Hand Surgeon, Peripheral Nerve Surgeon, Neurologist, Orthopaedic Specialist):
  • Known or Suspected Nerve Laceration/Transection: (e.g., deep cut with immediate complete motor/sensory loss in nerve distribution). Requires timely surgical exploration and repair.
  • Complete Motor and/or Sensory Loss in a specific nerve distribution after trauma (Axonotmesis/Neurotmesis suspected).
  • No signs of clinical or electrophysiological recovery after an expected period for Neurapraxia or Axonotmesis (e.g., 3-4 months, depending on injury).
  • Progressive Deterioration of Nerve Function despite conservative management.
  • Severe or Intractable Neuropathic Pain unresponsive to primary care management.
  • Suspected Brachial Plexus or Lumbosacral Plexus Injury: Complex injuries often requiring specialist MDT input.
  • PNI associated with Tumour or Mass compressing the nerve.
  • Entrapment Neuropathies failing conservative treatment or with severe/progressive symptoms.
  • Diagnostic Uncertainty: If unable to localise lesion or determine cause.
Considerations for GP Management Prior to Referral (if not an emergency):
  • Initial assessment and documentation of deficits.
  • Appropriate splinting and analgesia.
  • Basic investigations if indicated (e.g., X-ray for fracture).
  • Counselling patient about nature of injury and potential for recovery/need for specialist input.
  • Safety netting: advice on when to seek urgent review if symptoms worsen.
Time is Nerve!

For complete nerve transections (Neurotmesis), early surgical repair (ideally within weeks for sharp injuries) generally yields better outcomes than delayed repair, as muscle denervation atrophy becomes irreversible over time, and proximal nerve stump retraction/scarring occurs.

Flashcards: Peripheral Nerve Injuries

Click on each card to reveal the answer.

What are the three types of nerve injury in Seddon’s classification?

(Click to flip)

Answer

1. Neurapraxia
2. Axonotmesis
3. Neurotmesis

Which type of Seddon injury involves axonal damage but intact endoneurial tubes, allowing for good regeneration?

(Click to flip)

Answer

Axonotmesis.

Fracture of the surgical neck of the humerus commonly injures which nerve, and what is its autonomous sensory zone?

(Click to flip)

Answer

Nerve: Axillary Nerve.
Sensory Zone: “Regimental badge” area (lateral aspect of upper arm over deltoid).

“Wrist drop” is a characteristic sign of injury to which peripheral nerve?

(Click to flip)

Answer

Radial Nerve.

What is Froment’s sign, and which nerve injury does it indicate?

(Click to flip)

Answer

Froment’s sign: Flexion of the thumb IP joint when attempting to pinch/grip paper (due to adductor pollicis weakness, compensated by FPL – median nerve).
Indicates: Ulnar Nerve injury.

Injury to the common peroneal nerve at the fibular head typically results in what motor deficit?

(Click to flip)

Answer

Foot Drop (weakness of ankle dorsiflexion) and weakness of eversion.

What is Tinel’s sign, and what does a positive sign suggest?

(Click to flip)

Answer

Tinel’s Sign: Gently tapping over a nerve.
Suggests: Nerve irritability or regeneration at that point (reproduces paraesthesia in nerve distribution).

What are the “LOAF” muscles of the hand, and which nerve innervates them?

(Click to flip)

Answer

LOAF:
Lumbricals (1st and 2nd)
Opponens Pollicis
Abductor Pollicis Brevis
Flexor Pollicis Brevis (superficial head)
Innervated by: Median Nerve.

Peripheral Nerve Injuries Quiz

Test your knowledge on assessing and understanding peripheral nerve injuries.

1. A patient sustains a mid-shaft humeral fracture and develops an inability to extend their wrist and fingers. Which nerve is most likely injured?

A. Median Nerve.
B. Radial Nerve.
C. Ulnar Nerve.
D. Axillary Nerve.
Explanation: The radial nerve runs in the spiral groove of the humerus and is vulnerable in mid-shaft fractures. It supplies the wrist and finger extensors.

2. According to Seddon’s classification, which type of nerve injury involves axonal disruption but with intact endoneurial sheaths, allowing for good spontaneous regeneration?

A. Neurapraxia.
B. Axonotmesis.
C. Neurotmesis.
D. Sunderland Grade IV.
Explanation: Axonotmesis is characterised by axonal damage with preservation of the nerve’s connective tissue framework (including endoneurium), which guides regenerating axons.

3. A patient presents with weakness of thumb abduction (palmar abduction) and opposition, along with sensory loss over the palmar aspect of the thumb, index, and middle fingers. This pattern suggests injury to which nerve?

A. Median Nerve.
B. Ulnar Nerve.
C. Radial Nerve.
D. Musculocutaneous Nerve.
Explanation: The median nerve supplies the thenar muscles responsible for thumb abduction/opposition (APB, Opponens Pollicis) and provides sensation to the radial 3.5 digits palmarly.

4. “Winging of the scapula” when a patient pushes against a wall is caused by weakness of the Serratus Anterior muscle, due to injury of which nerve?

A. Axillary Nerve.
B. Suprascapular Nerve.
C. Long Thoracic Nerve.
D. Spinal Accessory Nerve.
Explanation: The Long Thoracic Nerve (from roots C5, C6, C7) innervates the Serratus Anterior, which holds the scapula against the thoracic wall and protracts it.

5. A patient has difficulty standing on their tiptoes and reports sensory loss on the sole of their foot. Which nerve is most likely affected?

A. Common Peroneal Nerve.
B. Tibial Nerve.
C. Femoral Nerve.
D. Obturator Nerve.
Explanation: The Tibial nerve supplies the plantarflexor muscles of the ankle (e.g., gastrocnemius, soleus) and provides sensation to the sole of the foot.

6. Nerve Conduction Studies (NCS) in a patient with pure demyelinating neuropathy (Neurapraxia) across a lesion would typically show:

A. Reduced amplitude of CMAP/SNAP with normal conduction velocity.
B. Slowed conduction velocity and/or conduction block across the lesion.
C. Normal NCS findings but abnormal EMG.
D. Fibrillation potentials and positive sharp waves on EMG.
Explanation: Demyelination (as in neurapraxia) primarily affects the speed of conduction, leading to slowed velocity or a block in conduction across the affected segment. Axonal loss causes reduced amplitude. Fibrillations are EMG signs of denervation.

7. What is the typical rate of peripheral nerve regeneration after axonotmesis?

A. 1 mm/week.
B. 1-3 mm/day.
C. 1 cm/day.
D. 1 cm/week.
Explanation: Axonal regeneration in the PNS typically occurs at a rate of approximately 1-3 mm per day (or about 1 inch per month).

8. A positive Trendelenburg sign (pelvis drops on the unsupported side when standing on one leg) indicates weakness of which muscle group, innervated by which nerve?

A. Quadriceps; Femoral nerve.
B. Gluteus Maximus; Inferior gluteal nerve.
C. Gluteus Medius & Minimus; Superior gluteal nerve.
D. Adductors; Obturator nerve.
Explanation: The Trendelenburg sign is caused by weakness of the hip abductors (Gluteus Medius and Minimus), which are innervated by the Superior Gluteal Nerve.

9. Which of these is a RED FLAG indicating an urgent need for specialist assessment in a patient with an acute peripheral nerve injury?

A. Mild paraesthesia in the nerve distribution.
B. Muscle weakness graded 4/5 MRC.
C. Associated vascular compromise (e.g., absent distal pulses).
D. Symptoms present for 1 week without improvement.
Explanation: Vascular compromise associated with a nerve injury is an emergency requiring urgent intervention to save the limb. While other factors may need referral, this is the most immediately limb-threatening.

10. The “Regimental Badge” area of sensory loss is characteristic of an injury to the:

A. Radial Nerve.
B. Median Nerve.
C. Axillary Nerve.
D. Ulnar Nerve.
Explanation: The Axillary nerve provides sensation to the skin over the lateral aspect of the upper arm (deltoid region), often referred to as the “regimental badge” area.

Tendinopathies (T&O Focus)

Tendinopathy is a broad term describing painful conditions occurring in and around tendons in response to overuse or overload. It encompasses a spectrum of pathology, from reactive tendinopathy to tendon disrepair and degenerative tendinopathy, rather than primarily an inflammatory process (hence “tendinopathy” is preferred over “tendinitis” for chronic conditions).

Overview & Pathophysiology of Tendinopathy

Tendons are strong, fibrous cords that connect muscle to bone, transmitting forces to produce movement. Tendinopathy arises from a failed healing response to repetitive microtrauma or overload.

Traditional vs. Current Understanding:

  • Historically “Tendinitis”: Implied inflammation was the primary driver. While inflammation may be present in acute/reactive phases, it’s often absent or minimal in chronic tendinopathy.
  • Currently “Tendinopathy”: Acknowledges a broader spectrum of pathology including:
    • Degenerative changes in collagen fibres (disorganisation, separation, microtears).
    • Increased ground substance (proteoglycans, water).
    • Neovascularisation and neural ingrowth (associated with pain).
    • Changes in tenocyte (tendon cell) activity and phenotype.
  • The term “tendinosis” is often used to describe these degenerative changes without significant inflammation.

Key Pathophysiological Features:

  • Overload/Overuse: Repetitive loading exceeding the tendon’s capacity to adapt and repair. This can be due to increased intensity, frequency, or duration of activity, or insufficient recovery.
  • Failed Healing Response: Instead of orderly repair, the tendon undergoes changes leading to altered structure and mechanical properties, making it less efficient at load transmission and more prone to pain.
  • Pain Mechanisms: Complex and not fully understood. May involve:
    • Mechanical stimulation of nociceptors in abnormal tendon tissue.
    • Biochemical irritation from substances released by tenocytes or inflammatory cells (in reactive phases).
    • Neural ingrowth into the tendon.
    • Central sensitisation in chronic cases.
💡 Load Management is Key!

Understanding that tendinopathy is primarily a load-related issue is crucial for management, which often involves modifying load (reducing aggravating activities) and then gradually reintroducing load through specific exercise programs to promote tendon adaptation.

Continuum Model of Tendinopathy (Cook & Purdam)

This model describes tendinopathy as a three-stage continuum, reflecting the tendon’s response to load. Not all tendons progress linearly through these stages, and stages can overlap or revert.

Placeholder for Continuum Model Diagram: Reactive -> Disrepair -> Degenerative
(Consider adding a simple diagram here)

Stage 1: Reactive Tendinopathy

  • Cause: Acute overload (e.g., sudden increase in training intensity/duration, unaccustomed activity).
  • Pathology: Non-inflammatory proliferative response in cells and matrix. Tendon thickens temporarily due to increased proteoglycans binding water. Collagen integrity largely maintained.
  • Clinical: Acute pain, swelling, tenderness. Often reversible with load reduction and appropriate management.
  • Analogy: Tendon “shouting” due to too much load too soon.

Stage 2: Tendon Disrepair (Failed Healing)

  • Cause: Continued overload beyond the tendon’s ability to adapt, or insufficient recovery from reactive phase.
  • Pathology: Greater matrix breakdown, increased vascularity (neovascularisation), neural ingrowth, some collagen fibre separation and disorganisation. Attempt at healing but disorganised.
  • Clinical: More persistent pain, often activity-related. May have palpable thickening. Can still be reversible with careful load management and rehabilitation.
  • Analogy: Tendon “struggling” to cope.

Stage 3: Degenerative Tendinopathy

  • Cause: Chronic overload, often in older individuals, or progression from disrepair.
  • Pathology: Areas of cell death (apoptosis), extensive collagen disorganisation and breakdown, matrix degradation, significant neovascularisation and neural ingrowth. May have macroscopic thickening, nodules, or partial tears. Little capacity for reversal of degenerative changes, but pain and function can still improve.
  • Clinical: Chronic, often grumbling pain, stiffness, reduced load tolerance, palpable nodules. Increased risk of tendon rupture.
  • Analogy: Tendon “giving up” in areas.

Clinical Implication: Management strategies should be tailored to the stage of tendinopathy. Early intervention in reactive/disrepair stages has better prognosis for full recovery. Load management is central to all stages.

Risk Factors for Tendinopathy

Development of tendinopathy is often multifactorial, involving both extrinsic and intrinsic factors.

Extrinsic Risk Factors (Related to Load & Environment):

  • Overuse / Excessive Load:
    • Sudden increase in training intensity, duration, or frequency.
    • Repetitive movements (occupational or sporting).
    • Insufficient recovery time between loading sessions.
    • High cumulative load over time.
  • Training Errors: Poor technique, inappropriate progression.
  • Equipment: Inappropriate footwear, incorrect sporting equipment (e.g., racquet grip size).
  • Playing Surface: Hard or uneven surfaces.
  • Environmental Factors: Cold weather (can reduce tendon elasticity).

Intrinsic Risk Factors (Patient-Specific):

  • Age: Increased incidence with age (tendons become less elastic, reduced healing capacity). Degenerative tendinopathy more common in older individuals.
  • Sex: Some tendinopathies more common in one sex (e.g., Achilles in men, gluteal in women). Hormonal influences may play a role.
  • Genetics: Predisposition to certain tendinopathies (e.g., collagen gene polymorphisms).
  • Previous Injury: History of tendinopathy or other MSK injury.
  • Biomechanics & Alignment:
    • Malalignment (e.g., pes planus/cavus, genu varum/valgum, leg length discrepancy).
    • Muscle weakness or imbalance (e.g., weak gluteals contributing to Achilles or patellar tendinopathy).
    • Reduced flexibility / joint stiffness.
  • Adiposity / Obesity: Increased mechanical load and potential metabolic effects.
  • Systemic Conditions:
    • Diabetes mellitus (altered collagen structure, impaired healing).
    • Inflammatory arthropathies (e.g., rheumatoid arthritis, spondyloarthropathies can cause enthesitis/tendinopathy).
    • Hypercholesterolaemia (may affect tendon structure/blood flow).
    • Hyperuricemia / Gout (urate crystal deposition in tendons).
  • Medications:
    • Fluoroquinolone antibiotics (e.g., ciprofloxacin) – increased risk of tendinopathy and rupture (especially Achilles).
    • Corticosteroids (systemic or repeated local injections) – can weaken tendon structure and increase rupture risk if injected directly into tendon substance.
    • Statins (potential association with tendinopathy, evidence less clear).
  • Smoking: Impairs healing and collagen synthesis.

Clinical Assessment Principles for Tendinopathy

Diagnosis is primarily clinical, based on a thorough history and focused physical examination.

History:

  • Pain Characteristics (SOCRATES):
    • Site: Usually well-localised to the affected tendon or its insertion.
    • Onset: Often gradual due to overuse, but can be acute after a specific incident or sudden increase in load (reactive tendinopathy).
    • Character: Aching, burning, sharp with specific movements.
    • Radiation: Usually not a primary feature unless nerve involvement.
    • Associated Symptoms: Stiffness (especially morning or after inactivity), weakness. Swelling or palpable thickening may be present. Avoid terms like “locking” or “giving way” which suggest intra-articular pathology.
    • Timing:
      • Pain often worse at the start of activity, may improve during activity (“warms up”), then worsens after activity and the next morning.
      • Night pain can occur in more severe cases.
      • Pain related to specific loads/activities.
    • Exacerbating/Relieving Factors: Specific movements, load, rest, ice/heat, analgesia.
    • Severity: Impact on ADLs, work, sport. Use pain scales (VAS, NPRS).
  • Training/Activity History: Detailed account of type, intensity, frequency, duration, recent changes, equipment, surfaces.
  • Previous episodes or other MSK issues.
  • Risk factors (extrinsic and intrinsic).
  • Patient’s goals and expectations.

Physical Examination (“Look, Feel, Move, Special Tests”):

  • Look: Swelling (localised over tendon), erythema (less common unless acute/reactive), muscle wasting (if chronic disuse), alignment, footwear.
  • Feel:
    • Localised tenderness: Directly over the affected portion of the tendon or its insertion. This is a key sign.
    • Palpable thickening, nodules, or defects in the tendon.
    • Crepitus (less common, may indicate tenosynovitis).
    • Temperature (may be warm in reactive phase).
  • Move (Range of Motion):
    • Active and passive ROM of adjacent joints may be normal or slightly restricted by pain.
    • Pain reproduced on resisted muscle contraction involving the affected tendon (isometric testing).
    • Pain on passive stretch of the affected tendon.
  • Special Tests / Provocative Tests: Specific tests for different tendinopathies (e.g., Empty Can/Jobe’s for supraspinatus, Finkelstein’s for De Quervain’s, Neer’s/Hawkins-Kennedy for shoulder impingement, painful arc for rotator cuff). These aim to selectively load the suspected tendon.
  • Functional Tests: Observe activities that reproduce pain (e.g., squat, jump, hop for lower limb tendinopathies).
  • Assessment of Kinetic Chain: Look for biomechanical faults or muscle imbalances proximally or distally that may contribute (e.g., weak glutes, foot pronation).

Common Upper Limb Tendinopathies

Shoulder:

  • Rotator Cuff Tendinopathy (Supraspinatus most common, also Infraspinatus, Subscapularis, Teres Minor):
    • AKA: Impingement syndrome (though impingement is a mechanism, not a diagnosis itself), subacromial pain syndrome.
    • Symptoms: Anterolateral shoulder pain, often radiating to deltoid insertion. Worse with overhead activities, reaching behind back, lying on affected side. Night pain common. Painful arc of abduction (typically 60-120°).
    • Signs: Tenderness over greater tuberosity/acromion. Positive impingement signs (Neer’s, Hawkins-Kennedy). Positive rotator cuff tests (e.g., Empty Can/Jobe’s for supraspinatus, resisted external/internal rotation).
  • Biceps Tendinopathy (Long Head):
    • Anterior shoulder pain, often radiating down biceps. Tenderness in bicipital groove. Pain with resisted elbow flexion/supination (Speed’s test, Yergason’s test). Often coexists with rotator cuff pathology.
  • Calcific Tendinopathy: Deposition of calcium hydroxyapatite in rotator cuff tendons (supraspinatus most common). Can cause acute severe pain (calcific “tendinitis”) or chronic ache. X-ray shows calcification.

Elbow:

  • Lateral Epicondylopathy / “Tennis Elbow” (Common Extensor Origin Tendinopathy):
    • Pain and tenderness over lateral epicondyle and common extensor origin. Worse with gripping, wrist extension (e.g., lifting a kettle, shaking hands). Cozen’s test, Mill’s test positive.
  • Medial Epicondylopathy / “Golfer’s Elbow” (Common Flexor Origin Tendinopathy):
    • Pain and tenderness over medial epicondyle and common flexor origin. Worse with gripping, wrist flexion, pronation. Resisted wrist flexion reproduces pain.

Wrist & Hand:

  • De Quervain’s Tenosynovitis:
    • Stenosing tenosynovitis of 1st dorsal compartment tendons (Abductor Pollicis Longus – APL, Extensor Pollicis Brevis – EPB).
    • Radial-sided wrist pain, radiating to thumb/forearm. Tenderness/swelling over radial styloid. Positive Finkelstein’s test.
  • Trigger Finger/Thumb (Stenosing Tenosynovitis):
    • Thickening of flexor tendon and/or A1 pulley, causing catching, locking, or painful clicking during finger flexion/extension. Palpable nodule often present.
  • Flexor Carpi Ulnaris (FCU) / Flexor Carpi Radialis (FCR) Tendinopathy: Pain at respective insertions.

Common Lower Limb Tendinopathies

Hip & Pelvis:

  • Gluteal Tendinopathy (Gluteus Medius/Minimus):
    • Major component of Greater Trochanteric Pain Syndrome (GTPS). Lateral hip pain, tenderness over greater trochanter. Worse with side-lying, walking, stairs, prolonged standing.
    • Common in middle-aged women. Weakness of hip abductors.
  • Proximal Hamstring Tendinopathy:
    • Pain deep in buttock, at ischial tuberosity. Worse with sitting (especially on hard surfaces), running, lunging, resisted hip extension/knee flexion.
  • Adductor Tendinopathy / “Groin Strain”:
    • Pain in medial thigh/groin, at adductor origin on pubic bone. Worse with resisted adduction, kicking, sprinting.
  • Rectus Femoris / Iliopsoas Tendinopathy: Anterior hip/groin pain.

Knee:

  • Patellar Tendinopathy / “Jumper’s Knee”:
    • Anterior knee pain, localised to inferior pole of patella or along patellar tendon. Overuse, common in jumping sports. Pain worse with loading (squatting, jumping, stairs), tender on palpation. Often associated with quadriceps tightness/weakness.
  • Quadriceps Tendinopathy: Pain at superior pole of patella or within quadriceps tendon.
  • Iliotibial Band (ITB) Syndrome: (Technically friction syndrome, not pure tendinopathy, but often grouped). Lateral knee pain, typically over lateral femoral epicondyle. Common in runners/cyclists. (See Knee Pain section).
  • Pes Anserine Tendinopathy/Bursitis: Medial knee pain, inferior to joint line. (See Knee Pain section).
  • Hamstring Tendinopathy (Distal): Posterior knee pain at insertions.

Ankle & Foot:

  • Achilles Tendinopathy (Mid-portion or Insertional):
    • Pain, stiffness, and often swelling in Achilles tendon. Worse with activity (running, jumping, stairs), morning stiffness common.
    • Mid-portion: 2-6cm proximal to calcaneal insertion. Palpable thickening.
    • Insertional: At calcaneal attachment. May have Haglund’s deformity (bony prominence).
    • Risk of rupture if chronic and severe.
  • Peroneal Tendinopathy (Longus/Brevis):
    • Posterolateral ankle/foot pain, behind lateral malleolus or along course of tendons. Worse with activity, resisted eversion. May have snapping or subluxation. Associated with ankle instability or cavovarus foot.
  • Tibialis Posterior Tendinopathy / Dysfunction (PTTD):
    • Medial ankle/hindfoot pain, along course of tibialis posterior tendon. Can lead to progressive flatfoot deformity (acquired flatfoot) if untreated. Pain/weakness on resisted inversion, difficulty with single heel raise (“too many toes” sign).
  • Flexor Hallucis Longus (FHL) Tendinopathy: Posteromedial ankle pain, common in dancers (“dancer’s tendinitis”).
  • Plantar Fasciitis: (Technically fasciopathy, not tendinopathy, but shares overuse principles). Inferior heel pain. (See MSK Pain section).

Imaging in Tendinopathy

Diagnosis of most tendinopathies is primarily clinical. Imaging is used selectively.

Indications for Imaging:

  • Diagnostic uncertainty / To exclude other pathology (e.g., fracture, tumour, intra-articular problem).
  • Failure to respond to appropriate conservative management.
  • To assess severity or confirm suspected tear if considering more invasive treatment (e.g., surgery).
  • Pre-operative planning.
  • To guide interventions (e.g., ultrasound-guided injections).

Common Imaging Modalities:

  • Ultrasound (USS) with Doppler:
    • Often first-line imaging for superficial tendons. Dynamic assessment possible.
    • Findings in Tendinopathy:
      • Tendon thickening (focal or diffuse).
      • Hypoechoic areas (reduced echogenicity due to matrix disorganisation, increased water).
      • Loss of normal fibrillar pattern.
      • Neovascularisation (increased blood flow on Doppler – often correlates with pain).
      • Calcifications (e.g., calcific tendinopathy).
      • Partial or full-thickness tears (anechoic/hypoechoic defects).
      • Associated peritendinous fluid or bursitis.
    • Can also assess for paratenon thickening (paratenonitis).
  • Magnetic Resonance Imaging (MRI):
    • Excellent for assessing tendon morphology, tears, and surrounding soft tissues/bone. More sensitive for subtle changes and deeper tendons.
    • Findings in Tendinopathy:
      • Increased intratendinous signal on T1 and T2/PD weighted images (intermediate signal in tendinosis, high signal with tears/fluid).
      • Tendon thickening.
      • Partial or full-thickness tears (high signal defect, fibre discontinuity, retraction).
      • Peritendinous oedema/fluid.
      • Bone marrow oedema at enthesis.
    • Useful if USS inconclusive, for deep tendons, or if intra-articular pathology also suspected.
  • Plain X-rays:
    • Generally limited for direct tendon assessment.
    • Can show:
      • Calcifications within tendon (calcific tendinopathy).
      • Bony spurs or enthesophytes at tendon insertions (e.g., Haglund’s deformity in Achilles, heel spur in plantar fasciitis).
      • Avulsion fractures.
      • Exclude other bony pathology or OA.
⚠️ Correlate Imaging with Clinical Findings!

Asymptomatic tendon abnormalities (thickening, signal changes, even small tears) are common on imaging, especially in older individuals or athletes. Imaging findings must always be interpreted in the context of the patient’s symptoms and clinical examination. Treat the patient, not just the scan!

Management Principles for Tendinopathy

Management is typically staged, starting with conservative measures. The cornerstone is appropriate load management and progressive rehabilitation.

Phase 1: Symptom Reduction & Load Management (Reactive/Early Disrepair)

  • Education: Explain diagnosis, pain mechanisms, importance of load management, expected recovery timeline. Address unhelpful beliefs.
  • Load Modification / Relative Rest:
    • Identify and reduce or temporarily avoid aggravating activities and loads. This does NOT mean complete immobilisation (which can be detrimental).
    • Modify training variables (intensity, duration, frequency, surface, equipment).
  • Pain Relief:
    • Analgesia: Paracetamol. Topical NSAIDs can be useful.
    • Oral NSAIDs: May be used for short periods (e.g., 7-14 days) in acute reactive phase if significant pain/inflammation, but limited role in chronic degenerative tendinopathy.
    • Ice: For acute pain/swelling (15-20 mins).
    • Isometrics: Pain-relieving in some tendinopathies (e.g., patellar, Achilles). Sustained contraction in mid-range.
  • Correction of Biomechanical Factors: Address muscle imbalances, flexibility deficits, footwear, orthotics if contributing.

Phase 2: Progressive Loading & Rehabilitation (Disrepair/Degenerative)

  • Progressive Exercise Therapy (Key to long-term success):
    • Aims to stimulate tendon matrix remodelling, improve tendon capacity, and restore muscle strength/function.
    • Supervised by physiotherapist. Tailored to individual tendon and stage.
    • Eccentric Exercises: Lengthening muscle contraction (e.g., heel drops for Achilles, slow lowering for patellar). Well-established for many tendinopathies.
    • Heavy Slow Resistance (HSR) Training: Slower concentric and eccentric contractions with significant load. Gaining evidence.
    • Gradual progression of load, speed, and complexity. Pain monitoring during exercises (e.g., pain acceptable up to 3-4/10 during, should settle by next day).
    • Include exercises for kinetic chain (e.g., core, hip, ankle).
  • Stretching: If associated muscle tightness.
  • Manual Therapy: Soft tissue techniques, joint mobilisation by physio if indicated.

Phase 3: Return to Function/Sport

  • Gradual, sport-specific or activity-specific training.
  • Maintenance of strength and load tolerance.
  • Strategies to prevent recurrence (e.g., ongoing conditioning, appropriate training progression).

Interventional Options (If conservative management fails after 3-6 months):

Evidence for many of these is variable, often used as adjuncts. Specialist input usually required.

  • Extracorporeal Shockwave Therapy (ESWT):
    • High-energy sound waves applied to tendon. May stimulate healing, reduce pain. Evidence for some tendinopathies (e.g., plantar fasciitis, Achilles, calcific rotator cuff).
  • Injections (Often ultrasound-guided):
    • Corticosteroid Injections: Can provide short-term pain relief, especially if significant peritendinous inflammation or bursitis. Generally avoid injecting directly into tendon substance due to risk of weakening/rupture. Controversial for chronic tendinopathy as may not address underlying pathology and could hinder long-term healing. Limit number of injections.
    • Platelet-Rich Plasma (PRP) Injections: Injecting concentrated platelets from patient’s own blood. Aims to deliver growth factors to promote healing. Evidence still evolving, variable results.
    • High Volume Injections / Hydrodissection: Injecting large volume of saline +/- anaesthetic/corticosteroid to break down adhesions or neovessels.
    • Sclerosant Injections (e.g., Polidocanol): To target neovessels.
  • Dry Needling / Tenotomy: Multiple percutaneous needle fenestrations into tendon to stimulate healing response.
  • Surgery (Rarely needed for most tendinopathies):
    • Considered for chronic, debilitating symptoms refractory to extensive conservative/less invasive treatment, or for significant structural tears.
    • Procedures: Debridement of abnormal tendon tissue, repair of tears, removal of calcific deposits, decompression (e.g., subacromial decompression for rotator cuff), tendon transfer or reconstruction if severe degeneration/rupture.

Tendon Ruptures (Brief Overview)

A complete or significant partial tear of a tendon, often occurring on a background of pre-existing tendinopathy or due to acute trauma.

Common Tendon Ruptures:

  • Achilles Tendon Rupture:
    • Often in middle-aged “weekend warriors” during sudden push-off/landing. Sensation of being “kicked” or “shot” in back of ankle, audible “pop.”
    • Inability to plantarflex strongly, palpable gap, positive Simmonds-Thompson test (calf squeeze – no plantarflexion if ruptured).
    • Management: Surgical repair vs. non-operative (functional bracing/casting). Debate continues, individualized decision. Early mobilisation important.
  • Rotator Cuff Tear (Supraspinatus most common):
    • Can be acute (trauma) or chronic degenerative. Shoulder pain, weakness (especially abduction/external rotation), night pain. Positive cuff tests, drop arm sign for large tear.
    • Management: Physio, analgesia, injections. Surgery (arthroscopic repair) for acute traumatic tears in younger/active individuals, or chronic symptomatic tears failed conservative Rx.
  • Biceps Tendon Rupture (Long Head or Distal):
    • Long Head: Often older individuals, may have pre-existing tendinopathy/impingement. Sudden pain, “Popeye” muscle deformity. Function often well-preserved, usually managed non-operatively.
    • Distal Biceps (at elbow): Usually traumatic, forceful eccentric contraction. Acute pain, bruising in antecubital fossa, weakness of supination/flexion. Hook test positive. Often requires surgical repair, especially in active individuals.
  • Patellar or Quadriceps Tendon Rupture:
    • Disruption of extensor mechanism. Inability to actively extend knee, palpable gap. Usually require urgent surgical repair. (See Patellofemoral section).
  • Other: Pectoralis major, triceps, hamstrings, tibialis posterior.

Suspected acute tendon ruptures usually require urgent T&O assessment and often imaging (USS or MRI) to confirm diagnosis and guide management.

Flashcards: Tendinopathies

Click on each card to reveal the answer.

What is the main difference in pathology between “tendinitis” and chronic “tendinopathy/tendinosis”?

(Click to flip)

Answer

“Tendinitis” implies primary inflammation. Chronic “tendinopathy/tendinosis” is characterized by degenerative changes (collagen disorganisation, neovascularisation) with minimal or absent inflammation.

According to Cook & Purdam’s continuum model, what are the three stages of tendinopathy?

(Click to flip)

Answer

1. Reactive Tendinopathy
2. Tendon Disrepair (Failed Healing)
3. Degenerative Tendinopathy

What type of exercise is a cornerstone of rehabilitation for many tendinopathies (e.g., Achilles, patellar)?

(Click to flip)

Answer

Eccentric exercises (lengthening muscle contraction). Heavy Slow Resistance (HSR) is also gaining evidence.

A positive Finkelstein’s test is indicative of which wrist/thumb tendinopathy?

(Click to flip)

Answer

De Quervain’s Tenosynovitis (affecting APL and EPB tendons in the 1st dorsal compartment).

What is the main risk associated with injecting corticosteroids directly into the substance of a load-bearing tendon?

(Click to flip)

Answer

Tendon weakening and potential for subsequent rupture.

A patient describes a sudden “kick” or “shot” sensation in the back of their ankle while playing sport, followed by inability to push off. What injury is suspected?

(Click to flip)

Answer

Achilles tendon rupture.

What is the “painful arc” sign in shoulder assessment often indicative of?

(Click to flip)

Answer

Rotator cuff tendinopathy / Subacromial impingement. (Pain typically between 60-120° of abduction).

Which class of antibiotics is notably associated with an increased risk of tendinopathy and tendon rupture?

(Click to flip)

Answer

Fluoroquinolones (e.g., ciprofloxacin, levofloxacin).

Tendinopathies Quiz (T&O Focus)

Test your knowledge on tendinopathies.

1. A 45-year-old recreational tennis player presents with pain over the lateral aspect of their elbow, worse with gripping and resisted wrist extension. On examination, there is marked tenderness over the lateral epicondyle. What is the most likely diagnosis?

A. Medial epicondylopathy (“Golfer’s elbow”).
B. Lateral epicondylopathy (“Tennis elbow”).
C. Olecranon bursitis.
D. Cubital tunnel syndrome.
Explanation: Pain over the lateral epicondyle, exacerbated by wrist extension and gripping, is characteristic of lateral epicondylopathy (common extensor origin tendinopathy).

2. According to the continuum model by Cook & Purdam, which stage of tendinopathy is characterized by a non-inflammatory proliferative response with tendon thickening, often reversible with load reduction?

A. Reactive tendinopathy.
B. Tendon disrepair.
C. Degenerative tendinopathy.
D. Calcific tendinopathy.
Explanation: Reactive tendinopathy is the initial response to acute overload, involving a non-inflammatory proliferation and tendon swelling, which is generally reversible.

3. A 30-year-old runner complains of pain in the mid-portion of her Achilles tendon, worse in the morning and at the start of runs. What type of exercise is a mainstay of her rehabilitation program?

A. Plyometric (jumping) exercises.
B. Isokinetic exercises.
C. Eccentric loading exercises (e.g., heel drops).
D. Passive stretching only.
Explanation: Eccentric exercises, such as controlled heel drops, are well-established as effective for treating mid-portion Achilles tendinopathy by promoting tendon remodelling and strength.

4. A positive Simmonds-Thompson test (calf squeeze test) indicates a rupture of which tendon?

A. Patellar tendon.
B. Achilles tendon.
C. Biceps femoris tendon.
D. Tibialis posterior tendon.
Explanation: Squeezing the calf normally causes passive plantarflexion of the foot. Absence of this plantarflexion (positive Simmonds-Thompson test) is indicative of an Achilles tendon rupture.

5. Which imaging modality is generally considered most appropriate for initial assessment of superficial tendinopathies, allowing dynamic evaluation and guidance for injections?

A. Plain X-ray.
B. MRI scan.
C. Ultrasound (USS) with Doppler.
D. CT scan.
Explanation: Ultrasound is excellent for visualizing superficial tendons, detecting thickening, tears, neovascularisation (Doppler), and can be used dynamically. It’s also useful for guiding injections.

6. Neovascularisation and neural ingrowth within a chronically painful tendon are typically associated with which stage of tendinopathy?

A. Acute reactive tendinopathy only.
B. Tendon disrepair and degenerative tendinopathy.
C. Normal healthy tendon.
D. Calcific tendinopathy only.
Explanation: Neovascularisation (new blood vessel formation) and associated neural ingrowth are characteristic features of more advanced tendinopathy stages (disrepair and degeneration) and are thought to contribute to pain.

7. Which of the following is a potential adverse effect of repeated corticosteroid injections around or into a tendon?

A. Tendon hypertrophy and strengthening.
B. Enhanced long-term healing of degenerative changes.
C. Tendon weakening and increased risk of rupture.
D. Prevention of neovascularisation.
Explanation: While corticosteroids can provide short-term pain relief by reducing inflammation, repeated injections, especially into the tendon substance, can weaken collagen structure and increase the risk of subsequent tendon rupture.

8. A “Popeye” sign (bunching of the biceps muscle distally) is associated with a rupture of which part of the biceps tendon?

A. Long head of biceps tendon (proximal rupture).
B. Short head of biceps tendon.
C. Distal biceps tendon (at the elbow).
D. Biceps muscle belly itself.
Explanation: Rupture of the long head of the biceps tendon proximally (at the shoulder) allows the muscle belly to retract distally, creating the characteristic “Popeye” deformity.

9. The primary goal of Extracorporeal Shockwave Therapy (ESWT) in tendinopathy is thought to be:

A. Direct dissolution of calcific deposits.
B. Stimulation of a healing response and pain modulation.
C. Numbing the nerve endings in the tendon.
D. Rapidly increasing tendon strength.
Explanation: ESWT is believed to work by inducing microtrauma, stimulating cellular activity, promoting neovascularization (in a controlled way), and modulating pain pathways, thereby kick-starting a healing response in chronic tendinopathies. While it can help break down some calcific deposits, that’s not its only or primary overall goal.

10. For chronic degenerative tendinopathy, which of the following management strategies has the strongest evidence for long-term improvement in pain and function?

A. Prolonged complete rest and immobilisation.
B. Regular oral NSAID use.
C. Progressive exercise therapy focusing on appropriate load management (e.g., eccentric or HSR).
D. Repeated corticosteroid injections.
Explanation: Progressive loading through specific exercise programs (like eccentric loading or heavy slow resistance) is the cornerstone of managing chronic tendinopathy, as it aims to stimulate tendon adaptation and improve its load-bearing capacity. Other options provide short-term relief or can even be detrimental long-term.

Developmental Dysplasia of the Hip (DDH) Assessment

Developmental Dysplasia of the Hip (DDH) encompasses a spectrum of abnormalities of the developing hip joint, ranging from mild acetabular dysplasia to complete dislocation of the femoral head. Early detection through screening and clinical examination is crucial to enable timely intervention and prevent long-term complications like osteoarthritis and gait abnormalities.

Overview & Definitions of DDH

DDH refers to a condition where the “ball and socket” joint of the hip does not form properly in infants and young children. It’s a developmental condition, meaning it can be present at birth or develop during infancy/early childhood.

Key Terms:

  • Dysplasia: Abnormal development or growth. In DDH, this primarily refers to a shallow acetabulum (socket).
  • Subluxation: The femoral head (ball) is partially out of the acetabulum but still in contact with it. The hip is unstable.
  • Dislocation: The femoral head is completely out of the acetabulum.
  • Instability: The hip can be subluxated or dislocated with manoeuvres (e.g., Barlow test).
  • Teratologic Dislocation: Hip dislocation that occurs in utero, often associated with neuromuscular conditions (e.g., arthrogryposis, spina bifida) or syndromes. These are usually rigid and difficult to reduce.

Importance of Early Detection:

  • The hip joint develops rapidly in early infancy. If DDH is not treated, it can lead to:
    • Limping gait (Trendelenburg gait).
    • Leg length discrepancy.
    • Limited range of motion.
    • Chronic pain.
    • Premature osteoarthritis of the hip in adulthood, potentially requiring hip replacement at a young age.
  • Treatment is most effective and less invasive when started early (ideally within first few months of life).
💡 Terminology Shift

The term “Developmental Dysplasia of the Hip” (DDH) has largely replaced “Congenital Dislocation of the Hip” (CDH) because the condition is not always present at birth and can develop or progress postnatally. It also encompasses a spectrum beyond just dislocation.

Relevant Hip Anatomy (Infant)

Understanding the infant hip’s unique features is key to DDH assessment.

  • Components of the Hip Joint:
    • Femoral Head (Ball): The proximal end of the femur. Largely cartilaginous at birth, ossification centre appears around 4-6 months.
    • Acetabulum (Socket): Formed by the ilium, ischium, and pubis. Also largely cartilaginous at birth. Its depth and coverage of the femoral head are crucial.
    • Acetabular Labrum: A fibrocartilaginous rim around the acetabulum that deepens the socket and contributes to stability. Can be inverted or hypertrophied in DDH.
    • Joint Capsule: Fibrous capsule enclosing the joint. Can become stretched (lax) or contracted in DDH. Ligamentum teres connects femoral head to acetabulum.
  • Cartilaginous Nature: Much of the infant hip joint is cartilage, which is not visible on X-ray in early infancy. This is why ultrasound is the preferred imaging modality in the first few months.
  • Growth and Development: The hip joint develops in response to the femoral head being concentrically located within the acetabulum. If the femoral head is not correctly positioned, both the acetabulum and the proximal femur may develop abnormally.
  • Muscular Influences: Muscles around the hip (e.g., adductors, iliopsoas) can contribute to instability or contractures if the hip is dysplastic or dislocated.

The goal of DDH treatment is to achieve and maintain a concentric reduction of the femoral head within the acetabulum to allow for normal development of both structures.

Pathophysiology & Spectrum of DDH

DDH is not a single entity but a spectrum of abnormalities affecting the hip’s development.

Spectrum of DDH:
  1. Acetabular Dysplasia (or Instability):
    • The mildest form. The acetabulum is shallow or “underdeveloped,” providing inadequate coverage for the femoral head.
    • The femoral head may be properly located but the hip can be unstable (demonstrable on Barlow test, or “clicky hip” on Ortolani if subluxable).
    • Often resolves spontaneously but can progress if untreated.
  2. Subluxation:
    • The femoral head is partially displaced from the acetabulum but still maintains some contact with it.
    • The joint capsule is often stretched.
  3. Dislocation:
    • The femoral head is completely displaced out of the acetabulum.
    • The capsule is significantly stretched. Over time, secondary changes can occur (e.g., inverted labrum, iliopsoas tendon constriction, formation of a “false acetabulum” on the ilium), making reduction more difficult.
Contributing Factors to Pathophysiology:
  • Ligamentous Laxity: Generalized ligamentous laxity (can be hormonally influenced by maternal relaxin in newborns, or genetic) can predispose to hip instability.
  • Mechanical Factors:
    • Intrauterine positioning: Breech presentation (especially frank breech with extended knees) is a major risk factor, as it places hips in adduction and flexion, stressing the joint. Oligohydramnios can also restrict foetal movement.
    • Postnatal positioning: Swaddling with hips and knees in extension and adduction can contribute to DDH or worsen existing instability. Traditional swaddling practices that allow hip flexion and abduction are protective.
  • Genetic Predisposition: DDH has a familial tendency. Specific genes related to connective tissue and joint development are implicated.
  • Acetabular Development: Normal acetabular development depends on the presence of a well-seated femoral head. If the head is subluxated or dislocated, the acetabulum may not deepen properly.

The natural history of untreated DDH varies. Some mild dysplasia or instability can resolve spontaneously. However, persistent subluxation or dislocation leads to abnormal joint mechanics and long-term problems.

Risk Factors for DDH

Identifying risk factors is key for targeted screening and maintaining a high index of suspicion.

Major Risk Factors (Often trigger targeted ultrasound screening):

  • Breech Presentation:
    • The single most significant risk factor (especially frank breech). Risk is ~10 times higher.
    • All babies with a history of breech presentation (at or after 36 weeks gestation, or at birth if earlier) should have a screening hip ultrasound.
  • Family History of DDH:
    • Having a parent or sibling with DDH increases the risk.
    • Screening ultrasound is usually recommended.
  • Female Sex:
    • DDH is 4-6 times more common in females, possibly due to greater ligamentous laxity in response to maternal hormones.
  • Firstborn Child:
    • Possibly due to a less stretched uterus and tighter abdominal muscles in primigravid mothers, leading to more constrained foetal positioning.

Other Potential Risk Factors / Associations:

  • Oligohydramnios: Reduced amniotic fluid restricts foetal movement.
  • Postnatal Swaddling Practices: Tight swaddling with hips and knees in extension and adduction.
  • Congenital “Packaging” Disorders / Syndromes:
    • Foot deformities (e.g., talipes equinovarus/clubfoot, metatarsus adductus).
    • Congenital torticollis.
    • Certain genetic syndromes (e.g., Down syndrome, Ehlers-Danlos syndrome, Larsen syndrome).
  • Large for Gestational Age.
  • Multiple Pregnancy.
🧠 DDH Risk Factors: “FEMALE HIP” (simplified)
F Female gender
E Eldest (Firstborn)
M Malpresentation (Breech)
A A Lot of fluid less (Oligohydramnios)
L Lax ligaments (generalized)
E External factors (tight swaddling)
H History (Family) of DDH
I In association with other packaging disorders (e.g. talipes)
P Positive clinical signs (Barlow/Ortolani)

The presence of multiple risk factors further increases the likelihood of DDH.

Clinical Examination: Newborn (NIPE – Newborn Infant Physical Examination)

The newborn hip examination is a screening tool performed within 72 hours of birth, and ideally again at 6-8 weeks. The aim is to detect hip instability.

Prerequisites for Examination:
  • Infant should be calm, relaxed, and undressed (nappy off).
  • Examiner should have warm hands.
  • Perform on a firm, flat surface.
  • Examine one hip at a time.
Inspection:
  • Look for gross asymmetry of leg length or skin folds (though skin fold asymmetry is common and non-specific).
Palpation & Provocative Manoeuvres:

These tests aim to detect a dislocatable (Barlow) or dislocated but reducible (Ortolani) hip.

  • Barlow Manoeuvre (Tests for a DISLOCATABLE hip – “Out”):
    1. Flex infant’s hips and knees to 90 degrees.
    2. Adduct the hip (bring thigh towards midline).
    3. Apply gentle posterior pressure along the line of the femur with the thumb on the inner thigh.
    4. Positive Barlow: A “clunk” or palpable sensation of the femoral head slipping out of (posteriorly dislocating from) the acetabulum. This indicates an unstable, dislocatable hip. A simple “click” without dislocation is usually benign.
  • Ortolani Manoeuvre (Tests for a DISLOCATED but REDUCIBLE hip – “In”):
    1. Start with hips and knees flexed to 90 degrees, often after a Barlow test which may have dislocated it.
    2. Gently abduct the hip (move thigh outwards/laterally).
    3. Simultaneously apply gentle upward/anterior pressure on the greater trochanter with the fingers.
    4. Positive Ortolani: A palpable “clunk” as the dislocated femoral head reduces back into the acetabulum. This indicates a dislocated but reducible hip.
Important Notes on Newborn Exam:
  • These tests are most reliable in the first few weeks of life. After this, muscle contractures can develop, making a dislocated hip irreducible (Ortolani negative) and less obviously unstable on Barlow.
  • “Clicks”: Soft, high-pitched clicks are common and usually benign, often due to ligaments or tendons moving. They are different from the “clunk” of subluxation/dislocation. However, persistent clicks or concern should prompt review/ultrasound.
  • Limited hip abduction: While more common in older infants, significantly limited abduction (<60 degrees with pelvis stabilised) can be a sign even in newborns.
⚠️ Gentle Manoeuvres

Barlow and Ortolani tests should be performed gently. Forceful examination can cause pain or iatrogenic injury. The aim is to feel for movement, not to force dislocation. If unsure, or if positive, refer for ultrasound.

Clinical Examination: Older Infant/Child (Beyond neonatal period, e.g. 6-8 week check, or later)

As infants get older, Barlow and Ortolani tests become less reliable. Other signs become more prominent if DDH is present and untreated.

Key Signs in Older Infants (e.g., >3 months):
  • Limited Hip Abduction:
    • This is often the most reliable sign in this age group.
    • With the infant supine, pelvis stabilised, and hips/knees flexed to 90 degrees, gently abduct both hips simultaneously.
    • Asymmetrical abduction or abduction limited to less than 60-75 degrees on one or both sides is suspicious for DDH (due to adductor muscle tightness/contracture around a subluxed/dislocated hip).
  • Apparent Leg Length Discrepancy (Galeazzi Sign / Allis Sign):
    • With infant supine, hips and knees flexed, and feet flat on the couch together, observe the height of the knees.
    • If one knee appears lower than the other, it suggests femoral shortening on that side, which can be due to a dislocated hip (femoral head displaced proximally).
    • More reliable if unilateral DDH.
  • Asymmetrical Thigh or Gluteal Skin Folds:
    • Observe for asymmetry in the number or depth of skin creases on the posterior thighs or buttocks.
    • This is a non-specific sign (can be present in normal infants) but may prompt closer examination. More concerning if combined with other signs.
  • Piston Mobility / Telescoping: Excessive up-and-down movement of the femur when pushing and pulling along its axis, suggesting instability (less commonly elicited now).
Signs in Walking Child (If DDH is missed):
  • Limp:
    • Painless limp is characteristic.
    • Trendelenburg Gait: If unilateral dislocation, the pelvis dips on the non-weight-bearing side due to weak hip abductor muscles (gluteus medius/minimus) on the dislocated side.
    • Waddling Gait: If bilateral dislocation.
  • Toe Walking on the affected side.
  • Increased Lumbar Lordosis (sway back) – especially with bilateral dislocation, to compensate for hip position.
  • Leg Length Discrepancy becomes more apparent.

If any of these signs are present, or if there is parental concern, referral for specialist assessment and imaging is warranted.

Imaging in DDH: Ultrasound & X-ray

Imaging is used to confirm suspected DDH, assess severity, and monitor treatment.

1. Hip Ultrasound (USS):
  • Preferred imaging modality for infants <4-6 months of age.
  • Why? Because the femoral head and much of the acetabulum are largely cartilaginous at this age and not well visualised on X-ray.
  • Techniques:
    • Static (Graf method): Standardised coronal view used to measure acetabular morphology (alpha and beta angles) and assess femoral head coverage. Classifies hips from Type I (normal) to Type IV (dislocated). Widely used in Europe.
    • Dynamic (Harcke method): Assesses hip stability with stress manoeuvres (similar to Barlow/Ortolani) under ultrasound guidance. More common in North America. Often combines morphological assessment with stability.
  • Indications for USS:
    • Positive or equivocal clinical examination findings (e.g., positive Ortolani/Barlow, limited abduction).
    • Presence of major risk factors (breech presentation, family history of DDH).
    • Follow-up of treated DDH.
  • Timing of Screening USS for Risk Factors: Typically around 4-6 weeks of age. Too early (<2-4 weeks) can lead to false positives due to physiological laxity.
2. Pelvic X-ray:
  • Used for infants and children >4-6 months of age, when the femoral head ossification centre has appeared, making bony landmarks more reliable.
  • Standard View: Anteroposterior (AP) view of the pelvis with legs in neutral position. Sometimes a frog-leg lateral view is also obtained.
  • Interpretation involves assessing various lines and angles:
    • Hilgenreiner’s Line: Horizontal line through triradiate cartilages.
    • Perkin’s Line: Vertical line perpendicular to Hilgenreiner’s, drawn from lateral edge of acetabulum. Femoral head ossific nucleus should be in inferomedial quadrant.
    • Shenton’s Line: Smooth arc along inferior border of superior pubic ramus and inferomedial aspect of femoral neck. Disrupted if hip subluxed/dislocated.
    • Acetabular Index (Angle): Angle between Hilgenreiner’s line and a line along the acetabular roof. Increased in dysplasia (normal <30° at birth, decreases with age).
    • Presence of femoral head ossific nucleus, its position relative to acetabulum.
  • Indications for X-ray:
    • Initial assessment in children >4-6 months with suspected DDH.
    • Monitoring DDH treatment (e.g., in cast or brace) in older infants/children.
    • Assessing for late complications.

CT and MRI are rarely used for primary diagnosis of DDH but may be used for complex cases, pre-operative planning, or assessing reduction after closed/open procedures, especially if concerns about soft tissue blocks.

Screening Programmes & Referral for DDH

The aim of DDH screening is early detection to facilitate timely and effective treatment.

Screening Approaches:

Screening strategies vary internationally. Common approaches include:

  • Universal Clinical Screening: All newborns undergo clinical hip examination (e.g., NIPE in UK) at birth and at routine child health surveillance checks (e.g., 6-8 week check).
  • Selective Ultrasound Screening: Hip ultrasound performed only for infants with:
    • Abnormal clinical examination findings (e.g., positive Barlow/Ortolani, limited abduction).
    • Presence of specific major risk factors (e.g., breech presentation, family history of DDH).
  • Universal Ultrasound Screening: All newborns undergo hip ultrasound (practiced in some European countries, e.g., Austria, Germany). Controversial due to cost and potential for over-treatment of physiologically immature hips if done too early.

In the UK (NICE CG1 / PHE Screening Programme):

  • Clinical Examination:
    • Within 72 hours of birth (NIPE).
    • At 6-8 weeks of age (GP check).
  • Targeted Ultrasound Scan at 4-6 weeks of age if:
    • First-degree family history of hip problems in early life.
    • Breech presentation at or after 36 weeks gestation, irrespective of mode of delivery, or breech at birth if <36 weeks.
    • Multiple pregnancy (some local guidelines).
  • Ultrasound Scan within 2 weeks if:
    • Clinical examination reveals a “clicky” hip (true clunk of Ortolani/Barlow) or any other concern (e.g., limited abduction).
Referral Guidelines (When to Refer to Paediatric Orthopaedics / Hip Clinic):
  • Positive Ortolani or Barlow test at any examination -> URGENT REFERRAL (aim for specialist assessment within few days to 2 weeks).
  • Persistently “clicky” hip on examination (even if Barlow/Ortolani negative) -> Refer for ultrasound; if USS abnormal, refer to orthopaedics.
  • Limited hip abduction (asymmetrical or bilateral restriction) -> Refer for ultrasound; if USS abnormal, refer to orthopaedics. (If >3-4 months old, X-ray may be considered instead of USS initially by GP, or refer for specialist imaging decision).
  • Abnormal screening ultrasound or X-ray results.
  • Presence of risk factors AND clinical concern, even if initial USS for risk factors alone was normal (DDH can develop).
  • Late signs of DDH (Galeazzi sign, limp, waddling gait) in an older infant or child -> Refer for assessment.
  • Parental concern about hips, even if examination seems normal (consider safety netting, re-examination, or low threshold for USS if persistent concern).

Clear communication of findings and reasons for referral is essential. Prompt referral for confirmed or highly suspected DDH is key to better outcomes.

Management Principles of DDH (Brief Overview)

Treatment aims to achieve and maintain a concentric and stable reduction of the femoral head in the acetabulum to promote normal hip joint development. Management is age-dependent and guided by specialists.

Newborns & Young Infants (typically <6 months):
  • Observation: Some very mild instability or minor ultrasound abnormalities (e.g., Graf Type IIa – physiologically immature hip) may be observed for a short period with repeat ultrasound, as many resolve spontaneously.
  • Pavlik Harness:
    • Most common initial treatment for DDH in this age group.
    • A dynamic flexion-abduction orthosis that holds hips in flexion (90-100°) and abduction (40-60°), guiding the femoral head into the acetabulum.
    • Allows some movement, promotes acetabular development.
    • Typically worn full-time for 6-12 weeks (or longer), then weaned.
    • Requires regular monitoring with ultrasound and adjustments.
    • High success rate (85-95%) if started early for appropriate indications.
    • Complications: Skin irritation, femoral nerve palsy (rare, if excessive flexion), failure of reduction, avascular necrosis (AVN) of femoral head (rare, if forced abduction).
Older Infants (e.g., 6-18 months):
  • Pavlik harness is less effective in this age group.
  • Closed Reduction and Spica Cast:
    • If Pavlik harness fails or if DDH diagnosed later.
    • Under general anaesthesia, hip is gently manipulated (reduced) into the acetabulum. Position confirmed by arthrogram (dye injected into joint) or MRI/CT.
    • A hip spica cast (covers torso down to ankles) is applied to maintain reduction.
    • Cast usually worn for ~3 months, may require changes.
Older Children (e.g., >18 months) or Failed Closed Reduction:
  • Open Reduction: Surgical procedure to directly visualise and reduce the hip joint. May involve removing obstacles to reduction (e.g., tight iliopsoas tendon, inverted labrum, ligamentum teres).
  • Often followed by spica cast.
  • Pelvic or Femoral Osteotomy: May be needed in older children or if significant residual dysplasia, to reshape the acetabulum (e.g., Salter, Pemberton osteotomy) or femur (e.g., varus derotation osteotomy) to improve stability and coverage.
Long-term Follow-up:
  • All children treated for DDH require long-term orthopaedic follow-up with clinical and radiological assessments until skeletal maturity to monitor hip development and detect any late complications (e.g., residual dysplasia, AVN, early OA).

The earlier DDH is diagnosed and treated, the simpler the treatment and the better the long-term outcome.

Flashcards: DDH Assessment

Click on each card to reveal the answer.

What do the Barlow and Ortolani manoeuvres test for in a newborn’s hip examination?

(Click to flip)

Answer

Barlow: Tests if a hip is DISLOCATABLE (femoral head slips out of acetabulum with adduction and posterior pressure – “Out”).
Ortolani: Tests if a dislocated hip is REDUCIBLE (femoral head clunks back into acetabulum with abduction and anterior pressure – “In”).

Name three major risk factors for DDH that often warrant targeted ultrasound screening.

(Click to flip)

Answer

1. Breech presentation.
2. Family history of DDH.
3. Female sex (though often combined with other factors for screening decision).

What is the preferred imaging modality for suspected DDH in an infant aged 2 months, and why?

(Click to flip)

Answer

Ultrasound (USS).
Because the femoral head and acetabulum are largely cartilaginous at this age and not well visualised on X-ray.

What is the Galeazzi (or Allis) sign, and what might it indicate?

(Click to flip)

Answer

With infant supine, hips and knees flexed, feet flat: one knee appears lower than the other.
It may indicate femoral shortening on that side, possibly due to a dislocated hip (unilateral DDH).

What is the most common initial treatment for DDH diagnosed in an infant under 6 months of age?

(Click to flip)

Answer

Pavlik Harness. (A dynamic flexion-abduction orthosis).

Limited hip abduction is often the most reliable clinical sign of DDH in an infant older than 3 months. What degree of abduction is typically considered restricted?

(Click to flip)

Answer

Abduction limited to less than 60-75 degrees on one or both sides (or significant asymmetry between sides).

What is Shenton’s line, and why is it assessed on a pelvic X-ray for DDH?

(Click to flip)

Answer

A smooth arc along the inferior border of the superior pubic ramus and the inferomedial aspect of the femoral neck.
It is assessed because a break or disruption in Shenton’s line suggests hip subluxation or dislocation.

What long-term complication is a major concern if DDH is left untreated or inadequately treated?

(Click to flip)

Answer

Premature Osteoarthritis of the hip, leading to pain, functional limitation, and potential need for early hip replacement.

DDH Assessment Quiz

Test your knowledge with these Orthopaedic scenarios.

1. During a newborn examination, you perform the Barlow test. You feel a distinct “clunk” as the femoral head seems to slip posteriorly out of the acetabulum. This finding indicates:

A. A normal, stable hip.
B. A dislocatable hip (hip instability).
C. A dislocated but reducible hip.
D. Normal ligamentous clicking.
Explanation: A positive Barlow test (palpable clunk of dislocation) indicates that the hip is unstable and can be dislocated with the manoeuvre.

2. Which of the following is the single most significant risk factor for DDH?

A. Female sex.
B. Firstborn child.
C. Breech presentation.
D. Family history of osteoarthritis.
Explanation: Breech presentation, especially frank breech, significantly increases the risk of DDH and is a key indication for targeted ultrasound screening.

3. What is the most appropriate imaging modality to confirm suspected DDH in a 6-week-old infant with a positive Ortolani sign?

A. Hip Ultrasound (USS).
B. Pelvic X-ray.
C. CT scan of the hips.
D. MRI scan of the hips.
Explanation: Hip ultrasound is the preferred imaging for infants under 4-6 months because the femoral head and acetabulum are largely cartilaginous. A positive Ortolani sign warrants urgent ultrasound.

4. At a 6-month well-child check, you note that an infant has significantly limited abduction of the left hip compared to the right. The Barlow and Ortolani tests are negative. What is the most appropriate next step?

A. Reassure parents and review in 2 months.
B. Refer for a pelvic X-ray and specialist orthopaedic assessment.
C. Attempt a Pavlik harness fitting in primary care.
D. Advise parents on stretching exercises.
Explanation: Limited hip abduction is a key sign of DDH in an infant >3 months. At 6 months, X-ray is appropriate as the femoral head ossification centre should be present. Specialist assessment is needed. Barlow/Ortolani are often negative at this age even with DDH.

5. The Pavlik harness aims to maintain an infant’s hips in which position to promote concentric reduction and acetabular development?

A. Extension and Adduction.
B. Flexion and Abduction.
C. Neutral rotation and Extension.
D. Internal rotation and Adduction.
Explanation: The Pavlik harness holds the hips in a position of flexion (typically 90-100 degrees) and controlled abduction, which directs the femoral head into the acetabulum.

6. A 2-year-old child presents with a painless limp and a waddling gait. On examination, there is increased lumbar lordosis. If DDH is suspected, which of the following is most likely?

A. Unilateral hip subluxation.
B. Bilateral hip dislocation.
C. Mild acetabular dysplasia only.
D. Septic arthritis of the hip.
Explanation: A waddling gait and increased lumbar lordosis in a walking child are characteristic signs of bilateral hip dislocation. A Trendelenburg limp is more typical of unilateral dislocation. Septic arthritis is usually painful.

7. According to UK NICE guidelines, a newborn with a confirmed breech presentation at or after 36 weeks gestation should routinely undergo:

A. A pelvic X-ray at 6 weeks.
B. A hip ultrasound scan at 4-6 weeks of age.
C. Clinical hip examination only, if normal no further action.
D. Referral to an orthopaedic surgeon by 2 weeks of age.
Explanation: Breech presentation is a major risk factor for DDH, and UK guidelines recommend targeted hip ultrasound screening at 4-6 weeks of age for these infants, even if the clinical newborn examination is normal.

8. All of the following are potential long-term complications of untreated DDH EXCEPT:

A. Premature osteoarthritis.
B. Leg length discrepancy.
C. Chronic hip pain.
D. Increased risk of femoral shaft fractures.
Explanation: While untreated DDH leads to significant hip morbidity (osteoarthritis, pain, limp, LLD), it does not directly increase the risk of femoral shaft fractures. The abnormal biomechanics affect the joint itself primarily.

9. The term “acetabular dysplasia” in the context of DDH primarily refers to:

A. An abnormally large femoral head.
B. A shallow or underdeveloped acetabulum (socket).
C. Inflammation of the hip joint capsule.
D. Avascular necrosis of the femoral head.
Explanation: Acetabular dysplasia specifically means that the socket (acetabulum) of the hip joint is shallow and does not provide adequate coverage for the femoral head.

10. When examining an infant’s hips with the Galeazzi (Allis) sign, what are you comparing?

A. The range of hip abduction on both sides.
B. The relative height of the knees when hips and knees are flexed with feet flat on the surface.
C. The presence of asymmetrical skin folds on the thighs.
D. The stability of the hip joint during Barlow manoeuvre.
Explanation: The Galeazzi sign compares the knee heights with the infant supine and hips/knees flexed. A difference in knee height suggests a leg length discrepancy, potentially due to a dislocated hip causing femoral shortening.

Trauma & Orthopaedics: Bone Tumours Approach

Bone tumours, while relatively uncommon, can be primary (originating in bone) or secondary (metastatic from another site). A systematic approach is crucial for early detection, appropriate investigation, and timely referral to specialist centres, as some bone tumours can be aggressive and life-threatening.

Overview & Broad Classification of Bone Tumours

Bone tumours are neoplasms arising from bone tissue. They can be benign (non-cancerous) or malignant (cancerous).

Broad Classification:

  • Primary Bone Tumours: Originate from cells within the bone or cartilage.
    • Benign: More common than malignant primary tumours. Usually slow-growing, well-defined, and do not metastasize. Examples: Osteochondroma, Enchondroma, Non-ossifying fibroma, Giant cell tumour (can be locally aggressive).
    • Malignant (Sarcomas): Rare but aggressive. Can metastasize, often to lungs. Examples: Osteosarcoma, Ewing’s sarcoma, Chondrosarcoma.
  • Secondary (Metastatic) Bone Tumours:
    • Cancer cells that have spread to the bone from a primary tumour elsewhere in the body.
    • Much more common than primary malignant bone tumours in adults.
    • Common primary sites that metastasize to bone: Prostate, Breast, Lung, Kidney, Thyroid (“PB KTL” – Lead Kettle or similar mnemonics).
  • Tumour-like Lesions: Conditions that can mimic tumours on imaging but are not true neoplasms (e.g., simple bone cyst, fibrous dysplasia, infection).

Importance of Age:

The age of the patient is a very important factor in the differential diagnosis of a bone lesion, as certain tumours are more prevalent in specific age groups.

  • Children & Adolescents: Osteosarcoma, Ewing’s sarcoma, Osteochondroma, Simple bone cyst.
  • Young Adults: Giant cell tumour.
  • Older Adults (>40-50 years): Metastatic disease, Multiple myeloma, Chondrosarcoma.
💡 OSCE/PLAB Tip

For a junior doctor, the key is not to diagnose the specific type of bone tumour, but to recognise suspicious features, perform a safe initial assessment, order appropriate first-line investigations, and know when to make an URGENT referral to a specialist sarcoma service or oncology team.

Red Flags & Common Presentations

Recognising red flag symptoms and signs is crucial for timely referral.

Key Symptoms Suggestive of a Bone Tumour:
  • Persistent Bone Pain:
    • Often deep, boring, or aching in nature.
    • Progressively worsening over time.
    • Night pain (wakes patient from sleep, not relieved by simple analgesia) is a significant red flag.
    • Pain unrelated to activity or injury, or pain at rest.
  • Palpable Mass or Swelling:
    • May be hard, fixed, or rapidly enlarging.
    • Tenderness over the swelling.
  • Pathological Fracture:
    • A fracture occurring with minimal or no trauma, through an area of weakened bone. This is a common presentation for both primary and metastatic tumours.
  • Systemic Symptoms (more common with malignant or metastatic disease):
    • Unexplained weight loss, fever, night sweats, fatigue, loss of appetite.
  • Neurological Symptoms:
    • If tumour affects spine or compresses nerves (e.g., radicular pain, weakness, numbness, sphincter disturbance – cauda equina syndrome is an emergency).
  • Restricted Joint Movement or Limp.
  • Hypercalcaemia Symptoms (nausea, vomiting, constipation, confusion, polyuria) – can occur with bone metastases or some primary tumours.
When to Suspect a Bone Tumour:
  • Any of the red flag symptoms listed above.
  • Unexplained bone pain, especially in children/adolescents or older adults.
  • A new or changing lump associated with bone.
  • Incidental finding of a suspicious bone lesion on imaging done for other reasons.
  • Known history of cancer presenting with new bone pain.
⚠️ Night Pain & Systemic Symptoms

Persistent, unexplained bone pain, especially night pain that disturbs sleep, and the presence of systemic symptoms like weight loss or fever, should always raise suspicion for malignancy and warrant urgent investigation/referral.

History Taking & Clinical Examination

A thorough history and examination can provide important clues.

History:
  • Presenting Complaint:
    • Pain: SOCRATES (Site, Onset, Character, Radiation, Associated symptoms, Timing/Triggers, Exacerbating/Relieving factors, Severity). Specifically ask about night pain, rest pain.
    • Swelling/Lump: Onset, duration, rate of growth, associated pain, skin changes.
    • Functional Limitation: Impact on mobility, ADLs.
    • History of Trauma: Even minor trauma if pathological fracture suspected.
  • Systemic Symptoms: Weight loss, fever, night sweats, malaise.
  • Past Medical History:
    • Previous cancer: Crucial for considering metastases.
    • Other bone conditions (e.g., Paget’s disease, previous benign lesions).
    • Conditions predisposing to pathological fractures (e.g., osteoporosis, renal osteodystrophy).
  • Family History: Some rare genetic syndromes predispose to bone tumours (e.g., Li-Fraumeni syndrome, hereditary retinoblastoma).
  • Social History: Smoking, alcohol, occupation (rarely relevant for primary bone tumours but part of general history). Impact on life.
Examination:
  • General Examination:
    • Cachexia, pallor, lymphadenopathy (suggestive of malignancy/metastasis).
    • Temperature.
  • Local Examination of Affected Area (Look, Feel, Move, Special Tests):
    • Look: Swelling (site, size, shape, skin changes – redness, warmth, venous distension, ulceration), deformity, muscle wasting.
    • Feel: Tenderness (localised or diffuse), temperature, consistency of any mass (hard, soft, fluctuant), fixity to underlying structures, local lymph nodes.
    • Move: Range of motion of adjacent joints (active and passive), pain on movement, crepitus. Assess muscle power.
    • Neurovascular Status: Distal to the lesion.
    • Measure: Size of any mass. Limb circumference if swelling.
  • Systemic Examination (guided by history):
    • Examine potential primary sites if metastases suspected (e.g., breast, prostate, lungs, thyroid).
    • Full neurological examination if spinal involvement or neurological symptoms.

Initial Investigations (in a Non-Specialist Setting like GP/A&E)

The primary aim of initial investigations is to confirm or exclude a bony lesion and identify features suggestive of malignancy that warrant urgent specialist referral.

1. Plain X-ray:
  • First-line imaging investigation for suspected bone tumour.
  • At least two orthogonal views (e.g., AP and lateral) of the entire affected bone/region. Include joints above and below if near a joint.
  • What to look for on X-ray (features suggesting aggression/malignancy):
    • Pattern of Bone Destruction:
      • Geographic: Well-defined, sclerotic margin (often benign).
      • Moth-eaten: Multiple small, irregular lytic areas (aggressive).
      • Permeative: Diffuse, ill-defined, small elongated lucencies (highly aggressive, e.g., Ewing’s, lymphoma).
    • Margins/Zone of Transition: Narrow, well-defined (benign) vs. Wide, ill-defined (malignant).
    • Periosteal Reaction: Reaction of the periosteum to underlying pathology.
      • Solid, smooth, thick: Benign, slow-growing.
      • Lamellated (“onion-skin”): Aggressive (e.g., Ewing’s).
      • Spiculated (“sunburst” or “hair-on-end”): Highly aggressive (e.g., Osteosarcoma).
      • Codman’s Triangle: Periosteum lifted off bone by tumour, forming a triangle (aggressive).
    • Soft Tissue Mass/Extension: Presence of a soft tissue component outside the bone suggests aggression.
    • Matrix Mineralisation: Type of matrix produced by tumour (e.g., osteoid – bone-forming, chondroid – cartilage-forming) can give clues to tumour type.
    • Location within bone: Epiphysis, metaphysis, diaphysis. Central or eccentric.
    • Pathological fracture.
  • NICE NG12 (Suspected Cancer Pathway) recommends urgent X-ray (within 2 weeks) for unexplained bone pain or swelling, especially if progressive or night pain.
2. Blood Tests:
  • FBC: Anaemia (chronic disease, marrow infiltration), raised WCC (infection, leukaemia).
  • ESR/CRP: Inflammatory markers. Often very high in Ewing’s sarcoma, osteomyelitis. Can be raised in other malignancies.
  • Alkaline Phosphatase (ALP): Often elevated in conditions with increased bone turnover (e.g., osteosarcoma, Paget’s disease, healing fractures, liver/bone metastases).
  • Calcium & Phosphate: Hypercalcaemia can occur with bone destruction (metastases, myeloma).
  • LDH (Lactate Dehydrogenase): Can be a marker of cell turnover/tumour burden (e.g., Ewing’s, lymphoma).
  • U&Es, LFTs: Baseline, assess for organ dysfunction.
  • If Myeloma Suspected (older adult, bone pain, pathological fracture, hypercalcaemia, renal impairment): Serum protein electrophoresis & urine Bence-Jones protein, immunoglobulins, free light chains.
  • If Metastases Suspected: Tumour markers if primary known or suspected (e.g., PSA for prostate cancer).

Further imaging (CT, MRI, bone scan, PET-CT) and biopsy are typically arranged by the specialist sarcoma/oncology service. DO NOT BIOPSY a suspected primary bone sarcoma in a non-specialist unit – it can compromise definitive surgery.

Common Benign Bone Tumours & Tumour-like Lesions

Many benign bone lesions are found incidentally. Most do not require treatment unless symptomatic or risk of pathological fracture.

Examples (for awareness, not detailed diagnosis):
  • Osteochondroma (Exostosis):
    • Most common benign bone tumour. Cartilage-capped bony projection arising from bone surface, often near physis.
    • Usually presents in childhood/adolescence as a painless, hard lump. Stops growing after skeletal maturity.
    • X-ray: Pedunculated or sessile bony outgrowth, cortex and medulla continuous with host bone. Cartilage cap not always visible.
    • Rarely undergoes malignant transformation to chondrosarcoma (<1%).
    • Treatment: Observation if asymptomatic. Excision if painful, causing mechanical symptoms, or concern for malignancy.
  • Enchondroma:
    • Benign tumour of hyaline cartilage within medullary cavity of bone. Common in small bones of hands/feet, also long bones.
    • Often asymptomatic, incidental finding. Can cause pain or pathological fracture.
    • X-ray: Well-defined lytic lesion with “rings and arcs” or “popcorn” calcification (chondroid matrix).
    • Malignant transformation to chondrosarcoma is rare but possible (Ollier’s disease, Maffucci’s syndrome have higher risk).
    • Treatment: Observation. Curettage and bone grafting if symptomatic or large.
  • Non-ossifying Fibroma (NOF) / Fibrous Cortical Defect:
    • Common developmental lesion in children/adolescents, usually in metaphysis of long bones (e.g., distal femur, proximal tibia).
    • Asymptomatic, incidental finding. Usually self-resolves (ossifies) with age.
    • X-ray: Eccentric, well-defined, cortically based lytic lesion with sclerotic rim.
    • Treatment: None usually needed. Rarely, curettage/grafting if very large or pathological fracture.
  • Simple (Unicameral) Bone Cyst:
    • Fluid-filled cavity, usually in metaphysis of long bones (proximal humerus/femur) in children/adolescents.
    • Often asymptomatic until pathological fracture (“fallen leaf” sign on X-ray if fragment in cyst).
    • X-ray: Central, well-defined lytic lesion, thinned cortex.
    • Treatment: Observation, aspiration/steroid injection, or curettage/bone grafting.
  • Giant Cell Tumour (GCT) of Bone:
    • Locally aggressive benign tumour, typically in epiphysis of mature long bones (e.g., distal femur, proximal tibia, distal radius) in young adults (20-40 years).
    • Can cause pain, swelling, pathological fracture. Can rarely metastasize to lungs (benign metastases).
    • X-ray: Eccentric, expansile, lytic lesion in epiphysis, extending to subchondral bone, often ill-defined margin.
    • Treatment: Curettage with adjuvant (e.g., phenol, cryotherapy) and bone cement/graft. Denosumab may be used. Resection if extensive. Requires careful follow-up due to recurrence risk.
  • Fibrous Dysplasia: Developmental anomaly where normal bone is replaced by fibrous tissue. Can be monostotic or polyostotic. X-ray: “Ground glass” appearance.

Primary Malignant Bone Tumours (Sarcomas)

These are rare but require urgent specialist management at a sarcoma centre.

1. Osteosarcoma:
  • Most common primary malignant bone tumour (excluding myeloma). Produces osteoid (immature bone).
  • Age: Bimodal peak – 75% in adolescents/young adults (10-25 years), smaller peak in older adults (often secondary to Paget’s or radiation).
  • Location: Metaphysis of long bones, commonly around the knee (distal femur, proximal tibia), proximal humerus.
  • Symptoms: Pain (often night pain), swelling, palpable mass. Pathological fracture less common initially.
  • X-ray: Aggressive lesion. Mixed lytic/sclerotic. Ill-defined margins. Periosteal reaction (Codman’s triangle, sunburst). Soft tissue extension.
  • Management: Neoadjuvant chemotherapy, then surgical resection (limb salvage or amputation), then adjuvant chemotherapy. High metastatic potential (lungs).
2. Ewing’s Sarcoma:
  • Small round blue cell tumour. Second most common primary malignant bone tumour in children/adolescents.
  • Age: Typically 5-25 years.
  • Location: Diaphysis or metaphysis of long bones (femur, tibia, humerus), pelvis, ribs.
  • Symptoms: Pain, swelling, systemic symptoms common (fever, weight loss, fatigue). Can mimic infection. ESR/CRP/LDH often very high.
  • X-ray: Permeative or moth-eaten lytic lesion, ill-defined. Lamellated (“onion-skin”) periosteal reaction. Large soft tissue mass.
  • Management: Chemotherapy (neoadjuvant and adjuvant), surgery and/or radiotherapy.
3. Chondrosarcoma:
  • Malignant tumour of cartilage-producing cells.
  • Age: Typically older adults (40-70 years). Rare in children.
  • Location: Pelvis, proximal femur, ribs, shoulder girdle. Can arise de novo or from pre-existing benign lesion (e.g., osteochondroma, enchondroma).
  • Symptoms: Insidious onset pain, swelling. Often slow-growing (low grade) but can be aggressive (high grade).
  • X-ray: Lytic lesion with characteristic chondroid matrix calcification (rings, arcs, popcorn). Endosteal scalloping, cortical thickening/destruction.
  • Management: Primarily surgical resection. Generally resistant to chemotherapy and radiotherapy (except some high-grade or dedifferentiated types).
Other Rare Primary Malignant Bone Tumours:
  • Adamantinoma, Chordoma, Fibrosarcoma, Malignant Fibrous Histiocytoma (MFH – now Undifferentiated Pleomorphic Sarcoma).

Metastatic Bone Disease

Secondary bone tumours (metastases) are the most common malignant bone tumours in adults.

Common Primary Cancers that Metastasize to Bone (“PB KTL”):
  • Prostate
  • Breast
  • Kidney (Renal Cell Carcinoma)
  • Thyroid
  • Lung
  • (Also: Multiple Myeloma – considered a primary haematological malignancy of bone marrow but presents with bone lesions).
Clinical Presentation:
  • Bone Pain: Most common symptom. Often insidious, progressive, worse at night.
  • Pathological Fractures: Especially in weight-bearing bones.
  • Spinal Cord Compression: If vertebral metastases. Neurological emergency – requires urgent MRI and management (steroids, radiotherapy, surgery).
  • Hypercalcaemia: Due to bone resorption. Symptoms: nausea, vomiting, constipation, confusion, polyuria.
  • Symptoms related to primary cancer (if not yet diagnosed).
  • Incidental finding on staging scans for known cancer.
Radiographic Appearance:
  • Lytic lesions (bone-destroying): Common with lung, kidney, thyroid, myeloma. Appear as “holes” in bone.
  • Sclerotic/Blastic lesions (bone-forming): Common with prostate cancer. Appear as dense areas.
  • Mixed lytic and sclerotic lesions: Common with breast cancer.
  • Multiple lesions are common. Axial skeleton (spine, pelvis, ribs, skull) and proximal long bones are frequent sites.
Management (Palliative Intent in most cases, aims to relieve pain, maintain function, prevent complications):
  • Treat the Primary Cancer: Systemic therapy (chemotherapy, hormone therapy, targeted therapy, immunotherapy).
  • Analgesia: Stepwise approach (paracetamol, NSAIDs cautiously, opioids).
  • Radiotherapy: Effective for localised bone pain from metastases. Single or multiple fractions.
  • Bisphosphonates (e.g., zoledronic acid) or Denosumab: Reduce bone pain, risk of skeletal-related events (SREs – pathological fractures, spinal cord compression, need for surgery/radiotherapy to bone, hypercalcaemia).
  • Surgical Fixation: Prophylactic fixation of impending pathological fractures (e.g., Mirels’ score used to assess risk) or fixation of actual pathological fractures. Stabilisation of spinal metastases if cord compression.
  • Management of hypercalcaemia (IV fluids, bisphosphonates).
  • MDT approach involving oncologists, palliative care, orthopaedic surgeons, radiologists.

Referral Pathways & Multidisciplinary Team (MDT)

Prompt and appropriate referral is critical for patients with suspected bone tumours.

When to Refer Urgently (e.g., 2-Week Wait Suspected Cancer Pathway – NICE NG12):
  • Children and Young Adults (any age up to 24 years): Unexplained bone pain or swelling, especially if progressive or night pain. Refer for urgent X-ray (within 48 hours) and if X-ray suggests possible sarcoma, refer immediately to a sarcoma specialist centre.
  • Adults (≥25 years):
    • X-ray findings suggestive of bone sarcoma.
    • Unexplained bone pain or swelling that is persistent, progressive, or associated with night pain, where X-ray is normal but clinical concern remains high (consider further imaging like MRI before referral, or refer for specialist opinion).
    • Soft tissue lump increasing in size (for suspected soft tissue sarcoma – often managed by same teams).
  • Suspected Spinal Cord Compression: EMERGENCY admission/referral to appropriate team (usually neurosurgery or oncology with spinal service).
  • Suspected Hypercalcaemia of Malignancy: Urgent assessment and management.
  • Pathological Fracture: Refer to orthopaedics. If underlying lesion suspicious for malignancy, they will involve sarcoma MDT.
Referral Destination:
  • Patients with suspected primary bone sarcoma should be referred to a Specialist Sarcoma Centre with an MDT.
  • Patients with suspected metastatic bone disease are usually managed by oncologists relevant to the primary tumour type, often with input from orthopaedics, palliative care, and specialist nurses.
  • Patients with clearly benign lesions on X-ray that are asymptomatic may not require referral, or may be referred to routine orthopaedics if symptomatic.
Role of the Multidisciplinary Team (MDT) in Sarcoma Care:

Essential for diagnosis, staging, and treatment planning for primary bone sarcomas. Includes:

  • Orthopaedic Sarcoma Surgeons
  • Medical Oncologists
  • Clinical Oncologists (Radiotherapists)
  • Radiologists (specialising in musculoskeletal imaging)
  • Pathologists (specialising in bone/soft tissue tumours)
  • Specialist Nurses
  • Physiotherapists, Occupational Therapists
  • Psychologists, Social Workers
  • Palliative Care Specialists
Biopsy Protocol

A biopsy of a suspected primary bone sarcoma should ONLY be performed at a specialist sarcoma centre by a surgeon experienced in sarcoma surgery. An inappropriately performed biopsy can contaminate tissue planes, spread tumour cells, and compromise limb salvage surgery or even curative outcome. If sarcoma is suspected, refer BEFORE biopsy.

Flashcards: Bone Tumours Approach

Click on each card to reveal the answer.

What is the most common type of malignant bone tumour in adults?

(Click to flip)

Answer

Metastatic bone disease (secondary tumours).

Name two “red flag” symptoms for a possible bone tumour.

(Click to flip)

Answer

1. Persistent, unexplained bone pain, especially night pain.
2. Palpable bone swelling or mass.
(Also: pathological fracture, systemic symptoms like weight loss).

What is the first-line imaging investigation for suspected bone tumour?

(Click to flip)

Answer

Plain X-ray (at least two orthogonal views).

Name two aggressive periosteal reactions seen on X-ray that suggest malignancy.

(Click to flip)

Answer

1. Lamellated (“onion-skin”) reaction.
2. Spiculated (“sunburst” or “hair-on-end”) reaction.
(Also: Codman’s triangle).

What is the most common primary malignant bone tumour in adolescents, and where does it typically occur?

(Click to flip)

Answer

Osteosarcoma. Typically in the metaphysis of long bones, commonly around the knee (distal femur/proximal tibia).

What does the mnemonic “PB KTL” refer to in the context of bone metastases?

(Click to flip)

Answer

Common primary cancers that metastasize to bone: Prostate, Breast, Kidney, Thyroid, Lung.

Where should a biopsy of a suspected primary bone sarcoma be performed?

(Click to flip)

Answer

ONLY at a specialist sarcoma centre by an experienced sarcoma surgeon/radiologist, as part of an MDT approach.

What is a pathological fracture?

(Click to flip)

Answer

A fracture that occurs through bone weakened by an underlying pathological process (e.g., tumour, osteoporosis, infection) with minimal or no trauma.

Bone Tumours Approach Quiz

Test your knowledge with these Orthopaedic questions.

1. A 16-year-old boy presents with a 2-month history of worsening pain in his right distal femur, particularly severe at night, waking him from sleep. There is a tender, firm swelling. What is the most important initial investigation?

A. MRI scan of the femur.
B. Plain X-ray of the right femur (AP and lateral).
C. Full blood count and ESR/CRP.
D. Bone biopsy.
Explanation: For suspected bone tumour, a plain X-ray is the first-line imaging. Night pain and swelling in an adolescent are red flags for primary bone sarcoma like osteosarcoma. Biopsy is a specialist procedure.

2. An X-ray of a 65-year-old man with back pain shows multiple well-defined lytic lesions in the vertebrae and pelvis. Blood tests show hypercalcaemia and renal impairment. Which diagnosis should be strongly considered?

A. Osteoarthritis.
B. Osteoporosis with vertebral fractures.
C. Multiple Myeloma or Metastatic Carcinoma.
D. Paget’s disease of bone.
Explanation: Multiple lytic bone lesions in an older adult, especially with hypercalcaemia and renal impairment, are highly suggestive of Multiple Myeloma or widespread metastatic disease.

3. Which of the following X-ray features is most characteristic of a benign bone lesion rather than a malignant one?

A. Ill-defined margins (wide zone of transition).
B. Well-defined sclerotic rim.
C. Permeative pattern of bone destruction.
D. Codman’s triangle periosteal reaction.
Explanation: A well-defined (narrow zone of transition) lesion with a sclerotic (dense white) rim is typical of a slow-growing, benign process. The other features suggest aggression/malignancy.

4. A 70-year-old woman with a known history of breast cancer presents with new, severe, localised thoracic back pain and difficulty walking. What is the most urgent concern?

A. Osteoporotic vertebral fracture.
B. Metastatic spinal cord compression.
C. Disc prolapse.
D. Shingles (herpes zoster).
Explanation: In a patient with known cancer, new severe back pain with neurological symptoms (difficulty walking) must be treated as suspected metastatic spinal cord compression until proven otherwise – this is an oncological emergency.

5. “Onion-skin” (lamellated) periosteal reaction on an X-ray is classically associated with which primary malignant bone tumour?

A. Osteosarcoma.
B. Ewing’s sarcoma.
C. Chondrosarcoma.
D. Giant cell tumour.
Explanation: A lamellated or “onion-skin” periosteal reaction is a characteristic, though not exclusive, feature of Ewing’s sarcoma, reflecting layers of new bone formation in response to tumour growth.

6. A GP suspects a primary bone sarcoma in a 20-year-old based on clinical features and an aggressive-looking X-ray. What is the most appropriate next step regarding biopsy?

A. Perform a needle biopsy in the GP surgery.
B. Refer urgently to a specialist sarcoma centre BEFORE any biopsy is attempted.
C. Refer to a general orthopaedic surgeon for biopsy.
D. Request a CT-guided biopsy from local radiology.
Explanation: Biopsy of a suspected primary bone sarcoma must be done at a specialist sarcoma centre by an experienced team to avoid compromising future treatment. Urgent referral to such a centre is paramount.

7. Which of these benign bone lesions typically occurs in the epiphysis of long bones in skeletally mature young adults (20-40 years) and can be locally aggressive?

A. Osteochondroma.
B. Non-ossifying fibroma.
C. Giant cell tumour (GCT) of bone.
D. Simple bone cyst.
Explanation: Giant cell tumours characteristically occur in the epiphysis of long bones after physeal closure (skeletal maturity) and are known for being locally aggressive with a risk of recurrence.

8. A patient with known prostate cancer develops bone pain. An X-ray shows dense, sclerotic lesions in the pelvis. This appearance is most consistent with:

A. Lytic metastases.
B. Sclerotic (blastic) metastases.
C. Paget’s disease of bone.
D. Osteomyelitis.
Explanation: Prostate cancer commonly produces sclerotic (bone-forming) metastases, which appear as dense, opaque areas on X-ray.

9. Which blood test is often markedly elevated in Ewing’s sarcoma and can also be raised in osteomyelitis, reflecting inflammation or high cell turnover?

A. Serum calcium.
B. Alkaline phosphatase (ALP).
C. ESR (Erythrocyte Sedimentation Rate) and/or LDH (Lactate Dehydrogenase).
D. PSA (Prostate-Specific Antigen).
Explanation: ESR and LDH are often significantly elevated in Ewing’s sarcoma, reflecting the systemic inflammatory response and high cell turnover associated with this aggressive tumour. ALP is more typical for osteosarcoma.

10. The most common benign bone tumour, often presenting as a painless, hard lump near a joint in an adolescent, is:

A. Osteochondroma.
B. Enchondroma.
C. Osteoid osteoma.
D. Giant cell tumour.
Explanation: Osteochondroma (exostosis) is the most common benign bone tumour, typically arising from the surface of bones near growth plates.

Orthopaedic Emergencies

Orthopaedic emergencies are conditions requiring prompt recognition and urgent, often immediate, intervention to prevent limb loss, permanent disability, or even death. This section covers key orthopaedic emergencies, focusing on their identification and initial management principles relevant to frontline clinicians.

Overview & Principles of Orthopaedic Emergencies

Time is critical in managing orthopaedic emergencies. The primary goals are to preserve life, limb, and function.

Key Principles:

  • High Index of Suspicion: Many orthopaedic emergencies can have subtle early signs. Maintain a high index of suspicion, especially in high-risk scenarios (e.g., high-energy trauma, immunocompromised patients).
  • ABCDE Approach: In trauma patients, always follow the Advanced Trauma Life Support (ATLS) principles. Address life-threatening injuries first. Orthopaedic injuries are often “D” (Disability) or “E” (Exposure/Environment) but can cause “C” (Circulation) problems.
  • Early Recognition: Promptly identify the specific emergency.
  • Rapid Assessment: Focused history and examination. Crucially, assess neurovascular status distal to any injury.
  • Immediate Intervention (if indicated): This may include gross realignment of deformed limbs, splinting, IV antibiotics, analgesia, and urgent specialist referral.
  • DO NOT DELAY REFERRAL: Contact the on-call orthopaedic team urgently. Clear communication of findings and concerns is vital.
  • Pain Management: Provide adequate analgesia early.
  • Documentation: Thoroughly document findings, interventions, and neurovascular status before and after any manipulation.
“Time is Tissue”

This mantra applies to many orthopaedic emergencies. Delays in diagnosis or treatment can lead to irreversible damage to nerves, blood vessels, muscles, or joints, resulting in permanent functional deficits or limb loss.

Open Fractures (Compound Fractures)

An open fracture is one in which there is a break in the skin and underlying soft tissues that communicates directly with the fracture or its haematoma.

Significance: SURGICAL EMERGENCY
  • High risk of infection (osteomyelitis, soft tissue infection).
  • Potential for significant soft tissue damage.
  • Associated neurovascular injuries are common.
Gustilo-Anderson Classification (Simplified – for general awareness):
  • Type I: Clean wound <1 cm, simple fracture pattern. Low energy.
  • Type II: Wound >1 cm, moderate soft tissue damage, no extensive stripping of periosteum. Moderate energy.
  • Type III: Extensive soft tissue damage, high energy injury. Subdivided:
    • IIIA: Adequate soft tissue coverage possible.
    • IIIB: Extensive soft tissue loss, periosteal stripping, bone exposure. Requires soft tissue reconstruction (e.g., flap).
    • IIIC: Associated arterial injury requiring repair, regardless of soft tissue damage extent. (Highest risk of amputation).
Clinical Features:
  • Visible break in the skin overlying a fracture site.
  • Bone may or may not be protruding.
  • Bleeding, soft tissue injury.
  • Pain, deformity, swelling.
Initial Management (Pre-hospital / ED):
  1. ABCDE assessment (if major trauma).
  2. Control Haemorrhage: Direct pressure. Avoid blind clamping. Tourniquet if life-threatening uncontrolled bleeding.
  3. Neurovascular Assessment: Document status distal to injury carefully.
  4. Remove Gross Contaminants: If easily removable. Do not extensively probe or wash wound in ED (this is for theatre).
  5. Cover Wound: Sterile saline-soaked gauze and sterile dressing.
  6. Photographs: Useful for documentation before covering.
  7. Realign & Splint: Grossly realign deformed limb to improve comfort and perfusion, then splint to immobilise. Recheck neurovascular status after splinting.
  8. Analgesia: IV opioids.
  9. IV Antibiotics: Administer ASAP (within 1 hour of injury if possible, as per local protocol). Typically broad-spectrum (e.g., co-amoxiclav or cefuroxime +/- metronidazole, depending on contamination grade).
  10. Tetanus Prophylaxis: Check status and give booster/immunoglobulin as needed.
  11. NBM (Nil By Mouth): In anticipation of surgery.
  12. URGENT ORTHOPAEDIC REFERRAL.

Definitive Management: Surgical debridement (excision of dead/contaminated tissue), irrigation, fracture stabilisation (internal or external fixation), and soft tissue coverage, usually within 24 hours (ideally sooner for high-grade injuries).

Compartment Syndrome

A condition where increased pressure within a closed fascial compartment compromises circulation and function of tissues (muscles, nerves) within that compartment. Can lead to irreversible muscle necrosis and nerve damage if not treated promptly.

Pathophysiology:
  • Increased intracompartmental pressure (due to oedema, bleeding, external compression).
  • Pressure exceeds capillary perfusion pressure -> tissue hypoxia and ischaemia.
  • Common sites: Lower leg (anterior, lateral, deep posterior, superficial posterior compartments), forearm, foot, thigh, buttock.
Causes:
  • Fractures (especially tibia, forearm).
  • Soft tissue crush injuries.
  • Burns.
  • Prolonged limb compression (e.g., unconscious patient, tight casts/bandages).
  • Reperfusion injury after revascularisation.
  • Intensive muscle use (exertional compartment syndrome – usually chronic, but can be acute).
  • Bleeding disorders or anticoagulation.
Clinical Features (The “6 P’s” – though not all present early):
  1. Pain out of Proportion: EARLY and MOST RELIABLE symptom. Severe, deep, throbbing pain, often poorly localised, not relieved by analgesia or immobilisation. Worsens with passive stretch of muscles in the affected compartment.
  2. Paraesthesia: Numbness, tingling (early sign of nerve ischaemia).
  3. Pallor: Skin may appear pale (less reliable).
  4. Pulselessness: LATE and OMINOUS sign. Peripheral pulses may be present until very late stages as compartment pressure rarely exceeds arterial pressure. Relying on absence of pulse delays diagnosis.
  5. Paralysis/Paresis: Weakness or inability to move digits/limb (late sign).
  6. Poikilothermia (coolness): Less reliable. Compartment may feel tense and warm initially due to inflammation.

Key Examination Finding: Pain on PASSIVE STRETCH of the muscles within the affected compartment. The compartment will also feel tense and tender on palpation (“wood-like” or “rock-hard”).

Diagnosis:
  • Primarily a CLINICAL DIAGNOSIS. High index of suspicion is crucial.
  • Compartment Pressure Measurement: Can confirm diagnosis if clinical signs are equivocal. Needle inserted into compartment connected to pressure monitor. Absolute pressure >30-40 mmHg, or differential pressure (Diastolic BP – Compartment Pressure) <20-30 mmHg is concerning. (Normal compartment pressure is 0-10 mmHg).
Management: LIMB-THREATENING EMERGENCY
  1. URGENT ORTHOPAEDIC REFERRAL / SURGICAL CONSULTATION.
  2. Remove all constricting dressings, casts, splints down to the skin.
  3. Elevate limb ONLY to level of the heart (not above, as this can reduce arterial inflow).
  4. Analgesia (opioids).
  5. Maintain systemic blood pressure (avoid hypotension).
  6. Definitive Treatment: Urgent Fasciotomy. Surgical incision of the fascia overlying all affected compartments to relieve pressure. Wounds often left open initially and closed later or skin grafted.

Delay in fasciotomy (>6-8 hours from onset of ischaemia) can result in irreversible muscle necrosis, nerve damage, Volkmann’s ischaemic contracture (forearm), rhabdomyolysis, renal failure, and potential need for amputation.

Neurovascular Injury with Fracture/Dislocation

Fractures and dislocations can directly injure or compress adjacent nerves and blood vessels, leading to ischaemia or neurological deficit.

Mechanisms of Injury:
  • Laceration by sharp bone fragments.
  • Stretching or traction injury.
  • Compression by displaced bone, haematoma, or swelling.
  • Vascular spasm.
  • Intimal tear leading to thrombosis.
High-Risk Injuries for Neurovascular Compromise:
  • Knee Dislocation: High risk of popliteal artery injury (up to 30-40%). Potential for peroneal nerve injury. THIS IS A VASCULAR EMERGENCY UNTIL PROVEN OTHERWISE.
  • Supracondylar Fracture of Humerus (Children): Risk to brachial artery, median nerve, radial nerve.
  • Shoulder Dislocation (Anterior): Axillary nerve injury common (regimental badge sensation loss, deltoid weakness). Axillary artery injury rare but serious.
  • Hip Dislocation (Posterior): Sciatic nerve injury.
  • Femoral Shaft Fracture: Potential for femoral artery injury (though often well-protected).
  • Tibial Plateau Fracture: Popliteal artery, peroneal nerve.
  • Any significantly displaced long bone fracture or major joint dislocation.
Clinical Features:
  • Vascular Injury (“Hard” signs mandate urgent surgical exploration):
    • Absent distal pulses.
    • Active haemorrhage / expanding haematoma.
    • Signs of distal ischaemia (Pallor, Pain, Pulselessness, Paraesthesia, Paralysis, Poikilothermia – the 6 P’s of acute limb ischaemia).
    • Palpable thrill or audible bruit.
    “Soft” signs (warrant further investigation e.g., Doppler USS, CT Angiogram):
    • Diminished pulses (compared to other side).
    • History of significant bleeding at scene.
    • Proximity of injury to major artery.
    • Associated nerve injury.
  • Nerve Injury:
    • Paraesthesia (numbness, tingling) in nerve distribution.
    • Weakness or paralysis of muscles supplied by the nerve.
    • Loss of sensation in dermatome.
    • Specific nerve palsy signs (e.g., wrist drop for radial nerve, foot drop for common peroneal nerve).
Management:
  1. ABCDEs in trauma.
  2. Careful and repeated neurovascular assessment distal to injury. Document findings meticulously. Compare with uninjured limb.
  3. Attempt gentle closed reduction and realignment of fracture/dislocation PROMPTLY. This may restore perfusion or relieve nerve compression. Reassess neurovascular status immediately after.
  4. If pulses do not return after reduction, or hard signs of vascular injury persist: URGENT VASCULAR SURGERY & ORTHOPAEDIC CONSULTATION. Further imaging (e.g., CT angiogram) may be needed but should not delay intervention if critical ischaemia.
  5. Splint limb after reduction.
  6. If open fracture, manage as per open fracture protocol.
  7. Nerve injuries are often monitored initially (neuropraxia/axonotmesis may recover). However, if nerve deficit occurs after manipulation, or is associated with an open injury or vascular injury requiring repair, surgical exploration may be needed.
⚠️ Knee Dislocation

Assume popliteal artery injury in ALL knee dislocations until proven otherwise. Even if pulses return after reduction, a CT angiogram is often mandatory due to risk of intimal tear and delayed thrombosis. Compartment syndrome is also a high risk.

Cauda Equina Syndrome (CES)

CES is a rare but serious neurological condition caused by compression of the cauda equina nerve roots (lumbar and sacral nerve roots) within the spinal canal, distal to the conus medullaris (L1/L2).

Causes:
  • Large Central Lumbar Disc Herniation (Most Common): Typically at L4/L5 or L5/S1.
  • Spinal stenosis.
  • Spinal tumours (primary or metastatic).
  • Trauma (fractures, haematoma).
  • Infection (epidural abscess, discitis).
  • Inflammatory conditions (e.g., ankylosing spondylitis).
  • Iatrogenic (e.g., post-spinal surgery or anaesthesia).
Clinical Features (Can be variable and evolve):

Suspect CES in patients with back pain/sciatica AND any of the following RED FLAG symptoms:

  • Bilateral Sciatica or Leg Weakness/Sensory Loss: Often severe and may be progressive.
  • Saddle Anaesthesia/Paraesthesia: Altered sensation in the perineal area (buttocks, inner thighs, genitalia – S2-S5 dermatomes). Ask specifically: “Does it feel different when you wipe yourself after using the toilet?”
  • Bladder Dysfunction:
    • Urinary Retention (Most common): Difficulty initiating micturition, poor stream, overflow incontinence, loss of sensation of bladder fullness. Post-void residual volume >200ml is significant.
    • Urinary Incontinence (less common initially).
  • Bowel Dysfunction:
    • Faecal incontinence (due to loss of anal sphincter tone).
    • Constipation or loss of sensation of rectal fullness.
  • Sexual Dysfunction: Erectile dysfunction, loss of vaginal sensation.
  • Loss of Anal Tone / Reduced Anal Sensation on digital rectal examination (DRE). (DRE is essential if CES suspected).

CES can be classified as:

  • CES-S (Suspected): Concerning symptoms but no objective neurological signs yet (e.g., bilateral leg pain, some subjective sensory change). Requires close monitoring/urgent review.
  • CES-I (Incomplete): Subjective urinary difficulties (altered stream, sensation) +/- altered perineal sensation, but still has voluntary bladder control. Sphincter disturbance is partial. Better prognosis with early surgery.
  • CES-R (Retention): Established urinary retention (painless, overflow incontinence) and absent perineal sensation. Poorer prognosis even with surgery.
Management: NEUROSURGICAL EMERGENCY
  1. High index of suspicion based on red flag symptoms.
  2. Focused neurological examination: Lower limb power, sensation (especially perineal), reflexes, anal tone (DRE). Assess for palpable bladder/post-void residual (bladder scan).
  3. If CES is suspected: URGENT (IMMEDIATE) REFERRAL to on-call Neurosurgery or Spinal Orthopaedic team.
  4. Patient should be kept NBM and flat (or as comfortable) if awaiting transfer/surgery.
  5. Urgent MRI scan of the lumbosacral spine is the investigation of choice to confirm diagnosis and identify cause of compression.
  6. Definitive Treatment: Urgent Surgical Decompression (e.g., laminectomy, discectomy) ideally within 24-48 hours of symptom onset for best chance of neurological recovery, especially for CES-I. For CES-R, surgery is still often done but recovery is less predictable.

Delay in diagnosis and treatment can lead to permanent paralysis, urinary/faecal incontinence, and sexual dysfunction.

Septic Arthritis

Septic arthritis is an infection within a joint space. It’s a destructive condition that can rapidly damage articular cartilage, leading to permanent joint dysfunction if not treated promptly.

Pathophysiology:
  • Bacteria (most common), fungi, or viruses enter the joint space.
  • Routes of Infection:
    • Haematogenous spread (most common): From a distant site of infection (e.g., skin, UTI, respiratory tract).
    • Direct inoculation: Trauma, surgery, intra-articular injection.
    • Contiguous spread: From adjacent osteomyelitis or soft tissue infection.
  • Inflammatory response within the joint -> pus formation, release of proteolytic enzymes -> cartilage destruction.
Common Causative Organisms:
  • Staphylococcus aureus (Most common overall).
  • Streptococcus species (e.g., S. pyogenes, S. pneumoniae).
  • Neisseria gonorrhoeae (common in sexually active young adults – often migratory polyarthralgia then monoarthritis, tenosynovitis, rash).
  • Gram-negative bacilli (e.g., E. coli, Pseudomonas) – more common in elderly, immunocompromised, IV drug users.
  • Kingella kingae – common in young children (<4 years).
Risk Factors:
  • Pre-existing joint disease (e.g., rheumatoid arthritis, osteoarthritis, gout).
  • Prosthetic joint (periprosthetic joint infection – PJI).
  • Diabetes mellitus.
  • Immunosuppression (steroids, biologics, HIV).
  • IV drug use.
  • Recent joint surgery or injection.
  • Skin infection or breach.
  • Extremes of age (very young, elderly).
Clinical Features:
  • Acute onset of a single hot, swollen, erythematous, and severely painful joint (classic monoarthritis). Most common joints: Knee, hip, shoulder, wrist, ankle.
  • Severe pain on ANY movement of the joint (active or passive). Patient holds joint rigidly still.
  • Fever and systemic upset (malaise, rigors) – may be absent in elderly or immunocompromised.
  • Inability to weight-bear if lower limb joint affected.
  • Effusion often present.

Kocher Criteria for Septic Arthritis in Children (Hip): (1 point for each; ≥3 points high probability)

  1. Non-weight bearing on affected side.
  2. ESR >40 mm/hr.
  3. Fever >38.5°C.
  4. WBC count >12,000/mm3.
Diagnosis:
  • Joint Aspiration (Arthrocentesis): CRUCIAL DIAGNOSTIC STEP.
    • Perform BEFORE starting antibiotics if possible (unless patient septic/critically ill).
    • Send synovial fluid for:
      • Gram stain and microscopy (WBC count, differential).
      • Culture and sensitivity (aerobic, anaerobic, consider gonococcal culture if relevant).
      • Crystal analysis (to exclude gout/pseudogout, which can mimic septic arthritis).
    • Typical synovial fluid findings in septic arthritis: Turbid/purulent appearance, WBC count often >50,000/mm3 (mostly neutrophils), positive Gram stain (~50-70%), positive culture (~70-90%).
  • Blood Tests: FBC (leukocytosis), ESR/CRP (markedly elevated). Blood cultures (may be positive in ~50%).
  • Imaging:
    • X-rays: Often normal in early stages. May show soft tissue swelling, joint space widening (effusion), or later, joint space narrowing, erosions, osteomyelitis. Useful to exclude fracture/other pathology.
    • Ultrasound: Useful to detect effusion and guide aspiration, especially for deep joints like hip.
    • MRI/CT: More sensitive for early changes, osteomyelitis, abscesses, but not usually first-line for diagnosis.
Management: ORTHOPAEDIC EMERGENCY
  1. URGENT ORTHOPAEDIC REFERRAL / ADMISSION.
  2. Joint Aspiration (as above).
  3. Empirical IV Antibiotics: Start immediately after joint aspiration (or before if patient is septic/critically ill or aspiration delayed). Choice based on likely organisms and local guidelines (e.g., Flucloxacillin +/- Gentamicin, or Vancomycin if MRSA risk, Ceftriaxone if gonococcal suspected). Tailor to culture results.
  4. Surgical Drainage/Washout: Often required, especially for large joints (hip, knee, shoulder), if purulent aspirate, positive Gram stain, or poor response to antibiotics. Can be done via arthroscopy or open arthrotomy. Repeated washouts may be necessary.
  5. Pain Relief: Opioids.
  6. Splinting: Immobilise joint in functional position for comfort initially.
  7. Prolonged Antibiotic Course: Typically several weeks (e.g., 2-4 weeks IV then oral, total 4-6 weeks or longer), guided by clinical response and inflammatory markers.
  8. Early mobilisation and physiotherapy once infection controlled to prevent contractures and regain function.

Major Joint Dislocations (Shoulder, Hip, Knee)

A dislocation is a complete displacement of the articular surfaces of a joint. Requires prompt reduction to relieve pain, prevent neurovascular compromise, and limit further soft tissue damage.

General Principles:
  • Mechanism: Usually significant trauma, but can occur with less force if underlying joint instability or ligamentous laxity.
  • Presentation: Severe pain, deformity, inability to move the joint.
  • Assessment: Careful neurovascular examination distal to the dislocation is paramount BEFORE and AFTER reduction. X-rays to confirm dislocation and rule out associated fractures.
  • Management: Prompt closed reduction, often under procedural sedation and analgesia (PSA) or general anaesthesia. Immobilisation post-reduction, followed by rehabilitation. Surgical intervention may be needed if closed reduction fails, if associated unstable fracture, or for recurrent dislocations.
1. Shoulder Dislocation:
  • Most common major joint dislocation.
  • Anterior Dislocation (~95%):
    • Mechanism: Fall on outstretched hand (FOOSH) with abduction, extension, external rotation.
    • Presentation: Arm held in slight abduction and external rotation. Squared-off shoulder (loss of deltoid contour), humeral head palpable anteriorly. Severe pain.
    • Neurovascular: Risk of axillary nerve injury (check regimental badge sensation, deltoid function). Axillary artery injury rare.
    • X-rays: AP and axillary lateral or scapular-Y view. Associated fractures: Bankart lesion (anteroinferior glenoid rim), Hill-Sachs lesion (posterolateral humeral head compression fracture).
    • Reduction: Various techniques (e.g., Kocher’s, Stimson’s, Milch).
    • Post-reduction: Sling, then physiotherapy.
  • Posterior Dislocation (~5%): Often missed.
    • Mechanism: Seizure, electric shock, direct trauma to anterior shoulder.
    • Presentation: Arm held in adduction and internal rotation. Anterior shoulder may look flat, coracoid prominent. Limited external rotation and abduction.
    • X-rays: AP may look normal (“lightbulb sign” of humeral head). Axillary view essential.
2. Hip Dislocation:
  • Requires significant force (e.g., MVA, fall from height). ORTHOPAEDIC EMERGENCY due to risk of AVN of femoral head.
  • Posterior Dislocation (~90%):
    • Mechanism: Axial load on flexed hip and knee (e.g., dashboard injury).
    • Presentation: Limb is shortened, adducted, and internally rotated. Severe pain.
    • Neurovascular: Risk of sciatic nerve injury (check foot dorsiflexion/plantarflexion, sensation).
  • Anterior Dislocation (~10%):
    • Mechanism: Forced abduction, external rotation, extension.
    • Presentation: Limb in abduction, external rotation, and may appear longer or flexed.
    • Neurovascular: Femoral nerve/vessel injury possible.
  • Management: URGENT closed reduction (ideally within 6 hours) under anaesthesia to reduce AVN risk. CT scan post-reduction to assess congruity and look for loose bodies/acetabular fractures.
3. Knee Dislocation:
  • High-energy injury, often multi-ligamentous. LIMB-THREATENING EMERGENCY.
  • High risk of POPLITEAL ARTERY INJURY (up to 30-40%), even if pulses palpable after spontaneous reduction.
  • Common peroneal nerve injury also frequent.
  • Presentation: Gross deformity, instability, severe pain. May have spontaneously reduced but knee remains grossly unstable or swollen.
  • Management:
    • Immediate assessment of vascular status. If signs of ischaemia, attempt immediate reduction.
    • Urgent orthopaedic and vascular surgery consultation.
    • Ankle-Brachial Pressure Index (ABPI/ABI): If <0.9, suggests arterial injury.
    • CT Angiogram often mandatory even if pulses present, to rule out intimal tear.
    • Surgical repair of artery if injured. External fixation of knee often initially. Later, complex ligament reconstruction.
    • High risk of compartment syndrome.

Pelvic Fractures (Unstable)

Unstable pelvic fractures are usually caused by high-energy trauma (e.g., MVA, fall from height, crush injury) and can be associated with life-threatening haemorrhage and injuries to pelvic organs.

Significance: POTENTIALLY LIFE-THREATENING
  • Pelvis is a ring structure; a break in one part often means a break or ligamentous disruption elsewhere.
  • Huge potential for retroperitoneal and intrapelvic bleeding (venous plexus, arterial branches).
  • Associated injuries: Bladder, urethra, rectum, vagina, nerves (lumbosacral plexus).
Classification (Simplified – based on stability):
  • Stable Fractures: e.g., isolated pubic rami fractures, isolated iliac wing fractures. Pelvic ring intact. Usually managed conservatively.
  • Unstable Fractures: Disruption of the pelvic ring in at least two places. High risk of haemodynamic instability.
    • Lateral Compression (LC): Side impact.
    • Anteroposterior Compression (APC – “Open Book”): Frontal impact or forced external rotation of hemipelvis. Symphysis pubis diastasis, sacroiliac joint disruption. Can cause massive bleeding.
    • Vertical Shear (VS): Fall from height landing on one leg. Vertical displacement of one hemipelvis. Highly unstable.
    • Combined Mechanism.
Clinical Features:
  • Severe pain in pelvis, hips, back, groin.
  • Inability to weight-bear or move legs.
  • Haemodynamic instability (hypotension, tachycardia) if significant bleeding.
  • Deformity, swelling, bruising over pelvis, perineum, scrotum.
  • Leg length discrepancy. Rotational deformity of limb.
  • Blood at urethral meatus, haematuria, inability to void (suggests urethral/bladder injury).
  • Vaginal or rectal bleeding.
  • Neurological deficits in lower limbs.
Initial Management (ATLS Principles – often in Resuscitation Bay):
  1. ABCDE approach. Prioritise airway, breathing, circulation.
  2. Haemorrhage Control:
    • Pelvic Binder or Sheet Wrap: Applied at level of greater trochanters to “close the book” and reduce pelvic volume, tamponading bleeding. Critical early intervention.
    • IV access (2 large bore cannulas), aggressive fluid resuscitation, blood transfusion (activate massive haemorrhage protocol if indicated).
  3. Avoid unnecessary movement or log-rolling once suspected, as this can dislodge clots and worsen bleeding. Limit pelvic examination to one careful assessment if needed.
  4. Analgesia.
  5. X-ray Pelvis (AP view): Part of primary survey in trauma.
  6. Further Imaging: CT scan of chest/abdomen/pelvis with contrast is gold standard for assessing fracture pattern, associated injuries, and identifying source of bleeding.
  7. FAST scan may show intra-abdominal free fluid.
  8. Catheterisation (avoid if blood at meatus – suspect urethral injury, may need suprapubic).
  9. URGENT ORTHOPAEDIC & TRAUMA TEAM INVOLVEMENT.
Definitive Management:
  • Haemodynamically unstable patients may require: Angioembolisation to control arterial bleeding, or direct surgical packing.
  • Fracture stabilisation: External fixation (initially), followed by internal fixation (ORIF) once patient stable.
  • Management of associated injuries.

Necrotising Fasciitis

Necrotising fasciitis (NF) is a rare but rapidly progressive, life-threatening bacterial infection of the fascia and subcutaneous tissues, leading to extensive tissue necrosis.

Pathophysiology:
  • Bacteria spread along fascial planes, causing thrombosis of subcutaneous blood vessels, leading to tissue ischaemia and necrosis.
  • Types:
    • Type I (Polymicrobial): Most common. Mixed aerobic and anaerobic bacteria (e.g., Streptococci, Staphylococci, Enterobacteriaceae, Bacteroides). Often in patients with comorbidities (diabetes, PVD).
    • Type II (Monomicrobial): Usually Group A Streptococcus (Streptococcus pyogenes – “flesh-eating bacteria”), sometimes with Staphylococcus aureus. Can occur in healthy individuals after minor trauma.
    • Type III: Gram-negative marine organisms (e.g., Vibrio vulnificus) from seawater exposure.
    • Type IV: Fungal (rare, immunocompromised).
Risk Factors:
  • Diabetes mellitus.
  • Peripheral vascular disease.
  • Immunosuppression (steroids, chemotherapy, HIV).
  • Chronic kidney disease, liver cirrhosis.
  • IV drug use.
  • Recent surgery or trauma (even minor).
  • Obesity, malnutrition.
Clinical Features:
  • Early Signs (often non-specific, can mimic cellulitis):
    • Area of erythema, warmth, swelling, tenderness.
    • Pain OUT OF PROPORTION to visible skin changes – KEY RED FLAG.
    • Flu-like symptoms, fever, malaise, tachycardia.
  • Later / Progressive Signs:
    • Skin becomes dusky, violaceous, or indurated (hardened).
    • Formation of bullae (blisters), often haemorrhagic.
    • Skin necrosis (black, gangrenous areas).
    • Crepitus (subcutaneous gas produced by anaerobic bacteria) – palpable or audible.
    • Loss of sensation over affected area (due to nerve destruction).
    • Rapidly spreading erythema/induration.
    • Systemic toxicity: Sepsis, hypotension, tachycardia, altered mental status, multi-organ failure.
Diagnosis:
  • Primarily a CLINICAL DIAGNOSIS requiring a very high index of suspicion. Do not wait for all signs to develop.
  • LRINEC Score (Laboratory Risk Indicator for Necrotizing Fasciitis): Uses CRP, WBC, Hb, Sodium, Creatinine, Glucose. Can help stratify risk but should not replace clinical judgement or delay surgery.
  • Imaging:
    • X-ray: May show subcutaneous gas (late sign).
    • CT/MRI: Can show fascial thickening, fluid collections, gas. Useful if diagnosis uncertain but should not delay surgery if high clinical suspicion.
  • Definitive Diagnosis: Surgical Exploration. Finding “dishwater pus,” necrotic fascia that does not bleed, and easy separation of tissue planes with blunt dissection confirms diagnosis. Tissue samples for Gram stain, culture.
Management: SURGICAL & MEDICAL EMERGENCY – HIGH MORTALITY
  1. URGENT (IMMEDIATE) SURGICAL REFERRAL/CONSULTATION (General Surgery or Plastics, often with Orthopaedics if limb involved).
  2. Early and Aggressive Surgical Debridement: This is the cornerstone of treatment. Wide excision of all necrotic fascia and overlying tissue until healthy, bleeding tissue is reached. May require multiple debridements (“second look” operations). Amputation may be necessary.
  3. Broad-Spectrum IV Antibiotics: Start empirically ASAP. Cover Gram-positives, Gram-negatives, and anaerobes (e.g., Piperacillin-tazobactam OR Meropenem + Clindamycin +/- Vancomycin). Clindamycin inhibits toxin production. Tailor to culture results.
  4. Haemodynamic Support: Aggressive fluid resuscitation, vasopressors if needed (manage septic shock).
  5. Intensive Care Unit (ICU) admission often required.
  6. Nutritional support, wound care (e.g., VAC dressings), pain control.
  7. Consider Intravenous Immunoglobulin (IVIG) for streptococcal toxic shock syndrome if present.
  8. Hyperbaric oxygen therapy (HBOT) is controversial, may be adjunct in some centres but should not delay surgery.

Flashcards: Orthopaedic Emergencies

Click on each card to reveal the answer.

What is the most important initial step in managing an open fracture in the ED?

(Click to flip)

Answer

Administer IV antibiotics ASAP (within 1 hour ideally), cover wound with sterile dressing, control haemorrhage, realign/splint, analgesia, tetanus prophylaxis, and URGENT orthopaedic referral.

What is the earliest and most reliable symptom of acute compartment syndrome?

(Click to flip)

Answer

Pain out of proportion to the injury, especially pain on passive stretch of the muscles in the affected compartment.

A patient has a knee dislocation. Even if pulses are palpable after reduction, what investigation is often mandatory due to high risk of vascular injury?

(Click to flip)

Answer

CT Angiogram of the affected limb to rule out popliteal artery intimal tear or other occult vascular injury.

Name three red flag symptoms for Cauda Equina Syndrome.

(Click to flip)

Answer

Any 3 of: Bilateral sciatica/leg weakness, Saddle anaesthesia/paraesthesia, Bladder dysfunction (especially retention), Bowel dysfunction (incontinence/constipation), Sexual dysfunction, Loss of anal tone.

What is the most crucial diagnostic investigation for suspected septic arthritis?

(Click to flip)

Answer

Joint Aspiration (Arthrocentesis) for synovial fluid microscopy (WBC count, differential, crystals), Gram stain, and culture & sensitivity.

What is the typical limb position in a posterior hip dislocation?

(Click to flip)

Answer

Limb is shortened, adducted, and internally rotated.

What is the cornerstone of treatment for necrotising fasciitis?

(Click to flip)

Answer

Early and aggressive surgical debridement of all necrotic tissue.

What is the immediate first aid measure for a suspected unstable pelvic fracture with haemodynamic instability in a trauma setting?

(Click to flip)

Answer

Application of a pelvic binder or sheet wrap at the level of the greater trochanters to reduce pelvic volume and tamponade bleeding.

Orthopaedic Emergencies Quiz

Test your knowledge with these urgent scenarios.

1. A patient presents with a tibial fracture and an overlying 2cm laceration after a motorcycle accident. The wound is contaminated with dirt. According to the Gustilo-Anderson classification, this is at least what type of open fracture?

A. Type I.
B. Type II.
C. Type IIIA.
D. Type IIIC.
Explanation: A wound >1cm with moderate soft tissue damage and contamination suggests at least a Type II open fracture. Type I is <1cm and clean. Type III involves extensive damage.

2. A patient with a tight cast on their forearm for a distal radius fracture complains of escalating pain unrelieved by opioids, and tingling in their fingers. Passive extension of their fingers elicits severe pain in the forearm. What is the most likely diagnosis?

A. Normal post-fracture pain.
B. Acute Compartment Syndrome.
C. Median nerve contusion.
D. Complex Regional Pain Syndrome.
Explanation: Pain out of proportion, paraesthesia, and severe pain on passive stretch of muscles in the compartment are classic signs of acute compartment syndrome, a surgical emergency. The tight cast is a major risk factor.

3. A patient involved in a high-speed MVA has a suspected knee dislocation. The knee has been reduced in ED, and distal pulses are now palpable. What is the most appropriate next step regarding vascular assessment?

A. Observation only, as pulses are present.
B. Urgent CT Angiogram of the limb.
C. Ankle-Brachial Pressure Index (ABPI) measurement only.
D. Doppler ultrasound of peripheral vessels.
Explanation: Due to the high risk (30-40%) of popliteal artery injury (including intimal tears which can thrombose later) with knee dislocations, a CT angiogram is often mandatory even if pulses are present post-reduction to definitively rule out vascular injury. ABPI and Doppler can be adjuncts.

4. A 50-year-old man presents with acute low back pain radiating to both legs, numbness in his perineal area (“saddle” area), and difficulty passing urine for the past 12 hours. What is the immediate management priority?

A. Prescribe strong analgesia and bed rest.
B. Arrange an outpatient MRI scan within 2 weeks.
C. Urgent (immediate) referral to neurosurgery/spinal surgery for suspected Cauda Equina Syndrome.
D. Catheterise and send MSU.
Explanation: The combination of bilateral leg symptoms, saddle anaesthesia, and bladder dysfunction is highly suggestive of Cauda Equina Syndrome, a neurosurgical emergency requiring urgent surgical decompression.

5. A 65-year-old diabetic patient presents with a 2-day history of a very painful, hot, swollen, and erythematous right knee, with inability to weight-bear and fever of 38.5°C. What investigation is most crucial for diagnosis?

A. X-ray of the knee.
B. MRI scan of the knee.
C. Urgent joint aspiration (arthrocentesis) for synovial fluid analysis.
D. Blood tests including FBC, ESR, CRP.
Explanation: This presentation is classic for septic arthritis. Joint aspiration for synovial fluid microscopy, Gram stain, and culture is the most critical diagnostic step to confirm infection and guide antibiotic therapy. Blood tests and X-ray are supportive.

6. A young adult presents after a fall onto an outstretched hand. Their arm is held in slight abduction and external rotation, and the shoulder contour appears “squared-off”. What is the most likely diagnosis?

A. Anterior shoulder dislocation.
B. Posterior shoulder dislocation.
C. Acromioclavicular joint separation.
D. Clavicle fracture.
Explanation: The mechanism (FOOSH with abduction/external rotation) and clinical presentation (arm position, squared-off shoulder) are classic for an anterior shoulder dislocation.

7. A patient is rescued from a collapsed building and has a suspected unstable “open book” (APC Type II/III) pelvic fracture. They are hypotensive and tachycardic. What is the most important initial intervention specific to the pelvic injury?

A. Log-rolling the patient to assess their back.
B. Application of a pelvic binder or sheet wrap.
C. Urgent X-ray of the pelvis.
D. Inserting a urinary catheter.
Explanation: For an unstable pelvic fracture with haemodynamic instability, immediate application of a pelvic binder or sheet wrap at the level of the greater trochanters is crucial to reduce pelvic volume and help tamponade bleeding. This should be done as part of “C” in ABCDE.

8. A patient presents with rapidly spreading erythema on their leg, severe pain described as “burning” and out of proportion to the visible skin changes, and some new blister formation. They feel systemically unwell with a fever. What serious infection must be urgently considered?

A. Simple cellulitis.
B. Erysipelas.
C. Necrotising Fasciitis.
D. Deep Vein Thrombosis.
Explanation: Pain out of proportion to skin signs, rapid progression, bullae formation, and systemic toxicity are red flags for necrotising fasciitis, a surgical emergency requiring urgent debridement and broad-spectrum antibiotics.

9. Which nerve is most commonly injured in an anterior shoulder dislocation?

A. Median nerve.
B. Radial nerve.
C. Axillary nerve.
D. Musculocutaneous nerve.
Explanation: The axillary nerve is most at risk with anterior shoulder dislocations, leading to potential deltoid weakness and loss of sensation over the “regimental badge” area.

10. The definitive treatment for acute compartment syndrome is:

A. Elevation of the limb above the heart.
B. High-dose intravenous corticosteroids.
C. Urgent surgical fasciotomy.
D. Application of a tighter compression bandage.
Explanation: Urgent surgical fasciotomy is the definitive treatment for acute compartment syndrome, involving incision of the fascia to relieve pressure within the affected compartment(s).

Splinting & Casting Overview (T&O Focus)

Splinting and casting are essential non-operative techniques used in Trauma & Orthopaedics to immobilize injured or surgically treated limbs. They provide support, protect healing tissues, reduce pain, and prevent further injury or deformity. A sound understanding of their principles, application, and potential complications is crucial for T&O trainees.

Principles & Indications for Splinting & Casting

General Principles:

  • Immobilisation: To stabilize fractures, reduce dislocations, support sprains/strains, or protect post-operative repairs.
  • Pain Relief: By restricting movement of injured parts.
  • Protection: Shielding injured area from further trauma.
  • Reduction of Swelling: By providing gentle compression and support (though primary aim is not compression). Elevation is key.
  • Maintenance of Alignment: Holding fractures or dislocations in an acceptable position for healing.
  • Rest: Allowing inflamed or injured tissues to heal.
  • Prevention of Deformity: In certain chronic conditions or during healing.

Indications for Splinting:

Splints are non-circumferential immobilizers, allowing for swelling. Often used for acute injuries or when regular inspection/access is needed.

  • Acute Fractures & Dislocations: Initial immobilization, especially when swelling is anticipated (e.g., in A&E before definitive casting or surgery).
  • Severe Sprains & Strains.
  • Soft Tissue Injuries requiring rest and support.
  • Post-operative immobilisation where swelling is expected or early ROM is planned by removing splint for exercises.
  • Tendonitis / Bursitis (to rest inflamed area).
  • Prophylactic use in certain conditions to prevent injury or deformity.

Indications for Casting:

Casts are circumferential immobilizers providing more rigid support. Typically applied once acute swelling has subsided or for definitive fracture management.

  • Definitive treatment of stable, non-displaced or acceptably reduced fractures.
  • Immobilisation after closed reduction of fractures or dislocations.
  • Correction of deformities (e.g., serial casting for clubfoot).
  • Prolonged immobilisation where rigid support is necessary.
  • Post-operative protection after certain surgical procedures.
💡 Splint First, Cast Later (Often)

In acute injuries with anticipated swelling (e.g., ankle fracture, distal radius fracture), a temporary splint (e.g., backslab) is often applied initially. A circumferential cast is typically applied once acute swelling has resolved (usually after 5-7 days) to avoid compartment syndrome or pressure sores.

Materials Used in Splinting & Casting

Padding Materials:

  • Stockinette:
    • Knitted cotton material applied directly to skin as first layer. Protects skin, absorbs moisture.
    • Available in various widths. Should be wrinkle-free.
  • Soft Roll / Cast Padding (e.g., Soffban, Orthoban):
    • Cotton, synthetic, or blended material applied over stockinette. Provides cushioning, protects bony prominences, absorbs exudate.
    • Applied in layers (usually 2-3), overlapping by ~50%. More padding over bony prominences.
    • Must be applied smoothly without constrictions or wrinkles.
  • Felt or Foam Padding: Denser material for extra protection over specific pressure points (e.g., malleoli, ulnar styloid, olecranon).

Immobilising Materials (Splint/Cast Material):

  • Plaster of Paris (POP):
    • Gypsum (calcium sulphate hemihydrate) impregnated bandages. Activated by dipping in water. Exothermic reaction as it sets.
    • Advantages: Molds very well to contours, relatively inexpensive, radiolucent (allows X-ray visualisation).
    • Disadvantages: Heavy, not water-resistant (disintegrates if wet), slower setting/curing time (takes ~24-72 hours to reach full strength), can crack/break more easily than fibreglass.
  • Fibreglass / Synthetic Casts (e.g., Scotchcast, Dynacast):
    • Polyurethane resin impregnated fibreglass or polyester tape. Activated by water.
    • Advantages: Lightweight, strong, durable, water-resistant (padding underneath still gets wet unless waterproof liner used), faster setting time (functional strength in ~20-30 mins), available in various colours.
    • Disadvantages: More expensive, less moldable than POP (requires more skill to apply well over complex contours), can have sharper edges if not finished properly. More difficult to remove with traditional cast saw if applied too thickly.
  • Thermoplastic Materials: Used for custom splints, especially in hand therapy. Become pliable when heated, rigid when cooled.

Other Supplies:

  • Water: Lukewarm for POP (affects setting time – warmer = faster), room temperature for fibreglass.
  • Gloves: Essential for hygiene and to protect hands from materials.
  • Apron: To protect clothing.
  • Bandage Scissors / Shears.
  • Cast Saw: For removing circumferential casts. Oscillating blade designed to cut rigid material but not soft skin (care still needed).
  • Cast Spreader.
  • Crepe Bandages / Elastic Bandages: To secure splints.

Common Types of Splints

Splints provide non-circumferential support and are often used for acute injuries.

Upper Limb Splints:

  • Volar Slab/Splint (Forearm/Wrist):
    • Applied to volar aspect of forearm and wrist.
    • Indications: Wrist sprains, carpal tunnel syndrome (night splint), stable distal radius/ulna fractures (e.g., Colles’ fracture initially), soft tissue injuries.
    • Position: Wrist usually in slight extension (10-20°) or neutral.
  • Dorsal Slab/Splint (Forearm/Wrist): Similar to volar but applied dorsally. Less common for wrist, sometimes used for extensor tendon injuries.
  • U-Slab / Sugar Tong Splint (Forearm/Wrist):
    • U-shaped slab from dorsal metacarpal heads, around elbow, to volar metacarpal heads. Prevents pronation/supination and wrist flexion/extension.
    • Indications: Distal radius/ulna fractures, forearm fractures.
  • Thumb Spica Splint:
    • Incorporates the thumb, immobilising CMC and MCP joints.
    • Indications: Scaphoid fractures (suspected or confirmed), thumb metacarpal/phalangeal fractures, De Quervain’s tenosynovitis, thumb UCL injury (Skier’s/Gamekeeper’s thumb).
    • Position: Thumb in slight abduction (“wine glass” position).
  • Ulnar Gutter Splint:
    • Immobilises ulnar aspect of hand/wrist, including 4th and 5th metacarpals/phalanges.
    • Indications: Boxer’s fracture (5th metacarpal neck), 4th/5th metacarpal shaft/phalangeal fractures.
  • Radial Gutter Splint: Immobilises radial aspect, 2nd/3rd metacarpals/phalanges.
  • Posterior Slab/Splint (Elbow/Humerus):
    • Applied to posterior aspect of arm, crossing elbow.
    • Indications: Elbow dislocations (post-reduction), supracondylar fractures, distal humerus fractures, olecranon fractures.
    • Position: Elbow usually at 90° flexion.
  • Collar and Cuff / Broad Arm Sling: For shoulder/proximal humerus injuries, clavicle fractures.

Lower Limb Splints:

  • Posterior Leg Slab / Backslab (Ankle/Lower Leg):
    • Applied to posterior aspect of lower leg, from below knee to toes.
    • Indications: Ankle fractures, severe ankle sprains, distal tibia/fibula fractures, foot fractures.
    • Position: Ankle usually at 90° (neutral).
  • Stirrup Splint / U-Slab (Ankle):
    • U-shaped slab from medial leg, under heel, up lateral leg. Often combined with posterior slab for more stability (Sugar-tong for ankle). Provides mediolateral stability.
  • Knee Immobiliser / Zimmer Splint / Cricket Pad Splint:
    • Prefabricated splint providing knee extension.
    • Indications: Acute knee injuries (ligament sprains, meniscal tears, patellar dislocation), post-operative immobilisation.
  • Long Leg Posterior Slab: From upper thigh to toes, for knee/femur injuries.
  • Thomas Splint (Historical/Traction): Used for femoral shaft fractures for transport/temporary traction. Less common now with modern external fixation/intramedullary nailing.

Common Types of Casts

Casts are circumferential and provide more rigid immobilisation, typically applied after acute swelling subsides.

Upper Limb Casts:

  • Short Arm Cast (SAC):
    • Extends from below elbow (proximal forearm) to metacarpal heads. Allows elbow flexion/extension.
    • Indications: Stable distal radius/ulna fractures, carpal bone fractures (excluding scaphoid), some metacarpal base fractures.
  • Long Arm Cast (LAC):
    • Extends from upper arm to metacarpal heads, immobilising elbow (usually at 90°).
    • Indications: Elbow fractures/dislocations, forearm fractures (both bones), complex wrist fractures requiring elbow immobilisation.
  • Thumb Spica Cast:
    • Incorporates the thumb, similar to thumb spica splint but circumferential.
    • Indications: Definitive treatment for scaphoid fractures, Bennett’s fracture, thumb ligament injuries.
  • Metacarpal Cast / Gutter Cast (Ulnar or Radial): Circumferential version of gutter splints for stable metacarpal/phalangeal fractures.

Lower Limb Casts:

  • Short Leg Cast (SLC) / Below Knee Cast (BKC):
    • Extends from below tibial tuberosity to metatarsal heads (or including toes – “toe plate”).
    • Indications: Stable ankle fractures, metatarsal fractures, some tarsal fractures. Can be weight-bearing (SLCWB) or non-weight-bearing (SLCNWB).
  • Long Leg Cast (LLC) / Above Knee Cast (AKC):
    • Extends from upper thigh to metatarsal heads, immobilising knee (usually in slight flexion, e.g., 10-20°).
    • Indications: Tibial shaft fractures, knee ligament injuries requiring immobilisation, some distal femur/proximal tibia fractures.
  • Patellar Tendon Bearing (PTB) Cast / Sarmiento Cast:
    • Specialised cast for tibial fractures, designed to allow some weight-bearing by distributing load through patellar tendon and tibial flares. Allows ankle/knee motion if hinged.
  • Cylinder Cast:
    • Immobilises knee only, from upper thigh to just above malleoli. Allows ankle motion.
    • Indications: Patellar fractures/dislocations, some knee ligament injuries.

Specialised Casts:

  • Hip Spica Cast: Immobilises hip and femur. Used mainly in children for femoral fractures or hip dysplasia.
  • Body Cast / Risser Jacket: For spinal conditions/fractures.
  • Serial Casts: Applied sequentially to gradually correct deformity (e.g., clubfoot, joint contractures).

General Application Principles for Splints & Casts

Proper technique is essential to ensure effective immobilisation and avoid complications.

Preparation:

  • Patient Consent & Explanation: Explain procedure, reason, duration, care, warning signs.
  • Gather Materials: Stockinette, padding, splint/cast material, water, gloves, apron.
  • Patient Positioning: Position patient comfortably with limb accessible and supported in desired position of function or reduction. Assistant may be needed.
  • Examine Limb: Check skin integrity, neurovascular status (distal pulses, sensation, motor function, capillary refill) BEFORE application. Document findings.

Application Steps (General):

  1. Stockinette: Apply smoothly, extending beyond planned splint/cast margins. Cut holes for thumb if needed. Avoid wrinkles.
  2. Padding (Soft Roll):
    • Apply evenly, overlapping each turn by ~50%. Usually 2-3 layers.
    • More padding over bony prominences (malleoli, olecranon, ulnar styloid, fibular head, anterior superior iliac spine) to prevent pressure sores.
    • Apply snugly but not too tight. Avoid constrictions.
  3. Splint/Cast Material Application:
    • POP: Submerge bandage in lukewarm water until bubbling stops, gently squeeze out excess water (don’t wring aggressively). Unroll onto limb, smoothing each layer.
    • Fibreglass: Dip in room temperature water (or as per manufacturer), squeeze. Apply with gentle tension, conforming to limb. Sets quickly.
    • Number of layers depends on desired strength and location (e.g., 8-12 layers POP for lower limb slab, 4-6 for upper limb).
    • Moulding (Crucial for POP): While setting, gently mould the cast to anatomical contours and to maintain fracture reduction (e.g., three-point moulding for certain fractures). Use palms, not fingertips, to avoid indentations.
  4. Positioning: Maintain desired joint position throughout application and setting (e.g., ankle at 90°, wrist in slight extension, elbow at 90°). Splint/cast should immobilise joint above and below injury where appropriate.
  5. Finishing:
    • Fold back stockinette and padding over proximal and distal edges for smooth, comfortable rim. Secure with final layer of cast material if needed.
    • Ensure no sharp edges or rough spots.
    • For splints: Secure with crepe bandage or elastic wrap (snugly, but not too tight, allowing for swelling).

Post-Application:

  • Recheck Neurovascular Status: Immediately after application and regularly. Document.
  • X-ray: If applied for fracture, check position in cast/splint (post-reduction films).
  • Patient Education: Provide detailed cast/splint care instructions and warning signs for complications (see Patient Advice section).
  • Arrange Follow-up.
⚠️ Avoid Constriction!

Padding should be adequate, especially over bony prominences. Splints should not be wrapped too tightly. Circumferential casts should only be applied once acute swelling has subsided or with extreme caution if applied acutely (consider bivalving or univalving if concerns).

Complications of Splinting & Casting

While effective, splints and casts can lead to complications if not applied or managed correctly.

Early Complications:

  • Compartment Syndrome: (ORTHOPAEDIC EMERGENCY)
    • Increased pressure within fascial compartment due to swelling constrained by tight cast/splint.
    • Symptoms: Pain out of proportion (especially on passive stretch), paraesthesia, tense compartment. Pulses often intact initially.
    • Action: Immediately remove/bivalve entire cast/splint down to skin, split padding. Elevate limb to heart level. Urgent senior review.
  • Pressure Sores / Ulcers:
    • From excessive pressure over bony prominences (malleoli, heel, olecranon, ulnar styloid, fibular head) due to inadequate padding or poor moulding.
    • Symptoms: Localised pain (“burning”), redness, blistering, skin breakdown.
    • Prevention: Adequate padding, smooth application, avoid indentations.
  • Nerve Compression / Palsy:
    • Pressure on superficial nerves (e.g., common peroneal nerve at fibular head causing foot drop; ulnar nerve at elbow).
    • Symptoms: Numbness, paraesthesia, weakness in nerve distribution.
    • Action: Loosen/adjust cast/splint, monitor. May require removal.
  • Vascular Insufficiency: From cast being too tight. Swelling, pain, pallor, coolness, diminished pulses, slow capillary refill. Action: Split cast immediately.
  • Contact Dermatitis / Skin Irritation: From materials or moisture.
  • Thermal Injury (Burns): From exothermic reaction of POP setting (especially if water too hot, cast too thick, or limb wrapped in impermeable material).
  • Loss of Reduction / Fracture Displacement: If cast is loose, poorly moulded, or broken. Requires X-ray monitoring.

Late Complications:

  • Joint Stiffness / Contractures: Due to prolonged immobilisation. Early controlled ROM and physiotherapy are key once safe.
  • Muscle Atrophy: Disuse of immobilised muscles.
  • Osteopenia / Disuse Osteoporosis: Reduced bone density from lack of weight-bearing/loading.
  • Complex Regional Pain Syndrome (CRPS): Chronic pain, swelling, stiffness, skin changes, vasomotor instability. Early mobilisation can help prevent.
  • Thromboembolism (DVT/PE): Especially with lower limb immobilisation. Consider thromboprophylaxis in high-risk patients.
  • Cast Dermatitis / Fungal Infections: Due to moisture under cast.
  • Non-union / Malunion of Fracture.
  • Psychological Issues: Frustration, anxiety, dependence.

Cast Removal & Aftercare

Cast Removal:

  • Timing: Determined by type of injury, healing progress (clinical and radiological), and treating surgeon’s plan.
  • Method: Cast Saw:
    • Oscillating blade cuts rigid cast material but generally not skin (if used correctly).
    • Explain procedure to patient (noisy, vibration, may feel warm). Reassure them.
    • Cut along two opposite sides of cast (bivalving). Protect underlying padding/skin, especially over bony prominences. Use cast spreaders to open.
    • Remove padding and stockinette.
  • POP casts: Can sometimes be soaked off if appropriate (e.g., small children, some wrist casts), but saw is standard.

Immediate Post-Removal:

  • Inspect Skin: Check for pressure areas, dryness, maceration.
  • Assess Joint: Range of motion (often stiff initially), stability, tenderness.
  • Neurovascular Status: Recheck.
  • Radiological Assessment: X-rays often taken out of cast to confirm healing/alignment.

Aftercare & Rehabilitation:

  • Skin Care: Gentle washing, moisturising. Skin may be dry, flaky, sensitive. Avoid vigorous scrubbing.
  • Swelling Management: Elevation, compression (e.g., tubigrip) may still be needed initially.
  • Pain Management: Simple analgesia as needed.
  • Physiotherapy / Rehabilitation: CRUCIAL.
    • Gradual restoration of range of motion.
    • Strengthening exercises for atrophied muscles.
    • Proprioception and balance training.
    • Functional activities, gradual return to normal ADLs and sport.
  • Patient Education: Expected recovery, importance of rehab exercises, gradual return to activity, signs of problems.
  • Follow-up: As per orthopaedic plan.

Patient Advice & Education with Splint/Cast

Clear instructions are vital for patient compliance and early detection of complications.

Care of Splint/Cast:

  • Keep it Dry:
    • POP casts must be kept completely dry (will weaken/disintegrate). Use waterproof covers for bathing/showering (e.g., Limbo, plastic bag sealed with tape).
    • Fibreglass casts are water-resistant, but padding underneath will get wet and can cause skin maceration/irritation. Advise to dry thoroughly if it gets wet (e.g., hairdryer on cool setting). Waterproof liners are an option but not always used.
  • Do NOT insert objects inside the cast/splint to scratch (risk of skin injury, infection). Use hairdryer on cool setting for itching, or tap on cast. Antihistamines may help.
  • Do NOT break off edges or modify the cast/splint.
  • Check skin around edges for rubbing or irritation.
  • Report any cracks, soft spots, or damage to the cast/splint.

Activity & Limb Use:

  • Elevation: Elevate the injured limb frequently, especially in first 48-72 hours, to reduce swelling (e.g., arm in sling above heart level, leg on pillows).
  • Movement of Unimmobilised Joints: Encourage regular movement of fingers/toes, shoulder/hip (if not immobilised) to prevent stiffness and aid circulation.
  • Weight-Bearing Status: Clarify if non-weight-bearing (NWB), touch weight-bearing (TWB), partial weight-bearing (PWB), or weight-bearing as tolerated (WBAT). Provide crutches/aids if needed and ensure safe use.
  • Avoid activities that could damage cast or re-injure limb.

Warning Signs / When to Seek Urgent Medical Attention: (RED FLAGS)

Instruct patient to seek immediate medical advice (e.g., return to A&E, contact fracture clinic/ortho team) if any of the following occur:

  • Severe or increasing pain not relieved by prescribed analgesia or elevation (could be compartment syndrome, pressure sore).
  • Swelling, numbness, tingling, or pins and needles in fingers/toes of affected limb.
  • Inability to move fingers/toes.
  • Fingers/toes becoming very cold, pale, blue, or dusky.
  • Foul smell or discharge/oozing from under the cast (suggests infection or sore).
  • Cast feels too tight, especially after swelling increases.
  • Cast feels too loose, slips, or is broken/damaged.
  • Fever, chills, or feeling generally unwell (could indicate infection).
  • Any new localised pain or burning sensation under the cast.

Follow-up:

  • Ensure patient knows date, time, and location of follow-up appointment (e.g., fracture clinic).
  • Reinforce importance of attending follow-up.

Provide written information leaflets if available.

Flashcards: Splinting & Casting

Click on each card to reveal the answer.

What is the main difference between a splint and a cast in terms of circumferential support?

(Click to flip)

Answer

Splints are non-circumferential (allowing for swelling). Casts are circumferential, providing more rigid immobilisation.

What is the most serious acute complication of a tight circumferential cast applied to a freshly injured limb?

(Click to flip)

Answer

Acute Compartment Syndrome.

Name two advantages of fibreglass cast material over Plaster of Paris (POP).

(Click to flip)

Answer

Any two of: Lightweight, stronger/more durable, water-resistant (material itself), faster setting time, various colours.

What does “bivalving” a cast mean and why is it done?

(Click to flip)

Answer

Cutting the cast longitudinally into two halves (anterior and posterior shells). Done to relieve pressure if compartment syndrome is suspected, to allow for swelling, or for removal.

A thumb spica cast/splint is commonly used for suspected or confirmed fractures of which carpal bone?

(Click to flip)

Answer

Scaphoid bone.

What is the primary purpose of soft roll/padding under a cast?

(Click to flip)

Answer

To provide cushioning and protect skin and bony prominences from pressure, and to absorb moisture.

If a patient in a leg cast complains of new numbness and tingling in their toes and inability to move them, what should be done?

(Click to flip)

Answer

Urgent medical review. The cast may be too tight (risk of neurovascular compromise or compartment syndrome) and may need to be split or removed.

What advice should be given about getting a Plaster of Paris (POP) cast wet?

(Click to flip)

Answer

It must be kept completely dry as water will weaken and disintegrate it. Use waterproof covers for bathing.

Splinting & Casting Quiz (T&O Focus)

Test your knowledge on splinting and casting principles.

1. A patient with an acute distal radius fracture has a volar backslab applied in A&E. What is the primary reason for using a splint (backslab) rather than a full circumferential cast at this stage?

A. Splints are cheaper than full casts.
B. To accommodate for anticipated swelling and reduce risk of compartment syndrome.
C. Splints provide more rigid immobilisation than casts.
D. Splints are easier for the patient to remove at home.
Explanation: Splints are non-circumferential, allowing space for swelling that typically occurs after an acute fracture, thereby reducing the risk of neurovascular compromise or compartment syndrome.

2. Which of the following is a key advantage of synthetic (fibreglass) cast material over Plaster of Paris (POP)?

A. It is more easily moulded to complex body contours.
B. It is significantly lighter and stronger once set.
C. It produces less heat during the setting process.
D. It is less expensive.
Explanation: Fibreglass casts are lighter, stronger, more durable, and set faster than POP. POP is generally better for moulding. Both produce exothermic heat. Fibreglass is more expensive.

3. A patient in a below-knee cast for an ankle fracture complains of severe, burning pain over their lateral malleolus, unrelieved by simple analgesia. The cast feels tight in that area. What is the most immediate concern?

A. Muscle atrophy.
B. Deep Vein Thrombosis (DVT).
C. Pressure sore or impending ulcer over a bony prominence.
D. Non-union of the fracture.
Explanation: Localised, severe, burning pain under a cast, especially over a bony prominence, is highly suggestive of excessive pressure leading to a pressure sore or ulcer. This requires urgent attention (windowing or splitting the cast).

4. To immobilise a scaphoid fracture effectively, which joint(s) must typically be included in the cast or splint?

A. Wrist only.
B. Wrist and elbow.
C. Wrist and thumb (CMC and MCP joints).
D. Wrist and all finger MCP joints.
Explanation: A thumb spica cast/splint immobilises the wrist and the carpometacarpal (CMC) and metacarpophalangeal (MCP) joints of the thumb to effectively stabilise the scaphoid.

5. A patient in a long arm cast (immobilising the elbow) reports new onset numbness and tingling in their little finger and ulnar half of the ring finger. This suggests potential compression of which nerve?

A. Median nerve.
B. Radial nerve.
C. Ulnar nerve.
D. Axillary nerve.
Explanation: The ulnar nerve supplies sensation to the little finger and ulnar half of the ring finger. It can be compressed at the elbow (cubital tunnel) by a poorly applied or tight long arm cast.

6. When applying padding (soft roll) for a cast, what is the recommended overlap for each turn?

A. No overlap, edge to edge.
B. Approximately 25%.
C. Approximately 50%.
D. Approximately 75-100%.
Explanation: Padding should be applied by overlapping each turn by about 50% to ensure even cushioning and avoid ridges or constrictions.

7. What is the main principle of “moulding” a Plaster of Paris cast during application?

A. To make the cast as thin as possible for lightness.
B. To conform the cast to anatomical contours and maintain fracture reduction using gentle, sustained pressure with palms.
C. To rapidly accelerate the setting time of the plaster.
D. To create decorative patterns on the cast surface.
Explanation: Moulding is crucial for POP casts to fit snugly, support anatomical arches, and maintain the position of a reduced fracture (e.g., three-point fixation). Using palms prevents indentations that could cause pressure points.

8. Which of these is a critical piece of advice to give a patient regarding care of their new POP cast?

A. It’s okay to get it slightly damp as it will dry out.
B. Insert a knitting needle to relieve itching.
C. Elevate the limb frequently, especially in the first 48-72 hours.
D. Try to remove the cast for short periods to air the skin.
Explanation: Elevation is crucial to reduce swelling and pain, especially in the acute phase. POP casts must be kept dry. Nothing should be inserted inside. Casts should not be removed by the patient.

9. A “sugar tong” splint for a forearm/wrist injury primarily aims to limit which movements?

A. Finger flexion and extension only.
B. Pronation and supination of the forearm, as well as wrist flexion/extension.
C. Elbow flexion and extension only.
D. Shoulder abduction and adduction.
Explanation: A sugar tong splint wraps around the elbow from dorsal to volar aspect of the forearm/wrist, effectively blocking pronation/supination and significantly limiting wrist motion.

10. What is a common late complication of prolonged immobilisation in a cast?

A. Acute compartment syndrome.
B. Thermal injury from cast setting.
C. Joint stiffness and muscle atrophy.
D. Skin maceration from getting the cast wet immediately.
Explanation: Prolonged immobilisation leads to disuse, resulting in joint stiffness, muscle weakness/atrophy, and potentially osteopenia. Compartment syndrome and thermal injury are acute complications. Skin maceration from immediate wetting is also an early issue.